HEALTH ASSESSMENT HESI

Pataasin ang iyong marka sa homework at exams ngayon gamit ang Quizwiz!

Which finding does the nurse expect when performing tactile fremitus? 1. A vibration of sounds that are equal bilaterally 2. A change in muscle tone when the patient inhales and exhales, indicating weakness 3. The symmetric rise of the thorax as the patient speaks, indicating equal expansion 4. Coughing triggered by patient speech, indicating bronchial irritation

1. A vibration of sounds that are equal bilaterally

which finding is considered abnormal during late pregnancy 1. watery vaginal discharge 2. hemorroids 3. lordosis 4. abdominal striae

1. watery vaginal discharge

A woman's waist circumference is 32 inches and her hip circumference is 29 inches. Her waist-to-hip ratio is _____.

1.10 Calculate waist-to-hip by dividing waist measurement by the hip measurement, 32 inches/29 = 1.10.

what is the nurse assessing when measuring from the patient's symphysis pubis to the top of the funds? 1. fetal development 2. fetal lie and position 3. attitude of the fetus 4. gestational age

4. gestational age

To obtain information that will help distinguish whether the client's fatigue is cardiac in nature, what question should the RN ask the client? "Why do you feel your fatigue is related to your age?" This question is unlikely to elicit information related to the cause of the fatigue. "Can you describe the quality of your fatigue?" The quality of fatigue is difficult to describe and typically non-specific to the cause of the fatigue. "What do you do when you feel tired?" The client's response to fatigue is unlikely to elicit information related to the cause of the fatigue. "At what time of day do you feel most fatigued?" Fatigue related to stress or depression may be worse in the morning or may be present all day, while fatigue related to decreased cardiac output may worsen in the evening. Tomas tells the RN that he gets progressively more fatigued throughout the day.

"At what time of day do you feel most fatigued?" Fatigue related to stress or depression may be worse in the morning or may be present all day, while fatigue related to decreased cardiac output may worsen in the evening. Tomas tells the RN that he gets progressively more fatigued throughout the day.

A 45-year-old woman tells the nurse she is distressed by the presence of dark, coarse hair on her face that has recently developed. What is the nurse's most appropriate response to this patient? a. "This is simple vellus hair and it will decrease in amount over time." b. "Some women in your cultural group normally have dark hair on their faces." c. "This is unusual; female hair distribution should be limited to arms, legs, and pubis." d. "Coarse dark hair could result from hormonal changes such as from menopause."

"Coarse dark hair could result from hormonal changes such as from menopause." A This response is not true. This example describes hirsutism, a condition associated with an increase in the growth of facial, body, or pubic hair in women. It does not decrease over time and the hair is not vellus. B Although it is true that women of some cultural groups normally have dark hair on the face, women in these cultural groups have darker facial hair most of their adult lives; the patient in this item has a new onset of hirsutism. C It is not true that female hair distribution should normally be limited to arms, legs, and pubis. Women do have hair on their faces and other areas. D Coarse, dark hair on the face describes hirsutism, an increase in the growth of facial, body, or pubic hair in women that can be associated with menopause or an endocrine disorder.

Which patient's description of pain is consistent with injury to a bone?

"Deep, dull, and boring"

Narrowing of the bronchi creates which adventitious sound? 1. Wheeze 2. Crackles 3. Rhonchi 4. Pleural friction rub

1. Wheeze

A female patient admitted with fluid retention has been in diuretic therapy to remove fluid. She weighed 187 lb on admission. Today she weighs 179 lb. Since admission, this patient has lost _____ L from fluid loss.

3.6 1 kg (2.2 lb) = 1 L; 187 - 179 = 8 lb weight loss divided by 2.2 = 3.6 L.

20. When the nurse is evaluating the reliability of a patient's responses, which of these statements would be correct? The patient: A. has a history of drug abuse and therefore is not reliable. B. provided consistent information and therefore is reliable. C. smiled throughout interview and therefore is assumed reliable. D. would not answer questions concerning stress and therefore is not reliable.

B. provided consistent information and therefore is reliable. Page: 50. A reliable person always gives the same answers, even when questions are rephrased or are repeated later in the interview. The other statements are not correct.

The nurse should use a(n) __to auscultate the chest and abdomen. A. Doppler B. stethoscope C. audiometer D. transilluminator

B. stethoscope

How should this finding be documented? Within normal limits. This is not a correct statement. Funnel chest. The client with a funnel chest will exhibit depression of the sternum, with narrowing of the anteroposterior diameter. Barrel chest. A barrel chest is the description for an increased anteroposterior (AP) to transverse ratio. The normal ratio is 1:2, so a 1:1 ratio represents an increased ratio. Thoracic scoliosis. Scoliosis is a lateral curvature of the spine.

Barrel chest. A barrel chest is the description for an increased anteroposterior (AP) to transverse ratio. The normal ratio is 1:2, so a 1:1 ratio represents an increased ratio.

When reporting to the supervisor, the nurse tells the supervisor that the client's pressure sore developed because the client had a stone in her shoe that she couldn't feel. How should the nurse summarize this initial report by the client? Foot paralysis bilaterally. Paralysis refers to loss of movement, rather than loss of sensation. Discomfort in both feet. Discomfort is overly vague and not the best description of what the client reported. Numbing pain in her feet. This description is inaccurate. Bilateral paresthesia in the feet. Paresthesia refers to abnormal sensation, such as numbness or tingling, so this is the best terminology to describe the client's report of numbness and lack of feeling in her feet.

Bilateral paresthesia in the feet. Paresthesia refers to abnormal sensation, such as numbness or tingling, so this is the best terminology to describe the client's report of numbness and lack of feeling in her feet.

55. The nurse has discovered decreased skin turgor in a patient and knows that this is an expected finding in which of these conditions? A) Severe obesity B) Childhood growth spurts C) Severe dehydration D) Connective tissue disorders such as scleroderma

C) Severe dehydration Page: 215. Decreased skin turgor is associated with severe dehydration or extreme weight loss.

99. When using a Doppler ultrasonic stethoscope, the nurse recognizes arterial flow when which sound is heard? A) Low humming sound B) Regular "lub, dub" pattern C) Swishing, whooshing sound D) Steady, even, flowing sound

C) Swishing, whooshing sound Pages: 515-516. When using the Doppler ultrasonic stethoscope, the pulse site is found when one hears a swishing, whooshing sound.

With the patient in a supine position, how does a nurse test the external rotation of the patient's right hip? a. Asking the patient to move the right leg laterally with the right knee straight b. Asking the patient to flex the right knee and turn medially toward the left side (inward) c. Asking the patient to place the right heel on the left patella d. Asking the patient to raise the right leg straight up and perpendicular to the body

C Placing the right heel on the left patella externally rotates the right hip.

What equipment should the nurse obtain prior to completing this measurement? Measuring tape. A measuring tape is not needed to measure the ankle brachial index. Blood pressure cuff. A blood pressure cuff along with a Doppler probe is used to obtain the systolic blood pressure in the lower extremity. To calculate the ankle brachial index (ABI), this value is compared with the systolic blood pressure in the upper extremity. Pulse oximeter. A pulse oximeter is not needed to measure the ankle brachial index. Tourniquet. A tourniquet is not needed to measure the ankle brachial index.

Blood pressure cuff. A blood pressure cuff along with a Doppler probe is used to obtain the systolic blood pressure in the lower extremity. To calculate the ankle brachial index (ABI), this value is compared with the systolic blood pressure in the upper extremity.

While assessing the client's nails, it is most important for the nurse to follow-up on which assessment finding? Brittle nail surface. Brittle or ridged nail surfaces may be the result of iron deficiency. This finding warrants follow-up assessment related to the client's nutritional status. Ragged cuticles. Ragged cuticles may indicate that the client chews her cuticles. Another finding is of higher priority for further assessment by the nurse. Firm nail base. A firm nail base is a normal finding. A spongy nail base is abnormal, and may be found if nailbed clubbing is present. Irregular nail edges. Irregular nail edges may indicate that the client chews her fingernails. Another finding is of higher priority for further assessment by the nurse.

Brittle nail surface. Brittle or ridged nail surfaces may be the result of iron deficiency. This finding warrants follow-up assessment related to the client's nutritional status.

Earlier, Ms. Yong reported that she often experiences unilateral left knee pain. The nurse palpates Ms. Yong's left knee and notes the presence of a small amount of swelling. Which sign should the nurse attempt to elicit? Bulge sign. A positive Bulge sign is found when very small amounts of fluid move across the joint. When swelling is palpated, the nurse may attempt to elicit this sign. Battle sign. A positive Battle sign is found in persons with certain brain injuries. Allis sign. A positive Allis sign may indicate hip dislocation in an infant. Tinel's sign. A positive Tinel's sign is found in persons with carpal tunnel syndrome.

Bulge sign. A positive Bulge sign is found when very small amounts of fluid move across the joint. When swelling is palpated, the nurse may attempt to elicit this sign.

A nurse performing an abdominal examination on a 37 year old woman would document which finding as abnormal?

Bulges observed when coughing

A patient has multiple solid, red, raised lesions on her legs and groin that she describes as "itchy insect bites." How does the nurse document these lesions? 1Wheals. 2Bullae. 3Tumors. 4Plaques.

Bullae

84. During auscultation of the lungs of an adult patient, the nurse notices the presence of bronchophony. The nurse should assess for signs of which condition? A) Airway obstruction B) Emphysema C) Pulmonary consolidation D) Asthma

C) Pulmonary consolidation Page: 446. Pathologic conditions that increase lung density, such as pulmonary consolidation, will enhance transmission of voice sounds, such as bronchophony. See Table 18-7.

Based on Mrs. Green's recent history of loss of consciousness and falling, what additional assessment takes priority? Blood pressure and heart rate and rhythm. Hypotension and bradycardia can cause a loss of consciousness. Bradycardia may also be a sign of increased intracranial pressure.If the client has hypertension, it places her at increased risk for a hemorrhagic stroke. If the client has cardiac irregularity, such as atrial fibrillation, she should be evaluated and treated to prevent an embolic stroke. Pedal pulse volume. The pedal pulse volume is unlikely to provide essential data related to the client's recent loss of consciousness.The carotid pulse would be a better choice for assessment because this would assess blood flow to the brain through the carotid arteries. The RN would hear a "bruit" if there was an occlusion in the carotid artery. Deep tendon reflexes. Altered deep tendon reflexes are of less priority upon initial assessment than other assessment data. Two-point discrimination. This assessment of sensory function is of less priority upon initial assessment than other assessment data.

Blood pressure and heart rate and rhythm. Hypotension and bradycardia can cause a loss of consciousness. Bradycardia may also be a sign of increased intracranial pressure.If the client has hypertension, it places her at increased risk for a hemorrhagic stroke. If the client has cardiac irregularity, such as atrial fibrillation, she should be evaluated and treated to prevent an embolic stroke.

What question does a nurse ask a patient with a history of pancreatitis who is complaining of abdominal pain?

How severe is the pain on a scale of 0-10?

When continuing to assess the abdominal area, the nurse hears a swishing sound. In what area would this sound be heard? Femoral artery. This area would produce a swishing sound that occurs during systole. Epigastric area. The midline areas would not produce this sound. Umbilical area. The umbilical area would not produce this sound. Right quadrants. The right quadrants would not produce this sound.

Femoral artery. This area would produce a swishing sound that occurs during systole.

While Ms. Yong moves her legs through the various forms of range of motion, the nurse grades her muscle strength. To indicate 100% muscle strength, the nurse assesses for movement against which? Gravity. When assessing muscle strength, the RN assesses for movement against gravity. Rest. Observation of the muscle at rest provides data about muscle tone. Light touch. Light touch is a level of stimuli used to assess neurological response rather than muscle strength. Pain. Pain is a level of stimuli used to assess neurological response rather than muscle strength. Resistance. When assessing muscle strength, the nurse assesses for movement against resistance. Grade 5, or 100% muscle strength, is present when the client demonstrates full range of motion against gravity and resistance.

Gravity. When assessing muscle strength, the RN assesses for movement against gravity. Resistance. When assessing muscle strength, the nurse assesses for movement against resistance. Grade 5, or 100% muscle strength, is present when the client demonstrates full range of motion against gravity and resistance.

Which is an expected finding of an abdominal examination of an adult?

High-pitched gurgles every 5 to 15 seconds on auscultation

What action should the nurse take? Feel the inguinal area with the back of the hand. The back of the hand is useful in assessing skin temperature but is not useful in assessing for a bruit. Firmly compress the artery with the fingertips. This action will not enable the RN to assess for a bruit. Position a stethoscope over the artery. A bruit is a swooshing sound heard when blood flow through an artery is turbulent. It is heard by placing a stethoscope over the artery. Observe the site for bulges or swelling. This action will not enable the RN to assess for a bruit.

Position a stethoscope over the artery. A bruit is a swooshing sound heard when blood flow through an artery is turbulent. It is heard by placing a stethoscope over the artery.

The nurse notes that the wound is round and 0.5 cm in diameter. To assess for the presence of any undermining tracts, what action should the nurse implement? Note the amount and appearance of any drainage to help determine the depth. The amount and appearance of any drainage does not provide data about the depth of the wound and any undermining tracts. Gently irrigate the wound with sterile saline to help determine the depth. Wound irrigation does not provide an effective measurement of the depth of the wound and any undermining tracts. Insert a sterile, cotton-tipped applicator to measure the depth. A sterile, cotton-tipped applicator can be gently inserted to measure the depth of the wound and any undermining tracts. Use sterile forceps to apply sterile packing to help determine the depth. The application of packing does not provide an effective measurement of the depth of the wound and any undermining tracts.

Insert a sterile, cotton-tipped applicator to measure the depth. A sterile, cotton-tipped applicator can be gently inserted to measure the depth of the wound and any undermining tracts.

The nurse observes the overall hair distribution on Amanda's face and arms. There is visible hair growth on her forearms. She has thin eyelashes and eyebrows, and otherwise fine, downy facial hair. What action should the nurse take in response to these observations? Ask the client if the excessive hair growth on her arms is of concern to her. The presence of visible hair on the arms is not considered excessive in the adolescent. Note the absence of normal hair growth patterns on the client's face. The observed hair growth patterns are within normal limits. Document the areas of alopecia as an indication of the client's poor nutrition. There is no evidence of alopecia, or hair loss. Move on to the next area of assessment since the findings are within normal limits. The findings are within normal limits, so the nurse should continue the assessment.

Move on to the next area of assessment since the findings are within normal limits. The findings are within normal limits, so the nurse should continue the assessment.

When examining a 16-year-old male patient, the nurse notes multiple pustules and comedones on the face. The nurse recognizes that increased activity of which cells or glands produce these manifestations? 1Epidermal cells. 2Eccrine glands. 3Apocrine glands. 4Sebaceous glands.

Sebaceous

Locating the Angle of Louis is important to guide the nurse in next locating what area? Locating the Angle of Louis is important to guide the nurse in next locating what area? <img src="http://coursewareobjects.elsevier.com/objects/apply/common/image_off.gif"> Xiphoid process. The xiphoid process is located at the lower end of the sternum. The Angle of Louis, or sternal angle, is not a helpful landmark in locating the xiphoid process. Erb's point. Erb's point is located at the third intercostal space to the left of the sternum. This would not be the next landmark located after the nurse finds the Angle of Louis, or sternal angle. Clavicle. Because the clavicles are connected to the top of the manubrium of the sternum, the suprasternal notch is the best landmark for palpating the clavicles, not the Angle of Louis, or sternal angle. Second rib. The second ribs attach to the sternum at the Angle of Louis, or sternal angle. This landmark is located at the bottom of the manubrium of the sternum, is felt as a bony ridge, and is the point where the treachea bifurcates into the right and left stem bronchi.

Second rib. The second ribs attach to the sternum at the Angle of Louis, or sternal angle. This landmark is located at the bottom of the manubrium of the sternum, is felt as a bony ridge, and is the point where the treachea bifurcates into the right and left stem bronchi.

Which of the following findings is considered abnormal when conducting an exam on a 68-year-old woman?

Serous nipple drainage

A client with chest pain, dizziness, and vomiting for the last 2 hours is admitted for evaluation for Acute Coronary Syndrome (ACS). Which cardiac biomarker should the registered nurse (RN) anticipate to be elevated if the client experienced myocardial damage? Creatine Kinase (CK-MB). Serum troponin. Myoglobin. Ischemia modified albumin.

Serum troponin. Troponin is the most sensitive and specific test for myocardial damage. Troponin elevation is more specific than CK-MB.

A 58-year-old woman has found a small lump in her breast. Which of the following data from her history are risk factors for breast cancer?

She underwent radiation treatment for Hodgkin's disease at age 17.

The nurse next assesses the client's elbows. When comparing these joints bilaterally, for what should the nurse observe? Skin color. This is a characteristic that can be observed and provides data related to joints. Tympany. Tympany is a clear hollow sound heard upon percussion and is not observed during joint assessment. Contour. This is a characteristic that can be observed and provides data related to joints. Resonance. Resonance, the vibratory sound produced upon percussion, is not observed during joint assessment. Size. This is a characteristic that can be observed and provides data related to joints.

Skin color. This is a characteristic that can be observed and provides data related to joints. Contour. This is a characteristic that can be observed and provides data related to joints. Size. This is a characteristic that can be observed and provides data related to joints.

The registered nurse (RN) is assessing a client who was discharged home after management of chronic hypertension. Which equipment should the RN instruct the client to use at home? Exercise bicycle. Sphygmomanometer. Blood glucose monitor. Weekly medication box.

Sphygmomanometer. Self-awareness is the best way for a client to manage chronic hypertension, so the client should obtain a sphygmomanometer and learn how to monitor blood pressure daily and maintain a record.

How should the nurse identify this sound? Diastolic murmur. A diastolic murmur coincides with the S2 heart sound. Systolic murmur. Murmurs are often heard as a swooshing sound. Systolic murmurs coincide with the S1 heart sound. S4 heart sound. The S4 heart sound is not heard at the same time as S1. S3 heart sound. The S3 heart sound is not heard at the same time as S1.

Systolic murmur. Murmurs are often heard as a swooshing sound. Systolic murmurs coincide with the S1 heart sound.

a 15-year-old boy approaches the school nurse and describes how uncomfortable he is around the girls because most of them are taller than he is. The nurse counsels the boy that:

The growth spurt during adolescence occurs in girls 18-24 months before it occurs in boys

A nurse is palpating the lymph nodes of an 18-month-old toddler and finds enlarged postauricular and occipital nodes. What is the significance of this finding?

This is an normal finding at this age

A patient reports nausea and vomiting; and the nurse observes hand tremors, agitation, and sweating. In view of these findings, which additional data would the nurse need to collect?

When the patient last drank alcohol and how much was consumed.

During a symptom analysis the patient reports a pain that radiates from the right lateral thigh, over the knee, and around the right medical ankle. The nurse refers to the dermatome map to determine that the patient's description of pain is consistent with dysfunction if which spinal nerve

fourth lumbar L4

which technique does a nurse use to access the mental status of patients?

have them calculate the change to expect after making a purchase

What are expected findings of a newborn's vision that the nurse teaches to the parents?

he newborn is nearsighted and cannot see items unless they are close

Which techniques does the nurse use to test the triceps reflex

holds the patient's relaxed arm with elbow flexed at a 90 degree angle in one hand and strike the appropriate tendon just above the elbow with either end of the reflex hammer

During the health history, a patient reports having difficulty swallowing. Based on this report, which assessment technique does the nurse use to collect more data about the patient's ability to swallow

observe the rising of the soft palate when the patient says "ahh"

The registered nurse (RN) assesses a client's results for arterial blood gases who has emphysema. Which finding is consistent with respiratory acidosis? pH 7.32, pCO2 46 mmHg, HCO3 24 MEq/L. pH 7.45 , pCO2 37 mmHg, HCO3 24 mEq/L. pH 7.34, pCO2 36 mmHg, HCO3 21 mEq/L. pH 7.46, pCO2 35 mmHg, HCO3 28 mEq/L.

pH 7.32, pCO2 46 mmHg, HCO3 24 MEq/L. Normal ABG ranges are pH 7.35 to 7.45; pCO2 35 to 45 mmHg; HCO3 21 to 28 mEq/L, and pO2 80 to 100 mmHg. An ABG of pH 7.32, pCO2 46 mmHg, HCO3 24 MEq/L represents a client with respiratory acidosis which is characterized by: low pH, pCO2 higher than normal, and HCO3 within normal limits.

10. A patient tells the nurse that her stools have bright red blood in them. The nurse suspects which problem? 1. Gallbladder disease 2. Hemorrhoids 3. Rectal polyps 4. Upper intestinal bleeding

2. Hemorrhoids

The nurse is listening to the patient's heart at the left sternal border (LSB) at the second intraclavicular space (ICS). Which area is being ausculatated? a. Erb's point b. Mitral area c. Aortic area d. Pulmonic area

d. Pulmonic area

Which communication technique conveys genuine interest in what the patient has to say? 1. active listening 2. sitting close to the patient 3. Maintaining professional dress and conduct 4. Holding the patient's hand during the interview

1. active listening

what does the nurse assess for during each prenatal visit? 1. blood pressure 2. hemorrgoids 3. personal habits (smoking, alcohol consumption) 4. visual acuity

1. blood pressure

A female has been admitted to the emergency department with severe abdominal pain. She is lying on a stretcher quietly, with very little movement. Which patient response should the nurse anticipate when palpating this patient's abdomen? 1. Flashing of the face and neck 2. Guarding over the abdomen 3. Redness on the lower abdominal quadrants 4. Decreased peristalsis

2. Guarding over the abdomen

To determine what happened to the client when she experienced a loss of consciousness, the RN should ask Mrs. Green which questions? "Do you ever feel lightheaded or dizzy?" This could indicate poor cerebral perfusion due to hypotension or carotid occlusion, which could cause loss of consciousness. "Does the dizziness occur when you change positions?" Postural hypotension occurs with position changes and may cause a client to fall when moving from a lying to sitting position. "Did it feel like the room was suddenly spinning before you fell? This indicates vertigo, which is related to alterations of vestibular apparatus in the ear. If the nerve is damaged, the client may experience equilibrium and balance issues. "Do you have any problems with smell" This question will not elicit information about loss of consciousness; instead it may indicate cranial nerve dysfunction, sinusitis, infection, allergic rhinitis or smoking. "Can you stick out your tongue?" This question would be asked to evaluate the hypoglossal nerve (CN XII).

"Do you ever feel lightheaded or dizzy?" This could indicate poor cerebral perfusion due to hypotension or carotid occlusion, which could cause loss of consciousness. "Does the dizziness occur when you change positions?" Postural hypotension occurs with position changes and may cause a client to fall when moving from a lying to sitting position. "Did it feel like the room was suddenly spinning before you fell? This indicates vertigo, which is related to alterations of vestibular apparatus in the ear. If the nerve is damaged, the client may experience equilibrium and balance issues.

When eliciting data about possible neurological problems, which questions would the RN ask the client? "Do you have any numbness,tingling, or weakness in your extremities?" Sensory function is an important component of a neurological assessment because loss of sensation may indicate a stroke or neuropathy. "When you passed out, did you hit your head? If so, what part of your head did you hit?" Loss of consciousness, confusion, and intracranial bleeding can occur as a result of a head injury, so the RN should determine whether the client sustained a head injury. The RN needs to examine the client for raccoon eyes or a battle sign to rule out a skull fracture. Also, the RN should note and report any drainage from eyes, ears,and/or nose to make sure that it is not spinal fluid leaking. Check for "halo sign" on bed linens, which could also indicate CSF leakage. "Do you have any difficulty speaking or swallowing?" Speech or swallowing difficulties are changes that are associated with an increased risk of stroke. "Have you ever heard voices inside your head that no one else hears?" This addresses psychosis, delusions, and hallucinations, which is are addressed in a psychosocial rather than a neurological assessment. "Have you had any headaches? If so, describe frequency and location." Headaches can indicate hypertension or intracranial bleeding.

"Do you have any numbness,tingling, or weakness in your extremities?" Sensory function is an important component of a neurological assessment because loss of sensation may indicate a stroke or neuropathy. "When you passed out, did you hit your head? If so, what part of your head did you hit?" Loss of consciousness, confusion, and intracranial bleeding can occur as a result of a head injury, so the RN should determine whether the client sustained a head injury. The RN needs to examine the client for raccoon eyes or a battle sign to rule out a skull fracture. Also, the RN should note and report any drainage from eyes, ears,and/or nose to make sure that it is not spinal fluid leaking. Check for "halo sign" on bed linens, which could also indicate CSF leakage. "Do you have any difficulty speaking or swallowing?" Speech or swallowing difficulties are changes that are associated with an increased risk of stroke. "Have you had any headaches? If so, describe frequency and location." Headaches can indicate hypertension or intracranial bleeding.

The nurse has already observed that both of Lourdes' feet are cool and pale. What questions should the nurse ask Lourdes to obtain additional supporting data? "Do your toes or toenails ever look blue?" Cool skin temperature and pallor are signs of diminished arterial circulation. Cyanosis, a bluish color, of the tips of the toes or nail beds, is also an indicator of decreased arterial circulation. "After a bump, do you bruise easily?" Cool skin temperature and pallor are signs of diminished arterial circulation. Bruising easily may be an indicator of a clotting problem, but it is not a sign of diminished arterial circulation. "Are any of your veins bulging or crooked?" Bulging or crooked veins are typical of varicose veins but are not a sign of diminished arterial circulation. "Have you ever had a blood clot?" A history of a blood clot might cause venous insufficiency, contributing to diminished circulation unilaterally, rather than bilaterally. "Do you feel tingling, numbness, or burning sensations in your legs and feet?" Cool skin temperature, paresthesia, and pallor are signs of diminished arterial circulation. Cyanosis, a bluish color of the tips of the toes or of the nail beds is also an indicator of decreased arterial circulation.

"Do your toes or toenails ever look blue?" Cool skin temperature and pallor are signs of diminished arterial circulation. Cyanosis, a bluish color, of the tips of the toes or nail beds, is also an indicator of decreased arterial circulation. "Do you feel tingling, numbness, or burning sensations in your legs and feet?" Cool skin temperature, paresthesia, and pallor are signs of diminished arterial circulation. Cyanosis, a bluish color of the tips of the toes or of the nail beds is also an indicator of decreased arterial circulation.

The nurse questions Amanda about the observable moles on her skin. What question is most important to ask the client? "Have you ever tried to cover up your moles with makeup?" This question may elicit information related to the client's body image, but it is of less priority than another question. "Do the moles on your arms make you feel self-conscious?" This question may elicit information related to the client's body image, but it is of less priority than another question. "Do you have any moles on your abdomen or chest?" Determining the location of moles on the client's body is of less priority than other information. "Have any of your moles changed in size or appearance?" Because a change in the size or appearance of a mole is a danger sign for skin cancer and warrants a referral for medical evaluation, this is the most important question for the nurse to ask. In response to the nurse's question, Amanda shrugs and tells the nurse that although she acknowledges the presence of moles on her body, she doesn't pay much attention to them. She lifts her tee shirt to show the nurse her abdomen and back.

"Have any of your moles changed in size or appearance?" Because a change in the size or appearance of a mole is a danger sign for skin cancer and warrants a referral for medical evaluation, this is the most important question for the nurse to ask. In response to the nurse's question, Amanda shrugs and tells the nurse that although she acknowledges the presence of moles on her body, she doesn't pay much attention to them. She lifts her tee shirt to show the nurse her abdomen and back.

To learn about any history of intermittent claudication, what question should the nurse ask? "When you first stand up, do you feel dizzy or light-headed?" This question will elicit information about possible orthostatic hypotension. "Can you feel your pulse pounding after vigorous activity?" This question will elicit information about cardiac function. "Have you experienced any leg cramping or pain in your legs?" Claudication is cramp-like calf pain, associated with diminished blood supply to the leg muscles. When this pain occurs only at specific times, such as during activities, it is referred to as intermittent claudication. "Do you have an urge to move your legs a lot during the night?" This question will elicit information about possible restless leg syndrome

"Have you experienced any leg cramping or pain in your legs?" Claudication is cramp-like calf pain, associated with diminished blood supply to the leg muscles. When this pain occurs only at specific times, such as during activities, it is referred to as intermittent claudication.

During a history the patient says that she is so uncomfortable with her life that she wished that it were over. Which is an appropriate follow-up question from the nurse?

"Have you thought of hurting yourself?"

Which question is appropriate for a nurse to ask at the beginning of a mental health history?

"How are you feeling about yourself?"

What follow-up question by the nurse provides the best information about the client's claudication distance? "When did you first notice you were having leg cramps?" This question will not provide information about claudication distance. "How long have you been walking this same distance?" This question will not provide information about claudication distance. "On a 10-point scale, how would you rank your pain?" This question will not provide information about claudication distance. "How far do you walk before the leg cramps begin?" Claudication distance refers to the distance, such as blocks walked, or stairs climbed, that produces pain.

"How far do you walk before the leg cramps begin?" Claudication distance refers to the distance, such as blocks walked, or stairs climbed, that produces pain.

Which question is most important for the nurse to ask the client? "Do you tend to bite or chew your nails?" This question may elicit information related to the client's habits that the nurse may want to address, but another question is of greater importance. "What do you use to cleanse your skin?" This question may elicit general information about the client's self-care routines that the nurse may want to address, but another question is of greater importance. "How often do you use a tanning booth?" Excessive use of a tanning booth increases the risk for skin cancer. Therefore, this is the most important question for the nurse to ask the client. Amanda states that she goes to a tanning booth once or twice a month. "Do you use a hair coloring product?" This question may elicit information about the client's self-care routines, but is of less importance than another question.

"How often do you use a tanning booth?" Excessive use of a tanning booth increases the risk for skin cancer. Therefore, this is the most important question for the nurse to ask the client. Amanda states that she goes to a tanning booth once or twice a month.

The nurse is obtaining the mental health history of a new patient. What should the nurse include in the mental health history? (Select all that apply.)

1. The patient's description of self 2. A past medical history 3. The current medications the patient is taking

7. Which finding is expected during a rectal exam? 1. The rectal wall is smooth. 2. Severe pain is reported when the finger is introduced through the anus. 3. Hard stool is present in the rectum. 4. The anus is surrounded by white flat lesions.

1. The rectal wall is smooth.

Which statement reflects an expected developmental task of a 40-year-old man?

"My wife and I have divided up the chores with the children since we are both working"

What are the characteristics of lymph nodes in patients who have an acute infection? 1. They are enlarged and tender 2. They are round, rubbery, and mobile 3. They are hard, fixed, and painless 4. They are soft, mobile, and painless

1. They are enlarged and tender

Which breath sounds are expected over the posterior chest of an adult? 1. Vesicular 2. Bronchovesicular 3. Bronchial 4. Bronchoalveolar

1. Vesicular

5. A patient has a herpes lesion on her vulva. While examining her, the nurse should take which measures? 1. Wear examination gloves while in contact with the genitalia. 2. Place the patient in an isolation room. 3. Wash the genitalia with alcohol or povidone-iodine (Betadine) before the examination. 4. Inspect the genitalia only; reschedule the patient for a full examination after the lesion has healed.

1. Wear examination gloves while in contact with the genitalia.

A 58-year-old man seeks treatment for "recent breast enlargement." On examination, the nurse notes bilateral enlargement of the breasts. Which of the following questions asked by the nurse is most appropriate base on this finding?

"What medications are you currently taking?"

A patient complains of leg pain. Which question is pertinent to ask to gain additional information?

"What were you doing when the pain first occurred?"

Which question that Lourdes ask the nurse, indicate that more teaching needs to be done? "Should I use sunblock and avoid extreme temperatures?" The client understands that warm rather than hot, and cool rather then cold water should be used for bathing, washing dishes, etc. Sun protection of at least SPF 30 should always be worn, and avoidance of direct sun is best when going outdoors. "Will I receive a diuretic for the swelling?" This answer requires more teaching because diuretics should not be used to help lymphedema. They draw off water in the interstitial spaces, and once the diuretic is out of the system, it will pull more water into the affected area. "Should I elevate my affected limb while sleeping?" This demonstrates understanding. The affected limb should be elevated while sleeping. "Is it possible that I will have to wear a compression sleeve?" This demonstrates understanding that there is a potential that a compression sleeve or device will be required.

"Will I receive a diuretic for the swelling?" This answer requires more teaching because diuretics should not be used to help lymphedema. They draw off water in the interstitial spaces, and once the diuretic is out of the system, it will pull more water into the affected area.

what would be an abnormal finding for a 7-year-old African American boy? 1. abdominal distension 2. umbelical hernia 3. abdominal breathing 4. tenseness of abdominal muscles

1. abdominal distension

during a sports physical of a 16 year old girl, the nurse asks which questions to collect data about drug use?

"many teens have tried street drugs. have you tried any?"

The school nurse is assessing the nutritional status of a healthy adolescent. Which assessment will the nurse include in this assessment? Select all that apply: 1) Anthropometrics 2) Biochemical tests results 3) Clinical evaluation of diet 4) Dietary assessment 5) Body Mass Index (BMI)

) Anthropometrics 2) Biochemical tests results 3) Clinical evaluation of diet 5) Body Mass Index (BMI)

The nurse is teaching adult male healthy eating guidelines. How many servings of dairy should the nurse recommend for this patient? 1) 2 to 3 2) 3 to 5 3) 5 to 6 4) 0 to 2

1) 2 to 3

The nurse is teaching a patient how to evaluate the percentage of fat in a serving of food. She explains that the label on a package of a toaster pastry states that there are 6 g of fat and 210 calories per serving, What is the percentage of fat per serving? 1) 26%. 2) 35%. 3) 54%. 4) 72%.

1) 26%.

The nurse is teaching a patient the importance of protein for healing. Which foods should the nurse include in the teaching plan? 1) Fish 2) Cereal 3) Bread 4) Oatmeal

1) Fish

A 52-year-old male patient is admitted to the hospital with a new diagnosis of rectal cancer. The nurse conducts which type of assessment on his admission? 1. A comprehensive assessment 2. A problem-based health assessment 3. An episodic assessment 4. A screening assessment for colorectal cancer

1. A comprehensive assessment

A nurse is obtaining a health history from a 52-year-old male patient with a red lesion at the base of the tongue. What additional data does the nurse specifically collect about this patient? 1. Alcohol and tobacco use 2. The date of his last dental examination 3. The presence of dentures 4. A history of pyorrhea

1. Alcohol and tobacco use

During inspection of the mouth of a 72-year-old male patient, the nurse notices a red lesion at the base of his tongue. What additional information does the nurse obtain from this patient? 1. Alcohol and tobacco use 2. Date of his last dental examination 3. How well his dentures fit 4. A history of gum disease

1. Alcohol and tobacco use

Auscultation is a component of which examination technique? 1. Blood pressure measurement 2. Visual acuity 3. Examination of the ears 4. Measurement of oxygen saturation

1. Blood pressure measurement

A patient describes a recent onset of frequent and severe unilateral headaches that last about 1 hour. Based on these symptoms, the nurse suspects which type of headache? 1. Cluster headaches 2. Migraine headaches 3. Tension headaches 4. Sinus headache

1. Cluster headaches

The nurse suspects altered thought processes. Which findings might suggest an altered thought process? (Select all that apply.)

1. Dress or appearance 2. Problems articulating words 3. Tone of voice

2. A 22-year-old white male comes to the emergency department with a concern about a mass in his testicle. In addition to his age and race, which fact is a known risk factor for testicular cancer? 1. He had an undescended testicle at birth. 2. His mother had breast cancer. 3. He was treated for gonorrhea 18 months ago. 4. He had a hydrocele during infancy.

1. He had an undescended testicle at birth.

While examining a patient with an infected abdominal incision, the nurse notices that it is very malodorous. Which technique does this represent? 1. Inspection 2. Palpation 3. Auscultation 4. Percussion

1. Inspection

During a physical examination the nurse is unable to feel the patient's thyroid gland with palpation. What is the appropriate action of the nurse at this time? 1. Recognize that this is an expected finding 2. Auscultate the thyroid area 3. Percuss the anterior neck for thyroid span 4. Refer the patient for follow up with an endocrinologist

1. Recognize that this is an expected finding

Which would be an abnormal finding during an abdominal examination of an older adult? 1. Report of incontinence when sneezing or coughing 2. Loss of abdominal muscle tone 3. Bowel sounds every 15 seconds in all quadrants 4. Silver-white striae and a very faint vascular network

1. Report of incontinence when sneezing or coughing

On auscultation of a patient's lungs, the nurse hears a low-pitched, coarse, loud, and low snoring sound. Which term does the nurse use to document this finding? 1. Rhonchi 2. Wheeze 3. Crackles 4. Pleural friction rub

1. Rhonchi

Select the example given below that represents information a nurse collects from a patient during a physical examination. 1. Shiny skin and lack of hair found on lower legs 2. Concerned about lack of money to pay for prescriptions 3. Complains of tingling in both feet while sleeping 4. Family history of colon and breast cancer

1. Shiny skin and lack of hair found on lower legs

A nurse is assessing a women whose religious beliefs do not allow blood transfusions. She has severe anemia, is very weak, and has altered mental status. What behavior by the nurse is needed to provide effective care to this women? 1. examine his or her own feelings about the importance of religious beliefs in making decisions about life. 2. reconzie that he or she cannot provide care to pts whose religious beliefs endanger their lives 3. try to convince the pt to have a blood transfusion to save her own life 4. determine whether the pt is competent to make her own decisions about health care

1. examine his or her own feelings about the importance of religious beliefs in making decisions about life.

A woman who is 4 feet 11 inches tall is told by her provider to lose weight so that she is closer to her desired body weight. She asks the nurse, "How can I find out what my desired body weight should be?" The nurse responds, "Let me show you how to calculate it. Your desired body weight (DBW) should be _____ lb."

103.25 4 feet 11 inches = 59 inches. DBW = 105 lb for the 60 inches + 5 lb for every other inch. However this woman is under 5 feet in height. Thus 105 lb/60 inches = 1.75 lb/inch. 1.75 ´ 59 inches = 103.25 lb.

The registered nurse (RN) is caring for a young adult who is having an oral glucose tolerance tests (OGTT). Which laboratory result should the RN assess as a normal value for the two hour postprandial result? 140 mg/dl. 160 mg/dl. 180 mg/dl. 200 mg/dl.

140 mg/dl. The two hour postprandial level should be less 140 mg/dl for a young adult client.

In which age group is skipping meals most commonly seen? 1) School-aged children 2) Adolescents 3) Adults 4) Older adults

2) Adolescents

A patient states that he has experienced "a lot" of unintentional weight loss over the past 4 months. The nurse measures his height and weight (5 feet 11 inches, 170 pounds) and determines that his body mass index is 22.7. Which of the following is the most appropriate action to better evaluate his recent weight loss? 1) Calculate his desirable body weight. 2) Ask, "What is your usual body weight?" 3) Record what he ate in the last 24 hours. 4) Determine his hip-to-waist ratio.

2) Ask, "What is your usual body weight?"

A man weighs 265 pounds and is 6 feet 4 inches tall. Based on these data, how does the nurse classify his weight? 1) Overweight 2) Class I obesity 3) Class II obesity 4) Class III obesity

2) Class I obesity

The nurse is assessing an elderly patient's risk of nutritional deficiency. An important risk factor for nutritional deficiency in the elderly is: 1) Increased blood pressure 2) Decreased activities of daily living 3) An allergy to shellfish 4) Exercise pattern

2) Decreased activities of daily living

32-year-old woman has a 4-day history of sore throat and difficulty swallowing. The nurse observes tonsils covered with yellow patches. The tonsils are so large that they fill the entire oropharynx and appear to be touching. How does the nurse document these findings? 1. "Tonsils yellow and swollen?" 2. "Enlarged tonsils 4+ with yellow exudate." 3. "Strep infection to tonsils with 3+ swelling." 4. "1+ edema of tonsils with pus."

2. "Enlarged tonsils 4+ with yellow exudate."

Which situation illustrates a screening assessment? 1. A patient visits a clinic for the first time and the nurse completes a history and physical examination. 2. A hospital sponsors a health fair in a community to measure blood pressure as well as cholesterol levels. 3. A nurse at an urgent care center checks the blood pressure, pulse, temperature, and respirations of a patient reporting leg pain. 4. A patient with diabetes mellitus comes to the laboratory to get her blood glucose tested prior to a visit with a health care provider.

2. A hospital sponsors a health fair in a community to measure blood pressure as well as cholesterol levels.

The nurse is caring for a patient with a femur fracture. An immobilization device is used to maintain the alignment of the femur. The nurse palpates the top of the foot to make which determination? 1. Amount of drainage from the wound 2. Adequacy of blood perfusion to the foot 3. Presence of air in the underlying tissue 4. Range of motion to the foot

2. Adequacy of blood perfusion to the foot

A nurse is auscultating the lungs of a healthy female patient and hears crackles on inspiration. WhT action can the nurse take to ensure this is an accurate finding - make sure the bell of the stethoscope is used rather than the diaphragm - ask the patient to cough then repeat the auscultation - ask the patient not to talk while the nurse is listening to the lungs - change the patient's position to ensure accurate sounds

2. Ask the patient to cough then repeat the auscultation

A nurse suspects a viral infection or upper respiratory allergies when the patient describes the sputum as being which color? 1. White 2. Clear 3. Yellow 4. Pink tinged

2. Clear

On inspection of the eye of an 82-year-old woman, the nurse notes which finding as normal? 1. Opaque coloring of the lens 2. Clear cornea with a gray-white ring around the limbus 3. Dilated pupils when looking at an item in her hand 4. Impaired perception of the colors yellow and red

2. Clear cornea with a gray-white ring around the limbus

In assessing the mood of older adult patients, a nurse documents which finding as abnormal? 1. Sadness and grief after returning from the funeral of a long-time friend 2. Depression that interferes with the ability to perform activities of daily living 3. Frustration about rearranging the day's schedule to attend a grandson's birthday party 4. Crying about the unexpected death of a pet that had been with the family 12 years

2. Depression that interferes with the ability to perform activities of daily living

1. Which finding does the nurse recognize as abnormal when examining a male patient? 1. Testes that are palpable and firm within the scrotal sac bilaterally 2. Discharge from the penis when the glans is compressed 3. Foreskin that lies loosely over the penis 4. Glans a lighter skin tone than the rest of the penis

2. Discharge from the penis

After collecting data, the nurse begins data analysis with which activity? 1. Documenting information from the history 2. Organizing the data collected 3. Reporting data to other care providers 4. Recording data from the physical examination

2. Organizing the data collected

4. While taking the health history of a 23-year-old female patient, the nurse considers risk factors for STD. Which data from the patient suggest a need for patient education? 1. She has been in a monogamous sexual relationship for 2 years; she uses a condom to prevent pregnancy. 2. She has been sexually involved with one man for the last 2 weeks; she uses spermicidal gel to prevent pregnancy. 389 3. She has a Pap test each year and the results have been negative. 4. She uses oral contraceptives to prevent pregnancy.

2. She has been sexually involved with one man for the last 2 weeks; she uses spermicidal gel to prevent pregnancy.

The nurse is interviewing an adult Navajo woman. Which statement demonstrates cultural sensitivity and acceptance of the patient? 1. How often do you visit the medicine man for your health care? 2. Tell me about your health care beliefs and practices 3. Many Navajo people are afraid of hospitals. Are you afraid? 4. Have you ever had a physical exam with a physician or a nurse practitioner?

2. Tell me about your health care beliefs and practices

The nurse examines a patient's auditory canal and tympanic membrane with an otoscope. Which finding is considered abnormal? 1. Presence of cerumen 2. Yellow or amber color to the tympanic membrane 3. Presence of a cone of light 4. Shiny, translucent tympanic membrane

2. Yellow or amber color to the tympanic membrane

A school nurse notices a boy with a bandage on his arm and black fluid under the edge of the bandage. She asks the teen what happened to his arm. He replies that his mother applied axle grease to a boil. What is the nurse's most appreciate response to this boy? 1. tell the teen to remove the bandage and wash his arm 2. ask the teen what the boil looks like, what it feels like and if the axle grease is helping it get better 3. advise the teen to tell his mother to use antibiotic cream rather than axle grease 4. suggest that the teen see a health care provider bc the axle grease will infect the boil

2. ask the teen what the boil looks like, what it feels like and if the axle grease is helping it get better

which finding of a preschooler during a cardiovasculr system examination is abnormal? 1. heart rate of 106 beats/min 2. failure to gain weight because of fatigue while eating 3. continuous low-pitched vibrations heard over the jugular vein 4. pulse increasing on inspiration and decreasing on expiration

2. failure to gain weight because of fatigue while eating

A nurse is teaching a family from Guatemala about the importance of exercise to reduce body weight. The husband asks, what exercise do we do? considering the time orientation of this family, which response by the nurse is most effective? 1. in the past research has shown that walking 30 min most days of the week is best 2. is there an exercise that you can do today for 30 min and make it part of your daily rountine? 3. if you exercise 30 min most days of the week, you can loose weight by your next visit 4. I have always found that resistance weight training each day for 30 min is effective

2. is there an exercise that you can do today for 30 min and make it part of your daily rountine?

a patient with a missed menstrual period and nausea has which signs and symptoms of pregnancy 1. questionable 2. presumptive 3. probable 4. positive

2. presumptive

A man who is 6 feet 9 inches tall is told by his provider to lose weight so that he is closer to his desired body weight. He asks the nurse, "How can I find out what my desired body weight should be?" The nurse responds, "Let me show you how to calculate it. Your desired body weight (DBW) should be _____ lb."

232 6 feet 9 inches = 81 inches. DBW = 106 lb for the first 60 inches + 6 lb for every other inch, which for this man equals 21 inches (81" - 60" = 21"). 21 ´ 6 = 126 +106 = 232 lb.

8. The nurse recognizes which symptom as commonly associated with prostate enlargement? 1. Constipation 2. Rectal bleeding 3 Weak urinary stream 4. Penile discharge

3 Weak urinary stream

A nurse is assessing an infant who is able to pull up to a sitting positioning, turn from prone to side position, laugh and babble, and show interest in her surroundings. These behaviors are consistent with an infant of what age?

3 months old

The nurse is assessing a patient's nutritional status and suspects the patient needs more macronutrients. Which are considered macronutrients? 1) Minerals 2) Vitamins 3) Fats 4) Water

3) Fats

The nurse is working with a patient to develop a nutritional plan for a patient newly diagnosed with diabetes. The nurse assesses what the patient's food preferences are because: 1) Food preferences can indicate a chronic disease that the nurse may be unaware of 2) Life expectancy can be predicted based on food preferences 3) Food preferences and dislike have a strong influence on what a person eats 4) A list of food preferences will help identify individuals who will not comply with special diets

3) Food preferences and dislike have a strong influence on what a person eats

Why does the nurse ask a patient which medications he takes as part of a nutritional assessment? 1) Medications must be taken with food to avoid irritation to the gastrointestinal system. 2) Many drugs affect nutritional intake requirements; thus adjustments to the diet must be made. 3) The absorption and bioavailability of some medications are affected by food. 4) Some medications taste bad and may interfere with the appetite.

3) The absorption and bioavailability of some medications are affected by food.

Which data from the health history of a 42-year-old man should be evaluated further as a possible risk for hearing loss? 1. "I watch TV in the evenings with my wife and children." 2. "When I was younger, I wore and earring." 3. "My primary hobby is carpentry work." 4. "I have been an accountant for 16 years for an insurance agency."

3. "My primary hobby is carpentry work."

Which set of vital signs should the nurse recognize as out of the expected range? 1. 42-year-old man: BP, 114/82; pulse, 74 beats/min; respiration, 16 breaths/min; temperature, 36.8° C 2. 11-year-old girl: pulse, 88 beats/min; respiration, 22 breaths/min; temperature, 36.7° C 3. 3-year-old boy: pulse, 130 beats/min; respiration, 44 breaths/min; temperature, 36.7° C 4. 1-month-old girl: pulse, 120 beats/min; respiration, 42 breaths/min; temperature, 36.7° C

3. 3-year-old boy: pulse, 130 beats/min; respiration, 44 breaths/min; temperature, 36.7° C

How does the nurse assess a patient's consensual reaction? 1. By touching the cornea with a small piece of sterile cotton and observing the change in the pupil size 2. By observing the patient's pupil size when she or her looks at an object 2 to 3 feet away and then looks at an object 6 to 8 inches away 3. By shining a light into the patient's right eye and observing the pupillary reaction of the left eye 4. By covering one eye with a card and observing the pupillary reaction when the card is removed

3. By shining a light into the patient's right eye and observing the pupillary reaction of the left eye

A nurse finds the patient's anteroposterior diameter of the chest to be the same as the lateral diameter. Based on their finding, what additional data would the nurse anticipate - bronchial breath sounds in the posterior thorax - decrease in respiratory rate - decreased breath sounds on auscultation -complaint of sharp chest pain on inspiration

3. Decreased breath sounds on auscultation

During the history the patient indicates that her eyes have been red and itching. Which additional question does the nurse ask? 1. Have you ever had a detached retina? 2. Have you had the pressure in your eyes checked? 3. Do you have seasonal allergies? 4. Do you also have double vision?

3. Do you have seasonal allergies?

9. During an examination the nurse palpates the Skene's glands. Which technique best describes this process? 1. Exerting pressure over the clitoris, slide the finger downward (posteriorly) toward the vaginal opening. 2. Palpate the fourchette and slide the finger forward (anteriorly) toward the vaginal opening. 3. Exert pressure on the anterior vaginal wall and slide the finger outward toward the vaginal opening. 4. Grasp the labia majora between the index finger and thumb and milk the labia outward.

3. Exert pressure on the anterior vaginal wall and slide the finger outward toward the vaginal opening.

A patient has an infection of the terminal bronchioles and alveoli that involves the right lower lobe of the lung. Which abnormal findings are expected? 1. Dyspnea with diminished breath sounds bilaterally 2. Asymmetric chest expansion on the bright side 3. Fever and tachypnea with crackles over the right lower lobe 4. Prolonged expiration with an occasional wheeze in the right lower lobe

3. Fever and tachypnea with crackles over the right lower lobe

Which question gives the nurse further information about the patient's complaint of chest pain? 1. Have you had your influenza immunization this year? 2. Are there environmental conditions that may affect your breathing at home? 3. How would you describe the chest pain? 4. Has the chest pain been interrupting your sleep?

3. How would you describe the chest pain?

Which finding is an expected age-related change for a woman 80 years old? 1. Kyphosis 2. Back pain 3. Loss of height 4. Depression

3. Loss of height

Which assessment data are determined by the use of a goniometer? 1. Auscultation of fetal heart tones 2. Inspection of the cervix 3. Measurement of joint flexion 4. Assessment of hearing

3. Measurement of joint flexion

A patient is brought to the emergency department with severe respiratory distress. Which method of temperature measurement would be most appropriate? 1. Oral temperature with an electric thermometer 2. Axillary with an electronic thermometer 3. Temporal artery 4. Rectal temperature

3. Temporal artery

What is the reason for palpating axillary lymph nodes during a clinical breast examination? 1. Axillary nodes fluctuate during the month in response to the menstrual cycle 2. Axillary node tenderness is the most common initial symptom of breast cancer 3. The lymph network in the breast primarily drains toward the axillary lymph nodes 4. this is a matter of convenience because of the close proximity of the axillae to the breast

3. The lymph network in the breast primarily drains toward the axillary lymph nodes

6. To inspect the glans penis of the uncircumcised male, the nurse retracts the foreskin. After inspection she is unable to replace the foreskin over the glans. The nurse recognizes that this situation could potentially lead to which complication? 1. Decreased sperm production 2. Urinary tract infection 3. Tissue necrosis of the penis 4. Testicular cancer

3. Tissue necrosis of the penis

A patient reports having abdominal distention and having vomited severAl times yesterday and today. What questions is appropriate for the nurse to ask in response to this information 1. Has there been a change in the amount of the distensión 2. Did you have heartburn before vomiting? 3. What did the vomitus look like? .4. Have you noticed a change in the color of your urine stool?

3. What did the vomitus look like?

A patient complaint of chest pain. Which questions has the highest priority to obtain additional information? 1. What were you doing when the pain first occurred 2. Do you have shortness of breath with the chest pain? 3. What does the pain feel like? 4. Has anyone in your family ever had similar pain?

3. What does the pain feel like?

During an interview, an elderly patient tells the nurse that she has periodic problems in keeping her balance. The nurse asks her what she is doing when the episodes occur. Which area of the symptoms analysis is the nurse pursuing with this question? 1. severity 2. frequency 3. aggravating factors 4. location

3. aggravating factors

An older man who is near death has been admitted to the hospital, and family members are at his bedside. During the admission assessment the nurse uses which question or statement to approciatly address spiritual needs? 1. what is your religion? ill make the appreciate spiritual arrangement? 2. tell me what death means to people from your culture. 3. are there any special needs or rituals that you and your family request at this time? 4. Ill call the hospital priest so he can administer last rites

3. are there any special needs or rituals that you and your family request at this time

an adolescent tells a nurse, that while he was riding in a friend's car, the friend was stopped by the police for driving while intoxicated. which assessment tool would be most appropriate to use with this adolescent 1. face pain scale 2. pediatric symptom checklist (PSC) 3. guidelines for adolescent prevention (GAP) 4. oucher scales

3. guidelines for adolescent prevention (GAP)

When should gloves be changed or discarded? A. After touching the genital skin B. After completing the internal vaginal exam C. After completing the rectal exam D. All of the above

A. After touching the genital skin B. After completing the internal vaginal exam C. After completing the rectal exam

The nurse is caring for a women who has just been pronounced dead. Her adult children are in the room. Which statement by the nurse indicates culturally competent care? 1. which funeral home would you like notified of your mothers death? 2. we will be moving her to the morgue in about 30 min 3. would you like time with your mother for any specific ceremonies? 4. here are some of her personal belongings that were in the drawer

3. would you like time with your mother for any specific ceremonies?

An older woman is 5 feet 2 inches tall and weighs 100 pounds. To best understand her dietary intake, which question is most appropriate? 1) "Who prepares your meals?" 2) "What are your favorite foods?" 3) "How do you get to the grocery store?" 4) "Could you describe what you eat on a typical day?"

4) "Could you describe what you eat on a typical day?"

According to the food plan, what represents one serving from the bread, cereal, and grain products group? 1) 1 cup cooked rice 2) 6 soda crackers 3) 1 hamburger bun 4) 1 slice of bread

4) 1 slice of bread

A 24-year-old female patient has a 2-day history of clear nasal drainage. Based on these data, which question is the most logical for the nurse to ask? 1. "Is there a foul odor coming from your nose?" 2. "Have you recently had nosebleeds?" 3. "Do you snore when sleeping?" 4. "Do you have allergies?"

4. "Do you have allergies?"

Which technique is used for palpating lymph nodes? 1. Apply firm pressure over the nodes with the pads of the fingers. 2. Apply gentle pressure over the nodes with the tips of the fingers 3. Apply firm pressure anterior to the nodes with the tips of the fingers 4. Apply gentle pressure over the nodes with the pads of the fingers.

4. Apply gentle pressure over the nodes with the pads of the fingers.

While talking with a patient, the nurse suspects that he was hearing loss. Which examination technique is most accurate for assessing hearing loss? 1. Whispered voice test 2. Rinne test 3. Weber's test 4. Audiometry test

4. Audiometry test

The nurse notes which finding as abnormal during a thoracic assessment of an older adult? 1. A skeletal deformity affecting curvature of the spine 2. Shortness of breath on exertion 3. An increase in anteroposterior diameter 4. Bronchovesicular breath sounds in the peripheral lung fields

4. Bronchovesicular breath sounds in the peripheral lung fields

A 62 year-old patient tells the nurse that he is in excellent health and does not take any medications. Why is the most appropriate response by the nurse to follow up on the patient's statement? 1. Do you avoid taking drugs because of bad experiences? 2. Which medications have you taken in the past? 3. That is hard to believe. Most men your age take medications 4. Do you use over-the-counter medications of herbal preparations?

4. Do you use over-the-counter medications of herbal preparations?

Which infection control intervention is most frequently applied? 1. Wearing gloves 2. Using masks 3. Wearing eye protection 4. Hand hygiene

4. Hand hygiene

3. Which data collected from the history of a 32-year-old female patient should be followed with a symptom analysis? 1. Has never had a mammogram. 2. Experiences light to moderate bleeding during menstrual cycle. 3. Periods began at age 12; has never been pregnant. 4. Has pelvic pain and vaginal discharge.

4. Has pelvic pain and vaginal discharge.

The nurse is conducting an interview with Jeremy, a 17 year-old accompanied by his mother. Which statement made by the nurse is an age-appropriate adjustment when conducting a health history with an adolescent? 1. Jeremy, do you have a girlfriend, and if so are you sexually active yet? 2. Mrs. Williams, is your son sexually active yet? 3. Jeremy, how do you incorporate safe sex practices into your daily life? 4. Mrs. Williams, would you wait outside while I discuss a few things with Jeremy?

4. Mrs. Williams, would you wait outside while I discuss a few things with Jeremy?

During inspection of the respiratory system, the nurse documents which finding as abnormal? 1. Skin color consistent with patient's ethnicity 2. 1:2 ratio of anteroposterior to lateral diameter 3. Anterior costal angle is 85 degrees 4. Patient leaning forward with arms braced against the knees

4. Patient leaning forward with arms braced against the knees

What is the most reliable way to assess pain in a patient who is cognitively intact? 1. Type and frequency of analgesic medications the patient takes 2. Patients most recent vital signs 3. Extent of tissue damage the patient has experienced 4. Report by he patient describing the pain experienced

4. Report by he patient describing the pain experienced

The nurse documents which information in the patient's history? 1. The patient is scratching his left arm. 2. The patient's skin feels warm. 3. The patient reports itching of her eyes. 4. The patient's temperature is 100°F.

4. The patient's temperature is 100°F

How does the nurse palpate the chest for tenderness, bulges, and symmetry? 1. Uses the fist of the dominant hand to gently tap the anterior, lateral, and posterior chest, comparing one side with another 2. Uses the ulnar surface of one hand to palpate the anterior, posterior, and lateral chest, comparing one side with another 3. With the tips of the fingers, palpates the skin over the chest and the alignment of vertebrae 4. With the palmar surface of fingers of both hands, feels the consistency of the skin over the chest and the alignment of vertebrae.

4. With the palmar surface of fingers of both hands, feels the consistency of the skin over the chest and the alignment of vertebrae.

which are expected findings of a newborn's respiratory assessment? 1. thoracic breathing 2. a 1:2 ration of anterior posterior to lateral diameter 3. flaring of the nares noted on inspiration 4. bronchovesicular breath sounds in the peripheral lung fields

4. bronchovesicular breath sounds in the peripheral lung fields

during an initial prenatal visit the nurse identifies which factor as a consistent with a high-risk pregnancy? 1. patient is 18 years old 2. patient height is 5 feet 4 inches 3. birth weight of infant with last pregnancy was 2800g 4. patient smokes one-half pack of cigarettes a day

4. patient smokes one-half pack of cigarettes a day

when examining the genitalia of a 3-year-old, which position is ideal? 1. prone position with legs flexed in a frog leg position 2. supine position with knees spread and ankles spread apart 3. lathotomy position with knees and ankles spread apart 4. sitting position with knees spread and ankles crossed

4. sitting position with knees spread and ankles crossed

on assessment of the neurologic status of a 4-month-old infant, the nurse notes which finding as abnormal 1. the infant abducts and extends arms and legs when startled 2. when the infant's sole is touched, the toes flex tightly in an attempt to grasp 3. when stroking the infant's foot from sole to great toes, there is fanning of the toes 4. the infant steps in place when held upright with feet on a flat surface

4. the infant steps in place when held upright with feet on a flat surface

Which is the most factual answer the nurse can provide to the client about lymph nodes? 25. This is not the most correct answer. 2,000,000. This is not the most correct answer. 650. A healthy adult has approximately 600 to 700 lymph nodes. 150. This is not the most correct answer.

650. A healthy adult has approximately 600 to 700 lymph nodes

Which patient does the nurse recognize as having the highest risk for ovarian cancer? a. A 24-year-old nulliparous woman who has a history of multiple sexual partners b. A 32-year-old woman who has had six live births and a history of human papilloma virus (HPV) infection c. A 55-year-old woman who reached menarche at age 12 and menopause at age 54 d. A 64-year-old nulliparous woman who has taken hormone replacement therapy for eight years

A This patient has no risk factor for ovarian cancer. B This patient has a risk factor for cervical cancer (HPV), but not ovarian cancer. C This patient has no risk factors for ovarian cancer. D This patient has a risk factor for ovarian cancer.

A male client is admitted after falling from his bed. The healthcare provider (HCP) tells the family that he has an incomplete fracture of the humerus. The family ask the RN what this means. Which type of fracture should the RN explain from these findings? Straignt fracture line that is also a simple, closed fracture. Nondisplaced fracture line that wraps around the bone. A complete fracture that also punctures the skin. A fracture that bends or splinters part of the bone.

A fracture that bends or splinters part of the bone. An incomplete fracture occurs when part of the bone is splintered (broken) and it has not gone completely through the thickness of the bone.

The registered nurse (RN) did not note that a prescription dose was recently changed and did not note the updated medication administration record (MAR). After giving the client the original dose, the RN reports the medication error to the nurse manager. What consequences will the RN experience due to this error in medication administration? The incident will be reported to the state's Board of Nursing (BON). A medication error report will be completed and risk management will be notified. The RN will be suspended from medication administration until the error is investigated. The incident will be documented in the RN's personnel file.

A medication error report will be completed and risk management will be notified. By reviewing quality of care internally, steps of care can be evaluated and staff can be educated where gaps are identified. The medication report and notification of management is the responsibility of the RN who made the mistake, so an internal review of the steps of the occurrence can be completed to determine further risk potentials.

6. The nurse is performing a health interview on a patient who has a language barrier, and no interpreter is available. Which is the best example of an appropriate question for the nurse to ask in this situation? A) "Do you take medicine?" B) "Do you sterilize the bottles?" C) "Do you have nausea and vomiting?" D) "You have been taking your medicine, haven't you?"

A) "Do you take medicine?" Page: 46 In a situation where there is a language barrier and no interpreter available, use simple words avoiding medical jargon. Avoid using contractions and pronouns. Use nouns repeatedly and discuss one topic at a time.

30. During a mental status examination, the nurse wants to assess a patient's affect. The nurse should ask the patient which question? A) "How do you feel today?" B) "Would you please repeat the following words?" C) "Have these medications had any effect on your pain?" D) "Has this pain affected your ability to get dressed by yourself?"

A) "How do you feel today?" Page: 74. Judge mood and affect by body language and facial expression and by asking directly, "How do you feel today?" or "How do you usually feel?" The mood should be appropriate to the person's place and condition and should change appropriately with topics.

73. The nurse is performing an ear examination of an 80-year-old patient. Which of these would be considered a normal finding? A) A high-tone frequency loss B) Increased elasticity of the pinna C) A thin, translucent membrane D) A shiny, pink tympanic membrane

A) A high-tone frequency loss Pages: 337-338. A high-tone frequency hearing loss is apparent for those affected with presbycusis, the hearing loss that occurs with aging. The pinna loses elasticity, causing earlobes to be pendulous. The eardrum may be whiter in color and more opaque and duller than in the young adult.

7. A female patient does not speak English well, and the nurse needs to choose an interpreter. Which of the following would be the most appropriate choice? A) A trained interpreter B) A male family member C) A female family member D) A volunteer college student from the foreign language studies department

A) A trained interpreter Page: 46 whenever possible, the nurse should use a trained interpreter, preferably one who knows medical terminology. In general, an older, more mature interpreter is preferred to a younger, less experienced one, and the same gender is preferred when possible.

Which of the following are risk factors for glaucoma? A. Age over 40 years B. Diabetes C. High blood pressure D. All of the above

A. Age over 40 years B. Diabetes C. High blood pressure All three answers, age over 40 years, diabetes, and high blood pressure are risk factors for glaucoma

10. During an assessment, the nurse notices that a patient is handling a small charm that is tied to a leather strip around his neck. Which action by the nurse is appropriate? A) Ask the patient about the item and its significance. B) Ask the patient to lock the item with other valuables in the hospital's safe. C) Tell the patient that a family member should take valuables home. D) No action is necessary.

A) Ask the patient about the item and its significance. Page: 21 The nurse should inquire about the amulet's meaning. Amulets, such as charms, are often seen as an important means of protection from "evil spirits" by some cultures.

41. A 6-month-old infant has been brought to the well-child clinic for a check-up. She is currently sleeping. What should the nurse do first when beginning the examination? A) Auscultate the lungs and heart while the infant is still sleeping. B) Examine the infant's hips because this procedure is uncomfortable. C) Begin with the assessment of the eye and continue with the remainder of the examination in a head-to-toe approach. D) Wake the infant before beginning any portion of the examination to obtain the most accurate assessment of body systems.

A) Auscultate the lungs and heart while the infant is still sleeping. Pages: 122-124. When the infant is quiet or sleeping is an ideal time to assess the cardiac, respiratory, and abdominal systems. Assessment of the eye, ear, nose, and throat are invasive procedures and should be performed at the end of the examination.

79. When assessing tactile fremitus, the nurse recalls that it is normal to feel tactile fremitus most intensely over which location? A) Between the scapulae B) Third intercostal space, MCL C) Fifth intercostal space, MAL D) Over the lower lobes, posterior side

A) Between the scapulae Page: 424. Normally, fremitus is most prominent between the scapulae and around the sternum. These are sites where the major bronchi are closest to the chest wall. Fremitus normally decreases as one progress down the chest because more tissue impedes sound transmission.

124. The nurse knows that determining whether a person is oriented to his or her surroundings will test the functioning of which of these structures? A) Cerebrum B) Cerebellum C) Cranial nerves D) Medulla oblongata

A) Cerebrum Pages: 621-622 | Page: 660. The cerebral cortex is responsible for thought, memory, reasoning, sensation, and voluntary movement. The other options structures are not responsible for a person's level of consciousness.

46. During a nutritional assessment, why is it important for the nurse to ask a patient what medications he or she is taking? A) Certain drugs can affect the metabolism of nutrients. B) The nurse needs to assess the patient for allergic reactions. C) Medications need to be documented on the record for the physician's review. D) Medications can affect one's memory and ability to identify food eaten in the last 24 hours.

A) Certain drugs can affect the metabolism of nutrients. Page: 183 Analgesics, antacids, anticonvulsants, antibiotics, diuretics, laxatives, antineoplastic drugs, steroids, and oral contraceptives are drugs that can interact with nutrients, impairing their digestion, absorption, metabolism, or use. The other responses are not correct.

9. The nurse is conducting an interview in an outpatient clinic and is using a computer to record data. Which is the best use of the computer in this situation? Select all that apply. A) Collect the patient's data in a direct, face-to-face manner. B) Enter all the data as the patient states it. C) Ask the patient to wait as the nurse enters data. D) Type the data into the computer after the narrative is fully explored. E) Allow the patient to see the monitor during typing.

A) Collect the patient's data in a direct, face-to-face manner. D) Type the data into the computer after the narrative is fully explored. E) Allow the patient to see the monitor during typing. Page: 32 The use of a computer can become a barrier. The nurse should begin the interview as usual by greeting the patient, establishing rapport, and collecting the patient's narrative story in a direct face-to-face manner. Only after the narrative is fully explored should the nurse type data into the computer. When typing, the nurse should position the monitor so that the patient can see it.

52. A patient tells the nurse that he has noticed that one of his moles has started to burn and bleed. When assessing his skin, the nurse would pay special attention to the danger signs for pigmented lesions and would be concerned with which additional finding? A) Color variation B) Border regularity C) Symmetry of lesions D) Diameter less than 6 mm

A) Color variation Pages: 212-213. Abnormal characteristics of pigmented lesions are summarized in the mnemonic ABCD: asymmetry of pigmented lesion, border irregularity, color variation, and diameter greater than 6 mm.

5. The nurse is interviewing a patient who has a hearing impairment. What techniques would be most beneficial in communicating with this patient? A) Determine the communication method he prefers. B) Avoid using facial and hand gestures because most hearing-impaired people find this degrading. C) Request a sign language interpreter before meeting with him to help facilitate the communication. D) Speak loudly and with exaggerated facial movement when talking with him because this helps with lip reading.

A) Determine the communication method he prefers. Pages: 40-41 The nurse should ask the deaf person the preferred way to communicate—by signing, lip reading, or writing. If the person prefers lip reading, then the nurse should be sure to face him or her squarely and have good lighting on the nurse's face. The nurse should not exaggerate lip movements because this distorts words. Similarly, shouting distorts the reception of a hearing aid the person may wear. The nurse should speak slowly and should supplement his or her voice with appropriate hand gestures or pantomime.

101. The nurse is percussing the seventh right intercostal space at the midclavicular line over the liver. Which sound should the nurse expect to hear? A) Dullness B) Tympany C) Resonance D) Hyperresonance

A) Dullness Page: 541. The liver is located in the right upper quadrant and would elicit a dull percussion note.

87. During auscultation of breath sounds, the nurse should use the stethoscope correctly, in which of the following ways? A) Listen to at least one full respiration in each location. B) Listen as the patient inhales and then go to the next site during exhalation. C) Have the patient breathe in and out rapidly while the nurse listens to the breath sounds. D) If the patient is modest, listen to sounds over his or her clothing or hospital gown.

A) Listen to at least one full respiration in each location. Pages: 426-427. During auscultation of breath sounds with a stethoscope, it is important to listen to one full respiration in each location. During the examination, the nurse should monitor the breathing and offer times for the person to breathe normally to prevent possible dizziness.

59. The nurse suspects that a patient has hyperthyroidism and laboratory data indicate that the patient's thyroxine and tri-iodothyronine hormone levels are elevated. Which of these findings would the nurse most likely find on examination? A) Tachycardia B) Constipation C) Rapid dyspnea D) Atrophied nodular thyroid

A) Tachycardia Thyroxine and tri-iodothyronine are thyroid hormones that stimulate the rate of cellular metabolism, resulting in tachycardia. With an enlarged thyroid as in hyperthyroidism, the nurse might expect to find diffuse enlargement (goiter) or a nodular lump, but not an atrophied gland. Dyspnea and constipation are not findings associated with hyperthyroidism.

45. The nurse is providing care for a 68-year-old woman who is complaining of constipation. What concern exists regarding her nutritional status? A) The absorption of nutrients may be impaired. B) The constipation may represent a food allergy. C) She may need emergency surgery for the problem. D) The gastrointestinal problem will increase her caloric demand.

A) The absorption of nutrients may be impaired. Page: 182. Gastrointestinal symptoms such as vomiting, diarrhea, or constipation may interfere with nutrient intake or absorption. The other responses are not correct.

120. During the assessment of an 80-year-old patient, the nurse notices that his hands show tremors when he reaches for something and his head is always nodding. There is no associated rigidity with movement. Which of these statements is most accurate? A) These are normal findings resulting from aging. B) These could be related to hyperthyroidism. C) These are the result of Parkinson disease. D) This patient should be evaluated for a cerebellar lesion.

A) These are normal findings resulting from aging. Page: 659. Senile tremors occasionally occur. These benign tremors include an intention tremor of the hands, head nodding (as if saying yes or no), and tongue protrusion. Tremors associated with Parkinson disease include rigidity, slowness, and weakness of voluntary movement. The other responses are incorrect.

90. The nurse is assessing voice sounds during a respiratory assessment. Which of these findings indicates a normal assessment? Select all that apply. A) Voice sounds are faint, muffled, and almost inaudible when the patient whispers "one, two, three" in a very soft voice. B) As the patient says "ninety-nine" repeatedly, the examiner hears the words "ninety-nine" clearly. C) When the patient speaks in a normal voice, the examiner can hear a sound but cannot distinguish exactly what is being said. D) As the patient says a long "ee-ee-ee" sound, the examiner also hears a long "ee-ee-ee" sound. E) As the patient says a long "ee-ee-ee" sound, the examiner hears a long "aaaaaa" sound.

A) Voice sounds are faint, muffled, and almost inaudible when the patient whispers "one, two, three" in a very soft voice. C) When the patient speaks in a normal voice, the examiner can hear a sound but cannot distinguish exactly what is being said. D) As the patient says a long "ee-ee-ee" sound, the examiner also hears a long "ee-ee-ee" sound. Page: 446. As a patient says "ninety-nine" repeatedly, normally, the examiner hears sound but cannot distinguish what is being said. If a clear "ninety-nine" is auscultated, then it could indicate increased lung density, which enhances transmission of voice sounds. This is a measure of bronchophony. When a patient says a long "ee-ee-ee" sound, normally the examiner also hears a long "ee-ee-ee" sound through auscultation. This is a measure of egophony. If the examiner hears a long "aaaaaa" sound instead, this could indicate areas of consolidation or compression. With whispered pectoriloquy, as when a patient whispers a phrase such as "one-two-three," the normal response when auscultating voice sounds is to hear sounds that are faint, muffled, and almost inaudible. If the examiners hears the whispered voice clearly, as if the patient is speaking through the stethoscope, then consolidation of the lung fields may exist.

85. The nurse is listening to the breath sounds of a patient with severe asthma. Air passing through narrowed bronchioles would produce which of these adventitious sounds? A) Wheezes B) Bronchial sounds C) Bronchophony D) Whispered pectoriloquy

A) Wheezes Page: 445. Wheezes are caused by air squeezed or compressed through passageways narrowed almost to closure by collapsing, swelling, secretions, or tumors, such as with acute asthma or chronic emphysema.

64. The nurse is performing an assessment on a 7-year-old child who has symptoms of chronic watery eyes, sneezing, and clear nasal drainage. The nurse notices the presence of a transverse line across the bridge of the nose, dark blue shadows below the eyes, and a double crease on the lower eyelids. These findings are characteristic of: A) allergies. B) a sinus infection. C) nasal congestion. D) an upper respiratory infection.

A) allergies. Page: 275. Chronic allergies often develop chronic facial characteristics. These include blue shadows below the eyes, a double or single crease on the lower eyelids, open-mouth breathing, and a transverse line on the nose.

86. An adult patient with a history of allergies comes to the clinic complaining of wheezing and difficulty in breathing when working in his yard. The assessment findings include tachypnea, use of accessory neck muscles, prolonged expiration, intercostal retractions, decreased breath sounds, and expiratory wheezes. The nurse interprets that these assessment findings are consistent with: A) asthma. B) atelectasis. C) lobar pneumonia. D) heart failure.

A) asthma. Page: 451. Asthma is allergic hypersensitivity to certain inhaled particles that produces inflammation and a reaction of bronchospasm, which increases airway resistance, especially during expiration. Increased respiratory rate, use of accessory muscles, retraction of intercostal muscles, prolonged expiration, decreased breath sounds, and expiratory wheezing are all characteristic of asthma. See Table 18-8 for descriptions of the other conditions.

89. The nurse is auscultating the lungs of a patient who had been sleeping and notices short, popping, crackling sounds that stop after a few breaths. The nurse recognizes that these breath sounds are: A) atelectatic crackles, and that they are not pathologic. B) fine crackles, and that they may be a sign of pneumonia. C) vesicular breath sounds. D) fine wheezes.

A) atelectatic crackles, and that they are not pathologic. Pages: 429-430. One type of adventitious sound, atelectatic crackles, is not pathologic. They are short, popping, crackling sounds that sound like fine crackles but do not last beyond a few breaths. When sections of alveoli are not fully aerated (as in people who are asleep or in the elderly), they deflate slightly and accumulate secretions. Crackles are heard when these sections are expanded by a few deep breaths. Atelectatic crackles are heard only in the periphery, usually in dependent portions of the lungs, and disappear after the first few breaths or after a cough.

Which possible signs and symptoms of testicular cancer should be reported to a physician? A. A lump, tenderness, or swelling in the scrotum or testicles B. Unexplained weight loss C. Breast development D. All of the above

A. A lump, tenderness, or swelling in the scrotum or testicles B. Unexplained weight loss C. Breast development A lump, tenderness, or swelling in the scrotum or testicles; unexplained weight loss; and breast development in a male can all be signs of testicular cancer. The testicles should be free of lumps and swelling and be nontender. Unexplained weight loss and breast development in a male can be a sign of testicular cance

17. The nurse recognizes that working with children with a different cultural perspective may be especially difficult because: A) children have spiritual needs that are influenced by their stages of development. B) children have spiritual needs that are direct reflections of what is occurring in their homes. C) religious beliefs rarely affect the parents' perceptions of the illness. D) parents are often the decision makers, and they have no knowledge of their children's spiritual needs.

A) children have spiritual needs that are influenced by their stages of development. Page: 20. Illness during childhood may be an especially difficult clinical situation. Children, as well as adults, have spiritual needs that vary according to the child's developmental level and the religious climate that exists in the family. The other statements are not correct.

37. When percussing over the liver of a patient, the nurse notices a dull sound. The nurse should: A) consider this a normal finding. B) palpate this area for an underlying mass. C) reposition the hands and attempt to percuss in this area again. D) consider this an abnormal finding and refer the patient for additional treatment.

A) consider this a normal finding. Pages: 116-117. Percussion over relatively dense organs, such as the liver or spleen, will produce a dull sound. The other responses are not correct.

76. When assessing a patient's lungs, the nurse recalls that the left lung: A) consists of two lobes. B) is divided by the horizontal fissure. C) consists primarily of an upper lobe on the posterior chest. D) is shorter than the right lung because of the underlying stomach.

A) consists of two lobes. Pages: 413-414. The left lung has two lobes, and the right lung has three lobes. The right lung is shorter than the left lung because of the underlying liver. The left lung is narrower than the right lung because the heart bulges to the left. The posterior chest is almost all lower lobe.

82. The nurse is percussing over the lungs of a patient with pneumonia. The nurse knows that percussion over an area of atelectasis in the lungs would reveal: A) dullness. B) tympany. C) resonance. D) hyperresonance.

A) dullness. Pages: 424-425. A dull percussion note signals an abnormal density in the lungs, as with pneumonia, pleural effusion, atelectasis, or tumor.

69. A 68-year-old woman is in the eye clinic for a checkup. She tells the nurse that she has been having trouble with reading the paper, sewing, and even seeing the faces of her grandchildren. On examination, the nurse notes that she has some loss of central vision but her peripheral vision is normal. These findings suggest that: A) she may have macular degeneration. B) her vision is normal for someone her age. C) she has the beginning stages of cataract formation. D) she has increased intraocular pressure or glaucoma.

A) she may have macular degeneration. Page: 285. Macular degeneration is the most common cause of blindness. It is characterized by loss of central vision. Cataracts would show lens opacity. Chronic open-angle glaucoma, the most common type of glaucoma, involves a gradual loss of peripheral vision.

50. The nurse is performing a nutritional assessment on an 80-year-old patient. The nurse knows that physiological changes that directly affect the nutritional status of the elderly include: A) slowed gastrointestinal motility. B) hyperstimulation of the salivary glands. C) an increased sensitivity to spicy and aromatic foods. D) decreased gastrointestinal absorption causing esophageal reflux.

A) slowed gastrointestinal motility. Page: 176. Normal physiological changes in aging adults that affect nutritional status include slowed gastrointestinal motility, decreased gastrointestinal absorption, diminished olfactory and taste sensitivity, decreased saliva production, decreased visual acuity, and poor dentition.

What will the nurse instruct nursing assistive personnel (NAP) to do regarding the management of a patient's pain? A. "Let me know at least 30 minutes before you transport her so I can administer her analgesics." B. "Be sure to keep the room temperature high and the TV on at all times." C. "Be sure to tell me if you notice grimacing, guarding, or any unusual behavior." D. "I've given her some medication; please report to me whether it seems to have relieved her pain within an hour or so."

A. "Let me know at least 30 minutes before you transport her so I can administer her analgesics." Rationale: This option provides the NAP with specific direction about the need to manage pain in relation to specific activities. Environmental conditions such as an excessively warm or noisy room can aggravate pain. Pain assessment is a nursing responsibility and may not be delegated to the NAP.

How often should normal bowel sounds be heard in each quadrant of the abdomen? A. 5-35 times per minute B. Less than 5 times per minute C. 15-20 times per minute D. 20-40 times per minute

A. 5-35 times per minute Normal bowel sounds should be heard 5-35 times per minute. Bowel sounds reflect peristalsis and should be heard irregularly

Which patient is most at risk of developing permanently impaired mobility? A. A 72-year-old woman hospitalized for anemia associated with diabetic nephropathy (kidney disease) B. A 55-year-old woman with mental illness who had become malnourished C. An 11-year-old boy who sustained a fractured pelvis during a fall from his tree house D. A 79-year-old man recovering from surgery to release a contracture of the connective tissue in his hand

A. A 72-year-old woman hospitalized for anemia associated with diabetic nephropathy (kidney disease) Rationale: Although the 72-year-old woman's anemia will not affect her mobility, she is the patient most at risk of mobility impairment. The fact that she has diabetes, a serious chronic condition, puts her at high risk of mobility impairment. In addition, her age is a risk factor, since mobility impairment is more prevalent among older adults. Poor nutritional status is a risk factor for mobility impairment; however, the 55-year-old female patient has no other known risk factors. Her mental illness is irrelevant to her risk of mobility impairment, except as it affects her ability to follow her provider's dietary instructions and comply with her medication schedule after discharge. The 11-year-old boy may experience adverse consequences of impaired mobility, such as altered self-concept, diminished self-esteem, and depression. He may become restless or even show signs of aggression. It is unlikely, however, that his mobility will be permanently compromised after he recovers from his traumatic injury. The 79-year-old male patient's age puts him at risk of mobility impairment; however, he has no other known risk factors

What is a Pap smear? A. A screening test for cervical cancer B. A screening test for colon cancer C. A screening test for sexually transmitted diseases D. None of the above

A. A screening test for cervical cancer A Pap smear is a screening test for cervical cancer and has contributed to a significant decline in the incidence of and mortality from cervical cancer. A Pap smear is not a screening test for colon cancer or sexually transmitted diseases.

The Rinne and Weber tests measure which of the following? A. Air and bone conduction B. Movement of the tympanic membrane C. Tenderness of the mastoid area D. Nodules in the auricles

A. Air and bone conduction A vibrating tuning fork is used to measure air and bone conduction. Movement of the tympanic membrane, tenderness of the mastoid area, and nodules in the auricles are not measured by the Rinne or Weber test.

Which of the following techniques is used to assess muscle strength in a patient? A. Apply an opposing force or resistance. B. Observe the patient at rest. C. Percuss the muscle. D. Palpate the muscle.

A. Apply an opposing force or resistance.

Which of the following statements is the most complete description of the tail of Spence? A. Breast tissue that extends into the axilla B. A common place to find breast lumps C. Not included in a breast exam D. Both A and B

A. Breast tissue that extends into the axilla B. A common place to find breast lumps The tail of Spence is breast tissue that extends into the axilla. It is a common place to find breast lumps upon palpation. The tail of Spence is part of a thorough breast examination.

Which of the following cranial nerves is assessed by observing the patient making specific facial movements? A. Cranial Nerve VII B. Cranial nerve XI C. Olfactory nerve D. Acoustic nerve

A. Cranial Nerve VII The facial nerve (cranial nerve VII) is assessed by observing the patient making specific facial movements. Cranial Nerve XI is assessed by having the patient shrug against resistance. The olfactory nerve is assessed by having a patient close his or her eyes, inhale deeply, and identifying the smell. The acoustic nerve is assessed by performing the whispered voice test.

Which test or tests assess accuracy of movement? A. Finger-to-finger test B. Finger-to-nose test C. Heel-to-shin test D. All of the above

A. Finger-to-finger test B. Finger-to-nose test C. Heel-to-shin test All of the above tests can be used to assess accuracy of movement. The finger-to-finger test is used to assess accuracy of movement. The patient's movements should be rapid, smooth, and accurate with no past pointing. The finger-to-nose test is used to assess accuracy of movement. The patient's movements should be rapid, smooth, and accurate, even with increasing speed. The heel-to-shin test is used to assess accuracy of movement. The patient should move his heel in a straight line without deviations to the side.

Which of the following methods is correct for examining the ear of an adult patient with an otoscope? A. Gently pull the auricle up and back. B. Gently pull the auricle down and back. C. Use the largest speculum that will fit comfortably in the patient's ear. D. Both A and C.

A. Gently pull the auricle up and back. C. Use the largest speculum that will fit comfortably in the patient's ear. Gently pulling the auricle up and back in the adult will straighten the auditory canal. Using the largest speculum that will fit comfortably is part of the otoscopic examination. Gently pulling the auricle down and back is the correct procedure for examining a child.

In which position should the patient be placed in order to palpate the popliteal pulse? A. Have the patient lie prone with the knee flexed. B. Have the patient lie prone with the leg straight. C. Have the patient lie supine with the knee flexed. D. Have the patient lie supine with the leg straight.

A. Have the patient lie prone with the knee flexed The best way to palpate the popliteal pulse is with the patient prone and the knee flexed. Pulsations of the popliteal pulse can be palpated deep in the popliteal fossa lateral to the midline. Having the patient lie prone or supine with the leg straight or supine with the knee flexed does not allow for deep palpation of the popliteal pulse.

What is included in the preparation for an assessment of the female genitalia? A. Having the patient empty the bladder B. Explaining the exam thoroughly if it is the patient's first exam C. Laying the head of the table flat D. Both A and B

A. Having the patient empty the bladder B. Explaining the exam thoroughly if it is the patient's first exam Having the patient empty her bladder and explaining the exam thoroughly are both part of preparation for assessment of the female genitalia. The female patient is instructed to empty the bladder before assessment of the female genitalia. If this is the patient's first exam, the exam is explained thoroughly. A model or illustration is used to show the patient what will happen and what will be looked for. The head of the table is elevated slightly.

A whispered voice test includes which of the following? A. Having the patient wiggle a finger in the opposite ear. B. Whispering while standing 1 to 2 feet from the patient's ear. C. Playing soothing music in the examination room. D. Both A and B.

A. Having the patient wiggle a finger in the opposite ear. B. Whispering while standing 1 to 2 feet from the patient's ear. Having the patient wiggle a finger in opposite ear ensures that patient is hearing the whisper in the ear being tested. A whispered voice test is performed while standing 1 to 2 feet from the patient's ear. Music can be a distraction or interference with a whispered voice test.

Where is the apical impulse located? A. In the fifth intercostal space at the midclavicular line B. In the third intercostal space at the left sternal border C. In the second intercostal space at the left sternal border D. In the second intercostal space at the right sternal border

A. In the fifth intercostal space at the midclavicular line Rationale: The apical impulse is located in the fifth intercostal space at the midclavicular line. The apical impulse is not located in the third intercostal space at the left sternal border. The apical impulse is not located in the second intercostal space at the left sternal border. The apical impulse is not located in the second intercostal space at the right sternal border.

Assessment of the ears includes which of the following? A. Inspection B. Palpation C. Examination with an otoscope D. All of the above

A. Inspection B. Palpation C. Examination with an otoscope

A nurse is assessing a patient's neck with the patient seated. Which of the following is considered an unexpected finding? A. Jugular vein distention B. Midline trachea C. Lack of bruits in carotid arteries D. Thyroid symmetry bilaterally

A. Jugular vein distention

Which complication of pregnancy does a nurse suspect when the patient reports painless vaginal bleeding at 32 weeks of gestation? a. Placenta previa b. Eclampsia c. Abruptio placentae d. Premature rupture of membranes

ANS: A Feedback A Placenta previa has painless vaginal bleeding as a manifestation. B Eclampsia does not have painless vaginal bleeding as a manifestation. C Abruptio placentae does not have painless vaginal bleeding as a manifestation. D Premature rupture of membranes does not have painless vaginal bleeding as a manifestation.

Which structures are included in a complete assessment of the mouth? A. Lips, mucosa, teeth, and gums B. Tongue and floor of the mouth C. Hard and soft palates D. All of the above

A. Lips, mucosa, teeth, and gums B. Tongue and floor of the mouth C. Hard and soft palates Lips, mucosa, teeth, gums, tongue, floor of the mouth, and hard and soft palates are all evaluated in a thorough assessment of the mouth. Lips, mucosa, teeth, and gums alone are only part of a thorough assessment of the mouth. Tongue and floor of mouth alone are only part of a thorough assessment of the mouth. Hard and soft palates alone are only part of a thorough assessment of the mouth.

When assessing the breasts, which of the following is considered normal? A. Long-standing, unchanging nevi B. Reddened Montgomery glands C. Peau d'orange skin texture D. Nipples that are not symmetrical

A. Long-standing, unchanging nevi Long-standing, unchanging nevi are a normal finding when assessing the breasts. Reddened Montgomery glands are not a normal finding when assessing the breasts. Montgomery tubercles should not be tender or suppurative. Peau d'orange skin texture is not a normal finding when assessing the breasts. Nipples that are not symmetrical is not a normal finding when assessing the breasts. The nipples should point forward similarly.

In which patient population does benign prostatic hypertrophy occur most commonly? A. Males over 50 years of age B. Adolescent males C. Males between 20 and 35 years of age D. Males between 35 and 50 years of age

A. Males over 50 years of age

Which of the following is considered an ABNORMAL finding in an older adult? A. Malignant melanoma B. Cherry angioma C. Seborrheic keratoses D. Lentigines

A. Malignant melanoma

What are the Snellen and Rosenbaum charts used to assess? A. Optic nerve B. Trigeminal nerve C. Abducens nerve D. Facial nerve

A. Optic nerve The Snellen and Rosenbaum charts are used to assess the optic nerve. The Snellen chart tests distance vision, and the Rosenbaum chart tests near vision

Which are considered basic techniques for physical assessment? Select all that apply. A. Palpation B. Medication reconciliation C. Inspection D. Auscultation E. History of present illness F. Percussion

A. Palpation C. Inspection D. Auscultation F. Percussion

Which of the following patients are considered at increased risk for colon cancer? A. Patients with a history of chronic inflammatory bowel disease B. Patients with a family history of adenomatous polyposis C. Patients with a history of appendicitis D. Both A and B

A. Patients with a history of chronic inflammatory bowel disease B. Patients with a family history of adenomatous polyposis Both a history of chronic inflammatory bowel disease and a family history of adenomatous polyposis put a patient at increased risk for colon cancer. Patients with a history of chronic inflammatory bowel disease are at increased risk for colon cancer. Patients with a family history of adenomatous polyposis are at increased risk for colon cancer. A history of appendicitis is not a risk factor for colon cancer; therefore these patients are not at an increased risk for colon cancer.

What is the correct way to palpate the frontal sinuses? A. Press the thumbs against the brow bones. B. Press the thumbs along the sides of the nose. C. Press the bridge of the nose between the thumb and first finger. D. Press the tip of the nose between the thumb and first finger.

A. Press the thumbs against the brow bones Pressing the thumbs against the brow bones is the correct way to palpate the frontal sinuses. Pressing the thumbs along the sides of the nose is not the correct way to palpate the frontal sinuses. Pressing the bridge or the tip of the nose between the thumb and first finger is not the correct way to palpate the frontal sinuses.

Which action would take priority if a patient's apical pulse has an irregular rhythm? A. Reassess the pulse for 1 full minute. B. Assess the patient's peripheral pulses. C. Wait 5 minutes, and then reassess the apical pulse. D. Review documentation regarding an irregular rhythm

A. Reassess the pulse for 1 full minute. A second measurement confirms the initial findings of an irregular heart rate. Irregular rate is more accurately assessed when measured over a longer interval. Peripheral pulse assessment is not a priority. Apical pulse is the most accurate pulse site. An irregular apical pulse already indicates an alteration in cardiac output. Waiting 5 minutes to reassess the apical pulse is not a priority action and could be dangerous in some unstable cardiac conditions. Reviewing documentation regarding an irregular rhythm is not the priority action, because establishing a history of irregular heartbeat is not essential.

Which of the following are included in the assessment of mental status? A. Speech and language B. Emotional stability C. Physical appearance and behavior D. All of the above

A. Speech and language B. Emotional stability C. Physical appearance and behavior

Which of the following indicates normal respiratory function? A. Symmetrical chest expansion B. Nasal flaring C. Use of accessory muscles D. Lip pursing

A. Symmetrical chest expansion Symmetrical chest expansion is a sign of normal respiratory function. Nasal flaring is not a normal finding and may be a sign of respiratory distress. Use of accessory muscles is not a normal finding and may be a sign of respiratory distress. Lip pursing is not a normal finding and is often taught to COPD patients to control shortness of breath.

Which of the following should be included in a male genital exam? A. Teaching the patient how to do self-exams B. Palpating for abnormalities C. Retracting and replacing the foreskin in an uncircumcised patient D. All of the above

A. Teaching the patient how to do self-exams B. Palpating for abnormalities C. Retracting and replacing the foreskin in an uncircumcised patient Palpating for abnormalities, retracting and replacing the foreskin in an uncircumcised patient, and teaching a patient how to do self-exams are all included in a male genital exam. Teaching the patient how to do self-exams is included in a male genital exam, including instructing the patient to use a mirror to help visualize the genital area. Palpating for abnormalities is included in a male genital exam. Retracting and replacing the foreskin in an uncircumcised patient is included in a male genital exam.

In inspecting the scrotum, the nurse documents which finding as normal? a. The epididymides are round, solid nodular masses. b. The scrotum is deeply pigmented with a rugous surface. c. The scrotal skin is a lighter color than the body skin. d. The vas deferens is palpable bilaterally.

ANS: B Feedback A Normally the epididymis is a tubular, comma-shaped structure. B This is an expected finding. C Normally the scrotal skin is more deeply pigmented than the body skin. D Normally the vas deferens is not palpable.

The gag reflex should be: A. Tested with a tongue blade on each side of the oropharynx B. Present on both sides C. Tested in the midline of the oropharynx only D. Both A and B

A. Tested with a tongue blade on each side of the oropharynx B. Present on both sides Testing with a tongue blade on each side of the oropharynx is part of assessing the gag reflex. The gag reflex should be present on both sides. Testing with a tongue blade on each side of the oropharynx is part of assessing the gag reflex. There should be a bilateral response to the gag reflex. Testing in the midline of the oropharynx is not part of assessing the gag reflex

Which of the following actions are part of the assessment of the glossopharyngeal and vagus nerves? A. Testing the gag reflex B. Having the patient swallow C. Touching the patient's face with dull and sharp objects D. Both A and B

A. Testing the gag reflex B. Having the patient swallow Both testing the gag reflex and having the patient swallow are part of a thorough assessment of the glossopharyngeal and vagus nerves. Testing the gag reflex is part of a thorough assessment of the glossopharyngeal and vagus nerves. When the posterior wall of the pharynx is touched, the patient should gag and the uvula should stay midline. Having the patient swallow is part of a thorough assessment of the glossopharyngeal and vagus nerves. Have the patient drink some water while you observe her ability to swallow. Touching the patient's face with dull and sharp instruments is not part of assessment of the glossopharyngeal and vagus nerves.

The nurse is assessing a superficial mass on a patient skin surface. Which part of the hand is used to palpate a superficial mass in the skin? A. The fingertips B. The heel of the hand C. The dorsal surface of the hand D. The ulnar surface of the hand

A. The fingertips

When using the Snellen chart, what does a vision evaluation of 20/50 mean? A. The patient has difficulty seeing far objects clearly. B. The patient can read at 20 feet what most people can read at 50 feet. C. The patient can read at 50 feet what most people can read at 20 feet. D. Both A and B.

A. The patient has difficulty seeing far objects clearly. B. The patient can read at 20 feet what most people can read at 50 feet. 20/50 on the Snellen chart means that the patient has difficulty seeing far objects clearly and can read at 20 feet what most people can read at 50 feet. 20/50 on the Snellen chart does not mean that the patient can read at 50 feet what most people can read at 20 feet.

How would you assess sensitivity to superficial pain? A. Touch the patient with the sharp side of a broken tongue blade. B. Have the patient keep his or her eyes open. C. Allow 2 seconds between stimuli. D. Both A and C.

A. Touch the patient with the sharp side of a broken tongue blade. C. Allow 2 seconds between stimuli. Touching the patient with the sharp side of a broken tongue blade and allowing 2 seconds between stimuli are both part of a thorough assessment for superficial pain sensation. Assessment of superficial pain can be done by touching the patient with the sharp side of a broken tongue blade. With the patient's eyes closed, ask the patient to identify if the sensation is dull or sharp. Assessment of superficial pain is performed with the patient's eyes closed. For assessment of superficial pain, randomly apply the sharp and dull stimuli, allowing 2 seconds between stimuli to avoid a summative effect.

A nurse is palpating the breasts of a patient. Which of the following are important aspects of proper palpation of the breasts? A. Using finger pads to palpate B. Using a consistent pattern C. Making small circles at each part of the breast and gliding from place to place D. All of the above

A. Using finger pads to palpate B. Using a consistent pattern C. Making small circles at each part of the breast and gliding from place to place Using the finger pads to palpate in a consistent pattern and making small circles at each part of the breast while gliding from place to place are all part of a thorough assessment of the breasts. Using finger pads to palpate is part of a thorough assessment of the breasts. Using a consistent pattern is part of a thorough assessment of the breasts. Making small circles at each part of the breast and gliding from place to place is part of a thorough assessment of the breasts.

Normal breath sounds include: A. Vesicular sounds B. Rhonchi C. Wheezes D. Crackles

A. Vesicular sounds esicular sounds are normal breath sounds heard over the periphery of the lung. Rhonchi, wheezes, and crackles are adventitious sounds.

The nurse suspects that a patient has a fungal infection of the skin. Which instrument helps confirm this suspicion? A. Wood's lamp B. Otoscope C. Sniff test D. Slit lamp

A. Wood's lamp

1. A patient's blood pressure using the posterior tibial pulse is 104/72 while blood pressure using the brachial pulse is 112/84. This patient's ankle-brachial index (ABI) is _____.

ANS: 0.92 Posterior tibial systolic pressure (104) divided by the brachial systolic pressure (112) = 0.92. The systolic pressures are the numbers used to calculate the ABI.

A pregnant patient's weight before pregnancy was 148 lb. Her expected weight during the first trimester is _____ to _____ lb.

ANS: 151 153 151, 153 3- to 5-lb weight gain is expected during the first trimester resulting in a total of 151 to 153 lb.

A pregnant patient's weight before pregnancy was 163 lb. The nurse expects the patient to weigh _____ to _____ lb during the second trimester.

ANS: 178; 183 178, 178 A 3- to 5-lb weight gain is expected during the first trimester, making the weight 166 to 168. The second trimester is 12 to 15 lb in addition to the first trimester weight. Thus 166 + 12 lb = 178 lb at the low end and 168 + 15 lb = 183 at the high end.

1. A patient tells the nurse that he has smoked 1 packs of cigarettes a day for 14 years. The nurse records this as _____ pack-years?

ANS: 21 1 packs of cigarettes ´ 14 years = 21 pack-years.

If a patient's last menstrual period was May 13, her estimated date of birth is ________.

ANS: February 20 May 13 minus 3 months = February 13 plus 7 days = February 20.

What is the patient's expected response when the nurse is assessing stereognosis? a. Identifies an object placed in the hand b. Distinguishes numbers or letters traced in the palm of the hand c. Touches the index finger of the nondominant hand to the nose d. Walks heel to toe in a straight line

ANS: A Feedback A A nurse tests stereognosis by asking the patient to close his or her eyes and placing a small, familiar object in the patient's hand, asking him or her to identify it. Stereognosis tests sensory nerve tracts and parietal lobe function. B This activity tests graphesthesia, a test of sensory nerve tracts and parietal lobe function. C This activity tests cerebellar function of the upper extremities. D This activity tests cerebellar function of the lower extremities.

How does a nurse describe abruptio placentae when teaching a class to pregnant women? a. A premature separation of the implanted placenta before birth of the fetus b. An accumulation of amniotic fluid associated with decreased fetal movement c. An attachment of the placenta in the lower uterine segment near the cervical os d. Related to premature rupture of membranes with purulent vaginal discharge

ANS: A Feedback A A premature separation of the implanted placenta before birth of the fetus is a description of abruptio placentae. B This is an incorrect description. C This is a description of placenta previa. D Abruptio placentae is premature separation of the implanted placenta rather than a premature rupture of membranes; there is no purulent vaginal discharge.

During the assessment, the nurse determines that the patient's Glasgow Coma Scale score is 15. What is the meaning of this number for this patient? a. This patient is fully conscious. b. This patient has movement but does not open the eyes or speak. c. This patient is unable to respond to any stimuli. d. This patient opens the eyes but does not speak or move.

ANS: A Feedback A A score of 15 is the expected value for the Glasgow Coma Scale. B This patient would score a 9 on the Glasgow Coma Scale. C This patient would score a 3 on the Glasgow Coma Scale. D This patient would score a 4 on the Glasgow Coma Scale.

Which behavior would be most indicative of hearing impairment in a 1-year-old child? a. Failure to respond to mother's voice b. Crying when a loud noise occurs unexpectedly c. Saying only single-syllable words d. Disinterest in playing with musical toys

ANS: A Feedback A By age 4 to 6 months, an infant should turn the head toward the source of the sound and should respond to the parent's voice. B This behavior indicates an ability to hear loud noises. C This is an expected finding for a child this age. D Although an infant by age 4 to 6 months should respond to music toys, a disinterest does not indicate a hearing problem.

During an office visit, a 78-year-old woman is upset because her height is "2 inches less than it was when I was 40!" How does the nurse explain this change to the patient? a. "Reduced height may occur as you age due to shortening of the vertebrae." b. "You may be experiencing this height change due to arthritis." c. "You need to improve your posture by performing stretching exercises." d. "This is a rare occurrence and warrants having a bone density test."

ANS: A Feedback A Decreased bone formation reduces height in most older adults, which may cause shortening of the vertebrae and thinning of the vertebral disks. B Decreased bone formation reduces height in most older adults and is not due to arthritis. C This is appropriate advice, but is not related the patient's height. D This is an expected occurrence and does not warrant concern.

In assessing a patient's deep tendon reflexes, a nurse finds a patient has a 4+ triceps response. How does the nurse interpret this finding? a. A hyperactive response b. A diminished response c. An absent response d. An expected response

ANS: A Feedback A Deep tendon reflexes are graded from 0 to 4+ and 4+ is a hyperactive response. B A diminished response is 1+. C An absent response is 0. D An expected response is 2+.

The nurse recognizes which patient has the highest risk of endometrial cancer? a. A 24-year old woman with menarche at age 9 b. A 30-year old woman who started menstruating at age 19 c. A 42-year old woman who reached menopause at age 40 d. A 64-year old woman who had irregular, heavy menstrual cycles

ANS: A Feedback A Early menarche is a risk factor. B This age of menarche is not a risk factor for endometrial cancer. C This age of menopause is not a risk factor for endometrial cancer. Patients who have late onset menopause are at risk. D An irregular, heavy menstrual cycle is not a risk factor for endometrial cancer.

A nurse documents as abnormal which finding of a pregnant patient? a. Facial swelling in a woman who is 20 weeks pregnant b. 1+ pitting ankle edema in a woman who is 26 weeks pregnant c. Pinkish-red blotches of the hands in a woman at 32 weeks gestation d. Blotchy, brownish pigmentation of the face in a woman at 36 weeks gestation

ANS: A Feedback A Excessive edema (particularly if noted in the hands and face in addition to the lower extremities) is considered pathologic and may be an indication of pregnancy induced hypertension. B Some edema is expected during pregnancy and 1+ is a modest amount of edema. C Pinkish-red blotches or diffuse mottling of the hands due to an increase in estrogen is termed palmar erythema and is considered an expected finding. D Blotchy, brownish pigmentation of the face—chloasma, or the mask of pregnancy—is an expected finding.

A 50-year-old patient asks the nurse about her risk of developing a cancer of the reproductive system. What is the appropriate response by the nurse? a. "Human papilloma virus infection and cigarette smoking are major risk factors for cervical cancer." b. "Some of the risk factors for endometrial cancer include being age 40 or older and having a history of infertility." c. "Ovarian cancer is not often seen in women under age 50 or those who have a family history of breast cancer." d. "Women who have had menstrual irregularities for many years are at lower risk of developing any of the reproductive system cancers."

ANS: A Feedback A Human papilloma virus infection and cigarette smoking are risk factors for cervical cancer. B These are not risk factors for endometrial cancer. C These are not risk factors for ovarian cancer. D These are not risk factors for gynecologic cancers.

A patient reports she has shortness of breath and peripheral edema. Under which category does the nurse document these data? a. Review of systems b. Present health status c. Past health history d. Functional ability

ANS: A Feedback A In the review systems part of the history, the nurse asks patients whether they have had symptoms from specific body systems. Patients either deny or admit to having the symptoms. B The present health status category contains data on chronic diseases, medications, and allergies. C The past health history category contains data on childhood diseases, surgeries and hospitalizations, and immunizations. D The functional ability category contains data on patient's activities, including maintaining a home and working full-time.

The nurse correlates which clinical manifestation with the diagnosis of polyhydramnios? a. Difficulty palpating fetal parts b. Increased fetal movement c. Weight gain of less than estimated by gestational age d. Increase of 2 cm in fundal height in 1 week

ANS: A Feedback A Inability to palpate the fetal position could be associated with polyhydramnios, an excessive quantity of amniotic fluid. B This is not a manifestation of excessive amniotic fluid. C This is not a manifestation of excessive amniotic fluid. D This is not a manifestation of excessive amniotic fluid.

A nurse asks the patient to stand with feet together, arms resting at the sides, with eyes open and then with the eyes closed. Which response by the patient indicates an expected cerebellar function? a. Sways slightly and maintains upright posture with feet together b. Is unable to stand upright after turning around in a circle once c. Steps sideways when standing with feet together and eyes closed d. Has to move arms horizontally to maintain balance

ANS: A Feedback A Maintaining balance indicates function of the cerebellum in the Romberg test. B Losing balance is an abnormal response, but turning in a circle is not a part of the Romberg test. C This is an abnormal response for the Romberg test (a positive Romberg test). D This is an abnormal response for the Romberg test (a positive Romberg test).

A nurse becomes suspicious that a patient may have breast cancer based on which abnormal finding? a. An irregularly shaped hard mass in one breast b. Bilateral, small, nontender nodes close to the surface c. Multiple rubbery-feeling lumps with well-defined borders d. A mobile, firm lump located in the upper outer quadrant of the left breast

ANS: A Feedback A Malignant masses are solitary, irregularly-shaped, unilateral, nontender, and immobile. B Malignant masses are unilateral. C Malignant masses have irregular, poorly defined borders. D Breast cancer often occurs in the upper outer quadrant, but is immobile.

The nurse correlates which factor to an increased risk of endometrial cancer in women with early menarche or late menopause? a. Total number of ovulatory cycles b. Less hormone stimulation c. Need for estrogen replacement in these patients d. Extended duration of the menstrual cycle in these patients

ANS: A Feedback A More ovulatory cycles increases risk. These risk factors represent an increased cumulative exposure to estrogen. B Hormone stimulation does not increase risk in endometrial cancer. C Estrogen replacement does not increase risk in endometrial cancer. D Extended duration of the menstrual cycle is not a risk factor.

Based on the history, a nurse determines that the patient with which finding requires further assessment? a. Occasional discharge from nipples b. Supernumerary nipples along the milk line c. Rash in the axillae associated with change in deodorant d. Mild breast swelling that fluctuates with the menstrual cycle

ANS: A Feedback A Nipple discharge is usually an abnormal finding. A specimen of the discharge should be collected. B Supernumerary nipples along the milk line are a normal finding in some women. C A rash in the axillae associated with change in deodorant can be solved by changing deodorant and treating the rash. D Mild breast swelling that fluctuates with the menstrual cycle is consistent with fibrocystic breast disease.

Which response does a nurse expect when testing ankle clonus of a healthy woman? a. No movement of the foot b. Plantar flexion of the foot c. Extension of the lower leg d. Dorsiflexion of the foot

ANS: A Feedback A No movement of the foot is the expected response from a healthy woman. B Plantar flexion of the foot is not a response to ankle clonus. C Extension of the lower leg is not a response to ankle clonus. D Dorsiflexion of the foot is an abnormal response of ankle clonus.

A nurse notices that an infant tries to reach for a toy that the mother has hidden in her hand. This illustrates that the child has developed an understanding of which concept? a. Object permanence b. Trust versus mistrust c. Autonomy d. Parallel play

ANS: A Feedback A Object permanence, meaning that objects and people still exist when they are out of sight. B This is Erikson's developmental stage for infants when they develop trust relationships with the mother or primary caregiver. Important criteria are the quality and consistency of the mother-child relationship. C This is Erikson's developmental stage for toddlers who yearn for independence; however, they lack judgment to maintain their safety. D Parallel play is a social skill of a 2-year-old toddler.

Which finding rules out defects in the cornea, lens, and vitreous chamber of an infant? a. Bilateral red reflex b. Symmetric corneal light reflex c. Bilateral blink reflex d. Symmetric eye movements

ANS: A Feedback A Presence of the red reflex eliminates the presence of most serious defects of the cornea, aqueous chamber, lens, and vitreous chamber. B Symmetric corneal light reflex tests for symmetric eye muscle function and eliminates the presence of strabismus. C Bilateral blink reflex tests the function of cranial nerve V (trigeminal). D Symmetry of extraocular muscles tests cranial nerves III (oculomotor), IV (trochlear), and VI (abducens).

On inspection of the internal structure of the vagina, the nurse notes a rounded protrusion on the posterior wall of the vagina. How does the nurse document this finding? a. Rectocele b. Cystocele c. Bartholin cyst d. Nabothian cyst

ANS: A Feedback A Rectocele is a hernia type of protrusion of the rectum against the posterior wall of the vagina. B Cystocele is a hernia type of protrusion of the bladder against the posterior wall of the vagina. C The Bartholin glands are external structures. D Nabothian cysts appear on the cervix.

During inspection of the mouth of an older adult, a nurse notices which finding as an expected change associated with aging? a. Exposed root surfaces of teeth b. Aphthous ulcers of the mucosa c. Collection of debris at the gingival margins d. Leukoplakia of the dorsal and ventral tongue

ANS: A Feedback A Root surfaces of the teeth are exposed to caries formation because of gingival recession. B Ulcers are not an expected finding. C Collection of debris at the gingival margins is not an expected finding. D This is not an expected finding; leukoplakia is a white patch or plaque that cannot be scraped off and often represents a premalignant lesion.

While giving a history, the patient reports having herpes genitalis. Based on this information, which finding does the nurse anticipate during the assessment? a. Small vesicles on the genitalia b. Single, firm, painless, open sore c. Pain when palpating the cervix d. Malodorous greenish-yellow vaginal discharge

ANS: A Feedback A Small vesicles on the genitalia are consistent with genital herpes. B Single, firm, painless, open sore is consistent with primary syphilis. C Pain when palpating the cervix is consistent with Chlamydia. D Malodorous greenish-yellow vaginal discharge is consistent with trichomonas.

After a rectal examination of a patient with obstructive jaundice, the nurse expects the stool to be what color? a. Tan b. Pale yellow c. Black d. Bright red

ANS: A Feedback A Tan stool indicates a lack of bile caused by obstructive jaundice. B Pale yellow stools indicate a malabsorption syndrome. C Black stools indicate upper intestinal tract bleeding or excessive iron or bismuth ingestion. D Bright red indicates bleeding from the lower rectum or hemorrhoids.

While giving a history, a patient reports having a weak urinary stream and feeling that his bladder is not empty after urination. Based on these data, what finding does the nurse anticipate upon examination? a. An enlarged prostate gland palpated on the anterior wall of the rectum b. An indirect hernia palpated through the inguinal ring when the patient coughs c. The foreskin of the penis cannot be returned to position after retraction behind the glans d. A nodular prostate gland palpated on the posterior wall of the rectum

ANS: A Feedback A The enlarged prostate compresses the urethra, causing difficulties with voiding. B A hernia would not interfere with voiding. C This is a description of paraphimosis. D The posterior prostate is palpated on the anterior surface of the rectum.

A nurse assesses a patient with a head injury who has slowing intellectual functioning, personality changes, and emotional lability. The nurse correlates these findings with which area of the brain? a. Frontal lobe b. Parietal lobe c. Thalamus d. Temporal lobe

ANS: A Feedback A The frontal lobe controls intellectual function, awareness of self, personality, and autonomic responses related to emotion. B The parietal lobe receives sensory input such as position sense, touch, shape, and texture of objects. C The thalamus is a relay and integration station from the spinal cord to the cerebral cortex and other parts of the brain. D The temporal lobe contains the primary auditory cortex. It also interprets auditory, visual, and somatic sensory inputs that are stored in thought and memory.

A nurse assessing a patient who had a cerebrovascular accident involving the Broca area suspects expressive or nonfluent aphasia. What communication abilities does the nurse anticipate from this patient? a. The patient understands speech but is unable to translate ideas into meaningful speech. b. The patient is unable to comprehend speech and thus does not respond verbally. c. The patient is able to understand speech but has difficulty forming words, creating muffled speech. d. The patient is unable to comprehend speech and responds inappropriately to conversation.

ANS: A Feedback A The inability to translate ideas into meaningful speech or writing is termed expressive aphasia or nonfluent aphasia and is associated with lesions in the Broca area in the frontal lobe. B The inability to comprehend the speech of others is called receptive aphasia or fluent aphasia and is associated with lesions in the Wernicke area in the temporal lobe. C This speech pattern is more consistent with patients who have involvement of muscles of speech rather than neurologic deficits. D This speech pattern is not relevant to this patient.

Which approach does a nurse use to assess neck range of motion of an older adult patient? a. Have the patient perform each neck movement separately. b. Defer range of motion examination if the patient has kyphosis. c. Ask the patient to turn the head against the resistance of the nurse's hand. d. Ask the patient to rotate the head starting with forward flexion and moving clockwise.

ANS: A Feedback A The nurse should assess range of motion of the neck with one movement at a time, rather than a full rotation of the neck, to avoid causing dizziness on movement. B Assessing range of motion is important data to gather to determine how limited the range is due to the kyphosis. C This technique tests muscle strength rather than range of motion. D This technique tests muscle strength rather than range of motion.

A nurse shines the light from the ophthalmoscope into the eyes of a newborn and observes a bright, round, red-orange glow seen through both pupils. How does the nurse document this finding? a. An expected red reflex b. Eyelid capillary hemangiomas c. Bilateral conjunctivitis d. Ophthalmia neonatorum

ANS: A Feedback A The red reflex appears as a bright, round, red-orange glow seen through both pupils. B Eyelid capillary hemangiomas are abnormalities that appear on the eyelid, but disappear spontaneously. C Redness, lesions, nodules, discharge, or crusting of conjunctiva indicate conjunctivitis. D Ophthalmia neonatorum is an eye disorder that produces purulent conjunctivitis and keratitis.

During a physical examination, a 12-year-old girl expresses concern to the nurse that her breasts are different sizes. Which response is most appropriate for the nurse? a. "This happens normally to many girls your age. Full breast development takes an average of 3 years." b. "I can talk with your mother about a referral to a physician who can perform further examination and tests." c. "Have you started your menstrual period yet, because breast development is irregular until menstruation begins?" d. "This is called 'precocious breast development' and your breasts will become more equal just before your growth spurt starts."

ANS: A Feedback A The right and left breasts may develop at different rates. It is important to reassure the patient that this is common and, in time, the development may equalize. Full development of the breast takes an average of 3 years (range 1.5 to 6 years). B Breasts may develop at different rates. C Menarche begins when the breasts reach Tanner stages 3 or 4, which is approximately age 12. D Precocious breast development is the development of breasts before 8 years old.

Which characteristics are expected during an assessment of a normal toddler? a. Half of adult height achieved by age two, potbelly, and sway back. b. Thirty-two erupted teeth by age 2, tripled birth weight by age 30 months. c. Desire for autonomy coupled with sufficient judgment to ensure safety. d. Head circumference greater than chest circumference, high frustration tolerance.

ANS: A Feedback A These are characteristics of toddlers. B Toddlers have 20 teeth erupted and infants triple their birth weight. C The Erikson developmental stage for toddlers is autonomy versus shame and doubt. D The first part is true for toddlers, but they do not have a high frustration tolerance.

A nurse expects to find which manifestations in the male patient who has both Chlamydia and gonorrhea? a. Painful urination and purulent urethral discharge b. A single, firm painless open sore on the shaft of the penis c. Red superficial vesicles on the shaft of the penis d. A single or a cluster of wartlike growth in the anal-rectal area

ANS: A Feedback A These are manifestations of Chlamydia and gonorrhea. Dysuria means painful urination. B This is a description of a lesion consistent with syphilis. C This is a description of lesions consistent with herpes genitalis. D This is a description of lesions consistent with genital warts caused by human papillomavirus

What is the meaning of "G5, T1, P0, A3, L1" found in a patient's history? a. One birth at term b. Three living children c. Five grown children d. One delivery not at term

ANS: A Feedback A These data report that this patient has had five pregnancies, one term birth, no preterm births, three abortions, and one living child. B This patient has had three abortions. C This patient has had five pregnancies. D This patient has had no preterm births.

In assessing a patient with suspected Chlamydia, the nurse's actions are guided by which characteristic of this disease? a. Chlamydia is frequently asymptomatic and requires screening. b. Chlamydia is associated with a yellow-green vaginal discharge. c. Chlamydia is accompanied by heavy bleeding and headache. d. Chlamydia is only seen in immunocompromised patients.

ANS: A Feedback A This answer is consistent with clinical findings of Chlamydia. B This answer is consistent with clinical findings of gonorrhea. C Chlamydia is not accompanied by heavy bleeding and headache. D Chlamydia is seen in patients with healthy immune systems who are not immunocompromised.

Which technique does a nurse use to assess hip location of a newborn? a. With newborn's knees flexed, the nurse adducts the legs, then abducts them, moving the knees apart and down to touch the table. b. With the newborn supine, the nurse flexes and extends the hips, and then passively moves each leg through internal and external rotation. c. The nurse holds the newborn in a vertical position with the feet flat on the table and palpates each hip for location. d. With the newborn supine, the nurse measures the length of each leg from the trochanter to the lateral malleolus (ankle).

ANS: A Feedback A This describes the Barlow-Ortolani maneuver to assess hip location and determine dislocation. B This describes an incorrect technique. C This describes an incorrect technique. D This describes an incorrect technique.

A nurse is performing a breast examination of a patient who complains of pain in both breasts that occurs around the time of her menstrual period. The nurse expects which findings during the breast examination? a. Masses in the breasts that are round, soft, mobile, and well-delineated b. Masses in the breasts that are round, firm, mobile, and well-delineated c. Masses in the breasts that are irregular, hard, and fixed d. Breast tissue that is red, edematous, tender, and warm to the touch

ANS: A Feedback A This finding is consistent with fibrocystic breast disease. The findings are almost the same as fibroadenoma (except for consistency of the masses), but the nurse must correlate the history (occurs during the menses) with the findings for fibrocystic disease. B These findings are consistent with fibroadenoma. C These findings are consistent with breast cancer. D These findings are consistent with mastitis.

During a breast examination of a male patient, the nurse recognizes which finding as normal? a. Bilateral nontender flat breasts with symmetric nipple and areolar areas b. A fibrous layer of subcutaneous breast tissue that is thicker than in women c. Breast tenderness on the dominant side but not on the other side d. Bilateral symmetry of breasts with absence of hair in the areolar areas

ANS: A Feedback A This is a description of a normal finding. B This is not a normal finding. C This is not a normal finding. D This is not a normal finding.

While obtaining a symptom analysis from a patient who had a transient ischemic attack, the nurse helps the patient distinguish between dizziness and vertigo. Which description by the patient indicates dizziness? a. "I felt lightheaded when I stood up." b. "It felt like I was on a merry-go-round." c. "The room seemed to be spinning around." d. "My body felt like it was revolving and could not stop."

ANS: A Feedback A This is a description of dizziness that is often associated with transient ischemia attacks. B This report is consistent with objective vertigo because it includes a sensation of motion. C This report is consistent with objective vertigo because it includes a sensation of motion. D This report is consistent with subjective vertigo because it includes a sensation of one's body rotating in space.

In assessing the external eyes of an older adult, a nurse documents which finding as abnormal? a. Gray-white circle where the cornea and the sclera merge b. Brown spots near the limbus in both eyes c. Lack of luster of the eye and dry bulbar conjunctiva d. Lower lid drops away from the globe

ANS: D Feedback A This is a description of arcus senilis. B This is a description of a normal variation. C This occurs because the lacrimal apparatus may function poorly, producing fewer tears. D This is a description of ectropion, an abnormal finding.

When performing a well woman examination, the nurse expects what findings? a. The inner surface of the vestibule is deep pink and moist with a smooth texture. b. The inguinal skin appears wrinkled and moist with sparse hair distribution. c. The labia minora is deeply pigmented, and the tissue is ragged and asymmetrical. d. Pubic hair is distributed evenly over the mons and shaped as a triangle with the apex over the mons.

ANS: A Feedback A This is a normal finding of female external genitalia. B The inguinal area would normally not be wrinkled. C The labia minora is not normally ragged and asymmetric. D Normally the base of the triangle is over the mons.

Which finding indicates to a nurse that a neonate has a cephalhematoma? a. Well-defined edematous area over one cranial bone b. Molding of the cranium that causes generalized cerebral edema c. Diffuse edema over two or more cranial bones d. Anterior fontanelle that is deeply depressed

ANS: A Feedback A This is a subperiosteal hematoma under the scalp that occurs secondary to birth trauma. The area, which appears as a soft, well-defined swelling over the cranial bone, generally is reabsorbed within the first month of life. B Molding is secondary to the head passing through the birth canal and generally lasts less than a week. C Cephalhematoma occurs over one cranial bone rather than several, and is well-defined rather than diffuse. D Anterior fontanelles are soft but not depressed.

In educating a male patient about testicular cancer, the nurse includes which statement? a. The highest incidence of this cancer is in men between 20 and 34 years of age. b. The incidence of this cancer is correlated with human papillomavirus (HPV) infection. c. The risk of this cancer increases with multiple sexual partners. d. This type of cancer more commonly affects uncircumcised males.

ANS: A Feedback A This is a true statement of risk. B HPV infection is correlated with cervical cancer, rather than testicular cancer. C The risk of sexually transmitted diseases increases with multiple sexual partners rather than testicular cancer. D This is not a risk factor.

What technique does the nurse use to obtain a cervical tissue sample for a Papanicolaou (Pap) test? a. A Cervex-Brush is inserted into the cervix and rotated to obtain a sample of ectocervical and endocervical cells. b. A wooden spatula scrapes the cervix to obtain a sample of endocervical cells. c. A pipette is placed inside the cervical os and rotated to obtain a thick layer of endocervical and ectocervical cells. d. A cotton-tipped applicator is used on the outside of the cervix to obtain ectocervical cells.

ANS: A Feedback A This is the correct technique. B A wooden spatula is not used and ectocervical cells are needed as well as endocervical cells. C A pipette is not used and the cells are not contained in a thick layer. D Using a brush as opposed to a cotton-tipped applicator has improved the quality of the sample of endocervical cells and ectocervical cells.

In inspecting the eyes and ears of an infant, the nurse documents which finding as normal? a. The external ear is in direct line with the outer margin of the eyelid. b. The ear lobe is within 10° of alignment with the outer margin of the eyelid. c. A lateral upward slant of the eyes aligns them with the helix of the ear. d. The inner margin of the eye is directly aligned with the helix of the ear.

ANS: A Feedback A This is the expected alignment of the ears and eyes of an infant. B This is not an expected finding. C This finding occurs in infants with Down syndrome. D This is not an expected finding.

The nurse documents which finding as expected on inspection of the anus? a. Skin tone darker and coarser than that of the surrounding skin b. Sphincter lightly closed when the patient is relaxed c. Large amount of stiff, curling hair surrounding the anus d. Slight protrusion under the skin when the patient strains or bears down

ANS: A Feedback A This is the normal finding. B The anal sphincter should be tight. C The anus is typically hairless. D A protrusion may be a hemorrhoid, which is not an expected finding.

During an internal examination of a patient's anus, the nurse notes that the patient has a hypertonic sphincter. What is the most relevant action for the nurse to take at this time? a. Ask the patient about anxiety or pain related to the examination. b. Inquire if the patient has had any neurologic injury that causes a hypertonic sphincter. c. Refer the patient to the physician for evaluation. d. Question the patient about a history of anal trauma.

ANS: A Feedback A This is the relevant action for the nurse to collect more data from the patient about the reaction to the examination. B This datum probably would have been gathered during the history, so that the nurse would have anticipated an abnormal finding. C This is not the action of most importance at this time. The nurse needs to collect more data from the patient before considering a referral. D This is not the action of most importance at this time. The nurse needs to collect more data from the patient before asking about trauma.

Which statement reflects a 21-year-old woman's achievement of an expected developmental task? a. "I am planning to get married next year." b. "I don't plan anything without asking my boyfriend first." c. "I don't know which direction I'll take after college." d. "I am living with my parents and may stay for a while."

ANS: A Feedback A This statement reflects an expected developmental task of a young adult. B This statement does not reflect an expected developmental task of a young adult. C This statement does not reflect an expected developmental task of a young adult. D This statement does not reflect an expected developmental task of a young adult.

A patient reports having difficulty swallowing. Based on this information, how does the nurse assess the appropriate cranial nerve? a. Ask the patient to stick out the tongue and move it in all directions. b. Ask the patient to move the head to the right and left. c. Observe the symmetry of the face when the patient talks. d. Assess for taste on the anterior part of the tongue.

ANS: A Feedback A This tests the hypoglossal cranial nerve (CN XII) that is involved in swallowing. The nurse must correlate difficulty swallowing with the cranial nerves involved with that function and how to test them. The cranial nerves involved are IX, X, and XII. B This tests the function of the spinal accessory cranial nerve (CN XI). C This tests the motor function of the facial cranial nerve (CN VII). D This tests the sensory portion of the facial cranial nerve (CN VII).

How does a nurse respond to parents of a 5-year-old who are worried that their child has a protruding abdomen? a. Assesses the child to differentiate a normal "potbelly" from a hernia b. Suggests that the parents administer an appropriate dose of a laxative at bedtime c. Refers the parents to a nutritionist to develop an appropriate weight-loss diet for the child d. Informs the parents that a protruding abdomen is always an abnormal finding in this age group

ANS: A Feedback A Toddlers normally exhibit a rounded (potbelly) abdomen while both standing and lying down. B The nurse needs to collect more data before making this recommendation. C The nurse needs to collect more data before making this recommendation. D This is an inappropriate statement based on the description of this child.

1. When does the health assessment begin? a. When the nurse first meets the patient b. When the patient tells the nurse his name and age c. When the nurse asks the patient the first health-related question d. When the patient consents to have a health assessment performed

ANS: A Feedback A When the nurse and patient first meet, the nurse begins collecting data about the patient. B Before this, the nurse began collecting data about the patient, such as gait, posture, and hygiene. C Before this, the nurse began collecting data about the patient, such as gait, posture, and hygiene. D Before this, the nurse began collecting data about the patient such as gait, posture, and hygiene.

A nurse performing a breast examination on a female patient places the patient in a supine position, places a pillow under the right shoulder, and asks the patient to place her right lower arm above her head. What is the reason for this position? a. Flatten the breast tissue evenly over the chest wall. b. Help the patient to relax and feel more comfortable. c. Reveal lumps deep in the breast more easily. d. Expose any drainage from the nipples.

ANS: A Feedback A This is the reason for the position. B Flattening the breast tissue, not relaxation, is the reason. C Lumps are detected by palpation rather than inspection. D Drainage is revealed by palpation of the nipples.

On examination, a nurse finds the patient has a productive cough with green sputum and inspiratory crackles. What other findings does this nurse expect during the examination? Select all that apply. a. Dull tones to percussion b. Increased vibration on vocal fremitus c. Fever d. Decreased diaphragmatic excursion e. A sharp, abrupt pain reported when patient breathes deeply f. Muffled sounds heard when the patient says "e-e-e"

ANS: A, B, C, E Correct: These abnormal findings are consistent with consolidation that may occur with pneumonia. Incorrect: Decreased diaphragmatic excursion occurs when the lung is overinflated as in emphysema. Muffled sounds when the patient says "e-e-e" is an expected finding. With a consolidation, the sound of "e-e-e" would be clear.

Which patient using respiratory equipment requires skin assessment? Select all that apply. a. A patient using a nasal cannula b. A patient with a tracheostomy c. A patient using an incentive spirometer d. A patient using a Ventimask e. A patient with an IV

ANS: A, B, D Correct: Patients using a nasal cannula need inspection of the nares and behind the helix of the ears. Patients with a tracheostomy need inspection of skin around the stoma where the tracheostomy tube enters the trachea. Patients using a Ventimask need inspection of skin where the mask comes in contact with the face and behind the helix of the ears. Patients with IVs need inspection of the skin to verify the catheter is secured and to assess for redness or edema. Incorrect: Using an incentive spirometer requires the patient to take deep breaths, thus a skin assessment is not indicated.

Alcoholism increases the risk of cancers of the gastrointestinal tract. Which cancer risk is increased in patients with alcoholism? Select all that apply. a. Esophageal cancer b. Stomach cancer c. Pancreatic cancer d. Liver cancer e. Colon cancer f. Bladder cancer

ANS: A, B, D, E Correct: The risk of esophageal, stomach, liver, and colon cancers are increased by heavy intake of alcohol. Incorrect: The risk of pancreatic and bladder cancers are increased with tobacco. However, the risk for esophageal, stomach, liver, and colon cancers are also increased with tobacco use.

Which movements does a nurse expect to find when assessing the ankle range of motion of a healthy person? Select all that apply. a. Inversion and eversion b. Plantar flexion and dorsiflexion c. Pronation and supination d. Adduction and abduction e. Rotation

ANS: A, B, D, E Correct: These are all expected motions for the ankle joint. Incorrect: Pronation and supination are not expected motions for the ankle joint.

Which characteristics are risk factors for cerebrovascular accident? Select all that apply. a. Excessive alcohol intake b. Smoking c. Eating large amounts of smoked foods d. Obesity e. Atherosclerosis f. High blood pressure

ANS: A, B, D, E, F Correct: All of these are risk factors for cerebrovascular accident. Incorrect: Eating large amounts of smoked foods is a risk factor for stomach cancer.

To complete a symptom analysis, which questions does a nurse ask patient who recently had a seizure for the first time? Select all that apply. a. "Did you have any warning signs before the seizure started?" b. "Did you lose consciousness during the seizure?" c. "Did the room seem to be spinning around before the seizure?" d. "Did you urinate during the seizure?" e. "What did you hear while you were seizing?" f. "How did you feel after the seizure?"

ANS: A, B, D, F Correct: These are all appropriate questions to ask to gather more data about this patient's first seizure. Incorrect: "Did the room seem to be spinning around before the seizure?" This question is about vertigo, which does not relate to this patient. "What did you hear while you were seizing?" The answer to this question is not needed in the data for this patient.

Which questions are appropriate for a symptom analysis of a patient with benign prostatic hyperplasia? Select all that apply. a. "How often have you found that you stopped and started again several times when you urinated?" b. "How often have you had to urinate again less than 2 hours after you finished urinating?" c. "How often have you been incontinent of urine?" d. "How often have you had constipation due to the enlarged prostate?" e. "How often have you had to push or strain to begin urination?" f. "How often have you had to get up during the night to urinate?"

ANS: A, B, E, F Correct: These questions from the American Urological Association Symptom Index for Benign Prostatic Hyperplasia are used to screen men for an enlarged prostate. Incorrect: The urinary problem is difficulty in starting the urinary stream due to the prostate gland compressing the urethra, thus incontinence is not a manifestation of an enlarged prostate. Although the enlarged prostate is palpated through the rectum, it does not become large enough to cause constipation.

Which techniques does a nurse routinely use to collect data when assessing the lower extremities of a patient? Select all that apply. a. Inspecting of legs, ankles, and feet for skin characteristics and hair distribution b. Assessing for knee stability with the drawer test, McMurray test, or Apley test c. Palpating lower legs and feet for temperature, pulses, and tenderness d. Assessing for nerve root compression with straight leg raises e. Palpating hips for stability and tenderness f. Testing for patellar and Achilles deep tendon reflexes bilaterally

ANS: A, C Correct: These techniques are performed in a routine head-to-toe assessment of the lower extremities. Incorrect: Assessing for knee stability with the drawer test, McMurray test, or Apley test is not performed unless indicated by knee instability. Assessing for nerve root compression with straight leg raises is not performed unless indicated. Palpating hips for stability and tenderness is not performed unless the patient has unstable hips. Testing for patellar and Achilles deep tendon reflexes bilaterally is not performed unless indicated.

What are the functions of the upper airways? Select all that apply. a. Conduct air to lower airway. b. Provide area for gas exchange. c. Prevent foreign matter from entering respiratory system. d. Warm, humidify, and filter air entering lungs. e. Provide transportation of oxygen and carbon dioxide between alveoli and cells.

ANS: A, C, D Correct: These are functions of the upper airway. Incorrect: Gas exchange occurs in the alveoli. The cardiovascular system provides transportation of oxygen and carbon dioxide between alveoli and cells.

Which patients have risk factors for endometrial cancer? Select all that apply. a. The patient who never had children. b. The patient who has given birth to nine children. c. The patient whose body mass index is 39. d. The patient whose menopause began at age 60. e. The patient whose father had colon cancer. f. The patient who has had human papillomavirus (HPV) infections.

ANS: A, C, D, E Correct: A family history of endometrial, breast, colon, or ovarian cancer is considered a risk factor. Incorrect: Nulliparity or infertility is a risk factor. HPV is a risk factor for cervical cancer rather than endometrial cancer.

Which findings are expected from a musculoskeletal assessment of a left-handed healthy adult? Select all that apply. a. Cervical concave, thoracic convex, and lumbar concave contours of the spine b. Muscle strength of 3/5 bilaterally c. Circumference of left upper arm larger than right upper arm d. Lumbar and thoracic spine flexion of 75 degrees e. External rotation and abduction of left arm of 90 degrees f. Flexion of right and left knees of 90 degrees

ANS: A, C, D, E Correct: Cervical concave, thoracic convex, and lumbar concave contours of the spine are expected findings of the spine. The circumference of the left upper arm larger than the right upper arm is considered an expected finding because this patient is left-handed, which may account for the increase in circumference. Lumbar and thoracic spine flexion of 75 degrees is an expected finding of the spine. Ninety-degree external rotation and abduction of the left arm is an expected finding of the spine. Incorrect: The expected muscle strength is 5/5. The expected flexion is 120 degrees.

112. When assessing muscle strength, the nurse observes that a patient has complete range of motion against gravity with full resistance. What Grade should the nurse record using a 0 to 5 point scale? A) 2 B) 3 C) 4 D) 5

D) 5 Pages: 578-579. Complete range of motion against gravity is normal muscle strength and is recorded as Grade 5 muscle strength.

Which life style behaviors do nurses ask about to identify patients with risk factors for breast cancer? Select all that apply. a. Obesity after age 50 b. Smoking more than one pack of cigarettes a day c. Never having given birth to a viable infant d. Drinking two to five alcoholic beverages a day e. Estrogen replacement therapy for more than 5 years f. High blood pressure for more than 3 years

ANS: A, C, D, E Correct: Obesity after age 50, never having given birth to a viable infant (nulliparity), drinking two to five alcoholic beverages a day, and estrogen replacement therapy for more than 5 years are all risk factors for breast cancer. Incorrect: Although smoking and high blood pressure are risk factors for many disorders, they are not risk factors for breast cancer.

What manifestations does a nurse anticipate when assessing a patient with preeclampsia? Select all that apply. a. Systolic blood pressure greater than 140 mm Hg b. Increase in systolic blood pressure of 20 mm Hg over baseline c. Diastolic blood pressure greater than 90 mm Hg d. Increase in diastolic blood pressure of 15 mm Hg over baseline e. 2 lb increase in weight in 1 week f. 9 lb increase in weight in 1 month

ANS: A, C, D, F Correct: Preeclampsia is precipitated by pregnancy-induced hypertension indicated by systolic blood pressure of 140 mm Hg or higher, diastolic blood pressure more than 90 mm Hg, and an increase of more than 15 mm Hg of diastolic blood pressure from baseline. A 9-lb weight gain in this time period contributes to hypertension. Incorrect: Increase in systolic blood pressure of 20 mm Hg over baseline is an increase of more than 30 mm Hg of systolic blood pressure from baseline. Two-pound increase in weight in 1 week is an expected weight gain during pregnancy.

Which comments by a male patient during a health history suggest erectile dysfunction? Select all that apply. a. "I have had type 1 diabetes mellitus since I was 8 years old." b. "I frequently have urinary tract infections." c. "I am taking medications to control my blood pressure." d. "I have an enlarged prostate gland." e. "I take a diuretic every morning."

ANS: A, C, E Correct: A chronic complication of diabetes can cause impotence. An adverse reaction of some types of antihypertensive and diuretic medications can cause impotence. Incorrect: Urinary tract infections are unrelated to impotence. Having an enlarged prostate causes problems with urinating, but not with erections.

Which techniques does a nurse use routinely to collect data when assessing a patient's posterior thorax? Select all that apply. a. Inspection of the thorax for symmetry of shoulders b. Percussion the costovertebral angle bilaterally c. Inspection of respiratory movement for symmetry, depth, and rhythm of respiration d. Percussion of the posterior and lateral thorax for resonance e. Palpation of vertebrae for alignment and tenderness f. Inspection of thorax for muscular development and scapular alignment

ANS: A, C, E, F Correct: These techniques are performed in a routine head-to-toe assessment of the posterior thorax. Incorrect: Percussion of the costovertebral angle bilaterally is not performed unless indicated. For example, when the patient has a kidney disorder. Percussion of the posterior and lateral thorax for resonance is not performed unless indicated.

Which data does a nurse collect during the general survey when meeting a patient for the first time? Select all that apply. a. Gait b. Muscle strength c. Heart sounds d. Hearing and speech abilities e. Mood or affect f. Position of the trachea

ANS: A, D, E Correct: These data are observed during the general survey as the patient enters the examination area and greets the nurse. Incorrect: Although the nurse could detect firmness in a patient's handshake, muscle strength testing is performed during the examination if indicated, not during the general survey. Data about heart sounds are collected during auscultation of the chest. Position of the trachea is determined by palpating the trachea during the examination.

A nurse is assessing the respiratory system of a healthy adult. Which findings does this nurse expect to find? Select all that apply. a. Thoracic expansion that is symmetric bilaterally b. Respiratory rate of 24 breaths/min c. Bronchophony revealing clear voice sounds d. Breath sounds clear with vesicular breath sounds heard over most lung fields e. Anteroposterior diameter of the chest about a 1:2 ratio of anteroposterior to lateral diameter f. Symmetric thorax with ribs sloping downward at about 45 degrees relative to the spine

ANS: A, D, E, F Correct: These are expected findings from a lung and respiratory assessment of a healthy adult. Incorrect: A respiratory rate of 24 breaths/min is considered tachypnea. Bronchophony revealing clear voice sounds is not performed unless there is an indication of consolidation of the lung, or if there was an abnormal finding of tactile fremitus. The expected finding is muffled voiced sounds rather than clear.

What normal finding does a nurse expect to find when palpating a male patient's prostate gland? a. Is approximately 4.5 cm in diameter and is highly mobile b. Feels smooth, firm, and slightly mobile c. Is deeply divided into three lobes, each approximately 2 cm in length d. Feels hard, asymmetrical, and has a palpable ridge that divides the gland into two lobes

ANS: B Feedback A A normal prostate is 1.5 inches in diameter and slightly mobile. B This is a normal finding. C The prostate consists of two lobes. D The prostate feels firm, smooth, and symmetric.

A pregnant patient presents to the clinic with a 3 lb/wk weight gain for 2 successive weeks. The nurse is most concerned that this patient is demonstrating signs of which condition? a. Gestational diabetes mellitus b. Preeclampsia c. Placenta enlargement d. Multiple gestations

ANS: B Feedback A A rapid increase in weight could indicate gestational diabetes, but a gain of this much in 2 successive weeks warrants considering preeclampsia. B If a woman gains more than 2 lb (0.9 kg) in any 1 week, preeclampsia should be suspected. C Rapid weight gain is not associated with placental enlargement. D A rapid increase in weight could indicate multiple gestation, but a gain of this much in 2 successive weeks warrants considering preeclampsia.

In reviewing the charts of several patients in the clinic, a nurse recognizes which patient as being at highest risk of breast cancer? a. A woman who had her first child at age 26 b. A woman who reached menopause at age 58 c. A woman who breastfed all four of her children d. A woman who states that she reached menarche at age 14

ANS: B Feedback A A woman who had her first child at age 26 has a low risk. B A woman who reached menopause at age 58. A long menstrual history (menopause after age 50) increases risk. C A woman who breastfed all four of her children has a low risk. D A woman who states that she reached menarche at age 14 has a low risk because the menarche was after 12 years of age.

A patient with testicular torsion is experiencing which abnormality? a. Abnormal dilation and tortuosity of the veins along the spermatic cord b. Twisting of the testicle and spermatic cord c. A cystic mass filled with sperm and seminal fluid in the epididymis d. An accumulation of fluid in the scrotum

ANS: B Feedback A Abnormal dilation and tortuosity of the veins along the spermatic cord describes a varicocele. B Twisting of the testicle and spermatic cord describes testicular torsion. C A cystic mass filled with sperm and seminal fluid in the epididymis describes a spermatocele. D An accumulation of fluid in the scrotum describes a hydrocele.

How does a nurse determine the Goodell sign? a. Assesses the softening of the lower uterine segment b. Palpates for softening of the cervix c. Assesses the breasts for fullness and tenderness d. Inspects the cervix for a bluish coloration

ANS: B Feedback A Assesses the softening of the lower uterine segment is a description of Hegar sign. B Palpates for softening of the cervix is a description of Goodell sign. C Assesses the breasts for fullness and tenderness is a presumptive sign of pregnancy rather than a probable sign. D Inspects the cervix for a bluish coloration is a description of Chadwick sign.

What technique does a nurse use when performing a breast examination on a patient who has had a mastectomy? a. Excludes palpation of the axillary area where there was lymph node dissection b. Inspects and palpates both the operative and the nonoperative sides c. Avoids palpating the scar to prevent causing the patient any discomfort d. Palpates only the muscle tissue on the affected side

ANS: B Feedback A Axilla is still palpated after node dissection. B Women who have had a mastectomy require the same breast assessment as all other women. C The scar tissue should not be tender and the technique of palpation on the operative side should be the same as the nonoperative side. D Women who have had a mastectomy require the same breast assessment as all other women.

The nurse would give immediate attention to the patient who presents with which complaint? a. Bilateral breast swelling b. Unilateral nipple discharge c. A breast lump that changes during the menstrual cycle d. Unequal breast size

ANS: B Feedback A Bilateral breast swelling is frequently related to fibrocystic breast disease. B A breast malignancy usually manifests in one breast, and a serosanguineous or clear nipple discharge may be present. C A breast lump that changes during the menstrual cycle is frequently related to fibrocystic breast disease. D Unequal breast size that does not include a mass in one breast is not an immediate concern.

In a presentation on breast cancer risk factors, a nurse would be accurate in making which statement? a. "Women who breastfeed their children are at increased risk of breast cancer." b. "Women who are more than 30% overweight are at increased risk of breast cancer." c. "African American women have the highest risk of breast cancer." d. "Women who have children before age 30 are at increased risk of breast cancer."

ANS: B Feedback A Breastfeeding is not a risk factor for breast cancer. B Obesity especially after age 50 or increased weight gain as an adult increases breast cancer risk. C White women have the highest incidence of breast cancer. D First full-term pregnancy after age 30 increases risk.

The nurse recognizes which clinical manifestation as a positive sign of pregnancy? a. Cessation of menstruation b. Visualization of the fetus by ultrasound c. Nausea and increased abdominal girth d. Positive pregnancy test (hCG)

ANS: B Feedback A Cessation of menstruation is a presumptive sign of pregnancy. B This is a positive sign of pregnancy. C Nausea and increased abdominal girth are presumptive signs of pregnancy. D Positive pregnancy test (hCG) is a probable sign of pregnancy.

Which data do nurses document under the category of past health history? a. Chronic diseases b. Immunizations received c. Allergies to medications or food d. Causes of death of the patient's parents

ANS: B Feedback A Chronic diseases are documented under the present health status category. B Immunizations are documented under the past health history category. C Allergies to medications or food are documented under the present health status category. D Causes of death of parents are documented under the family history category.

What is the best color for nurses to select when designing educational materials for older adults? a. Blue b. Yellow c. Violet d. Green

ANS: B Feedback A Color perception of blue, violet, and green may be impaired for older adults. B Color perception of blue, violet, and green may be impaired for older adults. C Color perception of blue, violet, and green may be impaired for older adults. D Color perception of blue, violet, and green may be impaired for older adults.

In prioritizing patient care, the nurse would give immediate attention to the pregnant patient with which clinical manifestation? a. Darkened eyelids b. Seeing spots c. Excessive eye dryness d. Pale conjunctiva

ANS: B Feedback A Darkened eyelids may occur normally and is not a reason for concern. B Chromatopsia may be noted, characterized by unusual color perception, seeing spots, or blindness in the lateral visual field, and requires immediate follow-up. C Excessive eye dryness occurs normally in pregnancy and can be treated with artificial tears. D Pale conjunctiva may indicate anemia, which needs to be treated, but it is not an urgent problem.

In assessing the mood of older adult patients, a nurse documents which finding as abnormal? a. Sadness and grief after returning from the funeral of a long-time friend b. Depression that interferes with the ability to perform activities of daily living c. Frustration about rearranging the day's schedule to attend a grandson's birthday party d. Crying about the unexpected death of a pet that had been with the family 12 years

ANS: B Feedback A Emotional experiences of sadness, grief, response to loss, and temporary "blue" moods are normal responses in older adults. B Persistent depression that interferes significantly with the ability to function is not an expected finding. C This is a normal response for any adult. D Emotional experiences of sadness, grief, response to loss, and temporary "blue" moods are normal responses in older adults.

The pregnant patient tells the nurse that she has had three pregnancies and two live births to date. How does the nurse record this in the patient's history? a. Gravida 3, para 3 b. Gravida 3, para 2 c. Gravida 2, para 3 d. Gravida 2, para 2

ANS: B Feedback A Gravida 3, para 3 represents three pregnancies and three that reached 20 weeks or longer. B Gravida 3, para 2 represents three pregnancies and two that reached 20 weeks or longer, which is consistent with what the patient reported. C Gravida 2, para 3 represents two pregnancies and three that reached 20 weeks or longer, which is not possible. D Gravida 2, para 2 represents two pregnancies and two that reached 20 weeks or longer.

In teaching a class of adolescents about sexually transmitted diseases, a nurse includes which information about the human papillomavirus (HPV)? a. HPV is fragile and not easily transmitted. b. Wartlike growths in the genital area are a sign of HPV infection. c. There is a specific blood test needed to screen for HPV. d. Heavy, purulent vaginal discharge is the primary sign of HPV.

ANS: B Feedback A HPV is highly contagious. B This is the correct description of the clinical findings of HPV. C There is no specific blood test to detect HPV. D HPV is a virus that develops warts.

In assessing a patient with damage to the occipital lobe, the nurse correlates which clinical manifestation to this injury? a. Intentional tremors b. Visual changes c. Decreased hearing d. Inability to formulate words

ANS: B Feedback A Intentional tremors are caused by cerebellar problems. B The occipital lobe contains the visual cortex. C The temporal lobe contains the auditory cortex. D The ability to formulate words comes from the Broca area in the frontal lobe.

When performing a neurologic assessment of a male patient, a nurse discovers that shouting and shaking are necessary to arouse the patient enough to assess his neurologic status. After the patient answers questions about who he is and squeezes the nurse's hand as requested, he returns to "sleep." How does the nurse document this patient's level of consciousness? a. Lethargic b. Obtunded c. Stuporous d. Semicomatose

ANS: B Feedback A Lethargic patients can be aroused by saying their names and touching them. B Obtunded patients require shouting and vigorous shaking to arouse them; they carry out requests while awake, but return to "sleep" when stimuli stops. C Stuporous patients require painful stimuli to respond and the response usually is a withdrawal from the source of pain. D Semicomatose patients require painful stimuli and respond with abnormal flexion or extension.

A patient had a left radical mastectomy last year. The nurse assesses for painless and nonpitting swelling of the arm on that side. Which complication of a mastectomy is the nurse assessing for? a. Infection b. Lymphedema c. Inflammation d. Lymphoma

ANS: B Feedback A Localized signs of infection include redness, heat, and pain; there can also be edema, which, when associated with infection, is pitting. B Lymphedema is a localized accumulation of lymph fluid in the interstitial spaces caused by removal of the lymph nodes. C Localized signs of inflammation include redness, heat, and pain; there can also be edema, which, when associated with inflammation, is pitting. D The sign of lymphoma is an enlarged lymph node rather than generalized swelling.

A pregnant woman who drinks alcoholic beverages while pregnant increases the risk for which disorder? a. Low infant birth weight b. Birth defects c. Abruptio placentae d. Gestational diabetes mellitus

ANS: B Feedback A Low infant birth weight is not directly caused by alcohol intake. B Alcohol is a teratogen; no safe level of alcohol ingestion has been identified for pregnant women. C Maternal hypertension is the risk factor for abruptio placentae. D Gestational diabetes mellitus is not caused by alcohol intake.

Which assessments are routine examination techniques of the upper extremities? a. Palpating the epitrochlear lymph nodes for size and tenderness b. Palpating the arms for skin characteristics, symmetry, tenderness, and deformities c. Testing the range of motion and muscle strength comparing one arm with the other d. Testing triceps, biceps, and brachioradialis deep tendon reflexes bilaterally

ANS: B Feedback A Lymph nodes are not palpated unless indicated. B Palpation of upper extremities is performed in a routine head-to-toe examination. C These data are not routinely assessed unless indicated. D These data are not routinely assessed unless indicated.

During assessment of an infant, the nurse notes that when the infant cries, the fontanelles bulge slightly. What is the most appropriate action for the nurse at this time? a. Note in the record that the child is microcephalic. b. Assess the fontanelles again when the child is not crying. c. Check the child for signs of malnutrition and dehydration. d. Use transillumination for further assessment of the skull.

ANS: B Feedback A Microcephaly occurs when the head circumference is less than expected, which is not the finding here. B The fontanelles frequently bulge when an infant is crying. The nurse will palpate the anterior and posterior fontanelles for fullness while the infant is in an upright position and calm. C An elevated fontanelle when the infant is calm may indicate overhydration or fluid volume excess. D Transillumination is not needed in this case.

What is the purpose of asking a female to lean forward during the breast examination? a. To accentuate the Montgomery glands b. To observe for symmetry of the suspensory ligaments c. To compare nipple symmetry d. To identify any breast masses in the subcutaneous tissues

ANS: B Feedback A Montgomery glands are located within the areolar surface and would be difficult to inspect when the patient leans forward. B Observing these ligaments can be accomplished with the patient leaning forward. C Nipple symmetry is better evaluated with the patient sitting up or lying supine. D Palpation is a better way to identify a breast mass in subcutaneous tissue.

In collecting a history from an older adult, which information does the nurse consider least essential for a patient of this age? a. Past health history b. Genogram c. Functional abilities d. Mental health

ANS: B Feedback A Past health history is important to document the patient's chronic illnesses. B A genogram is not routinely used to document the family history for an older adult. The health status and cause of death of the patient's parents and siblings lose value as the patient ages. C Assessing functional abilities provides data about how well the patient performs activities of daily living. D Mental health data are essential to collect about all patients regardless of age.

In assessing the eyes of a 4-month-old infant, a nurse shines a penlight in the infant's eyes and notices that the light reflection is not in the same location in each eye. What is the nurse's most appropriate response to this finding? a. Perform the cover-uncover test. b. Document it as an expected finding at this age. c. Document abnormal function of cranial nerves IV (trochlear) and VI (abducens). d. Refer the infant to an ophthalmologist.

ANS: B Feedback A Performing the cover-uncover test is not indicated unless there is asymmetric light reflex after 6 months of age. B Transient strabismus is common during the first few months of life due to a lack of binocular vision. If it continues after 6 months of age; however, a referral to an ophthalmologist is needed. C The data do not support this interpretation. D Referral is not needed unless the finding is present after the infant is 6 months old.

An older adult patient reports being able to see her granddaughter play basketball out of the sides of her eyes, but not in the center of her eyes. Based on this information, what vision disorder does the nurse suspect? a. Presbyopia b. Macular degeneration c. Pseudoptosis d. Entropion

ANS: B Feedback A Presbyopia is a decrease in near vision that usually occurs after age 40 and is treated with corrective lenses. B Gradual loss of central vision may be caused by macular degeneration due to changes in the retina. C Pseudoptosis is a relaxed upper eyelid. D Entropion is a disorder of the eyelid, in which the lower lid turns inward.

A patient complains of dysuria, yellow-green vaginal discharge, and vulvar itching. The nurse suspects which sexually transmitted disease? a. Syphilis b. Gonorrhea c. Genital warts d. Chlamydia

ANS: B Feedback A Primary syphilis produces a single, firm, painless open sore or chancre with indurated borders at the site of entry on the genitals. B Gonorrhea causes a yellow or green vaginal discharge, dysuria, pelvic or abdominal pain, and vaginal itching and burning. C Genital warts appear as soft, papillary, pink to brown, elongated lesions that may occur singularly or in clusters on the internal genitalia, the external genitalia, and the anal-rectal region. D Chlamydia infection is asymptomatic in up to 75% of women because it often does not cause enough inflammation to produce symptoms.

The nurse suspects respiratory distress in a newborn infant who exhibits which manifestation? a. Respiratory rate of 36 breaths/min b. Sternal retractions c. Nasal breathing d. Irregular breathing pattern

ANS: B Feedback A Respiratory rate of 36 per minute in the newborn and infant is within the expected range of 30 to 60 breaths/min. B Several respiratory findings indicate that an infant is in respiratory distress, including stridor, grunting, sternal or supraclavicular retractions, and nasal flaring. C Infants are obligate nose breathers until about 3 months old. D The respiratory pattern in the newborn may be irregular, having a Cheyne-Stokes type of pattern.

In performing a respiratory assessment of a 1-month-old infant, the nurse recognizes which finding as abnormal? a. Sneezing b. Coughing c. Abdominal breathing d. Predominantly nose breathing

ANS: B Feedback A Sneezing is a common finding for an infant and is therapeutic because it helps to clear the nose. B Coughing at this age is considered abnormal and indicates a problem. C Infants use abdominal breathing rather than diaphragmatic breathing. D Infants are obligate nose breathers until about 3 months old.

For which patient does the nurse make assessment of the oral mucous membrane a priority? a. The patient who has an arteriovenous (AV) fistula b. The patient who has a gastrostomy tube c. The patient who uses a Ventimask d. The patient who has a colostomy

ANS: B Feedback A The AV fistula is required by patients who need hemodialysis for kidney failure. They are able to drink fluids by mouth. B Which patient can drink fluids by mouth is the distinguishing fact. This patient has this gastrostomy tube because he or she has difficulty swallowing. Thus this patient may not have fluids by mouth, which increases the risk for dry mucous membranes and makes the assessment most important compared with the other listed patients. C The Ventimask fits over the nose and mouth to deliver oxygen. This patient is able to drink fluids by mouth. D This patient has had part or all of the colon removed, but this patient is able to drink fluids by mouth.

The nurse moves a wisp of cotton lightly across the anterior scalp, paranasal sinuses, and lower jaw to test the function of which cranial nerve? a. CN IV (trochlear nerve) b. CN V (trigeminal nerve) c. CN VI (abducens nerve) d. CN VII (facial nerve)

ANS: B Feedback A The CN IV (trochlear nerve) supplies downward and inward eye movement. B The CN V (trigeminal cranial nerve) supplies sensation to the cornea, iris, lacrimal glands, conjunctiva, eyelids, forehead, nose, nasal and mouth mucosa, teeth, tongue, ear, and facial skin. C The CN VI (abducens nerve) supplies lateral eye movement. D The CN VII (facial nerve) supplies movement of facial expression muscles except the jaw, closes the eyes, and allows labial speech sounds (b, m, w, and rounded vowels).

To assess the reflexes of a 5-month-old infant lying supine, the nurse turns the infant's head to the left side so that the chin is over the shoulder. What is the expected response for this reflex? a. Left arm and leg abduct and the right arm and leg adduct. b. Left arm and leg extend and the right arm and leg flex. c. Infant turns the chin from the left to the right side. d. Infant begins a sucking motion with the lips and tongue.

ANS: B Feedback A The arms and legs flex and extend, rather than abduct and adduct. B This is the expected response for the tonic neck reflex that appears from birth to 6 weeks old and disappears at 4 to 6 months old. C This is not an expected finding for any infant reflexes. D This is a description of the sucking reflex.

The parents of a 14-year-old boy express concern that their son's behavior ranges from clean-cut and personable to "grungy" and sullen. They have tried talking with him about this and have tried disciplining him, but he continues to show different sides, and they are confused. What is the nurse's assessment for the behavior of this teenager? a. The teenager is dangerously labile. b. This behavior is normal experimentation. c. This boy is being rebelliously hostile. d. This behavior may require hospitalization.

ANS: B Feedback A The behaviors described fall within expected behavior for adolescents. B Adolescents may behave in new and different ways, much to the chagrin of their parents, as they "try on" differing roles and values. Adolescents begin testing and evaluating previously accepted notions about life, living, spirituality, relating, and being. C The behaviors described fall within expected behavior for adolescents. D The behaviors described fall within expected behavior for adolescents.

The nurse assesses the glossopharyngeal nerve (CN IX) by testing which reflex? a. Corneal reflex b. Gag reflex c. Blink reflex d. Cough reflex

ANS: B Feedback A The corneal reflex is controlled by the trigeminal cranial nerve (CN V). B Movement of the posterior pharynx and gag reflex test is controlled by the glossopharyngeal cranial nerve (CN IX). C The blink reflex is another name for the corneal reflex. D The cough reflex is controlled from the medulla oblongata.

Which part of the nervous system is a nurse assessing when he places a vibrating tuning fork on a patient's wrist or ankle? a. Frontal lobe and motor tracts b. Parietal lobe and sensory tracts c. Hypothalamus and sensory tracts d. Cerebellum and motor tracts

ANS: B Feedback A The frontal lobe sends impulses governing movement through motor (efferent) tracts. B The parietal lobe receives sensory input, such as vibratory sense and pain, through sensory (afferent) tracts. C The hypothalamus functions include regulation of body temperature, hunger, and thirst and formation of autonomic nervous system responses. D The cerebellum provides equilibrium and coordination of movement.

A nurse who is assessing a patient's eyes finds that the pupils are equal, round, and react to light and accommodation (PERRLA). These findings verify the expected functioning of which cranial nerve? a. Optic cranial nerve (CN II) b. Oculomotor cranial nerve (CN III) c. Trochlear cranial nerve (CN IV) d. Abducens cranial nerve (CN VI)

ANS: B Feedback A The optic cranial nerve (CN II) provides vision. B The oculomotor cranial nerve (CN III) provides these eye functions. C The trochlear cranial nerve (CN IV) provides eye movement downward and inward. D The abducens cranial nerve (CN VI) provides lateral eye movement.

A patient asks when she can stop having Pap (Papanicolaou) tests. What is the nurse's most appropriate response? a. "Until you are no longer sexually active." b. "Through age 65." c. "Until you begin menopause." d. "Through the end of menopause."

ANS: B Feedback A The recommendation from the U.S. Preventive Services Task Force is through age 65. B This is the recommendation from the U.S. Preventive Services Task Force. All females should be screened after the onset of sexual activity through age 65. C The recommendation from the U.S. Preventive Services Task Force is through age 65. D The recommendation from the U.S. Preventive Services Task Force is through age 65.

The nurse holds the patient's relaxed arm with elbow flexed at a 90-degree angle, places a thumb over a tendon in the antecubital fossa, and strikes the thumb with the pointed end of the reflex hammer. Which deep tendon reflex is the nurse assessing? a. Brachioradialis b. Biceps c. Triceps d. Deltoid

ANS: B Feedback A The technique described is not the correct one for assessing the brachioradial deep tendon reflex. B This is the correct technique for assessing the biceps deep tendon reflex. C The technique described is not the correct one for assessing the triceps deep tendon reflex. D There is no reflex to test the deltoid muscle.

A nurse expects which normal findings when performing a bimanual palpation of the cervix and uterus? a. The uterus feels firm and slightly nodular. b. The cervix feels soft, smooth, and slightly rounded. c. The uterus of a nonpregnant patient cannot be felt with the internal fingers. d. The cervix is tender when moved laterally.

ANS: B Feedback A The uterus normally does not feel nodular. B These are the normal findings from a bimanual examination of the cervix. C The uterus of a nonpregnant woman can be palpated. D The cervix normally is not tender; if it is, it may indicate a sexually transmitted disease.

In assessing a patient with a tumor in the pons, the nurse expects to find which abnormalities due to pressure on cranial nerves? a. Dilated pupils and ptosis b. Facial asymmetry and impaired hearing c. Difficulty swallowing d. Impaired gag reflex

ANS: B Feedback A These abnormalities represent pressure on the oculomotor (CN III) that exits from the midbrain. B These abnormalities represent pressure on the facial and acoustic cranial nerves. The nurse correlates the cranial nerves that exit from the pons which are trigeminal (CN V), abducens (CN VI), facial (CN VII), and acoustic (CN VIII). C This abnormality represents pressure on the three cranial nerves that affect swallowing: glossopharyngeal (CN IX), vague (CN X), and hypoglossal (CN XII). These cranial nerves exit the brainstem in the medulla oblongata. D This reflex is controlled by the vagus cranial nerve (CN IX), which exits the brainstem in the medulla oblongata.

In assessing the nails of an older adult, which finding does a nurse expect to find? a. Transverse ridges b. Thick, brittle, and yellow nails c. Thin, brittle nails d. Lateral edges turned upward

ANS: B Feedback A These changes occur when the patient has had nail trauma. B Thick, brittle, and yellow nails are expected changes in the nails of older adults. C These changes occur when the patient has anemia. D This change, called spoon nail, occurs when the patient has anemia.

A 7-month-old infant weighs 11.6 lb compared with a birth weight of 7.1 lb and has a head circumference of 19 inches. What does the nurse document about this infant? a. Underweight and normal head circumference b. Underweight and larger-than-normal head circumference c. Overweight and smaller-than-normal head circumference d. Normal weight and larger-than-normal head circumference

ANS: B Feedback A This 7-month-old infant should weigh more than 14 lb and have a head circumference of 16.5 to 17.5 inches. B This 7-month-old infant should weigh more than 14 lb and have a head circumference of 16.5 to 17.5 inches. In general, infants double their birth weight by 4 to 5 months of age. Expected head circumference for term newborns averages from 13 to 14 inches (33 to 36 cm) and increases 0.5 inch (1.3 cm) monthly for the first six months. C This 7-month-old infant should weigh more than 14 lb and have a head circumference of 16.5 to 17.5 inches. D This 7-month-old infant should weigh more than 14 lb and have a head circumference of 16.5 to 17.5 inches.

During a counseling session, which statement by an adolescent indicates he is adjusting to expected developmental tasks? a. "I wish my parents would just leave me alone." b. "I'm hoping to go to college." c. "I don't have any friends." d. "It's terrible being taller than all my friends."

ANS: B Feedback A This comment does not demonstrate adjustment, although it is a problem some adolescents face. B This indicates a developmental task of adolescence. C This comment does not demonstrate adjustment. D This comment does not demonstrate adjustment, although it is a problem some adolescents face.

Which statement by a 40-year old man would be most indicative of possible breast cancer? a. "I had embarrassing breast enlargement when I was a teenager." b. "I think I felt a hard spot in my left breast, but it does not hurt." c. "My right breast has always been a little smaller than the left." d. "My father's breasts got larger after he was older."

ANS: B Feedback A This describes gynecomastia. B A breast malignancy usually manifests in one breast as a hard, painless, irregular nodule, often fixed. C One larger breast is not indicative of breast cancer and may be a normal variation. D This report is not indicative of breast cancer.

During the assessment of a newborn within hours after birth, a nurse determines which finding as abnormal? a. Capillary refill time of less than 1 second b. Apical pulse felt at the second intercostal space c. Splitting of heart sounds d. Cyanosis of the hands and feet

ANS: B Feedback A This is a normal finding; capillary refill in infants is rapid—less than 1 second. B This finding is abnormal because the apical pulse of the newborn normally is felt in the fourth or fifth intercostal space. C Splitting of heart sounds is common in infants until about 48 hours after birth because of the transition from fetal circulation to systemic and pulmonic circulation. D Acrocyanosis (cyanosis of hands and feet) without central cyanosis is of little concern and usually disappears within hours to days after birth.

As a patient is walking down the hall, the nurse notices the patient's staggering, unsteady gait. What findings does the nurse anticipate on the neurologic examination? a. When the patient stands with feet together, eyes open and then closed, an upright posture is maintained. b. When the patient touches the end of each finger to the thumb of the same hand, a tremor is observed in the fingers. c. When the patient is giving a history to the nurse, a tremor is noticed as the patient's hands rest in the lap. d. When lying supine, the patient is able to move the heel of one foot down the shin of the other leg.

ANS: B Feedback A This is a result of a negative Romberg test. This patient has a cerebellar problem, which would result in a positive Romberg test. B This patient has a cerebellar problem as evidenced by the staggering gait (noted at the beginning of the encounter) and the intention tremor on movement (noted during the examination). C This describes a tremor at rest that occurs in patients with parkinsonism rather than with cerebellar problems. D This describes a normal response on an examination of cerebellar function.

A patient has a herniated disk compressing the lumbar spine at L2, L3, and L4 that is impairing deep tendon reflexes. Which response does a nurse expect from this patient? a. Diminished contraction of the gastrocnemius muscle with plantar flexion of the foot b. Diminished contraction of the quadriceps muscle with extension of the lower leg c. Diminished plantar flexion of the toes d. Diminished dorsiflexion of the foot and flexion of the toes

ANS: B Feedback A This is an abnormal response from the Achilles tendon reflex that is innervated from S1 and S2. B This is an abnormal response from the patellar deep tendon reflex that is innervated from L2, L3, and L4. C This is an abnormal response from the plantar reflex or a positive Babinski sign. D This is an abnormal response of ankle clonus.

Which finding during inspection of the mouth of a 1-month-old infant requires further investigation? a. A small loose tooth in the lower jaw b. Tongue overlapping the floor of the mouth c. Whitish epithelial cells on the roof of the mouth d. White patches on the tongue that scrape off easily

ANS: B Feedback A This is not an abnormal finding; occasionally, a natal loose tooth may be found. B The infant's tongue should be appropriate to the size of the mouth and fit well into the floor of the mouth. C These small, white epithelial cells on the palate or gums are called Bohn nodules or Epstein pearls and are an expected finding. D Whitish patches seen along the mucosa that scrape off easily are milk deposits. White patches that scrape off but leave a red area that may bleed indicate candidiasis lesions.

During a well-baby check for several 4-month-old infants, a nurse recognizes that which infant needs further assessment of an abnormal finding? a. The infant who is unable to sit independently b. The infant whose head circumference and chest circumference are equal c. The infant whose weight has doubled since birth d. The infant whose length falls in the 90th percentile on growth charts

ANS: B Feedback A This is not an expected motor skill for a 4-month-old; it is expected at 6 months of age. B At four months of age, the head circumference should be larger than the chest circumference. C This is a normal finding; infants generally double their birth weight by age 4 to 5 months. D This is not an abnormal finding, especially if weight is normal; the height of the parents should also be considered.

What technique does the nurse use to test ankle clonus? a. Strokes the lateral aspect of the sole of the patient's foot from heel to ball with a reflex hammer b. Supports the patient's knee in flexed position and sharply dorsiflexes the foot and maintains the flexion c. Plantar flexes the ankle and strikes the appropriate tendon of the ankle with the hammer d. Everts the ankle and slowly moves the ankle into plantar flexion and quickly release the foot

ANS: B Feedback A This is the technique for testing plantar flex or the Babinski reflex. B This is the correct technique for assessing ankle clonus. C This is not a correct technique for testing any reflex. D This is not a correct technique for testing any reflex.

How does a nurse test the brachioradial deep tendon reflex? a. Uses the end of the handle on the reflex hammer to stroke the lateral aspect of the sole of the patient's foot from heel to ball b. Asks the patient to slightly pronate the relaxed forearm into the nurse's hand and strikes the appropriate tendon with the reflex hammer c. Holds the patient's relaxed arm with the elbow flexed at a 90-degree angle in one hand, and palpates and strikes the appropriate tendon just above the elbow with the flat end of the reflex hammer d. Holds the patient's relaxed arm with the elbow flexed at a 90-degree angle, places a thumb over the appropriate tendon in the antecubital fossa, and strikes the thumb with the pointed end of the reflex hammer

ANS: B Feedback A This is the technique to test plantar flexion, the Babinski reflex. B This is the technique to assess the brachioradial deep tendon reflex. C This is the technique to test the triceps deep tendon reflex. D This is the technique to test the biceps deep tendon reflex.

In assessing a neonate, the nurse notices that one testicle has not descended into the scrotal sac. What is the most appropriate response for the nurse as a result of this finding? a. Document the findings and refer this neonate for further examination for an undescended testicle. b. Place a finger over the upper inguinal ring and gently push downward to try to push the testicle into the scrotum. c. Use a light source to transilluminate the affected scrotal sack to determine if fluid is preventing the descent of the testicle. d. Insert the fifth finger into the inguinal ring to palpate for a hernia that may have prevented the testicle from descending.

ANS: B Feedback A This response is premature without determining if the testicle can be manually descended into the scrotum. B If the testicle can be pushed into the scrotum, it is considered descended even though it retracts into the inguinal canal. C Transillumination is used to detect a hydrocele, which is a common finding in infants but not related to an undescended testicle. D This response is not appropriate.

Which statement best illustrates Erikson's theory of development? a. The main goal is to establish equilibrium between self and environment. b. One progresses through stages that involve specific psychosocial tasks. c. There are four distinct, sequential levels of cognitive development. d. Cognitive development occurs from birth to around age 15.

ANS: B Feedback A This statement is not consistent with Erikson's theory of development. B This statement is consistent with Erikson's theory of development. C This is more consistent with Piaget's theory. D This is more consistent with Piaget's theory.

A patient reports having difficulty swallowing. Based on this information, how does the nurse assess the cranial nerve related to swallowing? a. Ask the patient about feeling the blunt end of a paper clip along the jaw line. b. Observe the rising of the soft palate when the patient says "Ahh." c. Observe the symmetry of the face when the patient talks. d. Assess taste on the anterior part of the tongue.

ANS: B Feedback A This tests the sensory function of the trigeminal cranial nerve (CN V). B This tests the glossopharyngeal cranial nerve (CN IX), which is involved in swallowing. The nurse must correlate difficulty swallowing with the cranial nerves involved with that function and how to test it. The cranial nerves involved are IX, X, and XII. C This tests the motor function of the facial cranial nerve (CN VII). D This tests the sensory portion of the facial cranial nerve (CN VII).

Which tool is most appropriate for testing the vision of a 5-year-old child? a. Denver II test b. Snellen E chart c. Allen picture cards d. Snellen standard chart

ANS: B Feedback A Vision can be assessed when performing developmental tests such as the Denver II, but the Snellen E provides more objective data. B Use the Snellen E chart for children 3 to 6 years of age (see Chapter 11). Have children point their fingers in the direction of the "arms" of the E. C Use the Allen picture cards to screen for visual acuity in 2.5- to 3-year-old children. D The Snellen standard chart is used for adolescents and adults; it is too difficult to use for children.

A patient who is 30 weeks pregnant tells the nurse, "I have had low blood pressure all my life, and now it is 136/74. What's wrong with me?" What is the most appropriate response by this nurse? a. "A woman's blood pressure usually drops several points during pregnancy, but yours hasn't." b. "The blood pressure increases because your blood volume increases to supply you and the baby with enough blood." c. "Yes, this is a significant change from your baseline, and I advise you to see your obstetrician at your earliest convenience." d. "If you spend more time lying down, I think your blood pressure should return to normal in a few days."

ANS: B Feedback A The blood pressure may decrease slightly in the second trimester and then return to the usual level during the third trimester. With an increase in blood volume, the patient may experience an increase in blood pressure above baseline. B Blood volume increases by 1500 mL to meet the needs of an enlarged uterus and fetal tissue, causing increased cardiac workload (increased heart rate and blood pressure). C This referral is unnecessary. D With an increase in blood volume, the patient may experience an increase in blood pressure above baseline.

Which behavior illustrates a developmental task for a "young-old" older adult? a. Adapting to living alone b. Adjusting to loss of physical strength, illness, and emotional stress c. Managing leisure time d. Accepting possible institutional living arrangements

ANS: C Feedback A This is a developmental task of those 85 years and older. B This is a developmental task of those older than 85 years of age. C This is a developmental task of the "young-old" adult (65 to 85 years of age). D This is a developmental task of those older than 85 years of age.

a patient reports a gnawing, burning pain in the mid-epigastric area that is aggravated by bending over or lying down. Which additional question odes the nurse ask for the symptom analysis?

Do you have any symptoms such as nausea with this pain?

On inspection of the external male genitalia, the nurse notes which finding as abnormal? a. The scrotum is covered with dark rugous skin. b. The skin covering the penis is hairless and loose. c. The urinary meatus is located on the upper surface of the penis. d. The left side of the scrotum hangs slightly lower than the right.

ANS: C Feedback A This is a normal finding. B This is a normal finding. C This is called epispadias. D This is a normal finding.

Which tube interferes with hearing lung sounds during auscultation? Select all that apply. a. Gastrostomy tube b. Chest tube c. Nasogastric tube d. Tracheostomy tube e. Oral endotracheal tube

ANS: B, C Correct: When attached to suction, chest and nasogastric tubes can create sounds that may mimic lung sounds. Incorrect: Gastrostomy tube is in the stomach and not attached to suction that might create a false sound similar to lung sounds. Tracheostomy tube is not attached to suction that might create a false sound similar to lung sounds. Oral endotracheal tubes are not attached to suction that might create a false sound similar to lung sounds.

1. A nurse suspects appendicitis in a patient with abdominal pain. Which findings are suggestive of appendicitis? Select all that apply. a. Pain radiating to the right shoulder b. Pain around the umbilicus c. Pain relieved by lying still d. Right lower quadrant pain e. Increased peristalsis

ANS: B, C, D Correct: These are all descriptions of pain related to appendicitis. Incorrect: Pain radiating to the right shoulder is associated with gallbladder disease. Increased peristalsis can be associated with gastroenteritis or diarrhea.

What expected physiologic changes of aging put older adults at risk for respiratory infections? Select all that apply. a. Breath sounds are bronchovesicular in the peripheral lung. b. Alveoli are less elastic. c. Weak intercostal muscles reduce effective coughing. d. Fewer cilia make mucociliary clearance less effective. e. Curvature of the spine limits chest wall expansion. f. Cough reflex is impaired due to deceased sensitivity of receptors.

ANS: B, C, D, E Correct: These are all expected findings of healthy older adults that impair their ability to breathe deeply and cough to prevent or recover from a respiratory infection. Incorrect: Breath sounds are the same as for younger adults—vesicular in the peripheral lungs. Cough reflex is not changed in the older adult.

Which movements does a nurse expect to find when assessing the hip range of motion of a healthy person? Select all that apply. a. Pronation and supination b. Flexion and extension c. Internal and external rotation d. Adduction and abduction e. Hyperextension

ANS: B, C, D, E Correct: These are all expected motions for the hip joint. Incorrect: Pronation and supination are not expected motions for the hip joint.

Which patients meet the criteria for Chlamydia screening? Select all that apply. a. A 40-year-old woman who is sexually active and uses barrier protection consistently b. A 15-year-old female woman who is sexually active with one partner c. A 22-year-old woman who is sexually active and uses barrier protection inconsistently d. A 23-year-old woman who has had four sexual partners in the last 3 months e. A 34 year-old woman who uses barrier protection inconsistently with multiple sexual partners f. A 36-year-old pregnant woman making the first prenatal visit

ANS: B, C, D, E, F Correct: These patients meet the criteria for screening for Chlamydia. Incorrect: This patient does not meet criteria based on age and use of protection.

On inspection, the nurse finds the patient's anteroposterior diameter of the chest to be the same as the lateral diameter. What other findings does this nurse expect during the examination? Select all that apply. a. Inspiratory wheezing found on auscultation b. Hyperresonance heard on percussion c. Decreased breath sounds heard on auscultation d. Deceased diaphragmatic excursion on percussion e. A sharp, abrupt pain reported when the patient breathes deeply f. Decreased to absent vibration on vocal fremitus

ANS: B, C, D, F Correct: These are all indications of enlargement or destruction of alveoli that occurs in emphysema. Air is trapped, which increases the anteroposterior to lateral diameter creating a barrel chest, and pushes the diaphragm down decreasing the excursion and causing hyperresonance. The destroyed alveoli decrease the breath sounds and create absent vibration on vocal fremitus. Incorrect: Inspiratory wheezing found on auscultation indicates narrowed airways as found in asthma. A sharp, abrupt pain reported when the patient breathes deeply is pleuritic chest pain associated with pleural lining irritation that may occur in a patient with pleurisy or pneumonia.

Which physiologic changes are associated with pregnancy? Select all that apply. a. Increase in brittleness of fingernails b. 1500 mL increase in blood volume c. Periodic shortness of breath later in pregnancy d. Edematous and bleeding gums e. Painless vaginal bleeding f. Waddling gait

ANS: B, C, D, F Correct: Blood volume increases by 1500 mL to meet the need of an enlarged uterus and fetal tissue, causing increased cardiac workload (increased heart rate). Uterine enlargement pushes up on the diaphragm, causing periodic shortness of breath. Increased estrogen increases vascularity and tissue proliferation of gums, resulting in edematous and bleeding gums. The increased size of the uterus and growing fetus results in the center of gravity moving forward, resulting in a waddling gait. Incorrect: Increased nail growth is reported by some individuals. Painless vaginal bleeding indicates a problem with the pregnancy, such as placenta previa.

Which techniques does a nurse use routinely to collect data when assessing a patient's anterior thorax? Select all that apply. a. Palpation of the thorax for fremitus b. Inspection of the skin for color, intactness, lesions, and scars c. Auscultation of breath sounds bilaterally d. Auscultation of heart sounds for rate, rhythm, frequency, and S1 and S2 e. Palpation the anterior chest wall for thoracic expansion f. Inspection of respiratory movement for symmetry and ease of respiration

ANS: B, C, D, F Correct: These techniques are performed in a routine head-to-toe assessment of the anterior thorax. Incorrect: Palpation of the thorax for fremitus and palpation of the anterior chest wall for thoracic expansion are not performed unless indicated.

Nurses inquire about lifestyle behaviors of patients with risk factors for osteoarthritis. Which risk factors for osteoarthritis does the nurse ask about? Select all that apply. a. Estrogen deficiency b. Physical inactivity c. Overuse of joints d. Smoking e. Obesity f. Age

ANS: B, C, E Correct: Lack of exercise weakens muscles that support joints. Overuse of joints damages cartilage in joints. Being overweight puts stress on joints. Incorrect: Estrogen deficiency, smoking, and age are risk factors for osteoporosis rather than osteoarthritis.

What does a nurse teach pregnant patients about the effects of smoking while pregnant? Select all that apply. a. Fluid retention increases. b. Prematurity risk increases. c. Low infant birth weight risk increases. d. Anemia develops. e. Vitamin C deficiency develops. f. Craving of starch, baking soda, or clay develops.

ANS: B, C, E Correct: These three are due to the effects of smoking while pregnant. Incorrect: Fluid retention increases may occur during pregnancy, but it is not caused by smoking. Anemia may occur during pregnancy, but it is not caused by smoking. Craving of starch, baking soda, or clay may occur during pregnancy, but it is not caused by smoking.

A patient with heart failure reports having a cough with frothy sputum and awakening during the night to urinate. Based on this information, what abnormal data might this nurse expect to find during an examination? Select all that apply. a. S4 heart sound b. Dyspnea c. Jugular vein distention d. Pericardial friction rub e. Edema of ankle and feet at the end of the day f. S3 heart sound

ANS: B, C, E, F Correct: All of these manifestations are consistent with fluid overload that occurs in heart failure because the cardiac output is decreased. Incorrect: S4 heart sounds signifies a noncompliant or "stiff'' ventricle. Hypertrophy of the ventricle precedes a noncompliant ventricle. Also, coronary artery disease is a major cause of a stiff ventricle. Pericardial friction rubs are caused by inflammation of the layers of the pericardial sac.

1. During a health fair, the nurse is alert for which risk factors for hypertension? Select all that apply. a. Excessive protein intake b. Having parents with hypertension c. Excessive alcohol intake d. Being Asian e. Experiencing persistent stress f. Elevated serum lipids

ANS: B, C, E, F Correct: These are all risk factors for hypertension. Incorrect: Excessive protein is not a risk factor for hypertension, but excessive sodium intake is a risk factor. Being Asian is not a risk factor, but being African-American is a risk factor.

Which techniques does a nurse use routinely to collect data when assessing the abdomen of a patient? Select all that apply. a. Testing for presence of abdominal reflexes b. Inspecting skin for contour, scars, lesions, vascularity, and bulges c. Percussing in all quadrants for tone d. Lightly palpating for tenderness, guarding, and masses e. Auscultating for bowel sounds, bruits, and venous hums f. Deeply palpating for tenderness, guarding, and masses

ANS: B, D, E Correct: These techniques are performed in a routine head-to-toe assessment of the abdomen. Incorrect: Testing for abdominal reflexes for presence, percussing in all quadrants for tone, and deeply palpating for tenderness, guarding, and masses are not performed unless indicated.

What risk factors for falls does a nurse teach a group of older adults? Select all that apply. a. Being a woman b. Taking more than six medications c. Having hypertension d. Having cataracts e. Muscle strength 3/5 bilaterally f. Incontinence

ANS: B, D, E, F Correct: Adverse effects of medications can contribute to falls. Cataracts impair vision, which is a risk factor for falls. Poor muscle strength is a risk factor for falls. Incontinence of urine or stool increases risk for falls. Incorrect: Men have a higher risk for falls than women. Hypertension itself does not contribute to falls. Dizziness does contribute to falls.

1. What data do nurses collect when assessing a patient's wound? Select all that apply. a. Skin turgor b. Width, length, and depth c. Presence of pulsations d. Wound color e. Presence of edema f. Drainage color

ANS: B, D, E, F Correct: These data are collected when assessing a wound. Incorrect: Skin turgor is assessed in intact skin rather than wounds. Presence of pulsations is not indicated when assessing a wound.

What findings does the nurse expect when assessing the cardiovascular system of a healthy adult? Select all that apply. a. Heart rate of 102 beats/min b. S1 and S2 present with regular rhythm c. Capillary refill greater than 3 seconds d. Blood pressure of 124/86 e. Warm, elastic turgor f. Pulse of smooth contour with 2+ amplitude

ANS: B, E, F Correct: These are all expected findings. Incorrect: A heart rate of 102 beats/min is tachycardia. Capillary refill should be 2 seconds or less. Blood pressure of 124/86 is prehypertension. Normal is less than 120 and less than 80 mm Hg.

Which data do nurses document under the category of personal and psychosocial health history? Select all that apply. a. Allergies to medications or food b. Diet and foods eaten on a regular basis c. Type of employment d. Address and date of birth e. Activities that promote health f. Use of tobacco and alcohol

ANS: B, E, F Correct: These data are documented under the heading of personal and psychosocial health. Incorrect: Allergies to medications or food is documented under the present health status category. Type of employment is documented under the heading of biographic data. Address and date of birth are documented under the heading of biographic data.

How does a nurse document a large, flat bluish capillary area on a neonate's cheek? a. Mongolian spot b. Stork bite (telangiectasis) c. Port-wine stain (nevus flammeus) d. Strawberry hemangioma

ANS: C Feedback A A Mongolian spot appears as an irregularly shaped, darkened, flat area over the sacrum and buttocks. B A stork bite (telangiectasis) is a common vascular birthmark that appears as a small red or pink spot often seen on the back of the neck. C Port-wine stains appear as large, flat, bluish purple capillary areas. They are frequently found on the face along the distribution of the fifth cranial nerve (trigeminal). D A strawberry hemangioma appears as a slightly raised, reddened area with a sharp demarcation line that may be 2 to 3 cm in diameter.

The nurse recognizes that a Papanicolaou (Pap) test is indicated for which patient? a. A 12-year-old who has not yet reached menarche. b. A 30-year-old who had a normal Pap test 12 months ago. c. A 45-year-old who had a total hysterectomy for cervical cancer. d. A 55-year-old who had a total hysterectomy to treat endometriosis.

ANS: C Feedback A A Pap test is not indicated for this patient. B A Pap test is not indicated for this patient. C In women who have undergone a hysterectomy in which the cervix was removed, Pap testing is not required unless the hysterectomy was performed because of cervical cancer or its precursors. D A Pap test is not indicated for this patient.

A nurse asks an older adult patient to rise from an arm chair without using the arms, stand with eyes closed, and turn around in a circle. What is the nurse assessing in this patient? a. Ability to follow instructions b. Muscle strength c. Balance d. Hearing

ANS: C Feedback A Although following instruction is required for this balance assessment, it is not the primary purpose of the assessment. B Muscle strength is tested by having the patient push or pull against resistance. C These are three of the activities of the Tinetti Balance and Gait Assessment Tool. D Although hearing is required for this balance assessment, it is not the primary purpose of the assessment.

A nurse uses the Glasgow Coma Scale to assess which patient? a. The patient who has a new onset of quadriplegia b. The patient who has tonic-clonic seizures c. The patient who requires stimuli for responses d. The patient who has dementia

ANS: C Feedback A Although this patient is paralyzed, he or she is conscious. The Glasgow Coma Scale would not yield useful data about this patient. B Although this patient may be unconscious during seizures, consciousness will return. The Glasgow Coma Scale would not yield useful data about this patient. C The Glasgow Coma Scale is applicable only to patients who are unconscious, meaning they do not respond unless stimulated in some way from touch to pain. D This patient is not unconscious. The Glasgow Coma Scale would not yield useful data about this patient.

A nurse refers which child for further assessment? a. A 2-year-old who has a jugular venous hum after playing b. A 4-year-old who has a resting heart rate of 100 c. A 5-year-old who positions herself in a squat after running a few feet d. A 7-year-old who has a strong femoral pulse readily detected on palpation

ANS: C Feedback A An expected finding in children is a venous hum in the jugular vein. B This is an expected resting heart rate for a child; the expected range for a toddler is from 80 to 110 and for a school-age child from 60 to 110 beats/min. C Squatting may be a compensatory position for a child with a heart defect. D This is an expected finding.

In teaching a patient about breast self-examination, why does the nurse emphasize palpation of the axillary areas? a. Because deep muscles in that area can mask changes b. Because some patients avoid this area because of tenderness c. Because most lymph draining from the breast flows through this area d. Because supporting ligaments in this area may present as tissue changes

ANS: C Feedback A Because deep muscles in that area can mask changes. This answer is incorrect. Palpating the axilla for enlarged lymph nodes is very important. B Because some patients avoid this area because of tenderness. This answer is incorrect. Palpating the axilla for enlarged lymph nodes is very important. C Because most lymph draining from the breast flows through this area. More than 75% of lymph drainage from the breast flows outward toward the axillary lymph node. D Because supporting ligaments in this area may present as tissue changes. This answer is incorrect. Palpating the axilla for enlarged lymph nodes is very important.

A patient has a compression fracture of the cervical spine at C7 to C8 that is impairing deep tendon reflexes. Which response will the nurse expect from the affected deep tendon reflex? a. Diminished to absent pronation of the arm b. Diminished to absent flexion of the elbow c. Diminished to absent extension of the elbow d. Diminished to absent adduction of the upper arm

ANS: C Feedback A Diminished to absent pronation of the arm is an abnormal response from the brachioradial deep tendon reflex that is innervated from C5 to C6. B Diminished to absent flexion of the elbow is an abnormal response from the biceps deep tendon reflex that is innervated from C5 to C6. C Diminished to absent extension of the elbow is an abnormal response from the triceps deep tendon reflex that is innervated from C6, C7, and C8. D Diminished to absent adduction of the upper arm is not a response of any deep tendon reflex.

The nurse correlates which patient complaint with suspected enlargement of the prostate gland? a. Constipation b. Change in bowel patterns c. Weak urine stream d. Increased mucus in urine

ANS: C Feedback A Enlargement of the prostate gland does not cause constipation. B This is a warning sign for colon cancer rather than a manifestation of an enlarged prostate. C Enlargement of the prostate gland compresses the urethra causing a weak urinary stream. D This may be an indication of a urinary tract infection rather than an enlarged prostate.

A woman who is 16 weeks pregnant with her first child is concerned because she has not felt the fetus move. What is the most appropriate explanation for a nurse to give this patient? a. Movement of a fetus in the first pregnancy often does not occur until the twenty-fourth week of pregnancy. b. A referral to an obstetrician should be made for further evaluation of this finding. c. Movement of the fetus is not expected until the nineteenth week of pregnancy. d. Movement of the fetus is related to fundal height; the greater the fundal height, the sooner the fetal movement.

ANS: C Feedback A Fetal movement is felt by the nineteenth week and movement observed by approximately the twenty-eighth week. B A referral is not warranted in this case. C This patient will likely not feel fetal movement for another 3 weeks. D Fetal movement is related to gestational age rather than fundal height.

In measuring fundal height, the nurse documents which finding as abnormal? a. 29 cm at week 30 b. 28 cm at week 26 c. 34 cm at week 38 d. 26 cm at week 24

ANS: C Feedback A From the twentieth to thirty-sixth week of gestation, the expected pattern of uterine growth is an increase in fundal height of about 1 cm per week. B Measurement of fundal height is an estimate and may vary among examiners by 1 to 2 cm. C This is a 4 cm difference; any discrepancy greater than 2 cm between fundal height and the estimate of gestational age (based on the last menstrual period) should be evaluated further. D Measurement of fundal height is an estimate and may vary among examiners by 1 to 2 cm.

A patient tells the nurse that he has been informed he has internal hemorrhoids and asks whether there are different types of hemorrhoids. What is the nurse's most appropriate response? a. "Internal hemorrhoids are usually seen outside the anus and appear blue." b. "Sometimes patients have other diseases, such as anal warts, that may be mistaken for internal hemorrhoids." c. "Internal hemorrhoids are found higher in the rectum and usually can't be felt unless they are infected or prolapsed." d. "Both internal and external hemorrhoids arise from the same general area and produce the same kinds of symptoms."

ANS: C Feedback A Internal hemorrhoids are found high in the rectum and may be felt if infected, but are not seen during an anal examination. B Anal warts have a distinct appearance that is different from hemorrhoids. C This is a correct statement. D Internal hemorrhoids are found higher in the rectum, while external hemorrhoids are found outside the external rectal sphincter.

When examining the lymph nodes of an adult female patient, the nurse recognizes which finding as normal? a. Visible superficial nodes b. Palpable supraclavicular nodes c. Nonpalpable lymph nodes in the axilla d. Enlarged, fixed nodes in the neck

ANS: C Feedback A Lymph nodes are not normally palpable or visible. B Lymph nodes are not normally palpable. Enlarged supraclavicular nodes often indicate a malignancy. C Lymph nodes are not normally palpable. D Lymph nodes are not normally palpable.

The nurse documents which finding as normal after performing the Barlow-Ortolani maneuver on an infant? a. The clavicles are immobile and without crepitus. b. Each shoulder remains in a "hunched up" position. c. No clicking is noted when the hips are abducted and adducted. d. Both feet are held in the varus position when stroked on the soles.

ANS: C Feedback A Normal findings include stable and smooth clavicles, without crepitus. B The Barlow-Ortolani maneuver assesses hip location. C The Barlow-Ortolani maneuver assesses hip location, and the movement should feel smooth and produce no clicking. D The feet should be flexible and not fixed. Normally, the hindfoot aligns with the lower leg and the forefoot turns inward slightly.

A nurse expects which normal findings when palpating a patient's ovaries? a. Nodular and nonmovable b. Smooth, fluid-filled, and nonmovable c. Smooth, firm, and about the size of a walnut d. Spongy, mobile, and about the size of a peanut

ANS: C Feedback A Normally the ovaries do not feel nodular and nonmovable. B Normally the ovaries do not feel fluid-filled and nonmovable. C This is a correct description of a normal finding when palpating an ovary. D Normally the ovaries do not feel spongy or mobile.

Which finding on cardiovascular assessment of an older adult patient warrants further evaluation? a. Occasional ectopic beats heard on auscultation of the heart b. Murmur heard over the mitral valve c. Systolic pressure of 156 in the right arm and 188 in the left arm d. Persistent S4 sound in a patient with a history of decreased ventricular function

ANS: C Feedback A Occasional ectopic beats are common and may or may not be significant. B Sclerosis of the mitral and aortic valves may cause murmurs. C These systolic pressures are above normal and require further evaluation. D The S4 heart sound is common in older adults and may be associated with decreased left ventricular compliance.

Which disorder, if any, does a nurse screen for when examining a healthy adolescent? a. Muscle weakness b. Limited joint range of motion c. Curvature of the spine d. No screening is needed when the adolescent is healthy

ANS: C Feedback A Screening for muscle weakness in a well adolescent is not indicated. B Screening for limited range of motion in a well adolescent is not indicated. C Adolescents are screened for scoliosis, kyphosis, and lordosis. Postural kyphosis is almost always accompanied by a compensatory lordosis, an abnormally concave lumbar curvature. D Adolescents are screened for scoliosis.

When assessing the skin of an older adult, a nurse notices pigmented, raised warty-appearing lesions on the trunk. How does a nurse document this finding? a. Solar lentigo b. Basal cell skin cancer c. Seborrheic keratosis d. Sebaceous hyperplasia

ANS: C Feedback A Solar lentigo appears as irregularly shaped, flat, deeply pigmented macules that may appear on body surface areas with repeated exposure to the sun. B Basal cell carcinoma appears as a nodular pigmented lesion with a depressed center and rolled borders found on sun-exposed areas. C Seborrheic keratosis is described as pigmented, raised warty-appearing lesions on the trunk. D Sebaceous hyperplasia appears as yellowish, flattened papules that have central depressions.

What finding does a nurse expect when assessing a one-month old's eyes and vision? a. The newborn distinguishes most colors b. Tears when the newborn cries c. The newborn following a bright toy or light d. The newborn's blink reflex is present

ANS: C Feedback A Testing for color vision occurs between ages 4 and 8 years. B There are no tears until about 2 to 3 months of age. C This is an accurate statement. D The blink reflex is present in normal newborns and infants before one month.

The nurse places an 8-year-old boy in which position for examination of his genitalia? a. Supine with legs extended to either side b. Lying on his left side with knees bent c. Reclining with knees flexed d. Standing with legs spread apart

ANS: C Feedback A The examination is easiest to perform if the child is sitting. B The examination is easiest to perform if the child is sitting. C The examination is easiest to perform if the child is sitting in either a slightly reclining position with his knees flexed or heels near the buttock or sitting with his knees spread and ankles crossed. D The examination is easiest to perform if the child is sitting.

The nurse hears in a report that a patient has receptive or fluent aphasia. What communication abilities does the nurse anticipate from this patient? a. The patient understands speech but is unable to translate ideas into meaningful speech. b. The patient is able to understand speech but has difficulty forming words creating muffled speech. c. The patient is unable to comprehend speech and thus does not respond verbally. d. The patient is emotionally liable and cries easily, which interferes with the ability to communicate.

ANS: C Feedback A The inability to translate ideas into meaningful speech or writing is termed expressive aphasia or nonfluent aphasia and is associated with lesions in the Broca area in the frontal lobe. B This speech pattern is more consistent with patients who have involvement of muscles of speech rather than neurologic deficits. C This deficit is called receptive aphasia or fluent aphasia and is associated with lesions in the Wernicke area in the temporal lobe. D This speech pattern is not relevant to this patient.

During the initial inspection of the female genitalia, the nurse recognizes which finding as normal? a. The labia minora are hair-covered and lying within the labia majora. b. The cervical os in the multiparous woman has the shape of a small circle. c. The vaginal vestibule lies between the labia minora and contains the urinary meatus. d. The openings of Skene and Bartholin glands are visible posteriorly.

ANS: C Feedback A The labia majora, rather than the labia minora, are covered with hair. B The os of parous women is the shape of a slit. C This description is of normal female anatomy. D The opening of these glands is on either side of the vaginal vestibule.

How does a nurse collect baseline measurements of a 6-month-old infant? a. Measure the chest circumference around the lower ribs. b. Ask the parent how much the infant's weight has changed since birth. c. Measure the head just above the ears and eyebrows. d. Ask the parent to hold the infant while the nurse measures the length.

ANS: C Feedback A The nurse measures chest circumference, but the tape measure is placed around the nipples rather than the ribs. B An infant platform scale covered with a paper drape is used for weighing newborns, infants, and small children. C The nurse measures head circumference using this procedure until about 2 years old. D An infant's height is measured while the infant is lying supine.

Which assessment technique does a nurse use to assess the inguinal region and femoral area of a male patient as he is standing and straining? a. Palpates the femoral artery b. Palpates the inguinal lymph nodes c. Observes for a bulge through the inguinal region d. Observes for discoloration of the inguinal ring

ANS: C Feedback A The nurse palpates the femoral artery when the patient is lying supine. B The nurse palpates the inguinal lymph nodes when the patient is lying supine. C The nurse observes for a bulge that may indicate a hernia; the normal finding is no bulge. D The nurse cannot see the inguinal ring; it must be palpated.

When assessing an infant, the nurse recognizes which finding requires immediate attention? a. Cheyne-Stokes type of respiratory pattern b. 1:1 anteroposterior to lateral chest diameter c. Stridor and nasal flaring d. Bronchovesicular lung sounds in the periphery

ANS: C Feedback A The respiratory pattern in the newborn may be irregular, having a Cheyne-Stokes type of pattern. B Unlike the adult, the infant has a round thorax with an equal anteroposterior and lateral diameter. C Stridor and nasal flaring warrants immediate medical attention. Stridor is a high-pitched, piercing sound that is primarily heard in a distressed infant during inspiration. D The predominant breath sound heard in the peripheral lung fields of infants is bronchovesicular.

During a breast examination of a healthy female, the nurse recognizes which finding as normal? a. Asymmetrical venous pattern b. Unequal nipple size c. Supernumerary nipples along the milk line d. Pink discharge from one nipple when manipulated

ANS: C Feedback A The venous patterns should be bilaterally similar. B Nipple size should be symmetric. C Supernumerary nipples are considered a normal variation, although they are uncommon. D Nipple discharge is usually considered an abnormal finding.

A patient's prepregnant weight was 131 lb, within the desirable range for her height. What is the expected weight for this pregnant patient? a. 131 lb at 1 week postpartum b. 140 lb at the end of the first trimester c. 145 lb at the end of the second trimester d. 176 lb at the beginning of the third trimester

ANS: C Feedback A There are no specific weight guidelines for after birth, but it is unlikely that she would be at her prepregnant weight 1 week after delivery. B The expected range for the first trimester is weight gain of 3 to 5 lb; the patient gained 9 lb. C This 14-lb weight gain is within expected ranges (12 to 15 lb [5.5 to 6.8 kg]) for the second trimester. D This 45-lb weight gain is 10 lb more than desired; the expected range for the third trimester is 12 to 15 lb greater than the second trimester, which would be 163 to 166 lb.

While taking a history of a patient with an enlarged prostate, the nurse expects the patient to report which symptom? a. Painful urination with each voiding b. Blood in the urine upon arising c. Waking from sleep to urinate d. Incontinence throughout the day

ANS: C Feedback A There is no pain associated with an enlarged prostate. B This is not a manifestation of a prostate disorder. C Compression of the urethra by the enlarged prostate may cause men to be awakened from sleep to urinate (nocturia). D The urinary problem associated with a prostate disorder is difficulty in starting the urinary stream because the enlarged prostate gland compresses the urethra, thus incontinence is not associated with a prostate disorder.

A nurse is assessing a preschooler who is able to draw a three-part human figure, hop on one foot, and recognize three colors. The nurse recognizes these characteristics as consistent for what age? a. 3 years old b. 4 years old c. 5 years old d. 6 years old

ANS: C Feedback A These characteristics are too advanced for 3-year-old children. B These characteristics are too advanced for 4-year-old children. C These are characteristics consistent with developmental expectations of 5-year-old children. D These characteristics are below those expected for 6-year-old children who are in first grade.

A patient at 20 weeks of gestation is concerned about a discharge from her nipples. What is the nurse's appropriate response to this patient? a. "That is expected. It is milk production that begins at the onset of pregnancy." b. "A nipple discharge is unusual. I advise you to consult your obstetrician." c. "I suggest you decrease your fluid intake for several days to see if this makes a difference." d. "After the first trimester a thin, yellow fluid called colostrum may be secreted from the nipples."

ANS: D Feedback A Colostrum cannot be expressed until after the first trimester. B This finding is not unusual. C This action is not necessary because this is an expected finding. D This is a correct statement by the nurse.

What suggestions does the nurse make to parents to support the development of their 8-year-old child? a. They buy the child a computer to foster a sense of self-worth. b. The emphasis is placed on the importance of being a success at all costs. c. The child is rewarded for cooperation and healthy competition with peers. d. Social relationships outside the home are limited to one or two close friends.

ANS: C Feedback A This action is not needed for the development of a child at this age. B This action is not needed for the development of a child at this age. C Cooperation and healthy competition are developmental tasks for school-age children. D This action would interfere with extending abilities to relate to others, a developmental task of this age group.

A 75-year old patient tells the nurse, "I just do not enjoy eating as much as I used to because the food does not have much taste or smell." Which statement by the nurse is most appropriate? a. "You should make an appointment with your health care provider." b. "Try eating small, frequent meals." c. "The senses of smell and taste decrease as we age." d. "Maybe you should use saline drops in your nose."

ANS: C Feedback A This action is not warranted in this case; these changes are expected with aging. B This is practical advice, but will not alter the change in smell and taste. C A decreased sense of smell is caused by a decrease in the number of sensory cells in the nasal lining. Taste perception may also diminish due to gradual atrophy of the tongue and a decrease in the number of papillae and taste buds. D This is practical advice, but will not alter the change in smell and taste.

A parent tells the nurse about having difficulty disciplining a 5-year-old child. What characteristic does the nurse teach this parent to improve the discipline of this child? a. Children at this age are incapable of delaying gratification. b. At age 5 years, children are not interested in attaining rights and privileges of individuality. c. Five-year-olds should demonstrate basic social skills and respond to others' expectations. d. At age 5 years, children use highly inappropriate methods of expressing frustration.

ANS: C Feedback A This behavior describes a toddler. B This behavior is not consistent with a preschooler. C This describes expected developmental tasks of preschoolers. D This behavior is more consistent with a toddler.

In reviewing the results of physical examination of a 25-year-old pregnant patient, a nurse recognizes which finding as expected? a. Small, round, oval cervix b. Pale, symmetrical cervix c. Smooth, bluish-colored cervix d. Slit-shaped, pink cervix

ANS: C Feedback A This describes a nulliparous cervix, but the patient referred to is pregnant, which would cause a bluish-colored cervix. B The patient referred to is pregnant, which would cause a bluish-colored cervix. C By the second month of pregnancy, the cervix, vagina, and vulva take on a bluish color (Chadwick sign) due to increased vascularity. D The patient referred to is pregnant, which would cause a bluish-colored cervix.

The nurse notices dimpling of the skin surrounding a palpable mass in the right breast of a female patient. What is the most appropriate action for the nurse to take next? a. Record this as an expected finding. b. Palpate the area of dimpling for pain. c. Palpate the borders of the area of dimpling for irregularity. d. Tell the patient that dimpling indicates the mass is benign.

ANS: C Feedback A This finding is not normal. B Malignant breast masses are not tender. C Malignant breast masses are irregular and poorly delineated. As the mass grows, there may be breast asymmetry, discoloration (erythema or ecchymosis), unilateral vein prominence, peau d'orange, ulceration, dimpling, puckering, or retraction of the skin. D Dimpling may indicate a malignant mass rather than a benign mass.

Which patient behavior indicates to the nurse that the patient's facial cranial nerve (CN VII) is intact? a. The patient's eyes move to the left, right, up, down, and obliquely. b. The patient moistens the lips with the tongue. c. The sides of the mouth are symmetric when the patient smiles. d. The patient's eyelids blink periodically.

ANS: C Feedback A This finding represents movement of the extraocular muscles, which are controlled by the oculomotor, trochlear, and abducens cranial nerves (CN III, IV, and VI, respectively). B This finding represents movement of the tongue, which is controlled by the hypoglossal cranial nerve (CN XII). C This finding represents facial symmetry, which is controlled by the facial cranial nerve (CN VII). D This finding represents function of the oculomotor cranial nerve (CN III).

Which documentation by a nurse is most descriptive? a. Heart sounds normal. b. Few ectopic beats heard during auscultation. c. S1 murmur is heard at second right sternal border. d. Pulse within normal limits.

ANS: C Feedback A This information is not specific without a description of what is considered "within normal limits" for hair color, texture, length, etc. B This description is not specific because the number of permanent teeth is not detailed. C This is the most descriptive data, specifying what is heard and the location. D This description would be more descriptive if the pain scale were used to indicate the level of tenderness and if the location on the abdomen were documented.

When does a nurse use transillumination of the scrotum? a. When the patient has tortuosity of the veins along the spermatic cord b. When the patient has an indirect hernia c. When there is a mass or fluid in the epididymis d. When there is twisting of the testicle and spermatic cord

ANS: C Feedback A This is a description of a varicocele, which does not transilluminate. B Hernias do not transilluminate. C This is a description of a spermatocele, which does transilluminate, as does a hydrocele. D This is a description of testicular torsion.

While obtaining a symptom analysis from a patient who has an inner ear infection, the nurse helps the patient distinguish between dizziness and vertigo. Which description by the patient indicates vertigo? a. "I felt lightheaded when I stood up." b. "I just could not keep my balance when I sat up." c. "It seemed that the room was spinning around." d. "I was afraid that I was going to lose consciousness."

ANS: C Feedback A This is a description of dizziness that is often associated with transient ischemia attacks. B This is a description of disequilibrium, a form of dizziness. C This report is consistent with vertigo because it includes a sensation of motion. D This is a description of presyncope, a form of dizziness.

Interviewing patients in middle adulthood, the nurse recognizes which behavior as an expected developmental task for this age group? a. Finding meaning in life b. Establishing autonomy as an individual c. Increased self-understanding d. Dissatisfaction with one's interpersonal relationships

ANS: C Feedback A This is a developmental task of older adults. B This is a developmental task of young adults. C This is a developmental task of middle adulthood. D This behavior would not be consistent with normal adaptation.

The nurse asks the patient to stand with feet together, arms resting at the sides, with eyes open and then with the eyes closed. Which response by the patient indicates a problem in the cerebellum? a. Maintains balance when eyes are open, but loses balance with eyes closed b. Is unable to stand upright after turning around in a circle once c. Steps sideways when standing with feet together and eyes closed d. Sways slightly and maintains upright posture with feet together

ANS: C Feedback A This is an abnormal response, but is indicative of a proprioceptive problem rather than a cerebellar problem. B Losing balance is an abnormal response, but turning in a circle is not a part of the Romberg test. C This is an abnormal response documented as a "positive Romberg" and indicates cerebellar dysfunction. D This is an expected response documented as a "negative Romberg," indicating appropriate cerebellar function for balance.

A nurse assessing a 3-month-old infant suspects hydrocephalus based on which finding? a. Soft anterior fontanelle b. Lack of head control while sitting c. Increasing head circumference d. Marked asymmetry of the head

ANS: C Feedback A This is an expected finding. Bulging fontanelle is a manifestation of hydrocephalus. B By 4 months old, most infants demonstrate head control by holding the head erect and midline when in an upright position. C The abnormal accumulation of cerebrospinal fluid increases the head circumference because the sutures of the skull are not ossified and the anterior fontanelle is not closed. D Marked asymmetry of the head may indicate craniosynostosis, a premature ossification of one or more of the cranial sutures.

Which statement by a mother makes the nurse assess the infant girl for cardiovascular problems? a. "She has gained 2 lb since our last visit." b. "She naps twice a day for almost 2 hours each time." c. "She gets so tired and out of breath when she takes her bottle." d. "She gets fussy after I feed her and seems to have lots of gas."

ANS: C Feedback A This is an expected weight gain for an infant. B This is an expected sleep pattern for an infant. C Heart problems are indicated when caregivers report the child stops eating to catch her breath. D This finding may be related to the food eaten, rather than a cardiovascular problem.

In assessing a patient with lymphedema after a mastectomy, the nurse expects which finding? a. Fragile, thin, pale skin covering the area of lymphedema b. Several brownish-red discolorations in the center of the affected arm c. Unilateral nonpitting edema of the affected arm d. Pitting edema of affected arm

ANS: C Feedback A This is an incorrect description of lymphedema. B This is more consistent with a description of chronic venous insufficiency than lymphedema. C Lymphedema is a localized accumulation of lymph fluid in the interstitial spaces caused by removal of the lymph nodes. D The fluid accumulation in lymphedema usually is not enough to cause pitting.

How does a nurse assess the head circumference of an infant? a. Places a ruler behind the infant's head, noting the head width. b. Uses a plastic headband placed around the infant's head from crown to chin. c. Places a measuring tape around the head above the eyebrows and occipital prominence. d. Uses a measuring tape to find the distance between the ears and eyes and between the eyes and occiput.

ANS: C Feedback A This is an incorrect technique. B This is an incorrect technique. C This is the correct technique for measuring head circumference. D This is an incorrect technique.

During a history, a patient reports rectal bleeding, a warning sign of colorectal cancer. The nurse correlates which clinical finding with colorectal cancer? a. Thick, blood-tinged mucus within the rectum b. A pus-filled cavity in the anorectal area c. An irregular mass with raised edges on the rectal wall d. A small, smooth nodule protruding from the rectum

ANS: C Feedback A This is not an indication of colorectal cancer. B This is an indication of an infection in the anal area. C This finding is indicative of colorectal cancer. D This may be a polyp protruding from the anus.

A mother asks a nurse when her daughter should get immunized again for human papilloma virus (HPV). What is the nurse's most appropriate response to this question? a. "Your daughter does not need this immunization until she becomes sexually active." b. "The recommended age for this immunization is between ages 25 and 30 years of age." c. "Between the ages of 11 and 26 years is the recommended time for this immunization." d. "When she begins having menstrual periods is the best time for this immunization."

ANS: C Feedback A This is not the recommendation of the CDC. B This is not the recommendation of the CDC. C This is the recommendation from the Centers for Disease Control and Prevention (CDC). D This is not the recommendation of the CDC.

In assessing the breast of a male patient, the nurse places him in which position? a. Standing with hands over the head b. Supine with the hand on the side being examined placed behind the head c. Sitting with arms at the side d. Bending forward 45 degrees at the waist

ANS: C Feedback A This is part of the examination for female patients. B This is part of the examination for female patients. C This is the appropriate position for the male breast examination. D This is part of the examination for female patients.

A nurse is assessing the pain of an 86-year-old man who had hip surgery recently. The patient has been slightly confused since his surgery, but he responds to simple questions. What is the best way to assess this patient's pain? a. Ask him to rate his pain on a scale of 0 to 10. b. Ask him to rate his pain using a list of descriptive adjectives. c. Ask him to rate his pain using a vertical numeric scale. d. Observe his behavior and measure his vital signs.

ANS: C Feedback A This scale is appropriate for adolescents and adults, but older adults are assessed more accurately with a scale that they can see. B This method is not effective for assessing pain because adjectives have different meanings to different people. It is best to use a scale or pictures of faces. C Pain assessment in older adults is significantly improved by using a vertical numeric scale or pain faces. D Patient behavior and vital signs are not accurate ways to assess a patient's perception of pain.

To perform Leopold maneuvers, the nurse uses which assessment technique? a. Percussing over the symphysis pubis b. Auscultating all four abdominal quadrants c. Palpating the fundus d. Measuring from symphysis pubis to the umbilicus

ANS: C Feedback A This technique may be used to determine a full bladder. B This technique determines presence of peristalsis. C This is the procedure for Leopold maneuvers to determine presentation. D This technique is used to determine the size of the fetus.

A pregnant woman's weight before becoming pregnant was 137 lb. What weight is expected for this woman during her second trimester? a. 149 lb b. 151 lb c. 155 lb d. 164 lb

ANS: C Feedback A This weight is obtained when you forget to include the 3 to 5 lb for the first trimester. B The prepregnant weight was 137 lb. Add 3 to 5 lb for the first trimester, making the ranges from 140 to 142 lb. The second trimester is 12 to 15 lb in addition to the first trimester weight. This 140 to 142 lb + 12 to 15 lb = a range from 152 to 157 lb. C The prepregnant weight was 137 lb. Add 3 to 5 lb for the first trimester, making the ranges from 140 to 142 lb. The second trimester is 12 to 15 lb in addition to the first trimester weight. This 140 to 142 lb + 12 to 15 lb = a range from 152 to 157 lb. D This is the weight for the third trimester when an additional 12 to 15 lb can be added.

When performing a physical assessment on a 7-month-old infant, the nurse notes that the child is able to smile responsively and unable to roll from the prone to the supine position. What is the most appropriate action for this nurse? a. Reassure the parents that the infant is "performing like an 8-month-old." b. Document the infant's growth and development as "within normal limits." c. Continue to assess the infant for other signs of developmental delay. d. Give the caretaker specific directions for specialized exercises.

ANS: C Feedback A This would be an incorrect statement because this infant has delayed development. B This would be an incorrect statement because this infant has delayed development. C An infant should be able to roll from prone to supine at 4 months. D This infant needs to be referred for additional evaluation.

While examining the genitalia of a 6-year-old girl, a nurse notices which finding as expected? a. Clear mucoid vaginal discharge b. Prepuce and clitoris are prominent c. Flat labia majora with thin labia minora d. Sparse pubic hair over the inner thighs

ANS: C Feedback A Vaginal discharge is an abnormal finding at this age. B Normally at this age the clitoris is relatively small. C Until approximately age 7, the labia majora are flat, the labia minora are thin. D Evidence of pubic hair may be seen by the time the child reaches pubescence, usually between ages 8 and 11.

Which assessment finding of older adult patients indicates expected respiratory function? a. Increased elasticity of the alveoli b. Flaccidity of the chest wall c. Reduced inspiratory and expiratory effort d. Decreased anteroposterior diameter

ANS: C Feedback A With aging, alveoli become less elastic and more fibrous. B With aging, the chest wall may become stiffer, possibly because of calcification at rib articulation points, resulting in decreased chest wall compliance. C Diminished strength of the respiratory muscles results in reduced maximal inspiratory and expiratory force. D The anteroposterior diameter increases with aging due to kyphoscoliosis.

Development of which complication is considered a never event? a. Fever b. Atelectasis c. Pressure ulcer d. Thrombophlebitis

ANS: C Feedback A Fever is a common occurrence in ill patients that may indicate inflammation or infection. B Atelectasis is collapse of alveoli that may occur due to the patient's hypoventilation, such as after surgery. C Pressure ulcer is termed a never event because it refers to preventable, medical errors that should never occur. D Thrombophlebitis is inflammation of veins that may occur due to immobility.

Which manifestations does a nurse correlate with a patient with suspected meningitis? Select all that apply. a. Ptosis b. Loss of balance when standing with feet together and the eyes closed c. Confusion, agitation, and irritability d. Severe headache e. Stiff neck f. Lethargy

ANS: C, D, E, F Correct: Confusion, agitation, and irritability; severe headache—this is a symptom of meningeal irritation due to inflammation of the meninges; stiff neck; lethargy. Patients may have changes in level of consciousness. Incorrect: Ptosis is drooping of eyelids controlled by the oculomotor cranial nerve. Loss of balance when standing with feet together and the eyes closed. This describes a positive Romberg test indicating a cerebellar problem.

What technique does a nurse use to inspect the ear canal of a 1-year-old child? a. Uses a light source without a speculum to minimize any trauma to the ear canal b. Places the child in an upright position with the head flexed slightly downward c. Applies gentle traction to the lower portion of the ear and pulls upward and laterally d. Uses an assistant to hold the child's arms down and keep the child's head turned to one side

ANS: D Feedback A A nurse uses the largest speculum that fits comfortably into the child's ear canal. B Because the nurse must have both hands free to hold the ear and maneuver the otoscope, another individual must act as a "holder." The child's head is turned to one side. C The correct technique is to grasp the lower portion of the pinna and apply gentle traction down and slightly backward (as opposed to pulling the pinna up and back for the adult). This maneuver straightens the canal of the ear. D Instruct the holder to secure the infant's arms at the sides with one hand, and turn and hold the infant's head to one side with the other hand.

Which behavior indicates successful achievement of the major developmental tasks of a young adulthood? a. Accepting physical changes b. Achieving emotional independence c. Mastering money management d. Getting an appropriate education

ANS: D Feedback A Accepting physical changes is a developmental task of adolescence. B Achieving emotional independence is a developmental task of adolescence. C Mastering money management is a developmental task of school-age children. D Getting an appropriate education is a developmental task of young adults.

In assessing a patient for modifiable risk factors, the nurse correlates which finding with a high-risk pregnancy? a. 15-year-old mother b. Low socioeconomic status c. Previous birth of infant with isoimmunization d. Weight less than 100 lb

ANS: D Feedback A Age cannot be modified. B Low socioeconomic status cannot be modified easily. C Previous birth of infant with isoimmunization cannot be modified. D This risk factor can be modified by diet.

The nurse documents which clinical finding as an expected change associated with advancing pregnancy? a. Slight persistent ankle edema b. Hypoplasia of the thyroid gland c. Increased diaphragmatic excursion d. Heart murmur after 20 weeks of gestation

ANS: D Feedback A Ankle edema may occur, but normally is not persistent. B This is not an expected finding in pregnancy. C The diaphragm is pushed up by the growing fetus, which decreases the diaphragmatic excursion, rather than increasing it. D Murmurs, splitting of S1 and S2, and the presence of S3 may be heard after the twentieth week of gestation.

After assessment of each child, the nurse determines which child needs to be referred for further evaluation? a. A 4-year-old child with a predominantly nasal breathing pattern b. A 6-year-old child with a 1:2 anteroposterior-to-transverse-chest ratio c. A 7-year-old child with a predominantly thoracic breathing pattern d. A 9-year-old child with bronchovesicular breath sounds in peripheral lungs

ANS: D Feedback A At this age, children have a nasal and abdominal breathing pattern and do not develop a thoracic breathing pattern until 6 or 7 years of age. B By age 5 or 6, the rounded thorax of the child approximates the 1:2 ratio of anteroposterior to lateral diameter of the adult. C By age 6 or 7, the child's breathing pattern should change from primarily nasal and abdominal to thoracic in girls and abdominal in boys. D Although bronchovesicular breath sounds in peripheral lung areas are expected in a young child, vesicular sounds are expected at this age.

The nurse observes that the urinary meatus is located on the under surface of the penis. How does the nurse document this finding? a. Balanitis b. Phimosis c. Epispadias d. Hypospadias

ANS: D Feedback A Balanitis is inflammation of the glans that occurs in patients with phimosis. B Phimosis is a very tight foreskin that cannot be retracted over the glans. C Epispadias occur when the urinary meatus is on the upper (dorsum) surface of the penis. D Hypospadias occur when the urinary meatus is on the undersurface of the penis.

While giving a presentation about breast health, a nurse informs patients about which recommendation? a. Women in their 30s should have annual clinical breast examinations. b. Women at high risk of breast cancer should have semiannual mammograms. c. Women who are postmenopausal require clinical breast examination every 5 years. d. A screening mammogram is recommended for all women beginning at age 50 years.

ANS: D Feedback A Clinical breast examinations are recommended as part of a periodic health examination at least every 3 years for average-risk, asymptomatic women in their 20s and 30s, and annually for asymptomatic women age 40 and older. B Having semiannual mammograms is more often than necessary. C Clinical breast examinations are recommended as part of a periodic health examination at least every 3 years for average-risk, asymptomatic women in their 20s and 30s, and annually for asymptomatic women age 40 and older. D This is the recommendation of the U.S. Preventive Services Task Force.

How does a nurse recognize when a patient has a testicular torsion? a. The nurse sees a light red glow on transillumination of the scrotum. b. The nurse palpates testicular edema that is painless. c. The patient reports a pulling sensation and dull ache of the scrotum. d. The patient complains of sudden onset of severe pain with edema of the scrotum.

ANS: D Feedback A This is a clinical finding of hydrocele. B This is a clinical finding of spermatocele. C This is a clinical finding of varicocele. D These are clinical findings of testicular torsion.

A nurse recognizes that which patient has not yet successfully completed Erikson's final developmental stage? a. A 78-year-old widower who has gone to the mental health clinic for counseling after the recent death of his wife b. A 60-year-old man who tells the nurse that he is feeling fine and really does not need any help from anyone c. An 81-year-old woman who states that she enjoys having her grandchildren visit but is usually glad when it is time for them to go home d. A 75-year-old woman who tells the nurse that she wishes her friends were alive and she wishes she could change the choices she made over the years

ANS: D Feedback A Dealing with the death of a spouse is a developmental task of this age group, and this patient is seeking assistance to help him cope. B This man is meeting the developmental tasks for his age. C Enjoying grandchildren is a developmental task of this age group. D Erikson's final stage is ego integrity versus despair, and this woman is voicing despair and regret about previous decisions made in her life.

A patient comes to the clinic because she found a mass in her left breast that is present during and after her menstrual periods. On palpation the nurse finds a mass in the left breast that is round, rubbery, mobile, and nontender. This finding is consistent with which breast disorder? a. Fibrocystic breast disease b. Invasive breast cancer c. Mastitis d. Fibroadenoma

ANS: D Feedback A Findings of this disorder affect both breasts and produce tenderness that subsides between menstrual periods. B Masses of breast cancer are irregular, hard, and fixed. C Mastitis is an inflammation of the entire breast that produces tissue that is red, edematous, tender, and warm to the touch. D These findings are consistent with fibroadenoma.

A nurse holds the patient's relaxed left arm, with elbow flexed at a 90-degree angle, in one hand. The nurse palpates and then strikes the appropriate tendon just above the elbow with either end of the reflex hammer. What is the expected response for this deep tendon reflex? a. Flexion of the left elbow b. Pronation of the left forearm c. Supination of the left arm d. Extension of the left elbow

ANS: D Feedback A Flexion of the left elbow would be a normal response for the biceps deep tendon reflex. B Pronation of the left forearm would be a normal response for the brachioradialis deep tendon reflex. C Supination of the left arm is not a response of any deep tendon reflex. D Extension of the left elbow is the normal response of the triceps deep tendon reflex.

What instructions does the nurse give a female patient when she is learning to perform breast self-examination? a. Press the pads of the fingers firmly to compress breast tissue against the rib cage. b. Lie in front of a mirror and observe for dimpling of the skin. c. Lift the fingers from the chest wall during palpation to better define the breast tissue. d. Apply gentle pressure while moving the fingers in a pattern across the breast.

ANS: D Feedback A Gentle rather than firm pressure is used. B A patient stands in front of a mirror to inspect symmetry; she does not lie in front of a mirror. C The fingers should not be lifted from the breast to prevent breaking the continuity of palpation. D This is appropriate technique.

A patient comes to the clinic complaining of a new onset of nipple discharge. After inspection of the breast and discharge, what action of the nurse has the highest priority? a. Palpating both breasts comparing amount of discharge b. Asking the patient about breast pain c. Asking the patient to raise her arms and comparing the movement of the breasts d. Obtaining a specimen of the discharge for cytology

ANS: D Feedback A Getting a specimen is more important than palpating breasts at this time. B Asking the patient about breast pain is not a priority action at this time. C Asking the patient to raise her arms and comparing the movement of the breasts is not a priority action at this time. D If a patient has nipple discharge, a specimen should be collected for cytologic examination to detect malignant cells.

A nurse examines a patient and finds a single, firm, painless open sore with indurated borders on the vulva. The nurse correlates this finding with which disorder? a. Human papillomavirus (HPV) infection b. Herpes infection c. Gonorrhea d. Syphilis

ANS: D Feedback A HPV infection causes wartlike growths. B Herpes infection forms vesicles rather than chancres. C Gonorrhea produces a yellow or green vaginal discharge. D The clinical finding is consistent with a chancre found in syphilis.

What data do nurses document under the category general survey? a. Mental health b. Functional ability c. Diet and nutrition d. Orientation

ANS: D Feedback A Mental health is described under the heading of personal and psychosocial history. B Functional ability data are collected during the history. C Diet and nutrition data are collected during the history. D The general survey is the beginning of the examination when the nurse is collecting data about the patient.

When assessing the pain level of an older adult, a nurse considers which factor? a. Neural transmission of pain is increased as a part of the aging process. b. Older adult patients are not reliable in their descriptions of pain and how it affects them. c. Physiologic indicators of pain that are unique to older adults are tachycardia and hypotension. d. The older adult may believe that pain is a factor of aging and not worth mentioning.

ANS: D Feedback A Neural transmission is the same for older and younger adults. B Becoming older does not diminish one's ability to describe pain. C The physiologic indicators are the same for older and younger adults. D Some older adults may perceive pain as an expected aspect of aging that they must endure.

A nurse instructs the patient about which expected skin changes during pregnancy? a. Nipples becoming thicker b. Hands and feet becoming pale and cool c. Blotchy, brown pigmentation of the abdomen d. Stretch marks on the expanding abdomen

ANS: D Feedback A Nipples and areolae become more prominent and deeply pigmented. B Blood volume increases and edema often develops in the feet. C The blotchy, brown pigmentation occurs on the face and is called chloasma, or the mask of pregnancy. D The increasing size of the abdomen contributes to striae gravidarum (stretch marks) over the abdomen.

A nurse holds the patient's relaxed arm with the elbow flexed at a 90-degree angle, places a thumb over the appropriate tendon in the antecubital fossa, and strikes the thumb with the pointed end of the reflex hammer. What is the expected response for this deep tendon reflex? a. Flexion of the left elbow b. Pronation of the left forearm c. Supination of the left arm d. Extension of the left elbow

ANS: D Feedback A Pronation of the left forearm is a normal response for the brachioradialis deep tendon reflex. B Supination of the left arm is not a response of any deep tendon reflex. C Extension of the left elbow is the normal response of the triceps deep tendon reflex. D Flexion of the left elbow is a normal response for the biceps deep tendon reflex.

What is the earliest and most sensitive indication of altered cerebral function? a. Unequal pupils b. Loss of deep tendon reflexes c. Paralysis on one side of the body d. Change in level of consciousness

ANS: D Feedback A Pupillary function represents function of the oculomotor cranial nerve and the midbrain. B Deep tendon reflexes represent function of the spinal cord and reflex arcs. C Movement represents function of the spinal cord and posterior frontal lobe. D Maintaining consciousness represents the functions of and communication between the frontal lobe and reticular activating system.

A nurse correlates a patient's altered stereognosis with a neurologic dysfunction in which part of the nervous system? a. Midbrain or pons b. Temporal lobe or ascending nerve tracts c. Frontal lobe or motor nerve tracts d. Parietal lobe or sensory nerve tracts

ANS: D Feedback A Sensory and motor tracts travel though the midbrain and pons, but they are not tested with stereognosis. B Ascending tracts carry sensory data, but the temporal lobe provides functions of hearing rather than perception of touch. C Motor tracts carry impulses for movement and they exit from the frontal lobe, which also helps to maintain consciousness. D A parietal lobe or sensory nerve tract dysfunction prevents a patient from identifying a familiar object by touch, which is a definition of stereognosis.

Using Nägele's rule, what is the estimated delivery date of a patient whose last menstrual period started on December 1? a. August 1 b. August 10 c. September 4 d. September 8

ANS: D Feedback A Subtract 3 months rather than 4, and add 7 days. B Subtract 3 months rather than 4, and add 7 days rather than 9. C After subtracting 3 months, add 7 days instead of 3. D September 8 is determined by using the first day of the last menstrual period, subtracting 3 months, and adding 7 days.

The patient is unable to tolerate a bimanual pelvic examination due to pain in ovaries and fallopian tubes. Which disorder does the nurse suspect? a. Tertiary syphilis b. Genital herpes c. Human papillomavirus (HPV) infection d. Pelvic inflammatory disease

ANS: D Feedback A Tertiary syphilis does not cause much pain. B Genital herpes does not cause much pain. C HPV infection does not cause much pain. D Typically, the pain is so severe that the patient with pelvic inflammatory disease is unable to tolerate bimanual pelvic examination.

The nurse examining the breasts of an older adult woman recognizes which finding as normal? a. Firm and rounded breasts of equal size and shape b. Relatively large size and number of mammary ducts c. Loose elasticity and puckering of the suspensory ligaments d. Flattened breasts with a slightly granular texture on palpation

ANS: D Feedback A The breasts in postmenopausal women may appear flattened. B This is not a finding in older women. C The suspensory ligaments in older woman are relaxed, but not puckering. D The breasts in postmenopausal women may appear flattened and elongated or pendulous secondary to a relaxation of the suspensory ligaments.

The nurse notes which finding as expected during a history and examination of an older adult patient's abdomen? a. Hyperactive bowel sounds in all quadrants b. Decreased fatty deposits over the abdomen c. Marked concavity of the abdominal contour d. Soft abdomen on palpation in all quadrants

ANS: D Feedback A The opposite is true; decreased peristalsis causes hypoactive bowel sounds. B The opposite is true; older adults may have increased fat deposits over the abdominal area. C The opposite is true due to the increased fat deposits over the abdominal area. D The abdomen of older adults may feel soft due to decreased abdominal muscle tone.

A patient asks when she should make an appointment for her first Pap (Papanicolaou) test to screen for cervical cancer. What is the nurse's most appropriate response? a. "There is no need for Pap tests until after you have become pregnant." b. "All women should have the first Pap test after reaching menarche." c. "All women should have the first Pap test after they are 19 years of age." d. "All women should have the first Pap test when they become sexually active or at age 21."

ANS: D Feedback A The recommendation is when females become sexually active or 21 years old. B The recommendation is when females become sexually active or 21 years old. C All females should be screened when they become sexually active or 21 years old, whichever happens first. D This is the recommendation from the U.S. Preventive Services Task Force. All females should be screened when they become sexually active or age 21, whichever happens first.

A nurse asks a 15-year-old boy to think of an explanation for a simple puzzle. When he is unable to come up with any answer at all, the nurse recognizes that he may not yet have successfully mastered which of Piaget's levels of cognitive development? a. Sensorimotor b. Preoperational c. Concrete operations d. Formal operations

ANS: D Feedback A The sensorimotor stage develops in children between the ages of birth to 2 years, when thoughts are dominated by physical manipulation of objects and events. B The preoperational stage develops in children between the ages of 2 to 7 years, when children function symbolically using language as a major tool. C The concrete operations stage develops in children between the ages of 7 to 11 years, when mental reasoning processes assume logical approaches to solving concrete problems. D The formal operations level develops in children between the ages of 11 and 15 years, during which true logical thoughts and manipulation of abstract concepts emerge.

On assessment of an infant's abdomen, the nurse notes which finding as normal? a. Easily palpable spleen b. Flat to slightly concave abdominal contour c. Lower liver border 2 inches below the costal margin d. Small protrusion between the rectus muscles when crying

ANS: D Feedback A The spleen is generally not palpable, although the tip may be felt in the left upper quadrant (far left costal margin). B Inspecting the abdomen of a healthy infant finds a symmetric, soft, and round abdomen with a slight protrusion. C The edge of the infant's liver should be 1 to 2 cm below the right rib cage (costal margin). D Diastasis swelling and a gap between the rectus muscles may be noted as an expect finding during crying.

When examining lymph nodes in a 7-year-old child, the nurse records which finding as abnormal? a. "Shotty" nodes in the cervical areas b. Palpable submandibular nodes c. Nodes that are tender 1 week after a tetanus vaccination d. Tender, fixed nodes greater than 1 cm

ANS: D Feedback A The term "shotty" may be used to describe small, firm, and mobile nodes, usually occurring as a normal variation in children. B Cervical and submandibular nodal enlargements are frequent in older children. C Enlarged, tender nodes may occur after immunizations or upper respiratory infection. D Abnormal findings are tender, fixed nodes greater than 1 cm.

Which cranial nerve is assessed when a nurse asks a patient to stick out the tongue and move it side to side? a. Vagus nerve (CN X) b. Facial nerve (CN VII) c. Abducens nerve (CN VI) d. Hypoglossal nerve (CN XII)

ANS: D Feedback A The vagus cranial nerve provides movement for voluntary muscles of phonation (guttural speech sounds) and swallowing. B The facial cranial nerve provides movement for facial expression muscles except the jaw, closes the eyes, and allows labial speech sounds (b, m, w, and rounded vowels). C The abducens cranial nerve provides for lateral eye movement. D The hypoglossal cranial nerve provides tongue movement for speech sound articulation (l, t, n) and swallowing.

What is the patient's expected response when the nurse is assessing graphesthesia? a. Lies supine and runs one heel along the opposite shin b. Identifies a familiar object placed in the hands c. Describes where a sensation of a vibrating tuning fork is felt d. Identifies a letter or number drawn in the hand

ANS: D Feedback A This activity tests cerebellar function of the lower extremities. B This is a test of stereognosis that tests the function of the parietal lobe and sensory tracts. C This is a test of vibratory sense that tests sensory tracts. D This is a test of graphesthesia that assesses the parietal lobe and sensory tracts.

After obtaining a history from the parents and inspecting the skin, the nurse determines which child needs further evaluation? a. The child who has a 1-cm red spot on the back of the neck, a fever of 100° F, and clear nasal drainage. b. The child who has a 2-cm slightly-raised, reddened area with a sharp demarcation line on the back of the neck. c. The child has a 2-cm abrasion on the right knee, a 3-cm abrasion on the left knee, and scrapes on both palms. d. The child who has several flat, bluish discolorations of the skin on the abdomen and back from 2 to 6 cm.

ANS: D Feedback A This child has a stork bite birthmark on the back of the neck and an upper respiratory allergy or viral infection. B This is a strawberry hemangioma, a birthmark that disappears by 5 years of age. C This child probably fell down while running, skinned both knees, and tried to break the fall with the hands. D Bruising in unusual areas (such as upper arms, back, buttocks, and abdomen) or multiple bruises found at different stages of healing should be further investigated to determine if there is abuse.

The mother of a 7-year-old boy takes him for a checkup at the local clinic. The nurse notes that the child has gained 4.9 lb and has grown 2.5 inches in 1 year. Based on these findings, what is the most appropriate action of this nurse? a. Recommend that the child be placed on a low-fat, high-protein diet. b. Counsel the mother to increase the amount of calcium in the child's diet. c. Ask the mother to return with the child next week for a more comprehensive growth and development study. d. Inform the mother that the child's developmental rate is within the expected ranges for his age.

ANS: D Feedback A This child's weight is within expected ranges. B This child's height is within expected ranges. C This child's weight and height are within expected ranges. D For school-age children, growth continues at a slow pace, with an average 5 lb weight gain and 2 inch height increase per year.

A 75-year-old man reports he stopped playing cards with his friends because their voices sounded mumbled. How does the nurse explain the cause of this change? a. Sudden low-frequency hearing loss b. Accumulation of earwax in the outer ear c. Damage to the middle ear from ear infections d. Gradual high-frequency hearing loss

ANS: D Feedback A This does not describe presbycusis and is not an expected change with aging. B This does not describe presbycusis and is not an expected change with aging. C This does not describe presbycusis and is not an expected change with aging. D This is a description of presbycusis, a sensorineural hearing loss, and an expected change with aging.

What procedure does a nurse use to assess the inguinal ring of a male patient for a hernia? a. Asks the patient to lie supine, lifts the scrotum, asks the patient to take a deep breath, and observes for a bulge b. Asks the patient to lean over the examination table, inserts a gloved finger into the lower part of the scrotum into the inguinal canal, asks the patient to cough, and palpates for a bulge c. Asks the patient to lie on the side not being assessed, inserts a gloved finger into the lower part of the scrotum into the inguinal canal, asks the patient to exhale completely, and palpates for a bulge d. Asks the patient to stand, inserts a gloved finger into the lower part of the scrotum into the inguinal canal, asks the patient to cough, and palpates for a bulge

ANS: D Feedback A This has the patient in the wrong position with the wrong technique and wrong instruction given. B This has the patient in the wrong position. C This has the patient in the wrong position with the wrong instruction given. D This describes the correct procedure.

A nurse assesses fetal heart rates when patients are examined in the maternity clinic. The nurse determines which fetus needs further assessment? a. The fetus with a heart rate of 150 beats/min b. The fetus with a heart rate of 140 beats/min c. The fetus with a heart rate of 130 beats/min d. The fetus with a heart rate of 110 beats/min

ANS: D Feedback A This heart rate is within the expected range of 120 to 160 beats/min. B This heart rate is within the expected range of 120 to 160 beats/min. C This heart rate is within the expected range of 120 to 160 beats/min. D This fetal heart rate is too slow and needs further assessment. The expected range is from 120 to 160 beats/min.

During middle adulthood, which immunization may be recommended? a. PPV (pneumococcal pneumonia vaccine) b. Hepatitis B virus vaccine, third dose c. Human papillomavirus (HPV) d. Td (tetanus and diphtheria toxoids)

ANS: D Feedback A This immunization is given to adults older than 65. B This immunization is given between birth and age 10. C This immunization is given between ages 11 and 24. D This is the only immunization recommended for this age group. Tetanus and diphtheria toxoid is recommended every 10 years.

The nurse documents which data under the category of present health status? a. Heart sounds normal. b. Few ectopic beats heard during auscultation. c. Apical pulse palpated at 5th left intercostal space, midclavicular line d. Pulse within normal limits.

ANS: D Feedback A This information is not specific without a description of what is considered "within normal limits" for heart sounds B This description is not specific C This is the most descriptive data, specifying what is heard and the location. D This description would be more descriptive such as which pulse and the rate

During the examination of the internal genitalia with the speculum, the nurse records which finding as normal? a. A healed laceration of the cervix in a nulliparous patient b. A large amount of thick white drainage from the cervical os c. Deviation of the cervix toward the posterior vaginal wall d. Pink cervix with a small ring of reddened tissue near the os

ANS: D Feedback A This is an abnormal finding; a laceration is not expected in nulliparous women. B This is an abnormal finding; this drainage may indicate a sexually transmitted disease. C This is an abnormal finding. D This is a normal finding.

A nurse refers which pregnant patient for additional assessment? a. A woman at 36 weeks of gestation who has 30% effacement of the cervix b. A woman at 19 weeks of gestation who has noticed fetal movement every day this week c. A woman at 20 weeks of gestation who has gained 4 lb in the last 2 weeks d. A woman at 28 weeks of gestation who has a systolic blood pressure of 40 mg Hg over baseline

ANS: D Feedback A This is an expected finding at this point in the third trimester. B This is an expected finding. C This is an expected weight gain. D This finding may indicate pregnancy-induced hypertension, characterized by systolic blood pressure of at least 30 mm Hg above baseline.

When palpating the epididymis, the nurse considers which finding to be abnormal? a. The epididymis is located on the posterolateral surface of each testis. b. The epididymis feels like a tubular, comma-shaped structure. c. The epididymis collapses on palpation. d. The epididymis has an irregular, nodular surface.

ANS: D Feedback A This is an expected finding. B This is an expected finding. C This is an expected finding. D The surface should be smooth and nontender.

Which technique does a nurse use to palpate the patient's axillary lymph nodes? a. With the patient sitting, the nurse places fingers of both hands deep into the axilla, one hand on either side, and firmly pushes the axillary tissue toward the center to feel for enlarged nodes. b. With the patient lying supine with arms at the sides, the nurse uses the tips of the fingers of one hand to palpate the axilla moving from the posterior to the anterior aspect of the axilla to feel for enlarged nodes. c. With the patient lying supine with the hand behind the head of the side being assessed, the nurse uses the pads of fingers of one hand to systematically palpate the axilla using small circular motions to feel for enlarged nodes. d. With the patient sitting, the nurse places fingers of one hand deep into the axilla and firmly slides the fingers along the patient's middle, anterior, and posterior of the axilla to feel for enlarged nodes.

ANS: D Feedback A This is incorrect technique. B This is incorrect technique. C This is incorrect technique. D This is the correct technique.

A nurse tests a reflex on a 9-month-old infant's right foot by stroking the surface of the infant's foot, moving from the sole laterally up and across to the great toe. What is the expected response? a. Flexion of the right toes b. Extension of the right ankle c. Dorsiflexion of the right foot d. Fanning of the toes of the right foot

ANS: D Feedback A This is not an expected response from any reflex. B This is not an expected response from any reflex. C This is part of the clonus reflex test. D This is the expected response for the Babinski reflex.

The nurse places a male patient in which position for rectal examination? a. Lithotomy position b. Prone with the knees fully extended c. Bending over the table, with feet everted d. Left lateral position with knees and hips flexed

ANS: D Feedback A This is the appropriate position for a rectal examination for a female patient. B This position is not used; a knee chest position may be used instead. C This is the appropriate position of a rectal examination for a male patient, except that the feet are inverted, rather than everted. D This is the appropriate position for a rectal examination for a male patient.

A nurse dorsiflexes a patient's right ankle 90 degrees and then uses a reflex hammer to strike the appropriate tendon. What is the expected response for this deep tendon reflex? a. Extension of the right lower leg b. Plantar flexion of the right toes c. Dorsiflexion of the right foot d. Plantar flexion of the right foot

ANS: D Feedback A This is the expected response for the patellar deep tendon reflex. B This is the expected response for the plantar reflex (Babinski). C This is an incorrect response because the nurse is holding the patient's foot in dorsiflexion, therefore dorsiflexion would not be an expected response. D Plantar flexion is the expected response of the Achilles deep tendon reflex.

What technique does the nurse use to test the patellar deep tendon reflex? a. Using the end of the handle on the reflex hammer, the nurse strokes the lateral aspect of the sole of the patient's foot from heel to ball. b. Ask the patient to flex one knee to 90 degrees, while the nurse dorsiflexes the ankle and strikes the appropriate tendon on the foot with the flat end of the reflex hammer. c. Ask the patient to flex one knee to 45 degrees, while the nurse plantar flexes the ankle and strikes the appropriate tendon of the ankle with the pointed end of the reflex hammer. d. Ask the patient to flex one knee to 90 degrees, while the nurse strikes the appropriate tendon in the knee with the blunt end of the reflex hammer.

ANS: D Feedback A This is the technique for testing plantar flex or the Babinski reflex. B This is the technique for testing the Achilles deep tendon reflex. C This is not a correct technique for testing any reflex. D This is the technique for testing the patella deep tendon reflex.

How does a nurse assess perfusion to the foot when a patient has a cast from the left middle calf to the toes? a. Palpate the popliteal pulse of the left leg. b. Palpate the posterior tibial pulse of the left leg. c. Assess movement and sensation of the left toes. d. Assess the capillary refill of the left toes.

ANS: D Feedback A This pulse is above the foot and does not indicate perfusion of the foot. B The pulse is not palpable because it is covered by the cast. C This assessment is important for this patient but assesses neurologic function rather than perfusion. D The presence of capillary refill in less than 2 seconds indicates perfusion of the left foot when the dorsalis pedis pulse cannot be palpated.

A mother who sees her newborn just after vaginal delivery is distraught because the child's head is elongated. Which response is most appropriate by the nurse? a. "This is due to a small area of bleeding that will go away in 1 to 2 months." b. "This may indicate a congenital deformity; the pediatrician will evaluate this." c. "This will require surgery to prevent hydrocephalus from developing." d. "This is not unusual after a vaginal delivery and will go away in about a week."

ANS: D Feedback A This response refers to cephalohematoma, which is not described here. B The newborn has molding that will resolve in a week. C The newborn has molding that will resolve in a week. D This is molding, which occurs when cranial bones override each other. Molding is secondary to the head passing through the birth canal and generally lasts less than a week.

During a musculoskeletal assessment of a school-age child, a nurse documents which finding as expected? a. A positive Trendelenburg sign on one side b. Lumbar lordosis, especially in African American children c. Varus rotation when the knees are greater than 1 inch apart d. Valgus rotation of less than 1 inch with the knees touching

ANS: D Feedback A Trendelenburg sign (or gait) tests for hip dysplasia and the function of the gluteus medius muscle. B Lordosis is seen more frequently in African American children but should not be seen in children older than 6 years of age. C Varus rotation (medial malleolus touching with knees greater than 1 inch [2.5 cm] apart) requires further evaluation for tibial torsion; it may be normal through 18 to 24 months of age. D Valgus rotation (medial malleolus greater than 1 inch [2.5 cm] apart with knees touching) is normal in 2- to 3.5-year-old children and may be present up to 12 years old.

What finding does a nurse look for when assessing the skin of an older adult with solar lentigo? a. Yellowish, thin papules with a central depression b. Pigmented, raised, wartlike lesions on the face or trunk c. Small, soft, pigmented tags of skin on the face and neck d. Irregular, flat, deeply pigmented macules on sun-exposed areas

ANS: D Feedback A Yellowish, thin papules with a central depression is a description of sebaceous hyperplasia. B Pigmented, raised, wart-like lesions on the face or trunk is a description of seborrheic keratosis. C Small, soft, pigmented tags of skin on the face and neck is a description of acrochordon (skin tags). D Irregular, flat, deeply pigmented macules on sun-exposed areas is a description of solar lentigo.

The parents of a toddler express concern that the child is not progressing the same way as their other children did at that age. What is the most appropriate suggestion the nurse can give the parents about monitoring the progress of the toddler? a. Advising the parents to take the toddler to the clinic every 2 months for reevaluation b. Teaching the parents how to use the Denver II test to assess for gross motor movement, language, fine motor movement, and personal-social skills c. Suggesting that the child needs more time to reach the milestones and that additional monitoring is not necessary d. Informing the parents about the ages and stages questionnaire (ASQ), which identifies developmental delays in children from 4 to 60 months

ANS: D Feedback A This suggestion is time-consuming, expensive, and not practical. B The Denver II test requires special equipment and training, and is designed for health care professionals. C The suggestion for the parents to take no action is not appropriate in this case. D This is the best suggestion for these parents at this time. The ASQ is designed for parents to report data on their children.

The patient with a respiratory rate that is within normal limits is the _____ whose respiratory rate is _____ breaths/min. a. 16-month-old; 36 b. 6-year-old; 20 c. 14-year-old;26 d. 40-year-old; 10

b. 6-year-old; 20 A A toddler's respiratory rate ranges from 24 to 32. B A school-age child's respiratory rate ranges from 18 to 26. C An adolescent's respiratory rate ranges from 12 to 16. D An adult's respiratory rate ranges from 12 to 20.

What movement from the patient does a nurse request to assess for hyperextension of the hip? a. Raise one leg at a time while lying prone. b. Raise one leg at a time while lying supine. c. Move one leg at a time laterally, away from midline, while lying prone. d. Move one leg at a time medially, toward midline, while lying supine.

Ans A A This procedure tests hyperextension of the hip. B This procedure tests hip flexion. C This procedure tests hip abduction. D This procedure tests hip adduction.

The nurse is planning to teach a group of patients stress reduction exercises to reduce the risk of depression. Which population group is at highest risk for depression?

Adolescents

How should the nurse respond? Continue the interview and assessment, avoiding further questions related to cigarette smoking. The nurse should not assume that the client does not want to respond to any further questions about his cigarette smoking, but that should be clarified before the nurse asks any further questions related to this topic. Restrict visitors so that the client can rest, and return later to complete the interview and assessment. The nurse should not restrict the client's visitors without the client's consent. Another intervention is correct. Advise the client to rest in the bed while the nurse performs a physical assessment of the client. Since it is important to obtain as much assessment data as possible to ensure the client's physiologic stability, the nurse should continue with the client's physical assessment but allow the client to rest by curtailing the interview. Document that the physical assessment could not be performed because of the client's level of fatigue. This action prevents the nurse from obtaining data needed to safely care for the client.

Advise the client to rest in the bed while the nurse performs a physical assessment of the client. Since it is important to obtain as much assessment data as possible to ensure the client's physiologic stability, the nurse should continue with the client's physical assessment but allow the client to rest by curtailing the interview.

During the client interview, the RN observes Mrs. Green's speech patterns. Ms. Green seems to have difficulty choosing and forming some of her words. What action should the RN take? Fill in the conversation with the words the client is attempting to say. This action does not promote the most effective client assessment. Allow the client to respond and ignore her difficulty to avoid embarrassment. This action does not promote the most effective client assessment. Affirm the client's difficulty and question her about when this first started. This action demonstrates caring and also enables the RN to obtain a more complete history related to the onset of the client's symptoms. Offer to complete the interview at a later time after the client has rested. This action prevents the RN from obtaining needed assessment data.

Affirm the client's difficulty and question her about when this first started. This action demonstrates caring and also enables the RN to obtain a more complete history related to the onset of the client's symptoms.

A patient asks, "Why is touching my toes necessary? This is a sports physical examination, not exercise class." What is the most appropriate response by the nurse? a. "This is the best way to check for symmetry of your arms." b. "I am looking at the stretch of your ham strings." c. "This allows me to see how straight your spinal column is." d. "I am assessing the flexion of your spine."

Ans C A This is an incorrect technique for assessing arm symmetry. B The hamstrings are not normally assessed. C This is the correct technique for inspecting the spine and for detecting scoliosis. D This is not a correct statement.

A female client is recently diagnosed with Sarcoidosis. The client tells the registered nurse (RN) that she does not understand why she has this. When teaching about the occurrence of sarcoidosis, the RN should include that sarcoidosis most commonly occurs with which ethnic group of women? African American women. Caucasian women. Asian women. Hispanic women.

African American women Sarcoidosis, an autoimmune inflammatory disease affecting multiple organs and has shown familial tendency due to multiple genes that together increase the susceptibility of developing the disease. In research studies it occurs more commonly in African American women (10-80 out of 100,000); compare to Caucasian women of the United States (8 out of 100,000).

The registered nurse (RN) is caring for a client with acute pancreatitis and assesses the admission laboratory results. What laboratory value should the RN anticipate being elevated with this diagnosis? Triglycerides. Amylase. Creatinine. Uric acid.

Amylase. An elevated amylase level is associated with acute pancreatitis.

The nurse observes multiple red circular lesions with central clearing that are scattered all over the abdomen and thorax. How does the nurse document the shape and pattern of these lesions? 1Gyrate and linear. 2Annular and generalized. 3Iris and discrete. 4Oval and clustered.

Annular and generalized

Which description of pain from the patient makes a nurse suspect the patient's pain is originating from a muscle? a. "Crampy" b. "Dull and deep" c. "Boring and intense" d. "Sharp upon movement"

Ans A A Muscle pain is often described as "crampy." B Bone pain typically is described as "deep" and "dull." C Bone pain typically is described as "boring" and "intense." D Muscle pain usually remains crampy on movement.

In teaching the group of patients about osteoporosis, the nurse identifies which one of these participants as having the highest risk for this disease? a. A small-boned, thin white American woman b. An American Indian man who smokes c. A Hispanic woman who has completed menopause d. An African American man with a family history of osteoporosis

Ans A A A small-boned, thin white American woman has three risk factors for osteoporosis: female gender, white race, and small body size. B This patient has one risk factor: smoking. C This patient has two risk factors: female gender and menopause. D This patient has one risk factor: family history.

A nurse palpates the patient's jaw movement by placing two fingers in front of each ear and asking the patient to slowly open and close the mouth. What movement does the nurse ask the patient to do next? a. Move the jaw side to side. b. Swallow. c. Smile. d. Clench the teeth together.

Ans A A Moving the jaw side to side assesses the range of motion of the jaw; asking the patient to protrude and retract the jaw also assesses range of motion. B Swallowing assesses cranial nerve IX (glossopharyngeal) and X (vagus). C Smiling assesses cranial nerve VII (facial). D Clenching the teeth together assesses cranial nerve VII (facial).

How does a nurse assess the eversion and inversion of a patient's ankle? a. For eversion, ask the patient to turn the sole of the foot away from the body and for inversion turn the sole of the foot toward the midline. b. For eversion, ask the patient to turn the sole of the inward toward the midline and for inversion turn the sole of the foot away from the body. c. For eversion, ask the patient to walk on his toes and, for inversion, to walk on his heels. d. For eversion, ask the patient to point the toes forward and, for inversion, to point the toes backward.

Ans A A This is the correct maneuver for eversion and inversion of the ankle. B This movement is the opposite of the correct movement. C This is the test for muscle strength of the ankles and feet. D This is the maneuver for dorsiflexion and plantar flexion of the ankle.

With the patient lying supine, a nurse raises the patient's leg to flex the hip. The patient complains of pain when the leg is raised to 40 degrees. The nurse correlates this finding with which disorder? a. Lumbar nerve compression b. Cervical disk herniation c. Osteoarthritis d. Bursitis

Ans A A To evaluate for nerve root irritation or lumbar disk herniation, perform straight leg raises. Pain in the back of the leg with 30 to 60 degrees of elevation indicates pressure on a lumbar peripheral nerve by an intervertebral disk. B Straight leg raises evaluate for herniated disks, but not in the cervical disks in the neck. C Osteoarthritis is a degenerative disease of articular cartilage that affects weight-bearing joints such as vertebrae, hips, knees, and ankles. Straight leg raises is not a technique to assess for osteoarthritis. D Bursa become inflamed by constant friction around joints and may be precipitated by arthritis or injury. The hip is a common site, but not vertebrae. Manifestations include painful range of motion, point tenderness, and erythema of the affected joint.

To assess the triceps and biceps muscle strength, the nurse applies resistance to the patient's arm. What should be done to ensure the appropriate muscle is being assessed? a. The patient pushes up against the nurse's hand to abduct the triceps muscle and pushes down against the nurse's hand to adduct the biceps muscle. b. The patient pushes forward against the nurse's hand to extend the triceps muscle and pulls backward against the nurse's hand to flex the biceps muscle. c. The patient pulls backward against the nurse's hand to flex the triceps muscle and pushes forward against the nurse's hand to extend the biceps muscle. d. The patient pushes up against the nurse's hand to abduct the biceps muscle and pushes down against the nurse's hand to adduct the triceps muscle.

Ans B A Abducting and adducting test range of motion rather than muscle strength. B This is the correct technique for assessing these muscles. C This is a reversal of the correct technique. D Abducting and adducting test range of motion rather than muscle strength.

The nurse in the figure below is assessing function and strength of which muscle? a. Sternocleidomastoid b. Trapezius c. Deltoid d. Pectoralis major

Ans B A The sternocleidomastoid is tested by the patient flexing the chin to the chest while the nurse tries to manually force the head upright. B The trapezius muscle is tested by the patient shrugging the shoulders while the nurse attempts to push them down. C The deltoid muscle is tested by the patient holding the arms up while the nurse tries to push them down. D The pectoralis major muscle is palpated for tenderness, but its strength is not usually assessed.

In assessing a patient with a history of poliomyelitis, the nurse suspects the right leg muscles are smaller than the left leg. What is the best approach for the nurse to confirm or reject this suspicion? a. Palpating both legs using the pads of the thumb and index fingers and comparing one side with another b. Using a tape to measure each leg's circumference at the same location, above or below the nearest joint c. Using a goniometer to measure the upper and lower legs with the patient in supine and standing positions d. Palpating the legs using the tips of the thumb and index fingers, and comparing the findings with the Lovett scale

Ans B A This describes an appropriate procedure to determine tenderness of muscles, but not muscle size. B This technique is correct, provides a baseline for future comparisons, and provides measurements for side-to-side comparisons. C The goniometer is used to measure the degree of joint flexion and extension rather than muscle size. D The Lovett scale is used to grade and record muscle strength, rather than muscle size.

With the patient in a supine position, how does a nurse test the external rotation of the patient's right hip? a. Asking the patient to move the right leg laterally with the right knee straight b. Asking the patient to flex the right knee and turn medially toward the left side (inward) c. Asking the patient to place the right heel on the left patella d. Asking the patient to raise the right leg straight up and perpendicular to the body

Ans C A Moving the right leg laterally with the right knee straight assesses abduction of the right hip. B Flexing the right knee and turning medially toward the left side (inward) internally rotates the right hip. C Placing the right heel on the left patella externally rotates the right hip. D Raising the right leg straight up and perpendicular to the body flexes the right hip.

A nurse asks a patient to describe his new onset of leg pain. He slept well through the night, but this morning he suddenly developed pain in his left lower leg that is red and too painful to touch. Nothing relieves the pain. Based on these data, the nurse suspects which disorder is causing this pain? a. Rheumatoid arthritis b. Osteoarthritis c. Gout d. Tendonitis

Ans C A Patients with rheumatoid arthritis often have morning stiffness lasting 1 to 2 hours. B Patients with osteoarthritis experience pain when bearing weight that is relieved by rest. C Sudden onset of pain and erythema in the great toe, ankle, and lower leg suggests gout (also called gouty arthritis). D Tendonitis may awaken the patient, especially when the patient is lying on the affected limb.

When a nurse asks a patient to place the right arm behind the back, so that the back of the hand is touching the lower spine, the nurse is testing for which range of motion? a. Pronation of the elbow b. Hyperextension of the elbow c. Internal rotation and adduction of the shoulder d. External rotation and abduction of the shoulder

Ans C A Pronation of the elbow is tested by pronating the palm on a flat surface. B Hyperextension of the elbow. This is not a motion of the elbow. C Internal rotation and adduction of the shoulder is tested by this maneuver. D External rotation and abduction of the shoulder is tested by asking the patient to place the hand behind the head.

On inspection of a patient's hands, the nurse notices ulnar deviation and swan-neck deformities bilaterally and correlates this finding with which disorder? a. Osteoarthritis b. Osteoporosis c. Rheumatoid arthritis d. Gout

Ans C A The findings are consistent with rheumatoid arthritis. B The findings are consistent with rheumatoid arthritis. C Ulnar deviation, swan-neck, and boutonnière deformities of interphalangeal joints are manifestations of rheumatoid arthritis. D The findings are consistent with rheumatoid arthritis.

When assessing the neck of a healthy adult, a nurse expects which findings? a. A convex contour of the posterior cervical spine b. Bending of the head to the right and left (ear to shoulder) 15 degrees c. Turning the chin to the right shoulder and then the left shoulder d. Hyperextension of the head 30 degrees from midline

Ans C A The posterior cervical spine should be concave. B The patient should be able to laterally bend the head 40 degrees from midline in each direction. C This is an expected finding. D The patient should be able to hyperextend the head 55 degrees from midline.

In assessing the joint range of motion of a patient's knees, the nurse notices the flexion is less than expected in both knees. What is the next appropriate action for the nurse? a. Documenting this finding as expected for this patient because it occurs in both knees b. Palpating the suprapatellar pouch on each side of the quadriceps for contour, tenderness, and edema c. Using a goniometer to measure the flexion in both knees and comparing the results with expected flexion d. Applying opposing force to the lower leg while the patient tries to maintain flexion and extension

Ans C A This is not an expected finding. Even when the finding is bilateral, when it is abnormal, additional examination is needed. B This is the procedure for palpating the knees, in which data are collected, but it may not have a direct bearing on the lack of flexion of the knees. C When a joint seems to have increased or decreased range of motion, use a goniometer to measure the angle. D This is the procedure for testing leg muscles for strength, in which data are collected, but it may not have a direct bearing on the lack of flexion of the knees.

While giving a history, the patient reports having carpal tunnel syndrome. Based on this information, what technique does the nurse include in a focused assessment? a. Ask the patient to press the pads of the right and left fingers against each other and hold for 1 minute. b. Ask the patient to push the hand against the nurse's forearm while attempting to flex the wrist. c. Ask the patient to flex both wrists and press the dorsal aspects of the hands together for 1 minute. d. Hold pressure to the radial and ulnar pulses and watch for blanching.

Ans C A This is not the correct technique for Phalen sign. B This is not the correct technique for Phalen sign. C This is the correct technique for Phalen sign. D This is the technique for Allen's test, which is used to detect arterial circulation of the hand, rather than Phalen sign, which is used to test for carpel tunnel syndrome.

When a nurse asks a patient to place the right arm behind the head, the nurse is testing for which range of motion? a. Flexion of the elbow b. Hyperextension of the shoulder c. Internal rotation and adduction of the shoulder d. External rotation and abduction of the shoulder

Ans D A Flexion of the elbow requires the patient to flex the elbow, but the elbow flexion is usually tested by asking the patient to bend the elbow so that the fingers are touching the shoulder. B Hyperextension of the shoulder is tested by moving the arm straight backward. C Internal rotation and adduction of the shoulder is tested by asking the patient to place the hand behind the back. D External rotation and abduction of the shoulder is tested by this maneuver.

How does a nurse document the finding from the patient shown below? a. Kyphosis b. Lordosis c. Osteoporosis d. Scoliosis

Ans D A Kyphosis is a posterior curvature (convexity) of the thoracic spine. B Lordosis is an anterior curvature (concavity) of the spine. C Osteoporosis occurs when the bones become porous and fracture easily. D Deviation of the spine or asymmetry of shoulder or iliac height is an abnormal finding. This figure shows a patient who has scoliosis, a lateral curvature of the spine.

The nurse asks the patient to hold the arms straight out, perpendicular to the floor, and the nurse tries to push the patient's arms down. This procedure tests the strength of which muscles? a. Triceps b. Biceps c. Trapezius d. Deltoid

Ans D A The nurse would test the triceps muscle strength by having the patient extend the arm and resist while the nurse tries to push the arm to a flexed position. B The nurse would test the muscle strength of the biceps by having the patient flex the forearm while the nurse applies resistance. C The nurse would test the trapezius by having the patient shrug the shoulders against resistance. D The patient uses the deltoid muscles to resist the action of the nurse.

A patient reports joint pain interfering with sleep and morning joint stiffness for the first hour after getting out of bed. Considering this report, what abnormal findings does the nurse anticipate during the examination? a. Abrupt onset of local tenderness, edema, and decreased range of motion of the shoulder and hip bilaterally b. Decreased range of motion of one hip and knee with pain on flexion and crepitus during movement of these joints c. Erythema in one great toe, ankle, and lower leg that is painful to the touch d. Hot, painful, deformed, and edematous wrists and peripheral interphalangeal joints bilaterally

Ans D A This examination finding is more consistent with bursitis. B This examination finding is more consistent with osteoarthritis. C This examination finding is more consistent with gout. D The history and these examination findings are consistent with rheumatoid arthritis. Joints are involved bilaterally in rheumatoid arthritis because it is a systemic autoimmune disorder.

A patient reports a history of compression of the left cranial nerve XI (spinal accessory nerve) from an old sports injury. Based on this information, what technique does the nurse include in the focused assessment? a. Asking the patient to rotate the head against resistance of the nurse's hand on the patient's chin b. Asking the patient to flex the chin to the chest against resistance of the nurse's hand on the patient's forehead c. Asking the patient to extend the head back against resistance of the nurse's hand on the back of the patient head d. Asking the patient to shrug the shoulders while the nurse attempts to push them down

Ans D A This technique tests the sternocleidomastoid muscle. B This technique tests the sternocleidomastoid muscle. C This technique tests the sternocleidomastoid muscle. D This is the technique to test strength of the trapezius muscle that is innervated by the cranial nerve XI.

The nurse notes that there is an audible clicking sound when the patient opens and closes the mouth. What is the appropriate response of the nurse at this time? a. Recording this as an abnormal finding, requiring additional assessment b. Measuring the distance between each side of the mandible and the eyes c. Applying resistance to the maxilla and asking the patient to repeat the motion d. Documenting this finding as expected if no other signs or symptoms are found

Ans D A This would be an incorrect action because clicking of the mandible with no other associated signs or symptoms is a normal finding. B This is not a correct assessment technique to use. C This is not a correct assessment technique to use. D The mandible should move smoothly and painlessly. An audible or palpable snapping or clicking in the absence of other symptoms is not unusual.

What sound does a nurse expect to hear when using the bell of the stethoscope over the epigastric area of the abdomen of a healthy patient? a. Bowel sounds b. Venous hum c. Soft, low-pitched murmur d. No sounds

Ans. D A Bowel sounds are heard with the diaphragm of the stethoscope. B Venous hum is not a normal finding. C Soft, low-pitched murmur is not a normal finding. D The bell is used to listen for vascular sounds and normally no vascular sounds are heard in the abdomen.

The nurse asks the patient to rest the left arm on a table and to move the lower arm so that the palm of the hand is up and then down. What motion is the nurse testing? a. Adduction and abduction of the wrist b. Supination and pronation of the wrist c. Adduction and abduction of the elbow d. Supination and pronation of the elbow

Ans D A The movements of the wrist are flexion, extension, and ulnar and radial deviation. B The movements of the wrist are flexion, extension, and ulnar and radial deviation. C This is not a movement of the elbow. The elbow moves in flexion, extension, pronation, and supination. D Supination and pronation of the elbow is tested by this maneuver.

A patient reports a change in the usual pattern of urination. What question does the nurse ask to determine if incontinence is the reason for these symptoms? a. "Do you have the feeling that you cannot wait to urinate?" b. "Are you urinating a large amount each time you go to the bathroom?" c. "Has the color of your urine changed lately?" d. "Have you noticed any swelling in your ankles at the end of the day?"

Ans. A A "Do you have the feeling that you cannot wait to urinate?" is a question that asks about urgency, a symptom of incontinence. B Are you urinating a large amount each time you go to the bathroom?" is not a question related to incontinence. Usually patients with incontinence void frequently in small amounts. C Has the color of your urine changed lately?" is a question that is asked when the nurse suspects the patient has gallbladder or liver disease. D Have you noticed any swelling in your ankles at the end of the day?" is a question that relates to patients who have renal or heart disease.

A patient reports that he has coronary artery disease with ventricular hypertrophy. Based on these data, what finding should the nurse expect during assessment? a. S4 heart sound b. Clubbing of fingers c. Splitting of the S1 heart sound d. Pericardial friction rub

Ans. A A An S4 heart sound signifies a noncompliant or "stiff'' ventricle. Coronary artery disease is a major cause of a stiff ventricle. B Clubbing of fingers occurs due to chronic hypoxia rather than a stiff ventricle. C Splitting of the S1 heart sound indicates a valve problem rather than ventricular hypertrophy. When the mitral and tricuspid valves do not close at the same time, S1 sounds as if it were split into two sounds instead of one. D Pericardial friction rubs are caused by inflammation of the layers of the pericardial sac.

What instructions does the nurse give a patient before palpating the abdomen? a. Bend the knees. b. Take a deep breath and hold it. c. Take a deep breath and cough. d. Place the hands over the head.

Ans. A A Bend the knees to relax the abdominal muscles. B This action is not needed to assess the abdomen. C This action is used to detect bulges in the abdomen, but not used before palpation. D This action is not needed to assess the abdomen.

A nurse notices a patient's chest wall moving in during inspiration and out during expiration. What additional assessment must the nurse perform immediately? a. Palpate for tracheal deviation. b. Auscultate for bronchovesicular breath sounds in the lung periphery. c. Palpate posterior thoracic muscles for tenderness. d. Auscultate for absence of breath sounds in the lung periphery.

Ans. A A Chest wall moving in during inspiration and out during expiration is paradoxical chest wall movement. It can be caused by a tension pneumothorax, which increases intrathoracic pressure in the thorax, causing tracheal deviation and indicating mediastinal shift. B Tension pneumothorax does not create bronchovesicular breath sounds in the lung periphery. C This is performed when the patient has air in the subcutaneous tissue or pleural friction rub. D Absent breath sounds may be found in pneumothorax, but if the patient has a tension pneumothorax, tracheal deviation is a more important sign.

A patient tells the nurse, "I've been having pain in my belly for several days that gets worse after eating." Which datum from the symptom analysis is consistent with the nurse's suspicion of peptic ulcer disease? a. Gnawing epigastric pain radiates to the back or shoulder that worsens after eating. b. Sharp midepigastric pain radiates to the jaw. c. Intermittent cramping pain in the left lower quadrant is relieved by defecation. d. Colicky pain is felt near the umbilicus with vomiting and constipation.

Ans. A A Gnawing epigastric pain that radiates to the back or shoulder and worsens after eating is a symptom that is consistent with peptic ulcer disease. B Sharp midepigastric pain that radiates to the jaw is not a symptom of peptic ulcer disease. C Intermittent cramping pain in the left lower quadrant relieved by defecation is a symptom of diverticular disease rather than peptic ulcer disease. D Colicky pain felt near the umbilicus with vomiting and constipation is a symptom of an intestinal obstruction rather than peptic ulcer disease.

A nurse determines that a patient has a heart rate of 42 beats per minute. What might be a cause of this heart rate? a. Sinoatrial (SA) node failure b. Atrial bradycardia c. A well-conditioned heart muscle d. Left ventricular hypertrophy

Ans. A A If the SA node is ineffective, the atrioventricular node may initiate contraction, but at a rate of 40 to 60 beats/min. B The heart rate reflects the ventricular rate rather than the atrial rate. C Although well-conditioned athletes may have slower heart rates, this rate is too slow for even an athlete. D Left ventricular hypertrophy alters the strength of contraction rather than the heart rate.

The nurse recognizes which clinical finding as expected on palpation of the abdomen? a. Inability to palpate the spleen b. Left kidney rounded at 2 cm below the costal margin c. Slight tenderness of the gallbladder on light palpation d. Bounding pulsation of the aorta over the umbilicus

Ans. A A Inability to palpate the spleen is the expected finding on palpating the abdomen. B A rounded left kidney at 2 cm below the costal margin is not an expected finding. Kidneys are usually not palpated. C Slight tenderness of the gallbladder on light palpation is not an expected finding; the gallbladder is usually not palpable. D Bounding pulsation of the aorta over the umbilicus would be an abnormal finding, perhaps indicating an aneurysm.

How does a nurse assess the competence of venous valves in patients who have varicose veins? a. Notes how quickly veins fill after lifting one leg above the level of the heart b. Assesses for Homan sign in both lower extremities while the patient is supine c. Assesses capillary refill on the toes of both feet while the patient is sitting in the chair d. Measures the circumference of both calves and compares the results

Ans. A A Noting how quickly veins fill after lifting one leg above the level of the heart is the procedure to test for incompetent veins. B Homan sign is an unreliable test for deep vein thrombosis. C Assessing capillary refill assesses perfusion (blood flow from arteries) rather than competence of venous valves. D Measuring the circumference of both calves and comparing the results is used to assess deep vein thrombosis.

A nurse learns from a report that a patient has aortic stenosis. Where does the nurse place the stethoscope to hear this stenotic valve? a. Second intercostal space, right sternal border b. Second intercostal space, left sternal border c. Fourth intercostal space, left sternal border d. Fifth intercostal space, left midclavicular line

Ans. A A Second intercostal space, right sternal border is the location for listening to the aortic valve. B Second intercostal space, left sternal border is the location for listening to the pulmonic valve. C Fourth intercostal space, left sternal border is the location for listening to the tricuspid valve. D Fifth intercostal space, left midclavicular line is the location for listening to the mitral valve.

During the problem-based history, a patient reports coughing up sputum when lying on the right side, but not when lying on the back or left side. The nurse suspects this patient may have a lung abscess. What additional question does the nurse ask to gather more data? a. "Does the sputum have an odor?" b. "Do you have chest pain when you take a deep breath?" c. "Have you also experienced tightness in your chest?" d. "Have you coughed up any blood?"

Ans. A A Sputum with odor and sputum production with change of position is associated with lung abscess or bronchiectasis. B Chest pain on deep breathing is associated with pleural lining irritation. C Tightness in the chest is associated with asthma. D Coughing up rust-colored sputum is associated with pneumonia, but coughing up blood may be associated with lung cancer.

After taking a brief health history, a nurse needs to complete a focused assessment on which patient? a. A male who works as a painter b. A male who plays basketball and hockey c. A female who recently moved into a college dormitory d. A female who has a history of gout

Ans. A A The fumes and chemicals from the paint may expose the patient to respiratory irritants. A baseline pulmonary assessment needs to be documented. B This patient is not at risk for pulmonary disease. C This patient is not at risk for pulmonary disease. D This patient is not at risk for pulmonary disease.

On palpation of the left upper quadrant of the abdomen of a female patient, the nurse notes tenderness. This finding may indicate a disorder in which organ? a. Spleen b. Gallbladder c. Sigmoid colon d. Left ovary

Ans. A A The spleen is located in the left upper quadrant of the abdomen. B The gallbladder is located in the right upper quadrant of the abdomen. C The sigmoid colon is located in the left lower quadrant of the abdomen. D The left ovary is located in the left lower quadrant of the abdomen.

Which patient should the nurse assess first? a. The patient whose respiratory rate is 26 breaths per minute and whose trachea deviates to the right. b. The patient who has pleuritic chest pain, bilateral crackles, a productive cough of yellow sputum, and fever. c. The patient who is short of breath, using pursed-lip breathing, and in a tripod position. d. The patient whose respiratory rate is 20 breaths/min, and has 8-word dyspnea and expiratory wheezes.

Ans. A A This is a description of a left tension pneumothorax. The key manifestation is deviation of the trachea from midline, which indicates high intrathoracic pressure from the left that is pushing the mediastinum out of alignment. The respiratory rate indicates tachypnea. B This is a description of a patient with pneumonia who needs to be examined, but this is not a life-threatening condition. C This is a description of a patient with emphysema, a chronic disease. This patient may have these manifestations frequently and does not need to be examined immediately. D This is a description of a patient who is having an asthma attack, but it is not a life threatening attack; the respiratory rate is the upper limits of normal; the dyspnea is abnormal, but not far from normal; and the wheezing is on expiration only.

A patient has right lower lobe pneumonia, creating a consolidation in that lung. In assessing for vocal fremitus, the nurse found increased fremitus over the right lower lung. What finding does the nurse anticipate when assessing vocal resonance to confirm the consolidation? a. Bronchophony reveals the patient's spoken "99" as clear and loud. b. No sounds are expected since sounds cannot be transmitted through consolidation. c. Egophony reveals indistinguishable sounds when the patient says "e-e-e." d. Whispered pectoriloquy reveals a muffled sound when the patient says "1-2-3."

Ans. A A This is an abnormal finding and occurs in consolidation. B The abnormal finding is hearing a clear sound. C This is a normal finding. D This is a normal finding.

A nurse notices abdominal distention when inspecting a patient's abdomen. What action does the nurse take next to gain further objective data? a. Place a measuring tape around the superior iliac crests. b. Assist the patient to turn on to the left side and then the right side. c. Ask the patient to cough while lying supine. d. Use the fingertips to sharply strike one side of the abdomen.

Ans. A A This is the procedure for measuring abdominal girth. B This procedure is unnecessary. The distention will remain in a side-lying position. C Having the patient cough is used to assess for bulges rather than distention. D This is part of the procedure to test for a fluid wave, which is not indicated in this patient.

Which sound does a nurse expect to hear when percussing a patient's abdomen? a. Tympany over all quadrants b. Resonance over the upper quadrants and tympany in the lower quadrants c. Dull sounds over the upper quadrants and hollow sounds over the lower quadrants d. Dull sounds over the stomach and resonant sounds over the bladder

Ans. A A Tympany over all quadrants is a normal finding, which represents the presence of gas. B Resonance in the upper quadrants and tympany over the lower quadrants are not normal findings. There would be tympany in the lower quadrants, but also in the upper quadrants. C Dull sounds over the lower quadrants and hollow sounds over the upper quadrants are not normal findings. There would be tympany over the lower quadrants, but also in the upper quadrants. D Dull sounds over the stomach and resonant sounds over the bladder are not normal findings.

A nurse had previously heard crackles over both lungs of a patient. As the patient improves, what lung sounds does the nurse expect to hear in the patient's lungs? a. Vesicular breath sounds heard in peripheral lung fields b. Bronchial breath sounds heard over the bronchi c. Bronchovesicular breath sounds heard over the apices d. Rhonchi heard over the main bronchi

Ans. A A Vesicular breath sounds heard in peripheral lung fields are an expected finding for healthy lungs. B Bronchial breath sounds are heard over the trachea. C Bronchovesicular breath sounds are heard anteriorly near the sternal border first and second intercostals space. D Rhonchi are adventitious sounds indicating secretions in the bronchi.

A nurse expects which finding during a cardiovascular assessment of a healthy adult? a. Visible, consistent pulsations of the jugular vein b. Pink nail beds with a 90-degree angle at the base c. Capillary refill of the toes greater than 5 seconds d. Bruits heard on auscultation of the carotid arteries

Ans. A A Visible, consistent pulsations of the jugular vein is an expected finding. B Pink nail beds with a 90-degree angle at the base is not a normal finding; the angle at the base should be 160 degrees. C Capillary refill of the toes greater than 5 seconds is not a normal finding. Capillary refills should be 2 seconds or less. D Bruits heard on auscultation of the carotid arteries is not a normal finding. Bruits indicate occlusion of a blood vessel.

A patient reports having abdominal distention. The nurse notices that the patient's sclerae are yellow. What question is appropriate for the nurse to ask in response to this information? a. "Has there been a change in your usual pattern of urination?" b. "Have you had any nausea or vomiting?" c. "Has there been a change in your bowel habits?" d. "Have you had indigestion or heartburn?"

Ans. B A "Has there been a change in your usual pattern of urination?" is not a question related to the abdominal distention and jaundice. B "Have you had any nausea or vomiting?" is an appropriate question because the nurse suspects the patient may have a liver disease based on the abdominal distention and jaundice. The nurse interprets the relationship with data gathered from the history and the observation. C "Has there been a change in your bowel habits?" is a question that may be related to the abdominal distention, but not the jaundice. D "Have you had indigestion or heartburn?" is not a question related to the abdominal distention and jaundice. It applies more to gastric disorders, such as gastroesophageal reflux disease or hiatal hernia.

In assessing a patient with renal disease, the nurse palpates edema in both ankles and feet. Based on this finding, what question does the nurse ask the patient? a. "Have you had any pain in your abdomen?" b. "Have you had an unexpected weight gain?" c. "Have you noticed a change in the color of your skin?" d. "Have you had any nausea or vomiting?"

Ans. B A "This question does not relate to renal disease. The pain experienced with renal disease is usually flank pain over the costovertebral angle. B This question relating to weight gain and edema suggests fluid retention that occurs with renal or heart disease, particularly renal failure. C This question does not relate to renal disease. It might relate to liver or gallbladder disease if the change in skin color was yellow, indicating jaundice. D This question usually relates to disorders within the gastrointestinal tract itself and not renal disease.

A nurse auscultates low-pitched, coarse snoring sounds in a patient's lungs during inhalation. What is the most appropriate action for the nurse to take at this time? a. Palpate the posterior thorax for vocal fremitus. b. Ask the patient to cough and repeat auscultation. c. Auscultate the posterior thorax for vocal sounds. d. Percuss the posterior thorax for tone.

Ans. B A An abnormal vocal fremitus (decreased or increased vibrations) is not expected for this patient. B The sounds indicate rhonchi, or secretions in the bronchi. The first action to take is to determine if the rhonchi clear with coughing. If the rhonchi clear, there is no need to further investigate this finding. C Abnormal vocal sounds (clear and loud sounds) are not expected for this patient. D An abnormal percussion tone (hyperresonance or dull) is not expected for this patient.

While taking a history, a nurse learns that a patient had rheumatic heart disease as a child. Based on this information, what abnormal data might this nurse expect to find during an examination? a. An extra beat just before the S2 heart sound heard during auscultation b. A raspy machine-like or blowing sound heard during auscultation c. A prominent thrust of the heart against the chest wall felt on palpation d. A visible indentation of pericardial tissue noted during inspection

Ans. B A An extra beat just before the S1 heart sound heard during auscultation is a description of the S4 heart sound that occurs when there is hypertrophy of the ventricle. B A raspy machine-like or blowing sound heard during auscultation is a description of a murmur that can develop after rheumatic heart disease. C A prominent thrust of the heart against the chest wall felt on palpation is a description of a heave, which may occur from left ventricular hypertrophy due to increased workload. D A visible indentation of pericardial tissue noted during inspection is a description of a retraction that begins in the intercostal spaces and occurs with increased respiratory effort.

The nurse is comparing pitch and duration of the various types of a patient's breath sounds and recognizes which one of these as an expected finding? a. Bronchial sounds are low-pitched and have a 2:1 inspiratory-versus-expiratory ratio. b. Bronchovesicular sounds have a moderate pitch and 1:1 expiratory-versus-inspiratory ratio. c. Vesicular breath sounds are high-pitched and have a 1:2 inspiratory-versus-expiratory ratio. d. Wheezes are low-pitched and have a 2.5:1 inspiratory-versus-expiratory ratio.

Ans. B A Bronchial sounds are high pitched with a duration of 1:2 inspiration-to-expiration is the correct statement. B Bronchovesicular sounds having a moderate pitch and 1:1 expiratory-versus-inspiratory ratio is a normal finding. C Vesicular sounds are low pitched with a duration of 2.5:1 inspiration-to-expiration is the correct statement. D Wheezes are high-pitched and have no specific duration because they are adventitious sounds.

Which patient's statement helps a nurse distinguish between chest pain originating from pericarditis rather than from angina? a. "No, I have not done anything to strain chest muscles." b. "If I take a deep breath, the pain gets much worse." c. "This pain feels like there's an elephant sitting on my chest." d. "Whenever this pain happens, it goes right away if I lie down."

Ans. B A Chest pain from muscle strain may be aggravated by movement of arms. B The chest pain from pericarditis is aggravated by deep breathing, coughing, or lying supine. C "This pain feels like there's an elephant sitting on my chest" is associated with a myocardial infarction. D Chest pain relieved by rest occurs with angina.

A patient is suspected of having a lung consolidation. A nurse uses the three techniques for assessing vocal resonance in this patient. What is the expected finding among the three procedures that will help eliminate consolidation as a problem? a. The nurse documents clearly hearing the patient say "99." b. The nurse documents hearing muffled sounds when the patient says "1-2-3." c. The nurse documents hearing no sounds when the patient says "e-e-e." d. The nurse documents clearly hearing the patient say "a-a-a."

Ans. B A Clear sounds are heard when a consolidation is present. B Muffled sounds of "1-2-3," "e-e-e," or "99" are heard when no consolidation is found. C Clear sounds are heard when a consolidation is present. D Clear sounds are heard when a consolidation is present.

A patient reports having abdominal distention. The nurse observes that the patient's sclerae are yellow. Which abnormal finding does the nurse anticipate on examination of this patient's abdomen? a. Decreased bowel sounds in all quadrants b. Glistening or taut skin of the abdomen c. Bulge in the abdomen when coughing d. Bruit around the umbilicus

Ans. B A Decreased bowel sounds in all quadrants may be present if the abdominal distention was from an intestinal obstruction, but the observation of jaundice suggests liver disease, which does not decrease bowel sounds. B Glistening or taut skin of the abdomen is consistent with ascites that appear as abdominal distention. Jaundice and ascites suggest liver disease. There would also be an increase in abdominal girth. C A bulge in the abdomen when coughing is a finding associated with abdominal or incisional hernias. D Bruit around the umbilicus is a finding associated with an abdominal aortic aneurysm.

A patient reports a productive cough with yellow sputum, fever, and a sharp pain when taking a deep breath to cough. Based on these data, what abnormal finding will the nurse anticipate on examination? a. Decreased breath sounds on auscultation b. Increased tactile fremitus and dull percussion tones c. Inspiratory wheezing found on auscultation d. Muffled sounds heard when the patient says "e-e-e"

Ans. B A Decreased breath sounds on auscultation is consistent with emphysema or atelectasis when alveoli are narrowed or destroyed. B The data describe purulent sputum and inflammation of the pleura that may occur in pneumonia. Additional findings include increased tactile fremitus and dull percussion tones, indicating congested or consolidated lung tissues. C Inspiratory wheezing found on auscultation is consistent with narrowing of bronchi that may occur in asthma. D Muffled sounds heard when the patient says "e-e-e" is a normal finding on vocal resonance (bronchophony or egophony).

Which assessment technique is the nurse performing in the figure below? a. Direct percussion b. Indirect percussion c. Light palpation d. Deep palpation

Ans. B A Direct percussion is performed with one hand. B Indirect percussion is the technique shown. C Light palpation is performed using the pads of the fingers depressing the tissue 1 to 2 cm, usually on the abdomen. D Deep palpation is performed using the pads of the fingers depressing the tissue 4 to 6 cm, usually on the abdomen.

A nurse is assessing a patient who was diagnosed with emphysema and chronic bronchitis 5 years ago. During the assessment of this patient's integumentary system, what finding should the nurse correlate to this respiratory disease? a. Dry, flaky skin b. Clubbing of the fingers c. Hypertrophy of the nails d. Hair loss from the scalp

Ans. B A Dry, flaky skin occurs with dehydration. B Clubbing of the fingers develops due to chronic hypoxemia, which occurs in chronic obstructive pulmonary disease. C Hypertrophy of the nails occurs with repeated trauma. D Hair loss from the scalp is alopecia, which occurs with many systemic diseases, but not chronic pulmonary disease.

When assessing the abdomen of a patient who has fluid in the peritoneal cavity, the nurse expects what change to occur when the patient turns from supine to the left side? a. Movement of the tympanic tones from lateral in the supine position to closer to midline when lying on the left side b. Movement of the dull tones from lateral in the supine position to closer to midline when lying on the left side c. Change in bowel sounds from hypoactive in the supine position to hyperactive when lying on the left side d. Change in bowel sounds from hyperactive in the supine position to hypoactive when lying on the left side

Ans. B A Movement of the tympanic tones from lateral in the supine position to closer to midline when lying on the left side is incorrect because the tone will be dull, rather than tympanic, due to the fluid. B Movement of the dull tones from lateral in the supine position to closer to midline when lying on the left side is the expected change when assessing for shifting dullness. C A change in bowel sounds from hypoactive in the supine position to hyperactive when lying on the left side is incorrect because bowel sounds would not be affected by the fluid. D A change in bowel sounds from hyperactive in the supine position to hypoactive when lying on the left side is incorrect because bowel sounds would not be affected by the fluid.

While assessing edema on a male patient's lower leg, the nurse notices that there is a slight imprint of his fingers where he palpated the patient's leg. How does the nurse document this finding? a. No edema b. 1+ edema c. 2+ edema d. 3+ edema

Ans. B A No pit left after palpation indicates no edema. B A barely perceptible pit is detected after palpation. C A deeper pit that rebounds in a few seconds after palpation is 2+ edema. D A deep pit that rebounds in 10 to 20 seconds after palpation is 3+ edema.

When inspecting a patient's abdomen, the nurse notes which finding as abnormal? a. Protruding abdomen with skin that is lighter in color than the arms and legs b. Marked rhythmic pulsation to the left of the midline c. Faint, fine vascular network d. Small shadows created by changes in contour

Ans. B A Obesity may cause a protruding abdomen and although obesity is not an indicator of health, it does not necessarily indicate a disease is present. B Marked rhythmic pulsation to the left of the midline is an abnormal finding that may indicate an abdominal aortic aneurysm. C A faint, fine vascular network is a normal finding. If the vessels were engorged, it would be an abnormal finding. D Small shadows created by changes in contour are a normal finding and they are seen by using a light source to inspect the contour.

A nurse inspects the abdomen for skin color, surface characteristics, and surface movement. What part of the abdominal assessment does the nurse perform next? a. Palpate lightly for tenderness and muscle tone. b. The tip of the middle finger of the dominant hand strikes the nail of the middle finger touching the skin of the abdomen. c. Palpate deeply for masses or aortic pulsation. d. Percuss for tones.

Ans. B A Palpating lightly for tenderness and muscle tone is performed after auscultation. B Auscultation for bowel sounds occurs before palpating and percussing the abdomen. C Palpating deeply for masses or aortic pulsation is performed after light palpation. D Percussion for tones is performed after palpation.

What technique does a nurse use when performing deep palpation of a patient's abdomen? a. Places the left hand under the ribs to lift them up b. Asks the patient to breathe slowly through the mouth c. Positions the patient on the right side with knees flexed d. Uses the heel of the hand to depress the abdomen

Ans. B A Placing the left hand under the ribs to lift them up is the technique for palpating the liver. B Asking the patient to breathe slowly through the mouth while the nurse uses the pads of the fingers to depress the abdomen is the correct procedure. C Positioning the patient on the right side with knees flexed is an alternate strategy for palpating the spleen. D Using the heel of the hand to depress the abdomen is not a correct technique; the pads of the fingers are used.

How does the nurse accurately assess bowel sounds? a. Press the diaphragm of the stethoscope firmly against the abdomen in each quadrant. b. Hold the diaphragm of the stethoscope lightly against the abdomen in each quadrant. c. Press the bell of the stethoscope firmly against the abdomen in each quadrant. d. Hold the bell of the stethoscope lightly against the abdomen in each quadrant.

Ans. B A Pressing the diaphragm of the stethoscope firmly against the abdomen in each quadrant is not the correct technique for listening to bowel sounds. B Holding the diaphragm lightly against the abdomen in each quadrant is the correct technique for listening to bowel sounds. C The bell is used to listen to vascular sounds of the abdomen, which are normally not heard. D The bell is used to listen to vascular sounds of the abdomen, which are normally not heard.

Which valve does a nurse auscultate when the stethoscope is placed on the fourth intercostal space at the left of the sternal border? a. Pulmonic b. Tricuspid c. Mitral d. Aortic

Ans. B A Pulmonic valve sounds are best heard in the second intercostal space at the left of the sternal border. B Tricuspid valve sounds are best heard in the fourth intercostal space at the left of the sternal border. C Mitral valve sounds are best heard in the fifth intercostal space at the midclavicular line. D Aortic valve sounds are best heard in the second intercostal space at the right of the sternal border.

The patient describes her chest pain as "squeezing, crushing, and 12 on a scale of 10." This pain started more than an hour ago while she was resting, and she also feels nauseous. Based on these findings, the nurse should assess for which associated symptoms? a. Tachycardia, tachypnea, and hypertension b. Dyspnea, diaphoresis, and palpitations c. Hyperventilation, fatigue, anorexia, and emotional strain d. Fever, dyspnea, orthopnea, and friction rub

Ans. B A Tachycardia, tachypnea, and hypertension are symptoms associated with cocaine-induced chest pain. B Dyspnea, diaphoresis, and palpitations are symptoms associated with unstable angina. C Hyperventilation, fatigue, anorexia, and emotional strain are symptoms associated with panic disorder. D Fever, dyspnea, orthopnea, and friction rub are symptoms associated with pericarditis.

A nurse informs a patient that her blood pressure is 128/78. The patient asks what the number 128 means. What is the nurse's appropriate response? The 128 represents the pressure in your blood vessels when: a. "The ventricles relax and the aortic and pulmonic valves open." b. "The ventricles contract and the mitral and tricuspid valves close." c. "The ventricles contract and the mitral and tricuspid valves open." d. "The ventricles relax and the aortic and pulmonic valves close."

Ans. B A The aortic and pulmonic valves open during systole, but ventricles fill during diastole. B During systole the ventricles contract, creating a pressure that closes the atrioventricular (AV) valves (mitral and tricuspid). C During systole the ventricles contract, creating a pressure that closes the AV valves (mitral and tricuspid). D The ventricles are relaxed and the aortic and pulmonic valves close during diastole, rather than systole.

What does the S2 heart sound represent? a. The beginning of systole. b. The closure of the aortic and pulmonic valves. c. The closure of the tricuspid and mitral values d. A split heard sound on exhalation

Ans. B A The beginning of systole is the S1 heart sound. B The second heart sound is made by the closing of these valves, which indicates the beginning of diastole. C The tricupid and mitral valves create the S1 heart sound. D A split sound on exhalation is not a correct statement.

Which location does a nurse select when palpating a patient's liver? a. A (right lower quadrant) b. B (right upper quadrant) c. C (left upper quadrant) d. D (left lower quadrant)

Ans. B A The majority of the liver is located in the right upper quadrant of the abdomen. B The majority of the liver is located in the right upper quadrant of the abdomen. C C is the left upper quadrant. D D is the left lower quadrant.

A nurse suspects a patient has a chest wall injury and wants to collect more data about thoracic expansion. Which is the appropriate technique to use? a. Place the palmar side of each hand against the lateral thorax at the level of the waist, ask the patient to take a deep breath, and observe lateral movement of the hands. b. Place both thumbs on either side of the patient's T9 to T10 spinal processes, extend fingers laterally, ask the patient to take a deep breath, and observe lateral movement of the thumbs. c. Place both thumbs on either side of the patient's T7 to T8 spinal processes, extend fingers laterally, ask the patient to exhale deeply, and observe lateral inward movement of the thumbs. d. Place the palmar side of each hand on the shoulders of the patient, ask the patient to sit up straight and take a deep breath, and observe symmetric movement of the shoulders.

Ans. B A The palms of the hands are not used and hands are not placed on the lateral thorax. B This is the correct technique to assess thoracic expansion. C The thoracic level is too high and the patient does not exhale. D The hands are not placed on the shoulders.

To document the palpation of a pulse, the nurse is correct in making which notation about the rhythm? a. "Rhythm 100 beats/min" b. "Irregular rhythm" c. "Rhythm noted at +2" d. "Bounding rhythm"

Ans. B A This notation refers to the rate rather than the rhythm. B The rhythm should be an equal pattern or spacing between beats. Irregular rhythms without any pattern should be noted. C This notation refers to the amplitude rather than the rhythm. D This notation refers to the contour rather than the rhythm.

Which question will give the nurse additional information about the nature of a patient's dyspnea? a. "How often do you see the physician?" b. "How has this condition affected your day-to-day activities?" c. "Do you have a cough that occurs with the dyspnea?" d. "Does your heart rate increase when you are short of breath?"

Ans. B A This question does not relate specifically to the patient's dyspnea. B This question provides data about the severity of the dyspnea and what actions the patient has taken to cope with the dyspnea on a daily basis. C This question provides data, but does not give additional facts about the patient's dyspnea. D This is a closed-ended question that does not collect additional data about this episode of dyspnea.

A patient is admitted to the emergency department with a tracheal obstruction. What sound does the nurse expect to hear as this patient breathes? a. Dull sounds on percussion b. Soft, muffled rhonchi heard over the trachea c. Bubbling or rasping sounds heard over the trachea d. High-pitched sounds on inspiration and exhalation

Ans. D A Dull sounds on percussion occur with pneumonia, pleural effusion, or atelectasis. B Soft, muffled rhonchi heard over the trachea is not a description of stridor. C Bubbling or rasping sounds heard over the trachea is not a description of stridor. D High-pitched sounds on inspiration and exhalation are consistent with stridor.

During an assessment for abdominal pain, a patient reports a colicky abdominal pain and pain in the right shoulder that gets worse after eating fried foods. What question does the nurse ask to confirm the suspicion of cholelithiasis? a. "Have you noticed any swelling in your ankles or feet at the end of the day?" b. "Have you noticed a change in the color of your urine or stools?" c. "Have you vomited up any blood in the last 24 hours?" d. "Have you experienced fever, chills, or sweating?"

Ans. B A This question is related to fluid retention, which may be asked if the patient has renal or heart failure. B Gallstones can obstruct the flow of bile to the gastrointestinal tract making urine darker and stools lighter in color. C This question applies if the patient has peptic ulcer disease or esophageal varices. D This question applies if the patient has gastroenteritis or a urinary tract infection.

A nurse is auscultating the lungs of a healthy male patient and hears crackles on inspiration. What action can the nurse take to ensure this is an accurate finding? a. Make sure the bell of the stethoscope is used, rather than the diaphragm. b. Hold stethoscope firmly to prevent movement when placed over chest hair. c. Ask the patient not to talk while the nurse is listening to the lungs. d. Change the patient's position to ensure accurate sounds.

Ans. B A Using the bell will provide inaccurate sounds, but not mimic crackles. B The stethoscope moving even slightly on chest hair can mimic the sound of crackles. C When the patient talks during auscultation, it does interfere with data collection, but the sound is a muffled voice. D Changing the position will not affect the outcome of the assessment if the initial problem remains.

During a history, the patient reports having gout. Based on this information, what findings does the nurse anticipate during a focused assessment? a. Warm, tender, and deformed wrists and peripheral interphalangeal (PIP) joints bilaterally b. Edema, warmth, and redness of one great toe and pea-like nodules in the ear lobes c. Enlarged and tender PIP or distal interphalangeal (DIP) joints on one or several fingers d. Tenderness with pronation and supination of the elbow and point tenderness on the lateral epicondyle

Ans. B A This is a description of findings of a patient who has rheumatoid arthritis. Bilateral joint involvement is common. B This is a description of gout. The pealike nodules are tophi, collections of uric acid in subcutaneous tissue. C This is a description of findings of a patient who has osteoarthritis. Enlarged and tender PIP joints refer to Heberden nodes and DIP joints refer to Bouchard nodes. D This is a description of epicondylitis (tennis elbow).

A patient reports having abdominal distention and having vomited several times yesterday and today. What question is appropriate for the nurse to ask in response to this information? a. "Has there been a change in your usual pattern of urination?" b. "Did you have heartburn before the vomiting?" c. "What did the vomitus look like?" d. "Have you noticed a change in the color of your urine or stools?"

Ans. C A "Has there been a change in your usual pattern of urination?" is not a question related to abdominal distention and vomiting. B Have you noticed a change in the color of your urine or stools?" is not a question related to abdominal distention and vomiting. It is related to elevated bilirubin from liver or gallbladder disease and is accompanied by jaundice. C "What did the vomitus look like?" is an appropriate question because the characteristics of the vomitus may help determine its cause. Acute gastritis leads to vomiting of stomach contents, obstruction of the bile duct results in greenish-yellow vomitus, and an intestinal obstruction may cause a fecal odor to the vomitus. D This is not a question related to the abdominal distention and vomiting. Heartburn applies more to gastric disorders, such as gastroesophageal reflux disease or hiatal hernia.

When assessing a patient with aortic valve stenosis, the nurse listens for which sound to detect a thrill? a. Sustained thrust of the heart against the chest wall during systole b. Visible sinking of the tissues between and around the ribs c. Fine, palpable vibration felt over the precordium d. Bounding pulse noted bilaterally

Ans. C A A sustained thrust of the heart against the chest wall during systole is a description of a lift. B A visible sinking of the tissues between and around the ribs is a description of a retraction. C A thrill is a palpable vibration over the precordium or artery. D A thrill feels like a palpable vibration rather than a bounding pulse.

Which patient does the nurse identify as the one at greatest risk for hypertension? a. Woman with coronary artery disease b. Hispanic male c. Obese male with diabetes mellitus d. Postmenopausal woman

Ans. C A Although hypertension is a risk factor for coronary artery disease, coronary artery disease is not a risk factor for hypertension. B Although male gender is a risk factor, African-American men have a greater risk than Hispanic men. C Obese men with diabetes mellitus have three risk factors: obesity, gender, and comorbidity of diabetes mellitus. D Postmenopausal women do not have an increased risk for developing hypertension.

A nurse palpating the chest of a patient finds increased fremitus bilaterally. What is the significance of this finding? a. An expected finding b. Chronic obstructive pulmonary disease c. Bilateral pneumonia d. Bilateral pneumothorax

Ans. C A An increase in fremitus from normal is not an expected finding. B Air trapping in chronic obstructive pulmonary disease causes a decreased fremitus. C Increased fremitus occurs when lung tissues are congested or consolidated, which may occur in patients who have pneumonia or a tumor. D Air in the pleural space causes a decreased fremitus.

A nurse examines a patient with a pleural effusion and finds decreased fremitus. What additional abnormal finding should the nurse anticipate during further examination? a. An increase in the anteroposterior to lateral ratio b. Hyperresonance over the affected area c. Absent breath sounds in the affected area d. Increased vocal fremitus over the affected area

Ans. C A An increase in the anteroposterior to lateral ratio occurs in overinflated lungs as in emphysema. B Hyperresonance over the affected area occurs in overinflated lungs as in emphysema. C Absent breath sound in the affected area is anticipated because the fluid in the pleural space prevents breath sounds from being heard. D Increased vocal fremitus over the affected area is associated with consolidation that occurs with pneumonia or tumor. Fremitus is decreased to absent in pleural effusion.

A nurse who is auscultating a patient's heart hears a harsh sound, a raspy machine-like blowing sound, after S1 and before S2. How does this nurse document this finding? a. An opening snap b. A diastolic murmur c. A systolic murmur d. A pericardial friction rub

Ans. C A An opening snap is caused by the opening of the mitral or tricuspid valve and is an abnormal sound heard in diastole when either valve is thickened, stenotic, or deformed. The sounds are high pitched and occur early in diastole. B A diastolic murmur is heard after the S2 heart sound at the beginning of diastole. C The blowing sound is a murmur. The nurse determines whether it is a systolic or a diastolic murmur based on where it is heard during the cardiac cycle. S1 indicates the beginning of systole; the sound is made by the closing of the mitral and tricuspid valves, which is followed by ventricular contraction or systole. D Pericardial friction rubs have a rubbing sound that is usually present in both diastole and systole, and is best heard over the apical area.

On inspection of a female patient's abdomen, the nurse asks the patient to raise her head without using her arms and notes a midline bulge. What is the appropriate response of the nurse at this time? a. Ask the patient to cough to see if the bulge reappears. b. Auscultate the patient's abdomen for hypoactive bowel sounds. c. Document this as a normal finding and continue the examination. d. Perform light and deep palpation of the abdomen.

Ans. C A Ask the patient to cough to see if the bulge reappears. A bulge that appears with coughing is an abnormal finding revealed by the increase in intrathoracic pressure during the cough. B Auscultating the patient's abdomen for hypoactive bowel sounds is not indicated because the bulge is a normal finding. C Document this as a normal finding and continue the examination. This is a normal finding on a patient D Performing light and deep palpation of the abdomen are not indicated because the bulge is a normal finding.

Which techniques does a nurse use to palpate a patient's right kidney? a. Asks the patient to take a deep breath, elevates the patient's eleventh and twelfth ribs with the left hand, and deeply palpates for the right kidney with the right hand b. Asks the patient to exhale, elevates the patient's eleventh and twelfth ribs with the left hand, and deeply palpates for the right kidney with the right hand c. Asks the patient to take a deep breath, elevates the patient's right flank with the left hand, and deeply palpates for the right kidney with the right hand d. Asks the patient to exhale, elevates the patient's right flank with the left hand, and deeply palpates for the right kidney with the right hand

Ans. C A Asking the patient to take a deep breath, elevating the patient's eleventh and twelfth ribs with the left hand, and deeply palpating for the right kidney with the right hand is incorrect because the flank is elevated rather than the ribs. B Asking the patient to exhale, elevating the patient's eleventh and twelfth ribs with the left hand, and deeply palpating for the right kidney with the right hand is incorrect because the flank is elevated rather than the ribs and the patient is asked to inhale rather than exhale. C Asking the patient to take a deep breath, elevating the patient's right flank with the left hand, and deeply palpating for the right kidney with the right hand is the correct technique. D Asking the patient to exhale, elevating the patient's right flank with the left hand, and deeply palpating for the right kidney with the right hand is incorrect because the patient is asked to inhale rather than exhale.

In reviewing the patient's record, the nurse notes that the patient has air in the subcutaneous tissue. The nurse validates that this patient has crepitus with which finding? a. Asymmetric expansion of the chest wall on inhalation b. Increased transmission of vocal vibrations on auscultation c. Crackling sensation under the skin of the chest on palpation d. Coarse grating sounds heard over the mediastinum on inspiration

Ans. C A Asymmetric chest expansion occurs with rib fracture or chest wall injury. B Increased vocal fremitus occurs with lung consolidation. C A crackling sensation is the finding when crepitus is present. D Coarse grating sounds heard over the mediastinum on inspiration does not validate crepitus.

A patient reports intermittent cramping abdominal pain that is relieved by having a bowel movement. The patient complains of having the pain at this time, which is why she is seeking care. Which abnormal finding does the nurse anticipate finding on examination of this patient's abdomen? a. Decreased bowel sounds b. Bulge in the abdomen when coughing c. Palpable mass in the left lower quadrant d. Bruit around the umbilicus

Ans. C A Decreased bowel sounds are not expected if the patient is having bowel movements. B Bulge in the abdomen when coughing is a finding associated with abdominal or incisional hernias. C Palpable mass in the left lower quadrant is expected when interpreted with other data—age of the patient, intermittent cramping abdominal pain relieved by a bowel movement—as consistent with diverticular disease. D Bruit around the umbilicus is a finding associated with an abdominal aortic aneurysm.

When assessing an adult's liver, the nurse percusses the lower border and finds it to be 5 cm below the costal margin. What is the nurse's appropriate action at this time? a. Document this as an expected finding for this adult. b. Palpate the gallbladder for tenderness. c. Palpate the upper liver border on deep inspiration. d. Use the hooking technique to palpate the lower border of the liver.

Ans. C A Documenting this as a normal finding for an adult patient is incorrect because this finding indicates an enlarged liver. B Palpating the gallbladder for tenderness is not indicated for an enlarged liver. C Palpating the upper border of the liver on deep inspiration is the correct technique to use when an enlarged liver is found (as indicated by the liver being percussed 5 cm below the costal margin). D Using the hooking technique to palpate the lower border of the liver is not needed because the liver is enlarged.

When auscultating a patient's abdomen using the bell of the stethoscope, the nurse hears soft, low-pitched murmurs over the right and left upper midline. What do these sounds indicate? a. Expected peristalsis b. Femoral artery stenosis c. Renal artery stenosis d. Hyperactive bowel sounds

Ans. C A Expected peristalsis would be heard using the diaphragm of the stethoscope and would be a gurgling sound. B Femoral artery stenosis is a vascular sound heard with the bell, but located in the lower abdomen. C Renal artery stenosis is a vascular sound heard with the bell and located in the upper abdomen. D Hyperactive bowel sounds would be heard using the diaphragm and would be present in all quadrants.

On inspection, a nurse finds the patient's anteroposterior diameter of the chest to be the same as the lateral diameter. Based on this finding, what additional data does the nurse anticipate? a. Increased vocal fremitus on palpation b. Dull tones heard on percussion c. Decreased breath sounds on auscultation d. Complaint of sharp chest pain on inspiration

Ans. C A Increased fremitus occurs when the vibrations feel enhanced. This is found when lung tissues are congested or consolidated, which may occur in patients who have pneumonia or a tumor. B Dull tones may be heard in patients with pneumonia, pleural effusion, or atelectasis. C The equal anteroposterior and lateral diameters of the chest indicate air trapping from enlarged or destroyed alveoli. This air trapping causes decreased to absent breath sounds on auscultation. D Complaint of sharp chest pain on inspiration is pleuritic chest pain associated with pleural lining irritation and may occur in a patient with pleurisy or pneumonia.

A nurse is having difficulty auscultating a patient's heart sounds because the lung sounds are too loud. What does the nurse ask the patient to do to improve hearing the heart sounds? a. Lie in a supine position. b. Cough. c. Hold his or her breath for a few seconds. d. Sit up and lean forward.

Ans. C A Lying in a supine position will not reduce the noise of breathing. B Coughing may clear some secretions, but when the lung sounds are so noisy that the heart sounds are difficult to hear, coughing is not sufficient to eliminate the noise from respirations. C Holding the breath for a few seconds eliminates the noise of breathing long enough to hear several cardiac cycles of heart sounds. The holding of the breath can be repeated if needed to hear the heart sounds again. D Sitting up and leaning forward brings the heart closer to the thoracic wall, but will not eliminate noise produced by the lungs.

A 75-year-old male patient asks how to reduce his risk of esophageal cancer. What is the nurse's most appropriate response? a. "Don't worry about it, esophageal cancers have a low incidence in men." b. "You should not be concerned about esophageal cancer at your age." c. "You should consider limiting your alcohol intake to two drinks per day." d. "Increasing the fiber and protein in your diet can help you lower your risk."

Ans. C A Men have a rate three times that of women. B The risk increases with age, with the peak between 70 and 80 years. C "You should consider limiting your alcohol intake to two drinks per day. Long-term alcohol intake increases your risk for esophageal cancer." D Although fiber and protein are important for the diet, their intake does not affect the risk of esophageal cancer.

When auscultating the heart of a patient with pericarditis, the nurse expects to hear which sound? a. A systolic murmur b. An S3 heart sound c. A friction rub d. An S4 heart sound

Ans. C A Most systolic murmurs are caused by obstruction of the outflow of the semilunar valves or by incompetent AV valves. B An S3 heart sound occurs when there is heart failure. C Two classic findings of pericarditis are pericardial friction rub and chest pain. D An S4 heart sound occurs when there is hypertrophy of the ventricle.

The nurse observes a patient rocking back and forth on the examination table in pain. Based on the patient's history, the nurse suspects kidney stones. What additional examination technique does the nurse perform to confirm this suspicion? a. Palpating the flank area for rebound tenderness b. Percussing the bladder for fullness c. Percussing the costal vertebral margins for tenderness d. Palpating McBurney point for tenderness

Ans. C A Palpating the flank area for rebound tenderness is the correct location (flank area), but rebound tenderness is performed on the abdomen to detect peritoneal inflammation. B Percussing the bladder for fullness would provide data about bladder distention, but is not a technique to detect for kidney stones. C Percussing the costal vertebral margins for tenderness is the appropriate technique to detect kidney stones. The nurse recognizes the relationship between the history and the observation with further assessment techniques needed to confirm kidney stones. D Palpating McBurney point for tenderness is a technique to detect appendicitis.

A nurse is assessing a patient's peripheral circulation. Which finding indicates venous insufficiency of this patient's legs? a. Paresthesias and weak, thin peripheral pulses b. Leg pain that can be relieved by walking c. Edema that is worse at the end of the day d. Leg pain that increases when the legs are lowered

Ans. C A Paresthesias and weak, thin peripheral pulses are characteristics of arterial insufficiencies rather than venous. B Pain caused by arterial insufficiency gets worse by walking, because walking requires additional arterial blood. C Dependent edema is an indication of venous insufficiency. D Arterial pain is relieved by lowering the leg and aggravated by elevating the legs.

A nurse inspects a patient's hands and notices clubbing of the fingers. The nurse correlates this finding with what condition? a. Pulmonary infection b. Trauma to the thorax c. Chronic hypoxemia d. Allergic reaction

Ans. C A Pulmonary infection is acute and not associated with chronic hypoxia. B Trauma to the thorax is acute and not associated with chronic hypoxia. C Clubbing develops due to chronic hypoxemia, which occurs in chronic obstructive pulmonary disease. D Allergic reaction is acute and not associated with chronic hypoxia.

Which pulse may be a challenge for a nurse to palpate? a. Temporal b. Femoral c. Popliteal d. Dorsalis pedis

Ans. C A The temporal pulse is palpated over the temporal bone on each side of the head. B For the femoral pulse, palpate below the inguinal ligament, midway between the symphysis pubis and anterior superior iliac. C For the popliteal pulse, palpate the popliteal artery behind the knee in the popliteal fossa to assess perfusion. This pulse may be difficult to find. D For the dorsalis pedis pulse, palpate on the inner aspect of the ankle below and slightly behind the medial malleolus (ankle bone).

After two separate office visits, the nurse suspects that a patient is developing Stage 1 hypertension based on which consecutive blood pressure readings? a. Visit 1, 118/78; Visit 2, 116/76 b. Visit 1, 130/88; Visit 2, 134/88 c. Visit 1, 144/92; Visit 2, 150/90 d. Visit 1, 162/100; Visit 2, 166/104

Ans. C A These readings are within normal limits. B These readings are prehypertension because the systolic pressures are 120 to 139 and diastolic pressures are greater than 80 mm Hg. C These readings are stage 1 because the systolic pressures are 140 to 159 and diastolic pressures are 90 mm Hg or greater. D These readings are stage 2 because the systolic pressures are greater than 160 and diastolic pressures are 100 mm Hg or greater.

To correctly percuss the abdomen, a nurse places the distal aspect of the middle finger of the nondominant hand against the skin of the abdomen, and the other fingers are spread apart and slightly lifted off the skin. How does the nurse use the fingers of the dominant hand? a. The pad of the middle finger strikes the distal interphalangeal joint of the middle finger touching the skin of the abdomen. b. The tip of the middle finger strikes the nail of the middle finger touching the skin of the abdomen. c. The tip of the middle finger strikes the distal interphalangeal joint of the middle finger touching the skin of the abdomen. d. The pads of the index and middle fingers strike the nail of the middle finger touching the skin of the abdomen.

Ans. C A This description is incorrect because the tip of the finger is used rather than the pad. B This description is incorrect because the distal joint is struck rather than the nail. C This is the correct technique. D This description is incorrect because the tip of the middle finger strikes the distal joint.

A patient complains of shortness of breath and having to sleep on three pillows to breathe comfortably at night. During the nurse's examination, what findings will suggest that the cause of this patient's dyspnea is due to heart disease rather than respiratory disease? a. Increased anteroposterior diameter b. Clubbing of the fingers c. Bilateral peripheral edema d. Increased tactile fremitus

Ans. C A This is seen with lung hyperinflation and may be associated with emphysema. B This is associated with chronic hypoxia and may be associated with cystic fibrosis or chronic obstructive pulmonary disease. C This indicates heart failure; dyspnea occurs because the heart cannot adequately perfuse the lungs. D This occurs when vibrations are enhanced and is associated with consolidation that may occur in pneumonia or tumor.

A nurse is assessing for vocal (tactile) fremitus on a patient with pulmonary edema. Which is the appropriate technique to use? a. Systematically percuss the posterior chest wall following the same pattern that is used for auscultation and listen for a change in tone from resonant to dull. b. Place the pads of the fingers on the right and left thoraces and palpate the texture and consistency of the skin feeling for a crackly sensation under the fingers. c. Place the palms of the hands on the right and left thoraces, ask the patient to say "99," and feel for vibrations. d. Place both thumbs on either side of the patient's spinal processes, extend fingers laterally, ask the patient to take a deep breath, and feel for vibrations.

Ans. C A This is the technique for percussing the thorax for tones. B This is the technique for detecting crepitus. C This is the correct technique for vocal fremitus. D This is not the correct technique.

A nurse determines that a patient's jugular venous pressure is 3.5 inches. What additional data does the nurse expect to find? a. Weight loss b. Tented skin turgor c. Peripheral edema d. Capillary refill greater than 5 seconds

Ans. C A Weight loss occurs with loss of fluid rather than fluid overload. B Tented skin turgor occurs with fluid loss rather than fluid overload. C The pressure should not rise more than 1 inch (2.5 cm) above the sternal angle. A pressure of 3.5 inches indicates fluid volume excess, which causes peripheral edema due to excessive fluid in blood vessels. D Capillary refill greater than 5 seconds occurs with arterial insufficiency rather than fluid overload.

When palpating the abdomen to determine a floating mass, a nurse presses on the abdomen at a 90-degree angle with the fingertips. Which finding indicates a mass? a. An increase in abdominal girth b. A complaint from the patient of a dull pain in the flank area c. A freely movable mass will float upward and touch the fingertips d. Fluid in the abdomen will shift upward and touch the fingertips

Ans. C A An increase in abdominal girth does not occur as a result of ballottement. B A complaint from the patient of a dull pain in the flank area is not an expected finding. C A freely movable mass floating upward and touching the fingertips is the expected finding (ballottement). D Fluid in the abdomen shifting upward and touching the fingertips does not occur; it is the mass on the abdomen that shifts upward.

Where does a nurse place a stethoscope to auscultate the mitral valve area? Choose the letter that corresponds to the correct stethoscope placement. a. A b. B c. D d. E

Ans. D A A is the location of the aortic valve area—second intercostal space, right sternal border. B B is the location of the pulmonic valve area—fifty-second intercostal space, left sternal border. C D is the location of the tricuspid valve area—fourth intercostal space, left sternal border. D E is the location of the mitral valve area—the fifth intercostal space, midclavicular line.

During a symptom analysis, a patient describes his productive cough and states his sputum is thick and yellow. Based on these data, the nurse suspects which factor as the cause of these symptoms? a. Virus b. Allergy c. Fungus d. Bacteria

Ans. D A A virus usually produces a nonproductive cough. B An allergy usually produces clear sputum. C A fungus usually produces few symptoms. The sputum used to diagnose the fungus is obtained from tracheal aspiration rather than the patient coughing up the sputum. D Bacteria usually produce sputum that is yellow or green in color.

A nurse expects which finding when assessing the abdomen of a patient who has been unable to void for 12 hours? a. Absent bowel sounds b. Hyperactive bowel sounds c. Tympanic tones over the lower abdomen d. Dull tones over the suprapubic area

Ans. D A Absent bowel sounds is incorrect because the bowel sounds would not be affected by a full bladder. B Hyperactive bowel sounds is incorrect because the bowel sounds would not be affected by a full bladder. C Tympanic tones over the lower abdomen is incorrect because tympany sound is created by gas in the abdomen. D Dull tones over the suprapubic area would be found. The urine in the bladder would create a dull sound when the bladder is percussed similar to the sounds when an abdominal mass is present.

Using deep palpation of a patient's epigastrium, a nurse feels a rhythmic pulsation of the aorta. Based on this finding, what is the nurse's most appropriate response? a. Auscultate this area using the bell of the stethoscope. b. Percuss the area for tones. c. Ask the patient if there is pain in this area. d. Document this as a normal finding.

Ans. D A Auscultating this area using the bell of the stethoscope is not necessary because this is a normal finding. Vascular sounds are usually not heard. B Percussing the area for tones is not necessary because this is a normal finding. C Asking the patient if there is pain in this area is not necessary because this is a normal finding. D Document this as a normal finding. The aorta is often palpable at the epigastrium.

A patient reports having leg pain while walking that is relieved with rest. Based on these data, the nurse expects which finding on inspection and palpation of this patient? a. 1+ edema of the feet and ankles bilaterally b. The circumference of the right leg is larger than the left leg c. Patchy petechiae and purpura of the lower extremities d. Cool feet with capillary refill of toes greater than 3 seconds

Ans. D A Edema of 1+ of the feet and ankles bilaterally is an indication of a venous problem rather than an arterial problem. B When one leg is larger in circumference than the other, it could be due to lymphedema or a deep vein thrombosis. C Petechiae and purpura of the lower extremities indicate a bleeding problem, such as low platelets, rather than an arterial problem. D The pain while walking that is relieved by rest is called intermittent claudication and is an indication of arterial insufficiency. Cool feet and prolonged capillary refill also occur due to arterial insufficiency.

When inspecting a patient's abdomen, which finding does the nurse note as normal? a. Engorgement of veins around the umbilicus b. Sudden bulge at the umbilicus when coughing c. Visible peristalsis in all quadrants d. Silver-white striae extending from the umbilicus

Ans. D A Engorgement of veins around the umbilicus is an abnormal finding. B Sudden bulge at the umbilicus when coughing is an abnormal finding and may indicate a hernia. C Visible peristalsis in all quadrants is an abnormal finding. D Silver-white striae extending from the umbilicus is a normal finding, particularly in women who have been pregnant or in any adult who has lost weight after having an obese abdomen.

While taking a history, a nurse learns that this patient experiences shortness of breath (dyspnea). If the cause of the dyspnea is a cardiovascular problem, the nurse expects which abnormal finding on examination? a. Flat jugular neck veins b. Red, shiny skin on the legs c. Weak, thready peripheral pulses d. Edema of the feet and ankles

Ans. D A Flat jugular veins indicate a fluid deficit, which is not associated with dyspnea. B Red, shiny skin on the legs is associated with peripheral arterial disease and is not associated with dyspnea. C Weak, thready peripheral pulses indicate fluid deficit, which is not associated with dyspnea. D This patient may have heart failure. Edema of the feet occurs with right ventricular heart failure. Dyspnea occurs with left ventricular heart failure.

A nurse in the emergency department is assessing a patient with a moderate left pneumothorax. What does this nurse expect to find during the respiratory examination? a. Increased fremitus over the left chest b. Tracheal deviation to the left side c. Hyporesonant percussion tones over the left chest d. Distant to absent breath sounds over the left chest

Ans. D A Increased fremitus occurs over lung consolidation as in lobar pneumonia or tumor. B If this patient had a tension pneumothorax, the trachea would deviate to the right. C Hyperresonant percussion tones are heard when the lung is overinflated as in emphysema. D The air separating the lung from the chest where the nurse is auscultating creates distant to absent breath sounds.

During a history, a nurse notices a patient is short of breath, is using pursed-lip breathing, and maintains a tripod position. Based on these data, what abnormal finding should the nurse expect to find during the examination? a. Increased tactile fremitus b. Inspiratory and expiratory wheezing c. Tracheal deviation d. An increased anteroposterior diameter

Ans. D A Increased tactile fremitus occurs when vibrations are enhanced and is associated with consolidation that may occur in pneumonia or tumor. B Inspiratory and expiratory wheezing is associated with asthma. C Tracheal deviation is associated with tension pneumothorax. D An increased anteroposterior diameter is consistent with emphysema.

Which patient has the lowest risk for colon cancer? a. Patient A is 50 years old, is obese, and has type 2 diabetes mellitus. b. Patient B is 60 years old, has alcoholism, and smokes a pack of cigarettes daily. c. Patient C is 55 years old, has ulcerative colitis, and inflammatory bowel disease. d. Patient D is 45 years old and has diverticulosis.

Ans. D A Patient A has three risk factors for colon cancer. B Patient B has three risk factors for colon cancer. C Patient C has two risk factors for colon cancer. D Patient D has the lowest risk of colon cancer. Ninety percent of colon cancers occur in adults older than 50 years of age. Although this patient does have a disorder of the colon, it is not linked to an increased risk of colon cancer.

A nurse is completing a symptom analysis with a patient complaining of chest pain. When asked what makes the chest pain worse, the patient reports that coughing and sneezing increase the chest pain. Based on these data, what does the nurse suspect as the cause of this patient's chest pain? a. Stable angina b. Esophageal reflux disease c. Mitral valve prolapse d. Costochondritis

Ans. D A Physical exertion, emotional stress, and cold worsen the chest pain associated with stable angina. B A spicy or acidic meal, alcohol, or lying supine may worsen the chest pain associated with esophageal reflux. C Only occasional position changes worsen the chest pain associated with mitral valve prolapse. D Coughing, deep breathing, laughing, and sneezing worsen the chest pain associated with costochondritis.

How is the first heart sound (S1) created? a. Pulmonic and tricuspid valves close. b. Mitral and aortic valves close. c. Aortic and pulmonic valves close. d. Mitral and tricuspid valves close.

Ans. D A The pulmonic and tricuspid valves are the valves of the right side of the heart, and they do not close simultaneously in the cardiac cycle. B The mitral and aortic valves are the valves of the left side of the heart, and they do not close simultaneously in the cardiac cycle. C The aortic and pulmonic valves are the semilunar valves that create the second heart sound. D The first heart sound (S1) is made by the closing of the mitral (M1) and tricuspid (T1) valves.

A patient reports having frequent heartburn. Which question does the nurse ask in response to this information? a. "Has your abdomen been distended when you feel the heartburn?" b. "What have you eaten in the last 24 hours?" c. "Is there a history of heart disease in your family?" d. "How long after eating do you have heartburn?"

Ans. D A The question "Has your abdomen been distended when you feel the heartburn?" is not related to the heartburn. Distention usually is related to intestinal obstruction or liver disease. B The question "What have you eaten in the last 24 hours?" relates more to gastroenteritis. Indigestion is usually caused by food eaten in the last meal rather than in the last 24 hours. C The question "Is there a history of heart disease in your family?" points to myocardial ischemia. Although heartburn may be a symptom of myocardial ischemia, asking the patient about the family history is not relevant in this case. D Asking "How long after eating do you have heartburn?" can aid in determining if the patient has gastroesophageal reflux disease or a hiatal hernia. Both are common disorders that cause indigestion a few hours after meals.

The patient reports right lower quadrant (RLQ) pain that is worse with coughing. Based on the patient's history, the nurse suspects appendicitis. What additional examination technique does the nurse perform to confirm this suspicion? a. Placing the hand over the lower right thigh and asking the patient to flex the knee while pushing down on the knee to resist it and noting if the patient complains of pain b. Palpating deeply a point of the abdomen, located halfway between the umbilicus and the left anterior iliac crest c. Asking the patient to flex the right hip and knee to 90 degrees, then abducting the leg and noting if the patient complains of pain d. Pressing down in an area away from the RLQ at a 90-degree angle to the abdomen, then releasing the fingers quickly and noting any complaint of pain

Ans. D A This is an incorrect description of the iliopsoas muscle test. B This is an incorrect description of the testing for McBurney point. McBurney point is located to the right of the umbilicus. C This is an incorrect description of the obturator muscle test. D This describes rebound tenderness, which is performed to detect peritoneal inflammation.

How does a nurse accurately palpate carotid pulses? a. Two fingers of each hand are placed firmly over the right and left temples at the same time. b. One finger is placed gently in the space between the biceps and triceps muscles. c. Two fingers are placed at the thumb side of the forearm at the wrist. d. One finger is placed along the right and then the left medial sternocleidomastoid muscle.

Ans. D A Two fingers of each hand placed firmly over the right and left temples at the same time is the correct procedure for palpating the temporal pulse. B One finger placed gently in the space between the biceps and triceps muscles is the correct procedure for palpating the brachial pulse. C Two fingers placed at the thumb side of the forearm at the wrist is the correct procedure for palpating the radial pulse. D One finger placed along the right and then the left medial sternocleidomastoid muscle is the correct procedure for palpating the carotid pulses, checking each side separately.

Where does a nurse expect to hear bronchovesicular lung sounds in a healthy adult? a. In the lower lobes b. Over the trachea c. In the apices of the lungs d. Near the sternal border

Ans. D A Vesicular breath sounds are normally heard in the lower lobes. B Bronchial sounds are normally heard over the trachea. C Vesicular breath sounds are normally heard in the apices of the lungs. D Bronchovesicular breath sounds are normally heard over the central area of the anterior thorax around the sternal border.

The nurse records the following general inspection findings on a patient: "41-year-old Hispanic male in no distress; very thin, skin tone slightly jaundiced, disheveled appearance, and appears older than stated age. Patient with flat affect and makes minimal eye contact." What additional information should be added to this general inspection? 1. Body movement 2. Family history 3. Estimated size of his liver 4. Palpation of pulses

Answer: 1. Body movement

The nurse obtains vital signs on a 42-year-old man having his annual physical examination. He has no medical conditions and states that his health is excellent. His blood pressure appears as 62/40 using an automated blood pressure device. Which action by the nurse is most appropriate? 1. Obtain a different cuff and take the blood pressure again. 2. Take the blood pressure again using the auscultation method. 3. Place the patient in a supine position and take the pressure on the leg. 4. Record the blood pressure and continue with the examination.

Answer: 2Take the blood pressure again using the auscultation method

A 62-year-old patient tells the nurse that he has recently had frequent fainting spells. After palpating the radial pulse, 13 pulsations are counted in 15 seconds. The nurse determines that the patient has a pulse rate of 52, with a regularly irregular rhythm. What is the most appropriate action for the nurse to take at this time? 1. Reassess the pulse rate after he walks around the room for several minutes. 2. Reassess the pulse rate for 15 seconds using the carotid artery. 3. Take an apical pulse for 5 full minutes, counting the number of skipped beats. 4. Palpate the pulse for a full minute and note whether there is a pattern to the irregularity.

Answer: 4. Palpate the pulse for a full minute and note whether there is a pattern to the irregularity.

Upon further questioning by the nurse, Ms. Yong reports that this buckling of her knee has occurred several times previously. What additional information is most important for the nurse to obtain? Whether Ms. Yong takes any pain medication for her knee pain. This information is useful but is of less priority than other information. The date Ms. Yong last had her bone density measured. This information is not related to Ms. Yong's immediate knee problem and can be obtained at a later time. Any recent history of trauma or injury to the affected knee. When the knee gives way suddenly, the nurse should determine if the client has experienced trauma to the area. This information is important in determining the cause of the symptom. How frequently Ms. Yong performs weight-bearing exercises. This information is useful in developing a teaching plan for prevention of further damage to the knee but is of less immediate priority than other information.

Any recent history of trauma or injury to the affected knee. When the knee gives way suddenly, the nurse should determine if the client has experienced trauma to the area. This information is important in determining the cause of the symptom.

A 48-year-old woman asks the nurse how to best protect herself from excessive sun exposure while at the beach. Which response would be most appropriate? 1"Limit your time in the sun to 5 minutes every hour." 2"Wear a wet suit that covers your arms and legs." 3"Apply a waterproof sunscreen (SPF 15 or higher) to exposed skin surfaces; reapply at least every 2 hours." 4"Apply sunscreen with a minimum SPF 50 to all skin surfaces before leaving for the beach; this will provide all-day coverage."

Apply a waterproof sunscreen spf 15 or higher to exposed skin surfaces, really at least every 2 hours

The RN tests cranial nerve XI by asking the client to shrug her shoulders. What action should the RN perform? Internally rotate each of the client's shoulders. Internally rotating the shoulders is not useful in testing cranial nerve XI. Observe the movement of the client's clavicles. Observing the movement of the clavicles is not useful in testing cranial nerve XI. Apply resistance to the client's shoulders. The RN should test the client's ability to shrug her shoulders against resistance with equal strength bilaterally. Slowly elevate both of the client's arms. Elevation of the client's arms is not useful in testing cranial nerve XI.

Apply resistance to the client's shoulders. The RN should test the client's ability to shrug her shoulders against resistance with equal strength bilaterally.

The nurse sees no evidence of pallor. What follow-up assessment should the nurse perform after observing the facial flushing? Observe the color of the sclerae. Rapid facial and neck flushing are not associated with conditions that would cause a change in the color of the sclerae of the eyes. Measure the oxygen saturation. Rapid facial and neck flushing are not manifestations of a change in oxygen saturation. Check for loss of skin integrity. Rapid facial and neck flushing are not associated with impaired skin integrity. If a cut or wound is present, the nurse should observe for inflammation directly around the wound site. Ask about any feelings of anxiety. Rapid facial and neck flushing are often the result of vasodilation secondary to stress or anxiety. The nurse talks calmly with Amanda to help her relax while continuing the assessment.

Ask about any feelings of anxiety. Rapid facial and neck flushing are often the result of vasodilation secondary to stress or anxiety. The nurse talks calmly with Amanda to help her relax while continuing the assessment.

After expressing concern about the bruises on Amanda's abdomen, how should the nurse begin the abuse assessment? Determine if the client is sexually active. This information may be relevant later in the assessment, but it is not immediately relevant to the finding of abdominal bruising. Ask the client if someone else caused the injuries. It is appropriate to first ask a direct question to elicit information about possible abuse. If the client is reluctant to respond to a direct question about possible abuse, the nurse may then choose to use an indirect approach to encourage further verbalization. Encourage the client to describe her family structure. This information may be relevant later in the assessment, but it is not immediately relevant to the finding of abdominal bruising. Advise the client of her right to legal counsel during the interview. Since the client is not being accused or charged with any criminal activity, there is no need to advise her about the right to legal counsel. Amanda tells the nurse that her boyfriend is a 21-year-old that she met in an online chat room. After they go out to eat, he complains that she is fat and sometimes punches her in the stomach so that she will throw up her dinner and remember to eat less the next time.

Ask the client if someone else caused the injuries. It is appropriate to first ask a direct question to elicit information about possible abuse. If the client is reluctant to respond to a direct question about possible abuse, the nurse may then choose to use an indirect approach to encourage further verbalization.

In response to the client's statement that she hurts a lot, what action should the nurse take first? Explain to the client that post-operative pain is normal. This is a non-therapeutic response to the client's current situation. Ask the client to describe her pain location and intensity. The nurse should begin by gathering further data about the pain, including location, intensity, and quality. Ask the client if she has passed gas since surgery. This question may be relevant, but other actions have priority. Assess the client's heart rate and blood pressure. This action may be warranted, but it is not the first action the nurse should implement when the client reports pain.

Ask the client to describe her pain location and intensity. The nurse should begin by gathering further data about the pain, including location, intensity, and quality.

To assess muscle strength in the foot, the nurse next asks the client to dorsiflex her foot. The client points her toes downward. What action should the nurse take next? Apply gentle pressure over the client's toes. This action is not useful in assessing muscle strength in the foot. Place one hand on the bottom of the client's foot. Another action should be taken before applying resistance to the bottom of the foot. Ask the client to flex her foot upward. Dorsiflexion involves pointing the toes upward, so the nurse should use language the client can understand to first position the foot correctly. Help the client evert and then invert her foot. This action allows assessment of range of motion rather than muscle strength in the foot.

Ask the client to flex her foot upward. Dorsiflexion involves pointing the toes upward, so the nurse should use language the client can understand to first position the foot correctly.

To assess chest excursion, what should the nurse do next? Ask the client to inhale deeply. To assess chest excursion, the nurse observes the movement of the hands placed on the lower posterior thorax as the client inhales. Encourage the client to cough. Encouraging the client to cough does not allow the nurse to assess for chest excursion. Tap lightly over the middle finger. This procedure may be used during percussion, but it is not useful when observing chest excursion. Instruct the client to hold his breath. Holding the breath will prevent the nurse from observing chest excursion.

Ask the client to inhale deeply. To assess chest excursion, the nurse observes the movement of the hands placed on the lower posterior thorax as the client inhales

While assessing the spine, the nurse assesses Ms. Yong's low back pain further. Which action will help determine the cause of her pain? Ask the client to lie supine and raise one leg, keeping it straight. If sciatic pain occurs when raising a straight leg, the nurse should suspect the presence of a herniated disc. Watch the client while she stands upright and slowly squats down. This action is not useful in differentiating the cause of the client's back pain. Instruct the client to balance on one foot with her arms at her sides. This action is not useful in differentiating the cause of the client's back pain. Help the client to a prone position, rotating both legs inward. This action is not useful in differentiating the cause of the client's back pain.

Ask the client to lie supine and raise one leg, keeping it straight. If sciatic pain occurs when raising a straight leg, the nurse should suspect the presence of a herniated disc.

To begin the assessment for vocal fremitus, what should the nurse do? Place one hand over each scapula. Vocal fremitus is assessed systematically, avoiding areas such as the scapulae where vibrations will not be transmitted. Locate the posterior axillary line. Vocal fremitus is assessed systematically, but the location of the posterior axillary line is not necessary. Assist the client to lie back in the bed. Because vocal fremitus can be assessed posteriorly, as well as anteriorly, there is no need to assist the client to lie back in the bed. Ask the client to repeat a phrase aloud. Vocal fremitus is assessed by palpating for vibrations on the thoracic wall beginning at the apex and ending at the base of the lungs while the client repeats a word or phrase aloud.

Ask the client to repeat a phrase aloud. Vocal fremitus is assessed by palpating for vibrations on the thoracic wall beginning at the apex and ending at the base of the lungs while the client repeats a word or phrase aloud.

Since Mrs. Green is lying in bed, which action should the RN take to observe small muscle movement and coordination? Ask the client to touch her thumb to each finger. While the client touches her thumb to each finger, the RN observes for smooth, coordinated movement of the small muscles. Stroke the lateral sides of the sole of each foot. This action is used to test for the Babinski reflex and is not useful in assessing small muscle movement. Use a reflex hammer to elicit arm movement. This action is useful in testing deep tendon reflexes, but it is not useful in assessing small muscle movement. Assist the client to sit on the side of the bed. It is not necessary to assist the client to a sitting position to assess small muscle movement.

Ask the client to touch her thumb to each finger. While the client touches her thumb to each finger, the RN observes for smooth, coordinated movement of the small muscles.

During an assessment of a young adult, the nurse notes that the patient's shoulders are uneven. Which further examination would the nurse perform for further data?

Ask the patient to bend forward at the waist while the nurse checks the alignment of the patient's vertebrae.

What actions should the nurse take in response to this finding? Document the assessment as within normal limits in the assessment record. The RN does not have sufficient data to document that the findings are within normal limits. Note the amount of pressure needed to occlude the nodes and prevent movement. This action is not taken when palpating lymph nodes. Apply pressure more firmly until all the nodes in the area can be palpated. All of the nodes may not be palpable. Excessive pressure may push the nodes downward and prevent effective palpation. Assess the nodes further for consistency and any palpable matting. If lymph nodes are palpable, the nurse should assess for mobility, size, shape, consistency, and whether the nodes are discrete or matted. These findings should then be documented in the assessment record. Ask Lourdes if any there is any tenderness upon palpation of the nodes. If lymph nodes are palpable, the nurse should assess for tenderness. This finding should then be documented in the assessment record.

Assess the nodes further for consistency and any palpable matting. If lymph nodes are palpable, the nurse should assess for mobility, size, shape, consistency, and whether the nodes are discrete or matted. These findings should then be documented in the assessment record. Ask Lourdes if any there is any tenderness upon palpation of the nodes. If lymph nodes are palpable, the nurse should assess for tenderness. This finding should then be documented in the assessment record.

The nurse uses the SBAR method when communicating with the primary HCP. Which are components of the SBAR method? Assessment. Communication of assessment findings is the second step of the SBAR method. Response. Response is not included when using the SBAR method. Recommendation. Communication of recommendations is the final step when using the SBAR method. Action. Action is not included when using the SBAR method. Situation. Description of the situation is the first step when using the SBAR method.

Assessment. Communication of assessment findings is the second step of the SBAR method. Recommendation. Communication of recommendations is the final step when using the SBAR method. Situation. Description of the situation is the first step when using the SBAR method.

The registered nurse (RN) uses the mini-mental state examination (MMSE) when assessing a client for admission to an assisted living facility. Which finding is the RN assessing when requesting the client to count by 7s? Recall of information. Orientation to surroundings. Attention to details. Ability to follow complex commands.

Attention to details. When conducting the MMSE and having the client count backwards by 7s; this evaluates their ability to do simple calculations and is specific to the client's attention to detail and staying focus and not getting distracted by external stimuli.

What assessment should the nurse perform? Palpate for changes in vocal fremitus. This assessment will not provide the most useful information at this time. Auscultate breath sounds bilaterally. Thick, purulent sputum is a sign of an infectious process. The nurse should auscultate the client's lungs to determine if a change from the previous assessment has occurred, reflecting a worsening of the client's condition. Observe the thoracic diameter ratio. This assessment will not change from the initial assessment. Percuss for diaphragmatic excursion. This assessment will not provide the most useful information at this time.

Auscultate breath sounds bilaterally. Thick, purulent sputum is a sign of an infectious process. The nurse should auscultate the client's lungs to determine if a change from the previous assessment has occurred, reflecting a worsening of the client's condition.

After palpating an irregular pulse rhythm at the left radial pulse site, what action should the nurse take to confirm the client's heart rate? Palpate both radial pulses simultaneously. This is not the best method to confirm the heart rate of a client with an irregular cardiac rhythm. Auscultate the apical pulse for 1 minute. Auscultation of the apical pulse is the most accurate method to determine heart rate and rhythm because the nurse is listening directly over the heart, rather than depending on the transmission of the pulse to a distal site, such as the radial pulse site. Compare the ulnar pulse to the radial pulse. The ulnar pulse is often difficult to palpate and is not the best site to use to assess cardiac rate and rhythm. Ask the client if he experiences palpitations. This action does not provide data that will confirm the client's heart rate. Palpitations may occur with either a regular or irregular cardiac rhythm.

Auscultate the apical pulse for 1 minute. Auscultation of the apical pulse is the most accurate method to determine heart rate and rhythm because the nurse is listening directly over the heart, rather than depending on the transmission of the pulse to a distal site, such as the radial pulse site.

What action should the nurse take? Encourage the client to cough and then auscultate these lung fields again. This action is not indicated at this time. Stop the assessment immediately and administer a PRN dose of an inhaler. This action is not indicated at this time. Continue the assessment after documenting the location of these abnormal sounds. Vesicular breath sounds are normally heard over the peripheral lung fields where air flows through smaller bronchioles and alveoli. Auscultate the lower lung fields to determine the presence of any adventitious sounds. Since vesicular breath sounds are normally heard in the peripheral lung fields, the nurse should continue to auscultate the remaining lung fields, listening for any abnormal, or adventitious, sounds.

Auscultate the lower lung fields to determine the presence of any adventitious sounds. Since vesicular breath sounds are normally heard in the peripheral lung fields, the nurse should continue to auscultate the remaining lung fields, listening for any abnormal, or adventitious, sounds.

115. In obtaining a history on a 74-year-old patient the nurse notes that he drinks alcohol daily and that he has noticed a tremor in his hands that affects his ability to hold things. With this information, what should the nurse's response be? A) "Does your family know you are drinking every day?" B) "Does the tremor change when you drink the alcohol?" C) "We'll do some tests to see what is causing the tremor." D) "You really shouldn't drink so much alcohol; it may be causing your tremor."

B) "Does the tremor change when you drink the alcohol?" Page: 632. Intention tremor/ senile tremor is relieved by alcohol, although this is not a recommended treatment. The nurse should assess whether the person is abusing alcohol in an effort to relieve the tremor.

31. During a mental status assessment, which question by the nurse would best assess a person's judgment? A) "Do you feel that you are being watched, followed, or controlled?" B) "Tell me about what you plan to do once you are discharged from the hospital." C) "What does the statement, 'People in glass houses shouldn't throw stones,' mean to you?" D) "What would you do if you found a stamped, addressed envelope lying on the sidewalk?"

B) "Tell me about what you plan to do once you are discharged from the hospital." Pages: 76-77. A person exercises judgment when he or she can compare and evaluate the alternatives in a situation and reach an appropriate course of action. Rather than testing the person's response to a hypothetical situation (as illustrated in the option with the envelope), the nurse should be more interested in the person's judgment about daily or long-term goals, the likelihood of acting in response to delusions or hallucinations and the capacity for violent or suicidal behavior.

108. During an abdominal assessment, the nurse is unable to hear bowel sounds in a patient's abdomen. Before reporting this finding as "silent bowel sounds" the nurse should listen for at least: A) 1 minute. B) 5 minutes. C) 10 minutes. D) 2 minutes in each quadrant.

B) 5 minutes. Pages: 539-540. Absent bowel sounds are rare. The nurse must listen for 5 minutes before deciding bowel sounds are completely absent.

107. During an abdominal assessment, the nurse would consider which of these findings as normal? A) The presence of a bruit in the femoral area B) A tympanic percussion note in the umbilical region C) A palpable spleen between the ninth and eleventh ribs in the left midaxillary line D) A dull percussion note in the left upper quadrant at the midclavicular line

B) A tympanic percussion note in the umbilical region Pages: 539-540. Tympany should predominate in all four quadrants of the abdomen because air in the intestines rises to the surface when the person is supine. Vascular bruits are not usually present. Normally the spleen is not palpable. Dullness would not be found in the area of lung resonance (left upper quadrant at the midclavicular line).

113. The nurse is assessing the joints of a woman who has stated, "I have a long family history of arthritis, and my joints hurt." The nurse suspects that she has osteoarthritis. Which of these are symptoms of osteoarthritis? Select all that apply. A) Symmetric joint involvement B) Asymmetric joint involvement C) Pain with motion of affected joints D) Affected joints are swollen with hard, bony protuberances E) Affected joints may have heat, redness, and swelling

B) Asymmetric joint involvement C) Pain with motion of affected joints D) Affected joints are swollen with hard, bony protuberances Page: 608. In osteoarthritis, asymmetric joint involvement commonly affects hands, knees, hips, and lumbar and cervical segments of the spine. Affected joints have stiffness, swelling with hard bony protuberances, pain with motion, and limitation of motion. The other options reflect signs of rheumatoid arthritis.

40. The nurse is examining an infant and prepares to elicit the Moro reflex at which time during the examination? A) When the infant is sleeping B) At the end of the examination C) Before auscultation of the thorax D) Halfway through the examination

B) At the end of the examination Page: 123. Elicit the Moro or "startle" reflex at the end of the examination because it may cause the infant to cry.

63. During an assessment of an infant, the nurse notes that the fontanels are depressed and sunken. The nurse suspects which condition? A) Rickets B) Dehydration C) Mental retardation D) Increased intracranial pressure

B) Dehydration Pages: 265-266. Depressed and sunken fontanels occur with dehydration or malnutrition. Mental retardation and rickets have no effect on fontanels. Increased intracranial pressure would cause tense or bulging, and possibly pulsating fontanels.

53. An elderly woman is brought to the emergency department after being found lying on the kitchen floor 2 days, and she is extremely dehydrated. What would the nurse expect to see upon examination? A) Smooth mucous membranes and lips B) Dry mucous membranes and cracked lips C) Pale mucous membranes D) White patches on the mucous membranes

B) Dry mucous membranes and cracked lips Page: 215. With dehydration, mucous membranes look dry and lips look parched and cracked. The other responses are not found in dehydration.

49. The nurse needs to perform anthropometric measures of an 80-year-old man who is confined to a wheelchair. Which of the following is true in this situation? A) Changes in fat distribution will affect the waist-to-hip ratio. B) Height measurements may not be accurate because of changes in bone. C) Declining muscle mass will affect the triceps skinfold measure. D) Mid-arm circumference is difficult to obtain because of loss of skin elasticity.

B) Height measurements may not be accurate because of changes in bone. Page: 191. Height measures may not be accurate in individuals confined to a bed or wheelchair or those over 60 years of age because of osteoporotic changes.

123. During an assessment of a 22-year-old woman who has a head injury from a car accident 4 hours ago, the nurse notices the following change: pupils were equal, but now the right pupil is fully dilated and nonreactive, left pupil is 4 mm and reacts to light. What does finding this suggest? A) Injury to the right eye B) Increased intracranial pressure C) Test was not performed accurately D) Normal response after a head injury

B) Increased intracranial pressure Pages: 662-663. In a brain-injured person, a sudden, unilateral, dilated, and nonreactive pupil is ominous. Cranial nerve III runs parallel to the brainstem. When increasing intracranial pressure pushes the brainstem down (uncal herniation), it puts pressure on cranial nerve III, causing pupil dilation. The other responses are incorrect.

29. The nurse is preparing to do a mental status examination. Which statement is true regarding the mental status examination? A) A patient's family is the best resource for information about the patient's coping skills. B) It is usually sufficient to gather mental status information during the health history interview. C) It takes an enormous amount of extra time to integrate the mental status examination into the health history interview. D) It is usually necessary to perform a complete mental status examination to get a good idea of the patient's level of functioning.

B) It is usually sufficient to gather mental status information during the health history interview. Page: 73. The full mental status examination is a systematic check of emotional and cognitive functioning. The steps described here, though, rarely need to be taken in their entirety. Usually, one can assess mental status through the context of the health history interview.

16. The nurse is reviewing concepts of cultural aspects of pain. Which statement is true regarding pain? A) All patients will behave the same way when in pain. B) Just as patients vary in their perceptions of pain, so will they vary in their expressions of pain. C) Cultural norms have very little to do with pain tolerance, because pain tolerance is always biologically determined. D) A patient's expression of pain is largely dependent on the amount of tissue injury associated with the pain.

B) Just as patients vary in their perceptions of pain, so will they vary in their expressions of pain. Page: 25 In addition to expecting variations in pain perception and tolerance, the nurse should expect variations in the expression of pain. It is well known that individuals turn to their social environment for validation and comparison. The other statements are incorrect.

93. The nurse is preparing to auscultate for heart sounds. Which technique is correct? A) Listen to the sounds at the aortic, tricuspid, pulmonic, and mitral areas. B) Listen by inching the stethoscope in a rough Z pattern, from the base of the heart across and down, then over to the apex. C) Listen to the sounds only at the site where the apical pulse is felt to be the strongest. D) Listen for all possible sounds at a time at each specified area.

B) Listen by inching the stethoscope in a rough Z pattern, from the base of the heart across and down, then over to the apex. Pages: 475-476. Do not limit auscultation of breath sounds to only four locations. Sounds produced by the valves may be heard all over the precordium. Inch the stethoscope in a rough Z pattern from the base of the heart across and down, then over to the apex. Or, start at the apex and work your way up. See Figure 19-22. Listen selectively to one sound at a time.

47. The nurse is reviewing the nutritional assessment of an 82-year-old patient. Which of these factors is most likely to affect the nutritional status of an elderly person? A) Increase in taste and smell B) Living alone on a fixed income C) Change in cardiovascular status D) Increase in gastrointestinal motility and absorption

B) Living alone on a fixed income Page: 176. Socioeconomic conditions frequently affect the nutritional status of the aging adult; these factors should be closely evaluated. Physical limitations, income, and social isolation are frequent problems that interfere with the acquisition of a balanced diet. A decrease in taste and smell and decreased gastrointestinal motility and absorption occur with aging. Cardiovascular status is not a factor that affects an elderly person's nutritional status.

116. During the neurologic assessment of a "healthy" 35-year-old patient, the nurse asks him to relax his muscles completely. The nurse then moves each extremity through full range of motion. Which of these results would the nurse expect to find? A) Firm, rigid resistance to movement B) Mild, even resistance to movement C) Hypotonic muscles as a result of total relaxation D) Slight pain with some directions of movement

B) Mild, even resistance to movement Page: 637. Tone is the normal degree of tension (contraction) in voluntarily relaxed muscles. It shows a mild resistance to passive stretch. Normally, the nurse will notice a mild, even resistance to movement. The other responses are not correct.

100. The nurse is reviewing an assessment of a patient's peripheral pulses and notices that the documentation states that the radial pulses are "2+." The nurse recognizes that this reading indicates what type of pulse? A) Bounding B) Normal C) Weak D) Absent

B) Normal Pages: 506-507. When documenting the force, or amplitude, of pulses, 3+ indicates an increased, full, or bounding pulse, 2+ indicates a normal pulse, 1+ indicates a weak pulse, and 0 indicates an absent pulse.

36. The nurse would use bimanual palpation technique in which situation? A) Palpating the thorax of an infant B) Palpating the kidneys and uterus C) Assessing pulsations and vibrations D) Assessing the presence of tenderness and pain

B) Palpating the kidneys and uterus Pages: 115-116. Bimanual palpation requires the use of both hands to envelop or capture certain body parts or organs such as the kidneys, uterus, or adnexa. The other situations are not appropriate for bimanual palpation.

98. The nurse is attempting to assess the femoral pulse in an obese patient. Which of these actions would be most appropriate? A) Have the patient assume a prone position. B) Ask the patient to bend his or her knees to the side in a froglike position. C) Press firmly against the bone with the patient in a semi-Fowler position. D) Listen with a stethoscope for pulsations because it is very difficult to palpate the pulse in an obese person.

B) Regular "lub, dub" pattern Pages: 510-511. To help expose the femoral area, particularly in obese people, the nurse should ask the person to bend his or her knees to the side in a froglike position.

When inspecting a patient's posterior wall of the pharynx and tonsils, a nurse documents which finding as abnormal? a. Both tonsils have a smooth surface. b. Left and right tonsils meet at the midline. c. Left and right tonsils extend beyond the posterior pillars. d. Both tonsils have a glistening appearance.

b. Left and right tonsils meet at the midline A A smooth surface is expected for the tonsils. B This indicates an enlargement documented as 4+. C This is an expected finding for the tonsils. D This is an expected finding for the tonsils.

111. The nurse suspects that a patient has appendicitis. Which of these procedures are appropriate for use when assessing for appendicitis or a perforated appendix? Select all that apply. A) Test for Murphy's sign. B) Test for Blumberg's sign. C) Test for shifting dullness. D) Perform iliopsoas muscle test. E) Test for fluid wave.

B) Test for Blumberg's sign. D) Perform iliopsoas muscle test. Pages: 543-544 | Page: 551. Testing for Blumberg's sign (rebound tenderness) and performing the iliopsoas muscle test should be used to assess for appendicitis. Murphy's sign is used to assess for an inflamed gallbladder or cholecystitis. Testing for a fluid wave and shifting dullness is done to assess for ascites.

109. A patient is suspected of having inflammation of the gallbladder, or cholecystitis. The nurse should conduct which of these techniques to assess for this condition? A) Obturator test B) Test for Murphy's sign C) Assess for rebound tenderness D) Iliopsoas muscle test

B) Test for Murphy's sign Page: 551. Normally, palpating the liver causes no pain. In a person with inflammation of the gallbladder, or cholecystitis, pain occurs as the descending liver pushes the inflamed gallbladder onto the examining hand during inspiration (Murphy's test). The person feels sharp pain and abruptly stops inspiration midway.

71. During an examination, a patient states that she was diagnosed with open-angle glaucoma 2 years ago. The nurse assesses for characteristics of open-angle glaucoma. Which of these are characteristics of open-angle glaucoma? Select all that apply. A) The patient may experience sensitivity to light, nausea, and halos around lights. B) The patient experiences tunnel vision in late stages. C) Immediate treatment is needed. D) Vision loss begins with peripheral vision. E) It causes sudden attacks of increased pressure that cause blurred vision. F) There are virtually no symptoms.

B) The patient experiences tunnel vision in late stages. D) Vision loss begins with peripheral vision. F) There are virtually no symptoms. Pages: 308-309. Open-angle glaucoma is the most common type of glaucoma; there are virtually no symptoms. Vision loss begins with the peripheral vision, which often goes unnoticed because individuals learn to compensate intuitively by turning their heads. The other characteristics are those of closed-angle glaucoma.

75. The nurse is testing the hearing of a 78-year-old man and keeps in mind the changes in hearing that occur with aging include which of the following? Select all that apply. A) Hearing loss related to aging begins in the mid 40s. B) The progression is slow. C) The aging person has low-frequency tone loss. D) The aging person may find it harder to hear consonants than vowels. E) Sounds may be garbled and difficult to localize. F) Hearing loss reflects nerve degeneration of the middle ear.

B) The progression is slow. D) The aging person may find it harder to hear consonants than vowels. E) Sounds may be garbled and difficult to localize. Page: 326. Presbycusis is a type of hearing loss that occurs with aging and is found in 60% of those older than 65 years. It is a gradual sensorineural loss caused by nerve degeneration in the inner ear or auditory nerve, and it slowly progresses after age 50. The person first notices a high-frequency tone loss; it is harder to hear consonants (high-pitched components of speech) than vowels. This makes words sound garbled. The ability to localize sound is impaired also.

8. The nurse is conducting an interview. Which of these statements is true regarding open-ended questions? Select all that apply. A) They elicit cold facts. B) They allow for self-expression. C) They build and enhance rapport. D) They leave interactions neutral. E) They call for short one- to two-word answers. F) They are used when narrative information is needed.

B) They allow for self-expression. C) They build and enhance rapport. F) They are used when narrative information Page: 32 Open-ended questions allow for self-expression, build rapport, and obtain narrative information. These features enhance communication during an interview. The other statements are appropriate for closed or direct questions.

66. The nurse is performing an eye assessment on an 80-year-old patient. Which of these findings is considered abnormal? A) A decrease in tear production B) Unequal pupillary constriction in response to light C) The presence of arcus senilis seen around the cornea D) Loss of the outer hair on the eyebrows due to a decrease in hair follicles

B) Unequal pupillary constriction in response to light Pages: 305-308. Pupils are small in old age, and the pupillary light reflex may be slowed, but pupillary constriction should be symmetric. The assessment findings in the other responses are considered normal in older persons.

39. When performing a physical examination, safety must be considered to protect the examiner and the patient against the spread of infection. Which of these statements describes the most appropriate action the nurse should take when performing a physical examination? A) There is no need to wash one's hands after removing gloves, as long as the gloves are still intact. B) Wash hands before and after every physical patient encounter. C) Wash hands between the examination of each body system to prevent the spread of bacteria from one part of the body to another. D) Wear gloves throughout the entire examination to demonstrate to the patient concern regarding the spread of infectious diseases.

B) Wash hands before and after every physical patient encounter. Page: 120. The nurse should wash his or her hands before and after every physical patient encounter; after contact with blood, body fluids, secretions, and excretions; after contact with any equipment contaminated with body fluids; and after removing gloves. Hands should be washed after gloves have been removed, even if the gloves appear to be intact. Gloves should be worn when there is potential contact with any body fluids.

91. During an assessment of a 68-year-old man with a recent onset of right-sided weakness, the nurse hears a blowing, swishing sound with the bell of the stethoscope over the left carotid artery. This finding would indicate: A) a valvular disorder. B) blood flow turbulence. C) fluid volume overload. D) ventricular hypertrophy.

B) blood flow turbulence. Page: 471. A bruit is a blowing, swishing sound indicating blood flow turbulence; normally none is present.

51. The nurse keeps in mind that a thorough skin assessment is very important because the skin holds information about a person's: A) support systems. B) circulatory status. C) socioeconomic status. D) psychological wellness.

B) circulatory status. Page: 211. The skin holds information about the body's circulation, nutritional status, and signs of systemic diseases as well as topical data on the integument itself.

11. In the majority culture of America, coughing, sweating, and diarrhea are symptoms of an illness. For some individuals of Mexican-American origin, however, these symptoms are a normal part of living. The nurse recognizes that this is true, probably because Mexican-Americans: A) have less efficient immune systems and are often ill. B) consider these symptoms a part of normal living, not symptoms of ill health. C) come from Mexico and coughing is normal and healthy there. D) are usually in a lower socioeconomic group and are more likely to be sick.

B) consider these symptoms a part of normal living, not symptoms of ill health. Page: 27 The nurse needs to identify the meaning of health to the patient, remembering that concepts are derived, in part, from the way in which members of the cultural group define health.

88. During palpation of the anterior chest wall, the nurse notices a coarse, crackling sensation over the skin surface. On the basis of these findings, the nurse suspects: A) tactile fremitus. B) crepitus. C) friction rub. D) adventitious sounds.

B) crepitus. Page: 424. Crepitus is a coarse, crackling sensation palpable over the skin surface. It occurs in subcutaneous emphysema when air escapes from the lung and enters the subcutaneous tissue, as after open thoracic injury or surgery.

34. The nurse is assessing a patient's skin during an office visit. What is the best technique to use to best assess the patient's skin temperature? Use the: A) fingertips because they're more sensitive to small changes in temperature. B) dorsal surface of the hand because the skin is thinner than on the palms. C) ulnar portion of the hand because there is increased blood supply that enhances temperature sensitivity. D) palmar surface of the hand because it is most sensitive to temperature variations because of increased nerve supply in this area.

B) dorsal surface of the hand because the skin is thinner than on the palms. The dorsa (backs) of hands and fingers are best for determining temperature because the skin there is thinner than on the palms. Fingertips are best for fine, tactile discrimination; the other responses are not useful for palpation.

96. During an assessment, the nurse uses the "profile sign" to detect: A) pitting edema. B) early clubbing. C) symmetry of the fingers. D) insufficient capillary refill.

B) early clubbing. Page: 506. The nurse should use the profile sign (viewing the finger from the side) to detect early clubbing.

33. When performing a physical assessment, the technique the nurse will always use first is: A) palpation. B) inspection. C) percussion. D) auscultation.

B) inspection. Pages: 115-116. The skills requisite for the physical examination are inspection, palpation, percussion, and auscultation. The skills are performed one at a time and in this order (with the exception of the abdominal assessment, where auscultation takes place before palpation and percussion). The assessment of each body system begins with inspection. A focused inspection takes time and yields a surprising amount of information.

105. A patient has hypoactive bowel sounds. The nurse knows that a potential cause of hypoactive bowel sounds is: A) diarrhea. B) peritonitis. C) laxative use. D) gastroenteritis.

B) peritonitis. Page: 561. Diminished or absent bowel sounds signal decreased motility from inflammation as seen with peritonitis, with paralytic ileus after abdominal surgery, or with late bowel obstruction.

62. The physician reports that a patient with a neck tumor has a tracheal shift. The nurse is aware that this means that the patient's trachea is: A) pulled to the affected side. B) pushed to the unaffected side. C) pulled downward. D) pulled downward in a rhythmic pattern.

B) pushed to the unaffected side. Pages: 262-263. The trachea is pushed to the unaffected side with an aortic aneurysm, a tumor, unilateral thyroid lobe enlargement, and pneumothorax. The trachea is pulled to the affected side with large atelectasis, pleural adhesions, or fibrosis. Tracheal tug is a rhythmic downward pull that is synchronous with systole and occurs with aortic arch aneurysm.

56. A 40-year-old woman reports a change in mole size, accompanied by color changes, itching, burning, and bleeding over the past month. She has a dark complexion and has no family history of skin cancer, but she has had many blistering sunburns in the past. The nurse would: A) tell the patient to watch the lesion and report back in 2 months. B) refer the patient because of the suspicion of melanoma on the basis of her symptoms. C) ask additional questions regarding environmental irritants that may have caused this condition. D) suspect that this is a compound nevus, which is very common in young to middle-aged adults.

B) refer the patient because of the suspicion of melanoma on the basis of her symptoms. The ABCD danger signs of melanoma are asymmetry, border irregularity, color variation, and diameter. In addition, individuals may report a change in size, development of itching, burning, bleeding, or a new-pigmented lesion. Any of these signs raise suspicion of malignant melanoma and warrant immediate referral.

68. A patient comes to the emergency department after a boxing match, and his left eye is swollen almost shut. He has bruises on his face and neck. He says he is worried because he "can't see well" from his left eye. The physician suspects retinal damage. The nurse recognizes that signs of retinal detachment include: A) loss of central vision. B) shadow or diminished vision in one quadrant or one half of the visual field. C) loss of peripheral vision. D) sudden loss of pupillary constriction and accommodation.

B) shadow or diminished vision in one quadrant or one half of the visual field. Page: 316. With retinal detachment, the person has shadows or diminished vision in one quadrant or one half of the visual field. The other responses are not signs of retinal detachment.

13. An individual who takes the magicoreligious perspective of illness and disease is likely to believe that his or her illness was caused by: A) germs and viruses. B) supernatural forces. C) eating imbalanced foods. D) an imbalance within his or her spiritual nature.

B) supernatural forces. Page: 21 The basic premise of the magicoreligious perspective is that the world is seen as an arena in which supernatural forces dominate. The fate of the world and those in it depends on the actions of supernatural forces for good or evil. The other answers do not reflect the magicoreligious perspective.

25. The nurse is performing a functional assessment on an 82-year-old patient who recently had a stroke. Which of these questions would be most important to ask? A. "Do you wear glasses?" B. "Are you able to dress yourself?" C. "Do you have any thyroid problems?" D. "How many times a day do you have a bowel movement?"

B. "Are you able to dress yourself?" Page: 67. Functional assessment measures how a person manages day-to-day activities. For the older person, the meaning of health becomes those activities that they can or cannot do. The other responses do not relate to functional assessment.

26. The nurse is conducting a developmental history on a 5-year-old child. Which questions are appropriate to ask the parents for this part of the assessment? Select all that apply. A. "How much junk food does your child eat?" B. "How many teeth has he lost, and when did he lose them?" C. "Is he able to tie his shoelaces?" D. "Does he take a children's vitamin?" E. "Can he tell time?" F. "Does he have any food allergies?"

B. "How many teeth has he lost, and when did he lose them?" C. "Is he able to tie his shoelaces?" E. "Can he tell time?" Page: 61. Questions about tooth loss, ability to tell time, and ability to tie shoelaces are appropriate questions for a developmental assessment. Questions about junk food intake and vitamins are part of a nutritional history. Questions about food allergies are not part of a developmental history.

Neck rotation on each side should be: A. 120 degrees B. 70 degrees C. 10 degrees D. 45 degrees

B. 70 degrees Neck rotation should be 70 degrees on each side. Neck flexion and extension should each be 45 degrees.

What questions can you ask a patient to assess his or her state of consciousness? A. Ask the patient about his or her thoughts, feelings, and emotions. B. Ask for the date, his or her name, and the location. C. Ask the patient to repeat a series of five numbers. D. Ask the patient to write his or her name and address.

B. Ask for the date, his or her name, and the location. Begin with asking the patient today's date, then ask the patient to state his or her name. A patient should be oriented to time, place, and person and be able to respond appropriately to questions about the environment. Thoughts, feelings, and emotions are not part of an assessment for state of consciousness. Repeating a series of numbers is not part of an assessment for state of consciousness. Having the ability to write his or her name and address is not part of an assessment for state of consciousness.

Which step must be taken to ensure accurate measurement of a patient's daily weight? A. Weigh the patient at a different time each day. B. Ask the patient to void before he or she is weighed. C. Ask the patient to list all food and beverages consumed since the last weight measurement was taken. D. Weigh the patient using two different scales, and compare the weights.

B. Ask the patient to void before he or she is weighed. To ensure accurate measurement of a patient's daily weight, the patient should be weighed at the same time of day, wearing the same clothes, and he or she should be asked to void first. The patient need not need list all food and drink consumed since the last time he or she was weighed. The nurse need not weigh the patient with two different scales in order to compare the weights.

Nocturia, urine dribbles, difficulty voiding, and a small urine stream are common symptoms of which of the following conditions? A. Colorectal cancer B. Benign prostatic hypertrophy C. Testicular cancer D. Hernias

B. Benign prostatic hypertrophy Benign prostatic hypertrophy is common in men over 50 years of age. Nocturia, urine dribble, difficulty voiding, and a small urine stream are common symptoms of benign prostatic hypertrophy.

Which description is consistent with normal vaginal secretions? A. Clear, thick, and with a fishy odor B. Clear or cloudy, and odorless or with a slight odor C. Yellow, thin, and with a strong odor D. Green, thick, and with a foul odor

B. Clear or cloudy, and odorless or with a slight odor Vaginal secretions that are clear or cloudy, and odorless or with a slight odor are a normal finding. Vaginal secretions that are thick with a fishy odor are not a normal finding and are a sign of vaginitis. Vaginal secretions that are yellow with a strong odor are not a normal finding and are a sign of vaginitis. Vaginal secretions that are green, thick, and with a foul odor are not a normal finding and are a sign of vaginitis.

What should the nurse do if a patient displays staggering or loss of balance during the Romberg test? A. Give the patient a gentle push to further assess balance. B. Delay other balance tests. C. Have the patient stand on one foot with the eyes closed. D. Have the patient hop on one foot.

B. Delay other balance tests. If a patient has staggering or loss of balance with the Romberg test, delay other balance tests. If a patient has staggering or loss of balance with the Romberg test, delay other balance tests; do not give the patient a gentle push to further assess balance. If a patient has staggering or loss of balance with the Romberg test, delay other balance tests; do not have the patient stand on one foot with eyes closed. If a patient has staggering or loss of balance with the Romberg test, delay other balance tests; do not have the patient hop on one foot.

What are the five danger signs to watch for when assessing nipples? A. Shape, tenderness, size, inversion, and mobility B. Discharge, depression, discoloration, dermatologic changes, and deviation C. Lymph nodes, breast size, symmetry, tail of Spence, and color D. Rash, nodules, warmth, redness, and pain

B. Discharge, depression, discoloration, dermatologic changes, and deviation

When examining the eyes, which of the following is an expected finding? A. Reddened conjunctivae B. Equal pupils C. Crusted eyelashes D. Periorbital edema

B. Equal pupils Equal pupils are an expected finding during an eye exam. Reddened conjunctivae are not an expected finding during an eye exam. Crusted eyelashes are not an expected finding during an eye exam. Periorbital edema is not an expected finding during an eye exam.

What is the primary purpose of initially assessing an apical pulse? A. Assessment of the patient's cardiac function B. Establishment of a baseline as part of the patient's vital signs C. Assessment of the patient's risk for cardiovascular disease D. Determination of oxygen saturation

B. Establishment of a baseline as part of the patient's vital signs The primary purpose for initially assessing an apical pulse is establishing a baseline for the patient against which future assessments of apical pulse rate can be compared. The apical pulse rate provides some information about the patient's cardiac function; however, obtaining such information is not the primary goal of conducting this assessment at this time. Measuring the apical pulse rate does not adequately assess the patient's risk for cardiovascular disease. Other factors (such as age, smoking, dietary patterns, and exercise habits) contribute to the risk for cardiovascular disease. Pulse oximetry is the measurement of arterial blood oxygen saturation, not an apical pulse

Which of the following tips will assist with eliciting the patellar and Achilles deep tendon reflexes? A. Have the patient sit with his or her feet flat on the floor. B. Have the patient focus on pulling his or her clasped hands apart. C. Have the patient flex his or her knees at a 45-degree angle. D. Strike the knee above the patella.

B. Have the patient focus on pulling his or her clasped hands apart. With the patient in a seated position, have him or her focus on pulling his or her clasped hands apart; this will take focus away from the test of deep tendon reflexes. The patient should sit so that the upper legs are supported and the lower legs hang loosely. The patient should flex his or her knees at a 90-degree angle. Strike the knee just below the patella.

What is the correct order for abdominal assessment? A. Inspection, palpation, auscultation, percussion B. Inspection, auscultation, percussion, palpation C. Auscultation, inspection, palpation, percussion D. Palpation, inspection, auscultation, percussion

B. Inspection, auscultation, percussion, palpation The correct order for abdominal assessment is inspection, auscultation, percussion, palpation. Palpation is the last step in abdominal assessment. Auscultation follows assessment because percussion and palpation can alter the frequency and intensity of bowel sounds.

Why is it important to have equipment and supplies organized before the exam? A. It makes you look smarter to the patient. B. It avoids interruptions and delays. C. In case you are out of a supply, you can skip that part of the exam. D. It lets you focus on the testing instead of the patient's response.

B. It avoids interruptions and delays Having equipment ready and supplies organized before the exam avoids interruptions and delays. Whether the patient thinks you are smart is not the focus of the exam. Skipping part of the exam is not a valid response to missing equipment; it is your responsibility to be prepared. You must always focus on the patient's response in case it is abnormal; that is why you are performing the exam!

What is the purpose of having the patient clench his teeth and smile? A. It tests two-point discrimination. B. It tests CN VII and lets you observe tooth occlusion. C. It tests the abdominal reflexes. D. It tests CN III, IV and VI.

B. It tests CN VII and lets you observe tooth occlusion Having the patient clench the teeth and smile tests CN VII and lets you observe tooth occlusion. In proper occlusion, the upper and lower molars interdigitate, and the premolars and canines interdigitate. Touching the skin with one or two sterile needles to determine the distance at which the patient can no longer distinguish two points is how two-point discrimination is tested. Stroking each abdominal quadrant tests the abdominal reflexes. Cranial nerves III, IV, and VI are tested by checking extraocular movements.

A nurse is assessing a patient's neck. Which of the following is considered an expected finding? A. Jugular vein distention B. Midline trachea C. Carotid artery prominence D. Thyroid enlargement

B. Midline trachea Midline trachea is an expected finding. Jugular vein distention, carotid artery prominence, and thyroid enlargement are not expected findings

Which of the following is the correct way to assess a patient's nose for patency? A. Have the patient breathe rapidly through both nares. B. Occlude one naris, and have the patient breathe through the open naris. C. Occlude both nares, and have the patient breathe through the mouth. D. Have the patient blow out through both nares.

B. Occlude one naris, and have the patient breathe through the open naris. Patency of nares is assessed by occluding one at a time. Having the patient breathe rapidly does not assess for patency. One nare is occluded at a time; both nares are not occluded at the same time. Having the patient blow out through the nares does not assess patency.

The nurse notes that a patient's left elbow is resistant to extension and flexion while performing range of motion exercises. What is the appropriate nursing action? A. Move the joint through the full range of motion exercises. B. Perform range of motion to the left elbow until resistance is met. C. Omit all the range of motion exercises until the health care provider is notified. D. Inform the health care provider that the patient is uncooperative with exercising

B. Perform range of motion to the left elbow until resistance is met. Rationale: The nurse would stop the range of motion exercises because resistance is met. The range of motion exercises should not continue until pain is felt by the patient. The nurse would not move the left elbow joint through the full range of motion because resistance is met. Range of motion exercises should not be omitted. When you note resistance within a joint, do not force the joint motion. Consult with the health care provider or a physical therapist. The nurse would not notify the health care provider without information to support the patient is uncooperative with exercising.

What is a depression that is left after pressing a finger or thumb on swollen tissue called? A. Cyanosis B. Pitting edema C. A thrill D. A varicosity

B. Pitting edema To assess a patient for pitting edema, press a finger or thumb over the swollen tissues of the tibia or medial malleolus where edema is present. A depression left in swollen tissue is not cyanosis; cyanosis is mottled blue color of the skin. A thrill is a soft vibratory sensation; it is not a depression caused by pressing a finger or thumb on swollen tissue. A varicosity is not determined by pressing a finger or thumb on swollen tissue; varicosities are distended veins.

What does full movement of the eyes in the six cardinal fields of gaze reflect? A. Both B and C B. Proper functioning of the oculomotor, trochlear, and abducens nerves C. Proper functioning of the extraocular muscles D. Proper functioning of the olfactory and optic nerves

B. Proper functioning of the oculomotor, trochlear, and abducens nerves C. Proper functioning of the extra ocular muscles Proper functioning of the oculomotor, trochlear, and abducens nerves plus proper functioning of the extraocular muscles is reflected by full movement of the eyes in the six cardinal fields of gaze. Proper functioning of the olfactory and optic nerves is reflected by an accurate sense of smell and accurate vision on the Snellen and Rosenbaum charts, not the movement of the eyes in the six cardinal fields of gaze

During vocalization, the soft palate: A. Lowers symmetrically B. Raises symmetrically C. Vibrates D. Has a gag reflex

B. Raises symmetrically During vocalization, the soft palate raises symmetrically. During vocalization, the soft palate raises symmetrically; it does not lower. The soft palate does not vibrate during vocalization. The soft palate does not have a gag reflex.

The nurse is assessing a patient who does not speak English. When checking the visual acuity of a non-English-speaking patient, the nurse should use which chart? A. Snellen chart B. Snellen E chart C. Rosenbaum pocket screener D. Pupil gauge chart

B. Snellen E chart

A nursing assistive personnel (NAP) is preparing to weigh a resident in a skilled nursing facility. The patient is usually weighed in street clothing and socks, with his shoes off. The patient is currently wearing street clothing with shoes and socks. What will the NAP do to ensure that the patient's weight is correctly measured? A. Take off the patient's street clothing. B. Take off the patient's shoes, but leave his socks on. C. Take off the patient's shoes and socks, and put on slippers. D. Take off the patient's street clothing and put on a hospital gown and nonskid socks.

B. Take off the patient's shoes, but leave his socks on. Since the patient is usually weighed in street clothing and socks, the NAP should remove the patient's shoes and leave his socks on. The patient's street clothing need not be removed. The patient need not wear slippers. The patient need not wear a hospital gown and nonskid socks; however, his shoes must be removed and his own socks left on.

Which observation indicates that a patient's analgesic has been effective in managing pain that she rated a 6 out of 10 on a pain rating scale before the intervention? A. The patient is seen quietly reading a magazine. B. The patient rates her current pain as 3 out of 10 on the pain rating scale. C. The patient is overheard telling her family that she is "feeling better today." D. The patient is observed sleeping, with a respiratory rate assessed at 18/minute, compared with 22/minute before the intervention

B. The patient rates her current pain as 3 out of 10 on the pain rating scale Rationale: The patient's self-report of less pain indicates that the analgesic has been effective. The patient's reading a magazine does not indicate that the analgesic has been effective, because she may be trying to distract herself as an alternative pain management technique. The patient's positive comment does not necessarily indicate that the analgesic has been effective. A lower respiratory rate does not necessarily indicate that the analgesic has been effective

When preparing to measure the height and weight of a newly admitted patient, why would the nurse ask about the patient's ability to stand? A. To determine if a wheelchair should be requested B. To determine if the patient is steady enough to stand without assistance C. To determine if a bed scale must be obtained to measure the patient's weight D. To establish how much help the patient will need with personal care

B. To determine if the patient is steady enough to stand without assistance Before measuring height and weight, the nurse asks about the patient's ability to stand in order to determine if the patient is steady enough to stand without assistance while having his or her height measured. Use of a wheelchair is determined only after the nurse determines if the patient can't stand without assistance, and that he or she is able to transfer to a wheelchair. Use of a bed scale is determined only after the nurse determines if the patient can't stand without assistance or get out of bed. Personal care needs of a patient do not come into consideration when measuring weight and height.

When should you check the patient's blood pressure to assess for orthostatic hypotension? A. After the patient has been walking around the room B. While the patient is sitting and standing C. After bloodwork has been done D. Immediately after the initial blood pressure reading

B. While the patient is sitting and standing If orthostatic hypotension is suspected, check the patient's blood pressure while the patient is sitting and standing. Orthostatic hypotension cannot be assessed after the patient has been walking around the room; it is assessed from a sitting to standing position. Bloodwork does not influence the assessment procedure for orthostatic hypotension. Orthostatic hypotension is assessed by checking the patient's blood pressure while the patient is sitting and standing

Pulse oximetry is used to: Select all that apply. A. detect pulsation in the veins. B. estimate the oxygen saturation of arterial blood. C. determine hemoglobin D. percentages of the blood. estimate the saturation of oxygen in the alveoli. E. assess heart rate.

B. estimate the oxygen saturation of arterial blood. E. assess heart rate

The nurse teaches Amanda about diphenhydramine (Benadryl). Which information should the nurse include? Benadryl blocks the effect of the histamine response to reduce itching. Benadryl is a H1 receptor antagonist and may be helpful to reduce itching associated with hives. Benadryl products contain aspirin, so observe for signs of bleeding. All Benadryl products are aspirin free and bleeding is not a side effect. This medication may cause drowsiness. Although considered a minor side effect, Benadryl may cause drowsiness. Blurred vision or loss of balance are potentially serious side effects of this medication. The client should be taught to notify a health care provider if these uncommon but potentially serious side effects occur. This medication may cause an increase in secretions and moisten mucous membranes. Benadryl blocks acetylcholine receptors which decreases secretions causing dry mucous membranes as a side effect.

Benadryl blocks the effect of the histamine response to reduce itching. Benadryl is a H1 receptor antagonist and may be helpful to reduce itching associated with hives. This medication may cause drowsiness. Although considered a minor side effect, Benadryl may cause drowsiness. Blurred vision or loss of balance are potentially serious side effects of this medication. The client should be taught to notify a health care provider if these uncommon but potentially serious side effects occur.

32. The nurse is performing a mental status examination. Which statement is true regarding the assessment of mental status? A) Mental status assessment diagnoses specific psychiatric disorders. B) Mental disorders occur in response to everyday life stressors. C) Mental status functioning is inferred through assessment of an individual's behaviors. D) Mental status can be assessed directly, just like other systems of the body (e.g., cardiac and breath sounds).

C) Mental status functioning is inferred through assessment of an individual's behaviors. Page: 71. Mental status functioning is inferred through assessment of an individual's behaviors. It cannot be assessed directly like characteristics of the skin or heart sounds.

65. A mother asks when her newborn infant's eyesight will be developed. The nurse should reply: A) "Vision is not totally developed until 2 years of age." B) "Infants develop the ability to focus on an object at around 8 months." C) "By about 3 months, infants develop more coordinated eye movements and can fixate on an object." D) "Most infants have uncoordinated eye movements for the first year of life."

C) "By about 3 months, infants develop more coordinated eye movements and can fixate on an object." Page: 284. Eye movements may be poorly coordinated at birth, but by 3 to 4 months of age, the infant should establish binocularity and should be able to fixate on a single image with both eyes simultaneously.

12. Among many Asians there is a belief in the yin/yang theory, rooted in the ancient Chinese philosophy of Tao. The nurse recognizes which statement that most accurately reflects "health" in an Asian with this belief? A) A person is able to work and produce. B) A person is happy, stable, and feels good. C) All aspects of the person are in perfect balance. D) A person is able to care for others and function socially.

C) All aspects of the person are in perfect balance. Page: 21 Many Asians believe in the yin/yang theory, in which health is believed to exist when all aspects of the person are in perfect balance. The other statements do not describe this theory.

78. A 65-year-old patient with a history of heart failure comes to the clinic with complaints of "being awakened from sleep with shortness of breath." Which action by the nurse is most appropriate? A) Obtain a detailed history of the patient's allergies and history of asthma. B) Tell the patient to sleep on his or her right side to facilitate ease of respirations. C) Assess for other signs and symptoms of paroxysmal nocturnal dyspnea. D) Assure the patient that this is normal and will probably resolve within the next week.

C) Assess for other signs and symptoms of paroxysmal nocturnal dyspnea. Pages: 419-420. The patient is experiencing paroxysmal nocturnal dyspnea: being awakened from sleep with shortness of breath and the need to be upright to achieve comfort.

60. During an examination, the nurse knows that Paget's disease would be indicated by which of these assessment findings? A) Positive Macewen sign B) Premature closure of the sagittal suture C) Headache, vertigo, tinnitus, and deafness D) Elongated head with heavy eyebrow ridge

C) Headache, vertigo, tinnitus, and deafness Paget's disease occurs more often in males and is characterized by bowed long bones, sudden fractures, and enlarging skull bones that press on cranial nerves causing symptoms of headache, vertigo, tinnitus, and progressive deafness.

94. The nurse is assessing a patient's apical impulse. Which of these statements is true regarding the apical impulse? A) It is palpable in all adults. B) It occurs with the onset of diastole. C) Its location may be indicative of heart size. D) It should normally be palpable in the anterior axillary line.

C) Its location may be indicative of heart size. Page: 473 | Page: 492. The apical impulse is palpable in about 50% of adults. It is located in the fifth left intercostal space in the midclavicular line. Horizontal or downward displacement of the apical impulse may indicate an enlargement of the left ventricle.

122. The nurse is caring for a patient who has just had neurosurgery. To assess for increased intracranial pressure, what would the nurse include in the assessment? A) Cranial nerves, motor function, and sensory function B) Deep tendon reflexes, vital signs, and coordinated movements C) Level of consciousness, motor function, pupillary response, and vital signs D) Mental status, deep tendon reflexes, sensory function, and pupillary response

C) Level of consciousness, motor function, pupillary response, and vital signs Pages: 660-661. Some hospitalized persons have head trauma or a neurologic deficit from a systemic disease process. These people must be monitored closely for any improvement or deterioration in neurologic status and for any indication of increasing intracranial pressure. The nurse should use an abbreviation of the neurologic examination in the following sequence: level of consciousness, motor function, pupillary response, and vital signs.

27. During an examination, the nurse can assess mental status by which activity? A) Examining the patient's electroencephalogram B) Observing the patient as he or she performs an IQ test C) Observing the patient and inferring health or dysfunction D) Examining the patient's response to a specific set of questions

C) Observing the patient and inferring health or dysfunction Page: 71. Mental status cannot be scrutinized directly like the characteristics of skin or heart sounds. Its functioning is inferred through assessment of an individual's behaviors, such as consciousness, language, mood and affect, and other aspects.

95. During an assessment of an older adult, the nurse should expect to notice which finding as a normal physiologic change associated with the aging process? A) Hormonal changes causing vasodilation and a resulting drop in blood pressure B) Progressive atrophy of the intramuscular calf veins, causing venous insufficiency C) Peripheral blood vessels growing more rigid with age, producing a rise in systolic blood pressure D) Narrowing of the inferior vena cava, causing low blood flow and increases in venous pressure resulting in varicosities

C) Peripheral blood vessels growing more rigid with age, producing a rise in systolic blood pressure Pages: 504-505. Peripheral blood vessels grow more rigid with age, resulting in a rise in systolic blood pressure. Aging produces progressive enlargement of the intramuscular calf veins, not atrophy. The other options are not correct.

58. A patient has been admitted for severe psoriasis. The nurse can expect to see what finding in the patient's fingernails? A) Splinter hemorrhages B) Paronychia C) Pitting D) Beau lines

C) Pitting Pages: 248-250. Pitting nails are characterized by sharply defined pitting and crumbling of the nails with distal detachment, and they are associated with psoriasis. See Table 12-13 for descriptions of the other terms.

42. The nurse is assessing an 80-year-old male patient. Which assessment findings would be considered normal? A) An increase in body weight from younger years B) Additional deposits of fat on the thighs and lower legs C) The presence of kyphosis and flexion in the knees and hips D) A change in overall body proportion, a longer trunk, and shorter extremities

C) The presence of kyphosis and flexion in the knees and hips Page: 149. Changes that occur in the aging person include more prominent bony landmarks, decreased body weight (especially in males), a decrease in subcutaneous fat from the face and periphery, and additional fat deposited on the abdomen and hips. Postural changes of kyphosis and slight flexion in the knees and hips also occur.

48. When the mid-upper arm circumference and triceps skinfold of an 82-year-old man are evaluated, which is important for the nurse to remember? A) These measurements are no longer necessary for the elderly. B) Derived weight measures may be difficult to interpret because of wide ranges of normal. C) These measurements may not be accurate because of changes in skin and fat distribution. D) Measurements may be difficult to obtain if the patient is unable to flex his elbow to at least 90 degrees.

C) These measurements may not be accurate because of changes in skin and fat distribution. Page: 191 Accurate mid-upper arm circumference and triceps skinfold measurements are difficult to obtain and interpret in older adults because of sagging skin, changes in fat distribution, and declining muscle mass. Body mass index and waist-to-hip ratio are better indicators of obesity in the elderly.

110. During an assessment the nurse notices that a patient's umbilicus is enlarged and everted. It is midline, and there is no change in skin color. The nurse recognizes that the patient may have which condition? A) Intra-abdominal bleeding B) Constipation C) Umbilical hernia D) An abdominal tumor

C) Umbilical hernia Page: 537. The umbilicus is normally midline and inverted, with no signs of discoloration. With an umbilical hernia, the mass is enlarged and everted. The other responses are incorrect.

67. The nurse notices the presence of periorbital edema when performing an eye assessment on a 70-year-old patient. The nurse should: A) check for the presence of exophthalmos. B) suspect that the patient has hyperthyroidism. C) ask the patient if he or she has a history of heart failure. D) assess for blepharitis because this is often associated with periorbital edema.

C) ask the patient if he or she has a history of heart failure. Page: 312. Periorbital edema occurs with local infections, crying, and systemic conditions such as heart failure, renal failure, allergy, and hypothyroidism. Periorbital edema is not associated with blepharitis.

97. When performing a peripheral vascular assessment on a patient, the nurse is unable to palpate the ulnar pulses. The patient's skin is warm and capillary refill time is normal. The nurse should next: A) check for the presence of claudication. B) refer the individual for further evaluation. C) consider this a normal finding and proceed with the peripheral vascular evaluation. D) ask the patient if he or she has experienced any unusual cramping or tingling in the arm.

C) consider this a normal finding and proceed with the peripheral vascular evaluation. Pages: 506-507. It is not usually necessary to palpate the ulnar pulses. The ulnar pulses are often not palpable in the normal person. The other responses are not correct.

15. An elderly Mexican-American woman with traditional beliefs has been admitted to an inpatient care unit. A culturally-sensitive nurse would: A) contact the hospital administrator about the best course of action. B) automatically get a curandero for her because it is not culturally appropriate for her to request one. C) further assess the patient's cultural beliefs and offer the patient assistance in contacting a curandero or priest if she desires. D) ask the family what they would like to do because Mexican-Americans traditionally give control of decisions to their families.

C) further assess the patient's cultural beliefs and offer the patient assistance in contacting a curandero or priest if she desires. Pages: 22-23 In addition to seeking help from the biomedical/scientific health care provider, patients may also seek help from folk or religious healers. Some people, such as those of Mexican-American or American Indian origins, may believe that the cure is incomplete unless the body, mind, and spirit are also healed (although the division of the person into parts is a Western concept).

43. When assessing the force, or strength, of a pulse, the nurse recalls that it: A) is usually recorded on a 0- to 2-point scale. B) demonstrates elasticity of the vessel wall. C) is a reflection of the heart's stroke volume. D) reflects the blood volume in the arteries during diastole.

C) is a reflection of the heart's stroke volume. Page: 134. The heart pumps an amount of blood (the stroke volume) into the aorta. The force flares the arterial walls and generates a pressure wave, which is felt in the periphery as the pulse.

80. The nurse is reviewing the technique of palpating for tactile fremitus with a new graduate. Which statement by the graduate nurse reflects a correct understanding of tactile fremitus? "Tactile fremitus: A) is caused by moisture in the alveoli." B) indicates that there is air in the subcutaneous tissues." C) is caused by sounds generated from the larynx." D) reflects the blood flow through the pulmonary arteries."

C) is caused by sounds generated from the larynx." Pages: 422-423. Fremitus is a palpable vibration. Sounds generated from the larynx are transmitted through patent bronchi and the lung parenchyma to the chest wall where they are felt as vibrations. Crepitus is the term for air in the subcutaneous tissues.

14. If an American Indian has come to the clinic to seek help with regulating her diabetes, the nurse can expect that she: A) will comply with the treatment prescribed. B) has obviously given up her beliefs in naturalistic causes of disease. C) may also be seeking the assistance of a shaman or medicine man. D) will need extra help in dealing with her illness and may be experiencing a crisis of faith.

C) may also be seeking the assistance of a shaman or medicine man. Page: 23 When self-treatment is unsuccessful, the individual may turn to the lay or folk healing systems, to spiritual or religious healing, or to scientific biomedicine. In addition to seeking help from a biomedical or scientific health care provider, patients may also seek help from folk or religious healers.

28. The nurse is assessing a 75-year-old man. As the nurse begins the mental status portion of the assessment, the nurse expects that this patient: A) will have no decrease in any of his abilities, including response time. B) will have difficulty on tests of remote memory because this typically decreases with age. C) may take a little longer to respond, but his general knowledge and abilities should not have declined. D) will have had a decrease in his response time because of language loss and a decrease in general knowledge.

C) may take a little longer to respond, but his general knowledge and abilities should not have declined. Page: 72. The aging process leaves the parameters of mental status mostly intact. There is no decrease in general knowledge and little or no loss in vocabulary. Response time is slower than in youth. It takes a bit longer for the brain to process information and to react to it. Recent memory, which requires some processing is somewhat decreased with aging, but remote memory is not affected.

77. During an assessment, the nurse knows that expected assessment findings in the normal adult lung include the presence of: A) adventitious sounds and limited chest expansion. B) increased tactile fremitus and dull percussion tones. C) muffled voice sounds and symmetrical tactile fremitus. D) absent voice sounds and hyperresonant percussion tones.

C) muffled voice sounds and symmetrical tactile fremitus. Pages: 429-430. Normal lung findings include symmetric chest expansion, resonant percussion tones, vesicular breath sounds over the peripheral lung fields, muffled voice sounds, and no adventitious sounds.

61. A woman comes to the clinic and states, "I've been sick for so long! My eyes have gotten so puffy, and my eyebrows and hair have become coarse and dry." The nurse will assess for other signs and symptoms of: A) cachexia. B) Parkinson's syndrome. C) myxedema. D) scleroderma.

C) myxedema. Pages: 276-277. Myxedema (hypothyroidism) is a deficiency of thyroid hormone that, when severe, causes a nonpitting edema or myxedema. The patient will have a puffy edematous face especially around eyes (periorbital edema), coarse facial features, dry skin, and dry, coarse hair and eyebrows. See Table 13-4, Abnormal Facial Appearances with Chronic Illnesses, for descriptions of the other responses.

104. While examining a patient, the nurse observes abdominal pulsations between the xiphoid and umbilicus. The nurse would suspect that these are: A) pulsations of the renal arteries. B) pulsations of the inferior vena cava. C) normal abdominal aortic pulsations. D) increased peristalsis from a bowel obstruction.

C) normal abdominal aortic pulsations. Pages: 538-539. Normally, one may see the pulsations from the aorta beneath the skin in the epigastric area, particularly in thin persons with good muscle wall relaxation.

114. During an assessment of an 80-year-old patient, the nurse notices the following: inability to identify vibrations at the ankle and to identify position of big toe, slower and more deliberate gait, and slightly impaired tactile sensation. All other neurologic findings are normal. The nurse should interpret that these findings indicate: A) cranial nerve dysfunction. B) lesion in the cerebral cortex. C) normal changes due to aging. D) demyelinization of nerves due to a lesion.

C) normal changes due to aging. Page: 629. Some aging adults show a slower response to requests, especially for those calling for coordination of movements. The findings listed are normal in the absence of other significant abnormal findings. The other responses are incorrect.

83. The nurse knows that auscultation of fine crackles would most likely be noticed in: A) a healthy 5-year-old child. B) a pregnant woman. C) the immediate newborn period. D) association with a pneumothorax.

C) the immediate newborn period. Pages: 436-437. Fine crackles are commonly heard in the immediate newborn period as a result of the opening of the airways and clearing of fluid. Persistent fine crackles would be noticed with pneumonia, bronchiolitis, or atelectasis.

74. During an examination, the patient states he is hearing a buzzing sound and says that it is "driving me crazy!" The nurse recognizes that this symptom indicates: A) vertigo. B) pruritus. C) tinnitus. D) cholesteatoma.

C) tinnitus. Pages: 328-329. Tinnitus is a sound that comes from within a person; it can be a ringing, crackling, or buzzing sound. It accompanies some hearing or ear disorders.

38. The nurse is unable to palpate the right radial pulse on a patient. The best action would be to: A) auscultate over the area with a fetoscope. B) use a goniometer to measure the pulsations. C) use a Doppler device to check for pulsations over the area. D) check for the presence of pulsations with a stethoscope.

C) use a Doppler device to check for pulsations over the area. Page: 120. Doppler devices are used to augment pulse or blood pressure measurements. Goniometers measure joint range of motion. A fetoscope is used to auscultate fetal heart tones. Stethoscopes are used to auscultate breath, bowel, and heart sounds.

3. A nurse is taking complete health histories on all of the patients attending a wellness workshop. On the history form, one of the written questions asks, "You don't smoke, drink, or take drugs, do you?" This question is an example of: A) talking too much. B) using confrontation. C) using biased or leading questions. D) using blunt language to deal with distasteful topics.

C) using biased or leading questions. Page: 36 This is an example of using leading or biased questions. Asking, "You don't smoke, do you?" implies that one answer is "better" than another. If the person wants to please someone, he or she is either forced to answer in a way corresponding to their implied values or is made to feel guilty when admitting the other answer.

81. When auscultating the lungs of an adult patient, the nurse notes that over the posterior lower lobes low-pitched, soft breath sounds are heard, with inspiration being longer than expiration. The nurse interprets that these are: A) sounds normally auscultated over the trachea. B) bronchial breath sounds and are normal in that location. C) vesicular breath sounds and are normal in that location. D) bronchovesicular breath sounds and are normal in that location.

C) vesicular breath sounds and are normal in that location. Pages: 428-429. Vesicular breath sounds are low-pitched, soft sounds with inspiration being longer than expiration. These breath sounds are expected over peripheral lung fields where air flows through smaller bronchioles and alveoli.

As the nurse is conducting an admission interview, the patient states, "I've lost 30 pounds over the last 4 months." Which question might the nurse ask to determine if the weight loss was intentional or unintentional? A. "Is your health care provider aware of this weight loss?" B. "Has your weight fluctuated like this before?" C. "Have you been following a specific diet?" D. "Is it easy for you to lose weight?"

C. "Have you been following a specific diet?" Asking if the patient has been following a specific diet will help determine if the patient's weight loss was intentional or unintentional because it asks the patient to offer a possible explanation for the weight loss. Whether the patient has reported the weight loss to a health care provider is irrelevant. Asking if the patient's weight has fluctuated before or if it's easy for the patient to lose weight will not help determine if the patient's weight loss was intentional or unintentional. It merely asks about the patient's history of weight loss.

The nurse is palpating the abdomen of a patient. How deep should the hands press while performing deep palpation? A. 1 cm B. 2 cm C. 4 cm D. 8 cm

C. 4 cm

What term refers to the constriction of the pupils when a patient focuses on an object held about 10 centimeters from the nose? A. Ptosis B. Glaucoma C. Accommodation D. Peripheral vision

C. Accommodation Accommodation is the correct term for constriction of the pupils when focusing on an object held about 10 centimeters from the nose. Ptosis is drooping of the upper eyelid, not constriction of the pupils. Glaucoma is a condition that causes damage to the eye's optic nerve, not constriction of the pupils when focusing on an object 10 centimeters from the nose. Peripheral vision is the part of vision that occurs outside the center of the gaze, not constriction of the pupils.

Hearing a bruit in an artery is a sign of which of the following conditions? A. Adequate blood flow B. A clot C. An obstruction D. A pulse deficit

C. An obstruction Presence of a bruit is a sign of arterial obstruction. Presence of a bruit is not a sign of adequate blood flow, a clot or a pulse deficit

Auscultatory sites of the heart include: A. Systolic and diastolic murmurs B. Bruits and thrills C. Aortic, pulmonic, tricuspid, and mitral areas D. Extra heart sounds and splitting

C. Aortic, pulmonic, tricuspid, and mitral areas Rationale: Aortic, pulmonic, tricuspid, and mitral areas are normal auscultatory sites of the heart. Systolic and diastolic murmurs are not auscultatory sites of the heart. Murmurs are long sounds often caused by the backflow of blood. Bruits and thrills are not auscultatory sites of the heart. A bruit can be a sign of turbulent blood flow, and thrills are fine, rushing vibrations. Extra heart sounds and splitting are not auscultatory sites of the heart. Extra heart sounds can include ejection clicks and pericardial friction rubs.

Which symptom found when examining the head would be a cause for concern? A. Symmetrical features at rest B. Even distribution of hair C. Bruits in the temporal arteries D. Symmetrical features with movement

C. Bruits in the temporal arteries Bruits in the temporal arteries may indicate a vascular anomaly in the brain. Symmetrical facial features at rest, even distribution of hair, and symmetrical facial features with movement are expected findings.

2. An ABNORMAL angle between the nail base and the nail is called clubbing and may indicate which of the following conditions? A. Poor hygiene B. Dehydration C. Cardiopulmonary disorder D. Skin cancer

C. Cardiopulmonary disorder

The nurse can best determine the effect of crying on a patient's apical pulse by doing what? A. Measuring the patient's apical pulse before and after crying B. Assessing the patient's apical pulse 30 minutes after crying C. Comparing the patient's post-crying apical pulse rate with her baseline or previous rate D. Measuring the patient's pulse deficit after crying

C. Comparing the patient's post-crying apical pulse rate with her baseline or previous rate The comparison of apical pulse rates at these times is the best means of evaluating the effect of crying on the patient's apical pulse rate. These values would be available data to compare. It is unlikely that the nurse will have the opportunity to measure the patient's apical pulse before and after crying. The time interval of 30 minutes is too long to effectively assess the effect of the crying on the apical pulse. Pulse deficit indicates alterations in cardiac output, not the effect of the emotional reaction.

What is the most important information to document if a mass is palpated in the breast or axilla? A. Color and tenderness of surrounding tissue B. Shape, tenderness, mobility, and size of the breasts C. Consistency, borders, mobility, location, size, shape, tenderness, and retraction of the mass D. Characteristics of the nipple and lymph nodes

C. Consistency, borders, mobility, location, size, shape, tenderness, and retraction of the mass Document consistency, delineate borders, determine mobility, and note characteristics of location, size, shape, tenderness, and retraction of the mass if a mass is palpated in the breast or axilla. Color and tenderness of surrounding tissue are not part of documentation when a mass is palpated in the breast or axilla. Size of breasts is not documented when a mass is palpated in the breast or axilla. Characteristics of the nipple and lymph nodes are not part of documentation for a mass found in the breast or axilla.

Which of the following are basic guidelines when assisting a patient with passive range of motion? A. Exercises should be continued until the point of fatigue and pain. B. Exercises should be done frequently to lessen pain for the patient. C. Each joint is exercised to the point of resistance but not pain. D. Exercises should be performed without the support to each joint

C. Each joint is exercised to the point of resistance but not pain. ationale: Joints should be exercised slowly, smoothly, and rhythmically to the point of resistance, but pain should not be felt by the patient. Uncomfortable reactions should be reported. Joints should never be exercised to the point of fatigue or pain. Exercises should be done twice a day to improve joint mobility and increase circulation. Pain will not be lessened with exercising. Pain should not be felt by the patient. Use a variety of support measures, cupping with your hand under joint or cradling the distal portion with arm. Support measures prevent muscle strain or injury to the patient

Which lymph nodes are located in the depression above and posterior to the medial condyle of the humerus? A. Axillary lymph nodes B. Inguinal lymph nodes C. Epitrochlear lymph nodes D. Parotid lymph nodes

C. Epitrochlear lymph nodes Epitrochlear lymph nodes are found in the forearms. Axillary lymph nodes are found in the arms, in the axillary region. Inguinal lymph nodes are found in the inguinal region near the medial thigh. Parotid lymph nodes are found near the mandible

Which of the following is not included in a head-to-toe assessment? A. Near vision test B. Testing the spinal reflexes C. Exercise stress test D. Balance tests

C. Exercise stress test An exercise stress test is not included in a head-to-toe assessment. Near vision is tested with the Rosenbaum card. Spinal reflexes and balance are tested as part of the neurologic system assessment.

Which of the following is an important part of performing an abdominal assessment? A. Completing the assessment as quickly as possible B. Stopping the assessment if the patient has any tenderness C. Explaining each step of the assessment to the patient D. Having the patient breathe normally at all times

C. Explaining each step of the assessment to the patient Explaining each step of the assessment demonstrates respect for the patient and allows the patient to be informed of the assessment process. Abdominal assessment should be performed in a thorough manner, not as quickly as possible. Complaints of tenderness from the patient should be noted, and the complete abdominal assessment should be continued. For most parts of the assessment, the patient will breathe normally. There are instances when the patient will need to take a deep breath, such as when assessing the spleen and gastric air bubble

What is an increased thoracic curvature, common in older adults, called? A. Scoliosis B. Lordosis C. Kyphosis D. Swayback

C. Kyphosis

Which of the following cranial nerves is assessed by holding a scented object under the patient's nose? A. Facial nerve B. Oculomotor nerve C. Olfactory nerve D. Acoustic nerve

C. Olfactory nerve The olfactory nerve is assessed by having a patient close his or her eyes, inhale deeply, and identifying the smell. The facial nerve is assessed by observing the patient making specific facial movements. The oculomotor nerve is assessed by inspecting the eyelids and by checking the pupils. The acoustic nerve is assessed by performing the whispered voice test.

A nurse is inspecting the patient's ears with an otoscope. Which of the following findings would be considered abnormal? A. Visible cone of light B. Pearly gray tympanic membrane C. Perforation of the tympanic membrane D. Small amount of cerumen

C. Perforation of the tympanic membrane The tympanic membrane should not have any perforations. A visible cone of light is a normal finding in an otoscopic examination. The tympanic membrane should appear pearly gray. A small amount of cerumen is a normal finding in an otoscopic examination.

A patient with a herniated disk is scheduled for surgery to fuse two vertebrae in her cervical spine. Which activity is most likely to be a palliative factor for this patient? A. Repainting her new apartment B. Lifting moving boxes on and off of a truck C. Performing neck, back, and shoulder exercises prescribed by a physical therapist D. Performing yoga exercises from the patient's favorite set of videotapes

C. Performing neck, back, and shoulder exercises prescribed by a physical therapist ationale: Performing moderate exercises appropriate for a patient with this kind of injury is likely to palliate the pain. Painting is likely to hyperextend the neck; this activity is likely to provoke the pain, not palliate it. Heavy lifting is likely to provoke the pain, not palliate it. Although physical activity in general and yoga in particular might be helpful, this patient's pain will most likely be palliated by exercises designed with her specific abilities and limitations in mind

The nurse notices that his patient has none of the signs and symptoms normally associated with pain, such as diaphoresis, tachycardia, and hypertension. The patient does, however, seem moody and a bit uncooperative. What conclusion does the nurse draw? A. It is likely the patient is a drug seeker and has little or no pain. B. The patient's problem is more mental than physical. C. The absence of physiological signs and symptoms is associated with chronic pain. D. The patient's pain cannot be accurately assessed until the patient has been treated for anxiety

C. The absence of physiological signs and symptoms is associated with chronic pain ationale: Patients with persistent, long-standing pain do not usually exhibit physical signs and symptoms of pain. The patient's self-report of pain is the single most reliable indicator of pain and cannot be dismissed. Furthermore, the nurse has no evidence that the patient is a drug seeker. Patients with chronic pain are often depressed and irritable, and the nurse has no evidence that the patient is mentally ill. This patient may be depressed, but does not appear to be anxious. Furthermore, a patient with depression or anxiety can nevertheless give an accurate self-report of his or her pain

What assessment should the nurse include? Check for jugular vein distention. An S3 heart sound may be an early indicator of the onset of heart failure, so the nurse should assess the client for other signs of heart failure, including jugular vein distention. Note the onset of nailbed clubbing. Nailbed clubbing is a sign of chronic hypoxemia, rather than a finding observed during an acute change in condition. Check for diminished skin elasticity. Decreased skin elasticity is typically related to loss of fluid volume. An S3 heart sound is not related to loss of fluid volume. Assess for orthostatic hypotension. The onset of an S3 heart sound is unlikely to be related to orthostatic hypotension.

Check for jugular vein distention. An S3 heart sound may be an early indicator of the onset of heart failure, so the nurse should assess the client for other signs of heart failure, including jugular vein distention

Why would the nurse ask a physical therapist to perform passive ROM exercises for a patient with lower extremity injuries sustained in a motor vehicle crash? A. The patient is an older adult or has a chronic condition. B. The patient is reluctant to perform the exercises because he is worried about reinjury. C. The patient has orthopedic trauma. D. The patient has pain exacerbated by exercise

C. The patient has orthopedic trauma Rationale: Specialized expertise is usually required to perform passive ROM exercises for a patient with orthopedic trauma or spinal cord injury. The patient's age or the existence of a chronic medical condition generally does not necessitate additional expertise. If a patient is concerned about injury, the nurse can address the patient's anxiety by explaining the procedure, easing into the exercises, and offering continual reassurance. If the provider is aware that the patient's pain is worse with exercise and the provider nevertheless orders the exercise, the nurse can offer pain medication before the intervention, as prescribed, and exercise the patient as tolerated

The nurse is percussing a patient's abdomen. What sound will the nurse most likely percuss? A. Resonance B. Dullness C. Tympany D. Flatness

C. Tympany

Screening for endometrial cancer consists of reinforcing the need to report: A. Bloody stools B. Painful bowel movements C. Unexpected vaginal bleeding or spotting D. Green or yellow vaginal discharge

C. Unexpected vaginal bleeding or spotting Unexpected vaginal bleeding or spotting can be a sign of endometrial cancer. Bloody stools are not a sign of endometrial cancer; this can be a sign of colon cancer. Painful bowel movements are not a sign of endometrial cancer; these can be a sign of colon cancer. Green or yellow vaginal discharge is not a sign of endometrial cancer; this can be a sign of vaginitis.

Which assessment provides the most useful data related to the client's current nutritional status? Calculate the client's body mass index (BMI). Body mass index (BMI) is a marker of the client's optimal weight for his height and provides important data related to the client's current nutritional status. Ask the client about any recent changes in his appetite. This subjective data supports more specific and objective data related to the client's current nutritional status, but it is not the most important data to obtain. Assist the client to complete a 24-hour diet recall. This subjective data supports more specific and objective data related to the client's current nutritional status, but it is not the most important data to obtain. Check the client's serum hemoglobin A1c level. The serum hemoglobin A1c level provides useful data about long term control of blood glucose levels, but it does not provide the most useful data about the client's overall nutritional status.

Calculate the client's body mass index (BMI). Body mass index (BMI) is a marker of the client's optimal weight for his height and provides important data related to the client's current nutritional status.

What stimuli should the RN use first to attempt to elicit a response from the client? Pinch the client's trapezius muscle. Another action should be taken first. Vigorously shake the client's shoulder. Another action should be taken first. Lightly touch the client's arm. Another action should be taken first. Call the client's name. The RN should begin with the least amount of stimulus and progress to the greatest amount of stimulus, observing the amount of stimulus needed to evoke a response by the client.

Call the client's name. The RN should begin with the least amount of stimulus and progress to the greatest amount of stimulus, observing the amount of stimulus needed to evoke a response by the client.

What questions should the nurse ask next? Can you identify which spicy foods cause a problem? This information will be helpful in planning interventions for meal preparation. How often do you eat spicy foods? Other information is more useful in assessing the client's inability to tolerate spicy foods. What happens when you eat spicy foods? Claudine's response is the most useful regarding the nature of her inability to eat spicy foods and any underlying problems. Do you remember when you developed this intolerance to spicy foods? This information is useful in assessing the client's inability to eat spicy foods. Why do you think spicy foods are a problem? This question not helpful in obtaining vital information

Can you identify which spicy foods cause a problem? This information will be helpful in planning interventions for meal preparation What happens when you eat spicy foods? Claudine's response is the most useful regarding the nature of her inability to eat spicy foods and any underlying problems. Do you remember when you developed this intolerance to spicy foods? This information is useful in assessing the client's inability to eat spicy foods

What is the best way for Lourdes to identify the affected or at risk extremity? Tie a string around the affected extremity. This is not an effective method of identification. If the string is too tight, it may constrict collateral circulation and/or make existing lymphedema worse. Put the affected extremity in a sling or brace. There is no evidence that this is an effective method of identification. Carry a wallet card or ID bracelet that identifies the affected extremity. This is the most effective way to remind HCP to avoid venipunctures, blood pressures, and fingersticks on the affected extremity. Have the HCP call first. While notification may be effective, this is not the best and most efficient way to identify the affected extremity.

Carry a wallet card or ID bracelet that identifies the affected extremity. This is the most effective way to remind HCP to avoid venipunctures, blood pressures, and fingersticks on the affected extremity.

What action should the nurse take? Reassure the client that his lung sounds are improving. This assessment does not reflect an improvement in the client's lung sounds. Document that the lung bases are clear upon auscultation. This is not accurate documentation of the findings. Chart what was heard both anteriorly and posteriorly. Although abnormal breath sounds in the lung bases may be heard only from the posterior, it is important to chart the sounds heard both anteriorly and posteriorly. Record only the location of the abnormal breath sounds. This does not provide the most complete documentation of the assessment findings. Recording the presence of normal breath sounds is also important.

Chart what was heard both anteriorly and posteriorly. Although abnormal breath sounds in the lung bases may be heard only from the posterior, it is important to chart the sounds heard both anteriorly and posteriorly.

The registered nurse (RN) is caring for a client with a newly placed nasogastric tube (NGT). Once the placement of the NG tube is verified by x-ray, which technique should the RN use as a reliable method to ensure the NGT is not displaced? Check pH of aspirated stomach contents obtained from the NGT. Auscultate over the epigastrium while injecting air into the NGT. Disconnect and place the end of NGT in water to see if bubbles appear. Listen for hyperactive bowel sounds in all four quadrants of abdomen.

Check pH of aspirated stomach contents obtained from the NGT. Checking the pH of the aspirate is the best method to validate that the NGT is not displaced and should reveal an acidic pH of 1.5 to 3.5 due to presence of gastric acid.

Which test result can the nurse review to obtain the same information that might be obtained during precordial percussion? Creatine phosphokinase (CPK). This enzyme level becomes elevated following muscle damage and may be used with other serum lab tests to assess for possible myocardial muscle damage. Percussion of the precordium does not provide information related to myocardial damage. Carotid ultrasound. Chest percussion helps outline the borders of the heart to detect enlargement. A carotid ultrasound provides information related to occlusion of the carotid arteries. Percussion of the precordium is not used to evaluate the carotid arteries. Serum liver enzymes. Serum liver enzymes are used to evaluate possible liver damage. This information is not obtained during percussion of the precordium. Chest x-ray. Chest percussion helps outline the borders of the heart to detect enlargement. Enlargement of the heart is more accurately detected with a chest x-ray. Echocardiogram. Chest percussion helps outline the borders of the heart to detect enlargement. Enlargement of the heart is more accurately detected using an echocardiogram.

Chest x-ray. Chest percussion helps outline the borders of the heart to detect enlargement. Enlargement of the heart is more accurately detected with a chest x-ray. Echocardiogram. Chest percussion helps outline the borders of the heart to detect enlargement. Enlargement of the heart is more accurately detected using an echocardiogram.

A client is newly diagnosed with diverticulosis. The registered nurse (RN) is assessing the client's basic knowledge about the disease process. Which statement by the client conveys the client's understanding of the etiology of diverticula? Over use of laxatives for bowel regularity result in loss of peristaltic tone. Inflammation of the colon mucosa cause growths that protrude into the colon lumen. Diverticulosis is the result of high fiber diet and sedentary life style. Chronic constipation causes weakening of colon wall which result in out-pouching sacs.

Chronic constipation causes weakening of colon wall which result in out-pouching sacs. A client who has chronic constipation often strains to pass constipated stool which increases intestinal pressure that weakens the intestinal walls and causes out-pouching sacs, called diverticula which commonly occur in the sigmoid.

While reviewing the client's electronic medical record (EMR), the registered nurse (RN) assesses a client who is at risk for a possible interaction with an over-the-counter (OTC) decongestant. Which client health history should the RN report to the healthcare provider concerning the OTC medication? (Select all that apply). Type I diabetes mellitus (DM). Closed angle glaucoma. Chronic hypertension. Rheumatoid arthritis. Crohn's disease.

Closed angle glaucoma. Chronic hypertension OTC decongestants can increase intraocular pressure and should be avoided in clients with closed angle glaucoma. Decongestants also can increase the heart rate and elevate blood pressure which can impact the client's management of chronic hypertension.

What additional assessment will provide supporting data related to hypoxemia? Color of palms and soles. Cyanosis, a bluish tinge typically signifying tissue hypoxia, is most evident in the nail beds, lips, and buccal mucosa. In dark-skinned persons, cyanosis may also be evident in the palms of the hands and the soles of the feet. Evidence of lower leg swelling. Edema, or swelling, indicates venous insufficiency, but it is not, by itself, a sign of hypoxemia. Presence and location of chest hair. The presence or absence of chest hair and its location on the chest are not indicators of the client's oxygenation status. Multiple thoracic hemangiomas. Hemangiomas are benign vascular lesions, and they are not reflective of the client's oxygenation. Shape of the fingers and fingertips. Nail clubbing is associated with chronic hypoxemia. It develops in stages including fluctuation and softening of the nail, loss of the normal angle, increased convexity of the nail fold, thickening of the distal end of the finger, and a shiny aspect and striation of the nail.

Color of palms and soles. Cyanosis, a bluish tinge typically signifying tissue hypoxia, is most evident in the nail beds, lips, and buccal mucosa. In dark-skinned persons, cyanosis may also be evident in the palms of the hands and the soles of the feet. Shape of the fingers and fingertips. Nail clubbing is associated with chronic hypoxemia. It develops in stages including fluctuation and softening of the nail, loss of the normal angle, increased convexity of the nail fold, thickening of the distal end of the finger, and a shiny aspect and striation of the nail.

During report, the nurse also describes the client's earlier emesis. The nurse should describe the emesis in terms of which characteristics? Color. It is important for the nurse to describe the appearance of the emesis, which includes the color. Odor. It is important for the nurse to describe any odor of the emesis, which could indicate the presence of blood, undigested foods, or fecal contaminant. Intensity. This is not characteristics of emesis. Volume. It is important for the nurse to describe the volume or amount of emesis. Duration. The duration will describe if the episodes of vomiting were short, sporadic, ongoing, or intermittent.

Color. It is important for the nurse to describe the appearance of the emesis, which includes the color. Odor. It is important for the nurse to describe any odor of the emesis, which could indicate the presence of blood, undigested foods, or fecal contaminant. Volume. It is important for the nurse to describe the volume or amount of emesis. Duration. The duration will describe if the episodes of vomiting were short, sporadic, ongoing, or intermittent.

How should the nurse report the assessment data? Provide the internist with a full report of the initial assessment data obtained upon the client's admission only. This report will not provide the internist with the best information about the client's status. Describe only the most current assessment data and the changes observed after the client's temperature increased. This report will not provide the internist with the best information about the client's status. Compare the current assessment of the client to the data obtained during the admission assessment of the client. This report will provide the most complete client data, enabling the internist to make the most effective decisions about any changes needed in the client's medical care. Notify the internist that the client's condition has changed, but avoid giving specific data until the internist assesses the client. This report will not provide the internist with the best information about the client's status.

Compare the current assessment of the client to the data obtained during the admission assessment of the client. This report will provide the most complete client data, enabling the internist to make the most effective decisions about any changes needed in the client's medical care.

How are expected findings for the musculoskeletal system determined during an examination?

Compare the patient's left side with the right side.

What follow-up action should the nurse implement Compare this finding with the location of the client's pneumonia seen on x-ray. Dullness upon percussion should be anticipated over areas of abnormal density, including pneumonia. The nurse can confirm this assessment finding by reviewing the location of the client's pneumonia found on x-ray. Review the client's medical history to determine how long he has had emphysema. Overly inflated alveoli found in clients with emphysema may cause hyperresonance upon percussion, rather than dullness. Document this normal assessment finding in the client's admission assessment. Resonance upon percussion is the predominant sound in healthy adult lung tissue. Notify the healthcare provider (HCP) that the client may have developed a pneumothorax. A pneumothorax may cause hyperresonance upon percussion rather than dullness.

Compare this finding with the location of the client's pneumonia seen on x-ray. Dullness upon percussion should be anticipated over areas of abnormal density, including pneumonia. The nurse can confirm this assessment finding by reviewing the location of the client's pneumonia found on x-ray.

Because of Ms. Yong's history of knee pain and current report of low back pain, which nursing action is most useful in developing an initial plan of care for the client? Obtain a family medical history. Gathering the client's family history is important, but another action is of greater priority in developing an initial plan of care of this client. Complete a functional assessment. A functional assessment provides information about the client's ability to function and includes areas such as mobility and self-care measures. It is most important to gather this information to determine the client's level of safety in basic functioning. Observe for callus formation. Calluses form because of prolonged pressure and are unlikely to occur as the result of low back pain or knee pain, so this assessment is of low priority in developing a plan of care for this client. Ask about any recent weight gain. Weight gain may be related to both low back pain and joint pain, but another assessment is of greater priority in developing a plan of care for this client.

Complete a functional assessment. A functional assessment provides information about the client's ability to function and includes areas such as mobility and self-care measures. It is most important to gather this information to determine the client's level of safety in basic functioning.

Although there is no visible swelling, Lourdes' legs are large, so the nurse gently depresses the tissue over the tibia for one second, noting that the tissue bounces back immediately. What action should the nurse take next? Document the presence of non-pitting edema. Large legs without visible swelling do not indicate the presence of non-pitting edema. Large legs may also be the result of excess fatty tissue. Note that there is currently no edema present. Another action should be taken before this action can be taken. Ask the client to elevate her legs and repeat. This action is not useful in determining the presence or absence of edema. Compress the tissue more firmly for 5 seconds. To effectively assess for pitting edema, the RN should firmly depress the tissue for 5 seconds, release, and measure any resultant indentation

Compress the tissue more firmly for 5 seconds. To effectively assess for pitting edema, the RN should firmly depress the tissue for 5 seconds, release, and measure any resultant indentation

What action should the nurse implement? Reassure Lourdes that the swelling has decreased slightly. This action provides Lourdes with incorrect information. Confirm that Lourdes' arm is more swollen than previously. Pitting edema of 1+ indicates mild pitting, or pitting of 2 mm.Pitting edema of 2+ indicates moderate edema, or pitting of 4 mm. This finding reflects that Lourdes' arm is more swollen than during the previous assessment. Document that the swelling has doubled since the last assessment. The depth of the pitting has doubled from 2 mm to 4 mm, but this is not documented as a "doubling" of the swelling. Note that the indentation is half as deep as the previous assessment. This action is incorrect.

Confirm that Lourdes' arm is more swollen than previously. Pitting edema of 1+ indicates mild pitting, or pitting of 2 mm.Pitting edema of 2+ indicates moderate edema, or pitting of 4 mm. This finding reflects that Lourdes' arm is more swollen than during the previous assessment.

The nurse observes that Amanda's skin pigmentation is deeply tanned. To evaluate the client for pallor, what area should the nurse assess? Earlobes. The ear lobes do not typically exhibit observable pallor. Hair follicles. The hair follicles do not exhibit pallor. Cheeks and chin. Although pallor can be assessed on the face, since Amanda's skin is deeply tanned, this is not the best site to use when assessing for paleness. Conjunctivae. Because paleness of the skin can be difficult to detect in persons with dark or tanned skin, the membranes that line the eyelids (conjunctivae) are a good area to assess for pallor.

Conjunctivae. Because paleness of the skin can be difficult to detect in persons with dark or tanned skin, the membranes that line the eyelids (conjunctivae) are a good area to assess for pallor.

A female client calls the clinic and talks with the registered nurse (RN) to inquire about a possible reaction after taking amoxicillin for 5 days. She reports having vaginal discomfort, itching, and a white discharge. The RN should discuss which action with the client? Discontinue the antibiotic because original symptoms have subsided. Continue taking medication until finished until the symptoms subside. Consult with healthcare provider about another treatment for this effect. Use an over-the-counter (OTC) vaginal wash to flush out the secretions.

Consult with healthcare provider about another treatment for this effect. A superinfection with normal flora yeast may occur during antibiotic therapy. If suspected, the new onset of findings should be reported to the healthcare provider for another prescribed treatment to treat the superinfection.

The registered nurse (RN) reviews the new prescription, phenelzine (Nardil), a monoamine oxidase inhibitor (MAOI), for a client on the psychiatric unit with depression. Which information is most important for the RN to assess? Consumptiion of any alcohol or tyramine-rich foods. Complaints of nausea or vomiting. Therapeutic serum drug levels. Blood pressure and pulse prior to taking each dose.

Consumptiion of any alcohol or tyramine-rich foods. The consumption of any type of tyramine containing foods such as aged cheeses, fermented fruits and vegetables, smoked or cured meats, dark wines and other alcoholic products should be avoided when a client is prescribed a MAOIs due to the a food-drug interaction causing a hypertensive crisis which can lead to a hemorrhagic stroke.

The nurse observes that the nail surface is slightly curved and the angle of the nail base is 160 degrees. What action should the nurse take in response to this finding? Ask the client about any current or past use of cigarettes. The assessment findings do not indicate use of cigarettes. Continue the assessment, noting the color of the nail surface. A slightly curved nail surface is a normal finding. The normal nail base angle is 160 degrees. Since these findings are within normal parameters, the nurse should continue the assessment by observing the color of the nail surface. Use a pulse oximeter to measure the oxygen saturation. The assessment findings do not indicate a problem with oxygenation. Assess for crepitus underneath and around the nail surface. Crepitus, a crackling or clicking noise, would not be heard under or around nail surfaces.

Continue the assessment, noting the color of the nail surface. A slightly curved nail surface is a normal finding. The normal nail base angle is 160 degrees. Since these findings are within normal parameters, the nurse should continue the assessment by observing the color of the nail surface

What action should the nurse take next? Move to the right upper quadrant (RUQ) to hear the sounds more distinctly. Irregular, high-pitched sounds are expected, so moving to another quadrant to hear more distinct sounds is not indicated. Continue to listen over the RLQ for 5 to 15 seconds. The pattern of bowel sounds is typically irregular and the duration of bowel sounds may range from 1 second to several seconds. Bowel sounds should be noted every 5 to 15 seconds. Change to the bell of the stethoscope to listen. The diaphragm of the stethoscope should be used to listen to bowel sounds. Listen for 5 minutes before documenting the activity of the bowel sounds. If bowel sounds are not heard, the nurse should listen for 5 minutes to allow sufficient time before documenting the absence of bowel sounds. If bowel sounds are heard, it is not necessary to continue to listen for 5 minutes.

Continue to listen over the RLQ for 5 to 15 seconds. The pattern of bowel sounds is typically irregular and the duration of bowel sounds may range from 1 second to several seconds. Bowel sounds should be noted every 5 to 15 seconds.

Ms. Yong is able to move her upper extremities through complete range of motion. In documenting full range of motion of the upper extremities, the nurse is able to note the absence of which abnormality? Arthritis. Arthritis may be present in individuals with full range of motion. Kyphosis. Kyphosis is an abnormal curvature of the spine. Flaccidity. Flaccid muscles are weak and fatigue easily; however, the client may still be able to demonstrate full range of motion. Contracture. A contracture is a shortening of a muscle resulting in limited range of motion. Full range of motion indicates that no contractures are present.

Contracture. A contracture is a shortening of a muscle resulting in limited range of motion. Full range of motion indicates that no contractures are present.

A woman in labor suffers from preeclampsia. Nurses in the labor and delivery unit need to assess her blood pressure. The nurse explains to the patient that: A. Using a cuff that is too narrow will give a reading that is inaccurate and high. B. Deflating the cuff too quickly will make the reading inaccurate and high. C. Deflating the cuff 5 mm Hg per second will make the reading inaccurate and high. D. Waiting 3 minutes before repeating the blood pressure measurement will result in a false-high blood pressure measurement.

Correct Answer: A. Using a cuff that is too narrow will give a reading that is inaccurate and high. Rationale: Cuff width should be 40% of the circumference of the limb to be used to assess blood pressure. Quick deflation (faster than 2 mm Hg to 3 mm Hg per second) makes the blood pressure measurement inaccurate and low. Waiting 3 minutes between repeat blood pressure measurements will help ensure an accurate second reading.

The nurse is working in a primary care clinic. She walks into the room, and the general inspection begins. What is not part of the general inspection? A. Patient's facial expressions are consistent with verbalized emotions. B. Patient is wearing clothes that are normally worn by whites. C. Patient is staring down at the floor through most of the interview. D. Patient's gait is strong and symmetrical.

Correct Answer: B. Patient is wearing clothes that are normally worn by whites. Rationale: Cultural assessment is important. However, nurses must be careful to make assumptions, generalizations, or both. In America, the common dress of people of many cultures is the same.

The nurse collects patient data through assessment of vital signs. Many nurses will delegate the performance of temperature data collection to unlicensed assistive personnel. As the nurse talks to the assistant, the nurse knows to teach that: A. Tympanic thermometers touch the tympanic membrane. B. Axillary temperatures are taken with the red probe on the electronic thermometer. C. Axillary temperatures are usually most accurate because of the local blood supply. D. Rectal thermometers are placed 2.8 cm to 3.5 cm into the rectum.

Correct Answer: D. Rectal thermometers are placed 2.8 cm to 3.5 cm into the rectum Rationale: Tympanic thermometers need to seal the ear canal but do not touch the tympanic membrane. Red tipped probes indicate rectal temperatures only. Axillary temperatures are considered the least accurate.

Which method of temperature measurement does a nurse choose when assessing children? Select all that apply. a. Axillary temperature b. Rectal temperature c. Temporal artery temperature d. Oral temperature e. Tympanic membrane temperature

Correct: Axillary, temporal artery, oral, and tympanic membrane temperatures are appropriate for children. Incorrect: Rectal temperature measurement is considered safe and accurate for adults only.

When recording the change in the client's assessment findings, how should the nurse document the breath sounds Adventitious breath sounds present in the middle and lower lungs bilaterally. This documentation does not provide the most complete assessment data. Client's posterior breath sounds have worsened from the earlier assessment. This documentation does not provide the most complete assessment data. Crackles heard bilaterally in the middle and lower lung fields posteriorly. This documentation provides a clear, concise picture of the current assessment findings. Bilateral normal breath sounds heard only in the upper lobes posteriorly. This documentation does not provide the most complete assessment data.

Crackles heard bilaterally in the middle and lower lung fields posteriorly. This documentation provides a clear, concise picture of the current assessment findings.

72. The nurse is taking the history of a patient who may have a perforated eardrum. What would be an important question in this situation? A) "Do you ever notice ringing or crackling in your ears?" B) "When was the last time you had your hearing checked?" C) "Have you ever been told you have any type of hearing loss?" D) "Was there any relationship between the ear pain and the discharge you mentioned?"

D) "Was there any relationship between the ear pain and the discharge you mentioned?" Pages: 327-328. Typically with perforation, ear pain occurs first, stopping with a popping sensation, and then drainage occurs.

18. When providing culturally competent care, nurses must incorporate cultural assessments into their health assessments. Which statement is most appropriate to use when initiating an assessment of cultural beliefs with an elderly American Indian patient? A) "Are you of the Christian faith?" B) "Do you want to see a medicine man?" C) "How often do you seek help from medical providers?" D) "What cultural or spiritual beliefs are important to you?"

D) "What cultural or spiritual beliefs are important to you?" Page: 17. The nurse needs to assess the cultural beliefs and practices of the patient. American Indians may seek assistance from a medicine man or shaman, but the nurse should not assume this. An open-ended question regarding cultural and spiritual beliefs is best used initially when performing a cultural assessment.

44. When assessing the quality of a patient's pain, the nurse should ask which question? A) "When did the pain start?" B) "Is the pain a stabbing pain?" C) "Is it a sharp pain or dull pain?" D) "What does your pain feel like?"

D) "What does your pain feel like?" Page: 164. To assess the quality of a person's pain, have the patient describe the pain in his or her own words.

118. During the history of a 78-year-old man, his wife states that he occasionally has problems with short-term memory loss and confusion: "He can't even remember how to button his shirt." In doing the assessment of his sensory system, which action by the nurse is most appropriate? A) The nurse would not do this part of the examination because results would not be valid. B) The nurse would perform the tests, knowing that mental status does not affect sensory ability. C) The nurse would proceed with the explanations of each test, making sure the wife understands. D) Before testing, the nurse would assess the patient's mental status and ability to follow directions at this time.

D) Before testing, the nurse would assess the patient's mental status and ability to follow directions at this time. The nurse should ensure validity of the sensory system testing by making sure the patient is alert, cooperative, comfortable, and has an adequate attention span. Otherwise, the nurse may obtain misleading and invalid results.

54. A 65-year-old man with emphysema and bronchitis has come to the clinic for a follow-up appointment. On assessment, the nurse might expect to see which assessment finding? A) Anasarca B) Scleroderma C) Pedal erythema D) Clubbing of the nails

D) Clubbing of the nails Pages: 217-218. Clubbing of the nails occurs with congenital cyanotic heart disease, neoplastic, and pulmonary diseases. The other responses are assessment findings not associated with pulmonary diseases.

92. During an assessment of a healthy adult, where would the nurse expect to palpate the apical impulse? A) Third left intercostal space at the midclavicular line B) Fourth left intercostal space at the sternal border C) Fourth left intercostal space at the anterior axillary line D) Fifth left intercostal space at the midclavicular line

D) Fifth left intercostal space at the midclavicular line Pages: 473-474. The apical impulse should occupy only one intercostal space, the fourth or fifth, and it should be at or medial to the midclavicular line.

19. When planning a cultural assessment, the nurse should include which component? A) Family history B) Chief complaint C) Medical history D) Health-related beliefs

D) Health-related beliefs Pages: 19-20. Health-related beliefs and practices are one component of a cultural assessment. The other items reflect other aspects of the patient's history.

119. In assessing a 70-year-old patient who has had a recent cerebrovascular accident, the nurse notices right-sided weakness. What might the nurse expect to find when testing his reflexes on the right side? A) Lack of reflexes B) Normal reflexes C) Diminished reflexes D) Hyperactive reflexes

D) Hyperactive reflexes Hyperreflexia is the exaggerated reflex seen when the monosynaptic reflex arc is released from the influence of higher cortical levels. This occurs with upper motor neuron lesions (e.g., a cerebrovascular accident). The other responses are incorrect

2. During an interview, the nurse states, "You mentioned shortness of breath. Tell me more about that." Which verbal skill is used with this statement? A) Reflection B) Facilitation C) Direct question D) Open-ended question

D) Open-ended question Page: 32 The open-ended question asks for narrative information. It states the topic to be discussed but only in general terms. The nurse should use it to begin the interview, to introduce a new section of questions, and whenever the person introduces a new topic.

103. The nurse suspects that a patient has a distended bladder. How should the nurse assess for this condition? A) Percuss and palpate in the lumbar region. B) Inspect and palpate in the epigastric region. C) Auscultate and percuss in the inguinal region. D) Percuss and palpate the midline area above the suprapubic bone.

D) Percuss and palpate the midline area above the suprapubic bone. Pages: 539-540. Dull percussion sounds would be elicited over a distended bladder, and the hypogastric area would seem firm to palpation.

102. Which structure is located in the left lower quadrant of the abdomen? A) Liver B) Duodenum C) Gallbladder D) Sigmoid colon

D) Sigmoid colon Page: 530. The sigmoid colon is located in the left lower quadrant of the abdomen.

35. The nurse is preparing to assess a patient's abdomen by palpation. How should the nurse proceed? A) Avoid palpation of reported "tender" areas because this may cause the patient pain. B) Quickly palpate a tender area to avoid any discomfort that the patient may experience. C) Begin the assessment with deep palpation, encouraging the patient to relax and take deep breaths. D) Start with light palpation to detect surface characteristics and to accustom the patient to being touched.

D) Start with light palpation to detect surface characteristics and to accustom the patient to being touched. Pages: 115-116. Light palpation is performed initially to detect any surface characteristics and to accustom the person to being touched. Tender areas should be palpated last, not first.

57. The nurse is assessing for clubbing of the fingernails and would expect to find: A) a nail base that is firm and slightly tender. B) curved nails with a convex profile and ridges across the nail. C) a nail base that feels spongy with an angle of the nail base of 150 degrees. D) an angle of the nail base of 180 degrees or greater with a nail base that feels spongy.

D) an angle of the nail base of 180 degrees or greater with a nail base that feels spongy. Pages: 217-218. The normal nail is firm at its base and has an angle of 160 degrees. In clubbing, the angle straightens to 180 degrees or greater and the nail base feels spongy.

121. While the nurse is taking the history of a 68-year-old patient who sustained a head injury 3 days earlier, he tells the nurse that he is on a cruise ship and is 30 years old. The nurse knows that this finding is indicative of: A) a great sense of humor. B) uncooperative behavior. C) inability to understand questions. D) decreased level of consciousness.

D) decreased level of consciousness. Pages: 660-661. A change in consciousness may be subtle. The nurse should notice any decreasing level of consciousness, disorientation, memory loss, uncooperative behavior, or even complacency in a previously combative person. The other responses are incorrect.

106. The physician comments that a patient has abdominal borborygmi. The nurse knows that this term refers to: A) a loud continuous hum. B) a peritoneal friction rub. C) hypoactive bowel sounds. D) hyperactive bowel sounds.

D) hyperactive bowel sounds. Pages: 539-540. Borborygmi is the term used for hyperperistalsis when the person actually feels his or her stomach growling.

70. An ophthalmic examination reveals papilledema. The nurse is aware that this finding indicates: A) retinal detachment. B) diabetic retinopathy. C) acute-angle glaucoma. D) increased intracranial pressure.

D) increased intracranial pressure. Pages: 319-320. Papilledema, or choked disk, is a serious sign of increased intracranial pressure, which is caused by a space-occupying mass such as a brain tumor or hematoma. This pressure causes venous stasis in the globe, showing redness, congestion, and elevation of the optic disc, blurred margins, hemorrhages, and absent venous pulsations. Papilledema is not associated with the conditions in the other responses.

117. When the nurse asks a 68-year-old patient to stand with feet together and arms at his side with his eyes closed, he starts to sway and moves his feet farther apart. The nurse would document this finding as a(n): A) ataxia. B) lack of coordination. C) negative Homans' sign. D) positive Romberg sign.

D) positive Romberg sign. Page: 638. Abnormal findings for Romberg test include swaying, falling, and widening base of feet to avoid falling. Positive Romberg sign is loss of balance that is increased by closing of the eyes. Ataxia is uncoordinated or unsteady gait. Homans' sign is used to test the legs for deep vein thrombosis.

4. During an interview, a parent of a hospitalized child is sitting in an open position. As the interviewer begins to discuss his son's treatment, however, he suddenly crosses his arms against his chest and crosses his legs. This would suggest that the parent is: A) just changing positions. B) more comfortable in this position. C) tired and needs a break from the interview. D) uncomfortable talking about his son's treatment.

D) uncomfortable talking about his son's treatment. Page: 37 Note the person's position. An open position with the extension of large muscle groups shows relaxation, physical comfort, and a willingness to share information. A closed position with the arms and legs crossed tends to look defensive and anxious. Note any change in posture. If a person in a relaxed position suddenly tenses, it suggests possible discomfort with the new topic.

1. Put in correct order the steps used to palpate the liver. A. Place your right hand parallel to the right costal margin. B. Ask the patient to take a deep breath. C. Place your right hand parallel to the right costal margin. D. Lift up the eleventh and twelfth ribs with the left hand. E. Press your right hand down and under the coastal margin. F. Ask the patient to take some deep breaths

D, C, A, E, B, F

22. The mother of a 16-month-old toddler tells the nurse that her daughter has an earache. What would be an appropriate response? A. "Maybe she is just teething." B. "I will check her ear for an ear infection." C. "Are you sure she is really having pain?" D. "Please describe what she is doing to indicate she is having pain."

D. "Please describe what she is doing to indicate she is having pain." Page: 60. With a very young child, ask the parent, "How do you know the child is in pain?" Pulling at ears alerts parent to ear pain. The statements about teething and questioning whether the child is really having pain do not explore the symptoms, which should be done before a physical examination.

Which statement demonstrates an understanding of the importance of communicating changes in the patient's apical pulse rate? A. "The patient's apical pulse is recorded as you asked." B. "The apical pulse is more difficult to hear when the patient is sitting up." C. "The apical pulse is usually slower in the morning than it is in the afternoon." D. "The apical pulse increased from 78 to 110, but the patient had just returned from the bathroom."

D. "The apical pulse increased from 78 to 110, but the patient had just returned from the bathroom." This statement identifies a significant change in the patient's apical pulse rate and the reason for the deviation in the rate. Recording the patient's apical pulse pertains to documentation, not to changes in apical pulse rate. While sitting up may make it more difficult to hear the apical pulse this may be true for an individual patient, it does not pertain to changes in the patient's apical pulse rate. The general statement that the apical pulse is usually slower in the morning than it is in the afternoon may or may not be accurate; it does not pertain to a significant change in the patient's apical pulse rate

Neck flexion and extension should be: A. 90 degrees B. 70 degrees C. 30 degrees D. 45 degrees

D. 45 degrees

The nurse is percussing the liver of an obese patient. Which percussion finding would be expected? A. Tones with a booming quality B. An enhanced tone quality C. A higher pitch tone than in patients of a normal weight D. A reduced intensity of tone

D. A reduced intensity of tone

For which reflex is the normal response plantar flexion of the foot? A. Brachioradialis B. Patellar C. Triceps D. Achilles

D. Achilles For the Achilles reflex, the examiner strikes the Achilles tendon at the level of the ankle malleoli; in response, the foot should plantar flex. The normal response for the brachioradialis reflex is forearm pronation and elbow flexion. The normal response for the patellar reflex is lower leg extension. The normal response for the triceps reflex is arm extension at the elbow.

Skin inspection and palpation includes assessment for: A. Color, uniformity, and symmetry B. Skin lesions C. Skin temperature D. All of the above

D. All of the above

The ABCD rule of melanoma includes: A. Asymmetry of shape B. Border irregularity and color variation C. Diameter larger than the eraser of a pencil D. All of the above

D. All of the above

The S1 heart sound: A. Is louder at the apex of the heart B. Results from closure of the mitral and tricuspid valves C. Marks the start of systole D. All of the above

D. All of the above All of these statements accurately describe the S1 heart sound. The S1 heart sound is heard best at the apex of the heart. The S1 heart sound results from closure of the mitral and tricuspid valves. The S1 heart sound marks the start of systole.

When palpating the thorax, which of the following would be an abnormal finding? A. Tenderness B. Pulsations C. Masses D. All of the above

D. All of the above All three of the answers are considered abnormal findings. Tenderness, pulsations, and masses are not a normal finding when palpating the thorax.

What information should be included when entering documentation of an enlarged lymph node? A. Location, size, and shape B. Consistency and tenderness C. Discreteness and movability D. All of the above

D. All of the above Documentation of location, size, shape, consistency, tenderness, discreteness, and movability are all aspects of a thorough lymph node assessment.

When auscultating the lungs, it is important to: A. Compare each side bilaterally. B. Note abnormal sounds. C. Ask the patient to take slow, deep breaths. D. All of the above

D. All of the above Rationale: All three of the answers are important considerations during auscultation of the lungs. Comparing each side of the lungs bilaterally during auscultation will allow the nurse to assess whether there is decreased aeration in any of the lung fields. Noting abnormal sounds during auscultation and documenting where the abnormal sounds are heard is part of a complete assessment of the lungs. Asking the patient to take slow, deep breaths will assist in thorough auscultation of all lung fields and help keep the patient from hyperventilating.

Moderate and deep palpation of the abdomen: A. May cause tenderness B. Should not detect masses C. May locate the margins of the liver D. All of the above

D. All of the above The patient may report tenderness with deep palpation that was not there during light palpation. Deep palpation may cause tenderness over the cecum, sigmoid colon, aorta, and xiphoid process. In a healthy patient, deep palpation should not detect masses. Palpate with the side of your hand over the liver and spleen; these organs should bump into your hand with inspiration

What does a goniometer measure? A. Muscle strength B. Joint stability C. Cranial nerve function D. Angles of extension and flexion

D. Angles of extension and flexion

Which of the following is part of monitoring and care? A. Explain that no additional screening will ever be needed. B. Stay in the room while the patient is returning to street clothes. C. Keep the patient's glasses until they are back in street clothes. D. Ask the patient if she/he has any questions or concerns about the exam.

D. Ask the patient if she/he has any questions or concerns about the exam. It is vital to allow the patient to ask any questions! Additional screenings may be needed, based on the findings of the exam. Explaining which screenings are needed is part of monitoring and care. Provide privacy for the patient while she/he returns to street clothes. It is appropriate to return corrective lenses to the patient promptly at the conclusion of the exam.

Which of the following findings in a musculoskeletal assessment would be considered abnormal? A. Nodules B. Bogginess C. Symmetry D. Both A and B

D. Both A and B

When palpating the carotid arteries: A. Palpate one artery at a time. B. Feel for thrills. C. Use the thumb to palpate. D. Both A and B

D. Both A and B Palpating one artery at a time and feeling for thrills are part of a complete assessment of the carotid arteries. Palpating one carotid artery at a time is part of a complete assessment of the carotid arteries. One artery is palpated at a time to avoid occlusion of blood flow in the carotid arteries. Palpation for the presence of thrills is part of a complete assessment of the carotid arteries. Palpation is performed with the index and middle fingers.

When percussing the thorax, which of the following would be a normal finding? A. Dullness over the lung fields B. Resonance over the lung fields C. Dullness over the ribs, heart, and diaphragm D. Both B and C

D. Both B and C Rationale: Both B and C are normal findings. Dullness over the lung fields during percussion is not a normal finding. Resonance over the lung fields during percussion is a normal finding. Dullness over the ribs, heart, and diaphragm during percussion is a normal finding.

When assessing lymph nodes, it is important to do which of the following? A. Compare lymph nodes bilaterally. B. Use the thumbs to palpate. C. Provide privacy for the patient. D. Both comparing the lymph nodes bilaterally and providing privacy for the patient.

D. Both comparing the lymph nodes bilaterally and providing privacy for the patient. Both comparing the lymph nodes bilaterally and providing privacy for the patient are important for lymph node assessment. Comparison to determine abnormality on one or both sides is important for lymph node assessment. Lymph nodes are palpated using the pads of the second and third fingers. Providing patient privacy is an important part of physical assessment

Which are the best places to check the skin for tenting, which is a sign of dehydration? A. Top of the hand and foot B. Neck and top of the head C. Shoulder and thigh D. Forearm and sternum

D. Forearm and sternum

24. The nurse is preparing to do a functional assessment. Which statement best describes the purpose of a functional assessment? A. It assesses how the individual is coping with life at home. B. It determines how children are meeting developmental milestones. C. It can identify any problems with memory the individual may be experiencing. D. It helps to determine how a person is managing day-to-day activities.

D. It helps to determine how a person is managing day-to-day activities. Page: 67. The functional assessment measures how a person manages day-to-day activities. The other answers do not reflect the purpose of a functional assessment.

In which arteries are bruits considered normal? A. Carotid arteries B. Temporal arteries C. Aortic artery D. None of the above

D. None of the above

In which arteries are bruits considered normal? A. Carotid arteries B. Temporal arteries C. Aortic artery D. None of the above

D. None of the above Presence of a bruit is a sign of arterial obstruction. It is not considered normal at any arterial site

21. In recording the childhood illnesses of a patient who denies having had any, which note by the nurse would be most accurate? A. Patient denies usual childhood illnesses. B. Patient states he was a "very healthy" child. C. Patient states sister had measles, but he didn't. D. Patient denies measles, mumps, rubella, chickenpox, pertussis, and strep throat.

D. Patient denies measles, mumps, rubella, chickenpox, pertussis, and strep throat. Page: 51. Childhood illnesses include measles, mumps, rubella, chickenpox, pertussis, and strep throat. Avoid recording "usual childhood illnesses" because an illness common in the person's childhood may be unusual today (e.g., measles).

For which patient would the nurse instruct nursing assistive personnel (NAP) to weigh a patient with a bed scale? A. Patient with an ostomy device B. Patient with chronic renal failure who receives hemodialysis three times a week C. Patient who is using a walker after knee replacement surgery D. Patient who has heart failure and a consequent inability to bear weight

D. Patient who has heart failure and a consequent inability to bear weight A bed scale would be needed for a patient with heart failure and a consequent inability to bear weight. The patient is unable to stand without assistance and most likely would not be able to tolerate sitting for a chair weight. A bed scale would not be needed for the patient with an ostomy device. There is no reason why this patient cannot stand on a scale. A bed scale would not be needed for a patient with chronic renal failure who receives hemodialysis. If the patient is weak, a chair scale can be used. A bed scale would not be needed for a patient who is using a walker after knee replacement surgery. If the patient is unable to stand without assistance, a chair scale can be used.

What should you do if a patient is ticklish when you are palpating the abdomen? A. Distract the patient by talking to him or her. B. Do not palpate the abdomen in the upper quadrants. C. Do only deep palpation of all four quadrants. D. Place your hand over the patient's hand during palpation.

D. Place your hand over the patient's hand during palpation. Rationale: Place your hand over the patient's hand during palpation, leaving your fingers free to palpate. Palpate with a firm hand or place your hand over the patient's during palpation. All quadrants are palpated for a thorough abdominal assessment. The abdominal assessment begins with light palpation.

What instruction should the nurse give nursing assistive personnel (NAP) regarding the appropriate technique when measuring the adult patient's apical pulse? A. Document the patient's pulse rate and rhythm. B. Place the patient in the right lateral position before measuring the apical pulse. C. Review the patient's previous apical pulse measurements. D. Place your stethoscope at the fifth intercostal space over the left midclavicular line

D. Place your stethoscope at the fifth intercostal space over the left midclavicular line These anatomical landmarks allow correct placement of the stethoscope over the apex of the heart. Documenting the patient's pulse rate and rhythm pertains to documentation, not technique. The patient may assume a sitting or supine position before measuring the apical pulse. The heart is located to the left of the sternum. If unable to locate the point of maximal impulse (PMI), reposition the patient on the left side to hear the sounds more clearly. Reviewing the patient's previous apical pulse measurements does not pertain specifically to the technique or procedure itself.

Normal capillary refill is less than 2 seconds and is assessed by: A. Pressing on the radial artery until a pulse is no longer felt B. Palpating all superficial veins in the legs to check for tenderness C. Palpating all of the pulses bilaterally to check that they are equal D. Pressing on the nail bed until it blanches, and observing how quickly full color returns

D. Pressing on the nail bed until it blanches, and observing how quickly full color returns Pressing on the nail bed until it blanches and observing how quickly full color returns is the correct method for assessing normal capillary refill. Pressing on the radial artery does not assess capillary refill. Palpation of superficial veins or pulses do not assess capillary refill.

23. A 5-year-old boy is being admitted to the hospital to have his tonsils removed. Which information should the nurse collect before this procedure? A. The child's birth weight B. The age at which he crawled C. Whether he has had the measles D. Reactions to previous hospitalizations

D. Reactions to previous hospitalizations Assess how the child reacted to hospitalization and any complications. If the child reacted poorly, he or she may be afraid now and will need special preparation for the examination that is to follow. The other items are not significant for the procedure.

Which of the following findings during a cardiac assessment of an adult patient are considered normal? A. Ejection or systolic clicks B. Pericardial friction rubs C. Murmurs D. S1 and S2 sounds

D. S1 and S2 sounds Rationale: S1 and S2 sounds are normal findings in a cardiac assessment. S1 and S2 sounds are heart sounds that always occur and are the most distinct. Ejection and systolic clicks are not normal findings in a cardiac assessment. Ejection clicks result from faulty opening of the semilunar valves. Pericardial friction rubs are not normal findings in a cardiac assessment. These are extra heart sounds. Murmurs are not normal findings in a cardiac assessment. They are relatively long sounds caused by a backward flow of blood

A nurse is inspecting the patient's ears with an otoscope. Which of the following findings would be considered normal? A. A small plastic bead B. Brownish black tympanic membrane C. Perforation of the tympanic membrane D. Small amount of cerumen

D. Small amount of cerumen A small amount of cerumen is a normal finding in an otoscopic examination. The tympanic membrane should appear pearly gray and should not have any perforations. Insects or other objects may be inside the ear canal, but are not normal. Children may put objects into the ear canal.

The nurse is performing passive shoulder and elbow exercises for a patient who is recovering from surgery to remove a soft-tissue tumor in her upper arm. Why does the nurse cup one hand around the patient's elbow and support the forearm and wrist during the ROM exercises? A. To keep the arm above the level of the heart B. To assess the patient's muscle tension C. To listen for crepitus in the joint D. To ensure stability while exercising the join

D. To ensure stability while exercising the join Rationale: The nurse supports the distal portion of the extremity in order to ensure joint stability. It is not necessary to keep the arm above the level of the heart. Muscle tension is not assessed while performing passive ROM exercises. This patient is recovering from surgery to remove a soft-tissue tumor, which would not cause crepitus, a grating sound produced as the ends of long bones rub together in a patient with arthritis

The nurse is preparing to assess a patient in the hospital setting. What is the first step for the nurse? A. Don gloves B. Wear a mask C. Don goggles D. Wash hands

D. Wash hands

A patient with darkly pigmented skin has been admitted to the hospital with hepatitis. What is the best way for the nurse to assess for jaundice in this patient? A patient with darkly pigmented skin has been admitted to the hospital with hepatitis. What is the best way for the nurse to assess for jaundice in this patient? 1Jaundice is best seen in the sclera. 2In dark-skinned persons, jaundice results in a darkening of genitalia. 3Jaundice is best determined by blanching the fingernails. 4Jaundice cannot be assessed in patients with darkly pigmented skin.

Jaundice is best seen in the sclera

The registered nurse (RN) notifies the spouse of a client who was admitted to hospice with shallow respirations, of a change in the client's condition. Over the past hour, the client's respiratory pattern has changed to a Cheyne Stokes pattern. After receiving this information, the client's spouse begins vacuuming around the bed. Which stage of grief is the spouse displaying during the visit? Acceptance. Denial. Bargaining. Depression

Denial. The spouse is exhibiting the first stage of denial of Kubler-Ross's grief model by ignoring that the client's death is imminent.

Which approach is best for the nurse to use when assessing for capillary refill? Observe the nailbed angle. This action is used to observe for nailbed clubbing and is not necessary when assessing capillary refill. Depress the client's nailbed. To assess capillary refill, the RN first compresses the nailbed for 3 seconds. This results in blanching of the nailbed. The RN then measures the amount of time necessary for return of normal color of the nailbeds: the capillary refill time. Place the client's fingertips in a dependent position. The hands should be near the level of the heart, rather than dangling in a dependent position. Count the radial pulse for 30 seconds. This action is not necessary when assessing capillary refill.

Depress the client's nailbed. To assess capillary refill, the RN first compresses the nailbed for 3 seconds. This results in blanching of the nailbed. The RN then measures the amount of time necessary for return of normal color of the nailbeds: the capillary refill time.

Which technique does the nurse use to palpate a patient's abdomen?

Depresses the abdomen 1 to 2 inches for light palpation

Based on Mr. Kapur's report of increasingly frequent periods of dyspnea, dizziness, and minor chest discomfort, what assessment should the nurse perform next? Ask the client to stand and then recheck the blood pressure. Assessment of the client for orthostatic hypotension is important, but another assessment takes priority. Place the client in a supine position and observe for orthopnea. Assessment for orthopnea is important, but another assessment takes priority. Measure the apical and radial pulse rates at the same time. Assessment of the client for an apical-radial deficit is important, but another assessment should be completed first. Determine if the client is currently experiencing any angina. Because the client has a history of chest discomfort, the nurse should first determine if the client is currently experiencing angina. Angina should be treated immediately to reduce the risk for myocardial damage. Mr. Kapur denies any current symptoms, including angina.

Determine if the client is currently experiencing any angina. Because the client has a history of chest discomfort, the nurse should first determine if the client is currently experiencing angina. Angina should be treated immediately to reduce the risk for myocardial damage. Mr. Kapur denies any current symptoms, including angina.

What additional action should the nurse take to correctly stage the wound? Determine the depth of the wound and underlying tissue damage. Pressure ulcers are staged based on the depth of tissue damage to the dermis and underlying tissues, which may include underlying tendons, joint capsules, bones, and muscles. Measure the width of the wound from front-to-back and side-to side. This assessment provides useful information, but it is not used in staging a pressure ulcer. Note the amount, color, and character of the wound drainage. This assessment provides useful information, but it is not used in staging a pressure ulcer. Observe the tissue to determine the phase of wound healing. This assessment provides useful information, but it is not used in staging a pressure ulcer.

Determine the depth of the wound and underlying tissue damage. Pressure ulcers are staged based on the depth of tissue damage to the dermis and underlying tissues, which may include underlying tendons, joint capsules, bones, and muscles.

It is most important for the nurse to obtain further information related to which aspects of the client's care? Hygiene practices. There are no commonly held beliefs related to hygiene practices in Hinduism. Sleep patterns. There are no commonly held beliefs related to sleep patterns in Hinduism. Exercise habits. There are no commonly held beliefs related to exercise habits in Hinduism. Dietary needs. While there are few commonly held beliefs in Hinduism, many Hindus are vegetarians, so the nurse should assess the client's dietary needs. Herbs or purgatives used. Ayuveda, a traditional Hindu system of medicine, uses a combination of herbs, purgatives, and rubbing oils to treat illnesses.

Dietary needs. While there are few commonly held beliefs in Hinduism, many Hindus are vegetarians, so the nurse should assess the client's dietary needs. Herbs or purgatives used. Ayuveda, a traditional Hindu system of medicine, uses a combination of herbs, purgatives, and rubbing oils to treat illnesses.

The registered nurse (RN) palpates a weak pedal pulse in the client's right foot. Which assessment findings should the RN document that are consistent with diminished peripheral circulation? (Select all that apply.) Diminished hair on legs. Bruising on extremities. Skin cool to touch. Capillary refill less than 3 seconds. Darkened skin on extremities.

Diminished hair on legs Skin cool to touch. Diminished hair on the legs and skin that is cool to touch are expectant signs of decreased arterial blood flow.

The nurse palpates a weak pedal pulse in the client's right foot. Which assessment findings should the RN document that are consistent with diminished peripheral circulation? (Select all that apply.) Diminished hair on legs Bruising on extremities Skin cool to touch Capillary refill less than 3 seconds Darkened skin on extremities

Diminished hair on legs Skin cool to touch Diminished hair on the legs and skin that is cool to touch are symptoms of decreased arterial blood flow. The other options are not indicators for impaired peripheral circulation

The nurse begins the client interview, focusing on the gastrointestinal system. For the nurse to learn about Claudine's bowel patterns, which questions are most important to ask Claudine? Have you had any recent onset of heartburn? While the onset of heartburn may be important, this is not the most important information to obtain. Do you take any prescription or over-the-counter medications? Medications can cause adverse GI effects. Have you noticed any change in your stool pattern? Changes in bowel habits can be due to various etiologies, such as diet, stress, activity and medications. Do you have any difficulty with defecation? To fully assess the client's bowel patterns, it is essential to obtain information related to any difficulty with defecation, such as straining or pain. Do you have frequent vomiting episodes? This information is an important part of the client's history.

Do you take any prescription or over-the-counter medications? Medications can cause adverse GI effects. Have you noticed any change in your stool pattern? Changes in bowel habits can be due to various etiologies, such as diet, stress, activity and medications. Do you have any difficulty with defecation? To fully assess the client's bowel patterns, it is essential to obtain information related to any difficulty with defecation, such as straining or pain. Do you have frequent vomiting episodes? This information is an important part of the client's histor

What action should the nurse take next? Palpate the area below the medial condyle. The epitrochlear node is not located in this area. Ask the client to slowly flex her elbow. This will not impact the ability to assess the node. Document that the node is not palpable. The epitrochlear lymph node is located in the groove between the biceps and triceps muscles, above the medial epicondyle of the humerus and is not normally palpable. Apply pressure over the antecubital fossa. The epitrochlear node is not located in this area.

Document that the node is not palpable. The epitrochlear lymph node is located in the groove between the biceps and triceps muscles, above the medial epicondyle of the humerus and is not normally palpable.

What action should the nurse take? Use a stethoscope to auscultate the pulsation. It is not necessary to auscultate a venous pulsation. Palpate the pulsation again, using less pressure. Venous pulsations are not palpable. Reposition the client's head and attempt to palpate again. Venous pulsations are not palpable. Document the level at which the pulsation is observed. Venous pulsations are not palpable. The nurse should document the level at which the pulsations are observed.

Document the level at which the pulsation is observed. Venous pulsations are not palpable. The nurse should document the level at which the pulsations are observed.

The nurse observes areas of petechiae surrounding some of the bruises. How should the nurse respond to this finding? Ask the client how these burns occurred. Petechiae are not signs of burn injury. Palpate the areas for warmth and swelling. Petechiae are not signs of inflammation. Immediately measure and record the vital signs. The presence of petechiae surrounding bruises that have been present for a period of time does not warrant immediate vital sign measurement. Document the location of the bruises and petechiae. Petechiae are very small areas of hemorrhage from superficial capillaries. They may be the result of a bleeding or clotting problem as well as an indication of superficial trauma.The presence of bruising and petechiae on Amanda's abdomen causes the nurse to suspect that Amanda may be the victim of abuse.

Document the location of the bruises and petechiae. Petechiae are very small areas of hemorrhage from superficial capillaries. They may be the result of a bleeding or clotting problem as well as an indication of superficial trauma.The presence of bruising and petechiae on Amanda's abdomen causes the nurse to suspect that Amanda may be the victim of abuse.

The nurse observes symmetric chest excursion. What action should the nurse take? Ask the client to cough before repeating the assessment. This action is not necessary. Document the normal finding on the assessment record. Because chest excursion should be symmetric, i.e., equal on both sides, the nurse should document this normal finding on the assessment record. No additional intervention is warranted. If the chest excursion is asymmetric, the nurse should take further action to determine the cause of the asymmetry. Question the client about a recent history of rib fractures. This action is not necessary. Stop the assessment and measure the client's vital signs. This action is not necessary.

Document the normal finding on the assessment record. Because chest excursion should be symmetric, i.e., equal on both sides, the nurse should document this normal finding on the assessment record. No additional intervention is warranted. If the chest excursion is asymmetric, the nurse should take further action to determine the cause of the asymmetry.

When observing the client from the side, the nurse observes a slightly convex thoracic curve and a slightly concave lumbar curve. What action should the nurse take in response to these findings? Ask the client how long she has had a "Dowager's hump." The curvatures observed are not indicative of a "Dowager's hump," a term sometimes used to describe a severe thoracic curvature resulting in a hump-like appearance of the upper back. Record these symptoms of osteoporosis in the client's chart. The curvatures observed are not symptomatic of osteoporosis. Document the normal spinal curvature on the assessment form. The curvatures observed are normal spinal curvatures, so no action is needed other than documentation of the finding. Note the client's poor posture as a possible cause of her back pain. The curvatures observed do not indicate poor posture.

Document the normal spinal curvature on the assessment form. The curvatures observed are normal spinal curvatures, so no action is needed other than documentation of the finding.

What action should the nurse take? Notify the healthcare provider (HCP) immediately of the lack of a pulse. This action is not indicated. Move the end of the transducer closer to the toes and listen again. It is not necessary to take this action. Remove the excess gel, apply pressure more gently, and try again. It is not necessary to take this action. Document the presence of the pulse heard by Doppler ultrasound. A regular swooshing sound indicates that a pulse is heard with the Doppler ultrasound stethoscope. This finding should be documented

Document the presence of the pulse heard by Doppler ultrasound. A regular swooshing sound indicates that a pulse is heard with the Doppler ultrasound stethoscope. This finding should be documented

What action should the nurse take? Document the presence of wheezes in the upper lobes and complete the assessment. Wheezes may be present in clients with chronic emphysema when diffuse airway obstruction occurs. Immediately assist the client to lean forward to reduce his respiratory effort. Leaning forward will not impact these breath sounds. Note the location of these bronchial breath sounds before completing the assessment. Bronchial, or tracheal, breath sounds are normal breath sounds heard as a harsh, hollow tubular sound over the trachea and larynx. Wait to assess these heart sounds until the respiratory assessment is complete. These high-pitched musical sounds are not heart sounds.

Document the presence of wheezes in the upper lobes and complete the assessment. Wheezes may be present in clients with chronic emphysema when diffuse airway obstruction occurs.

What action should the nurse take in response to this finding? Document this finding on the physical assessment form. Dependent rubor, a deep blue-red color when the legs are in a dependent position, occurs with severe arterial insufficiency. The absence of dependent rubor is a normal assessment finding, and should be documented in the physical assessment but requires no further intervention. Immediately help the client sit down and elevate her legs. This finding does not warrant this action. Lightly palpate the calves for warmth or tenderness. This finding does not warrant this action. Assess for range of motion in the lower legs and feet. This finding does not warrant this action

Document this finding on the physical assessment form. Dependent rubor, a deep blue-red color when the legs are in a dependent position, occurs with severe arterial insufficiency. The absence of dependent rubor is a normal assessment finding, and should be documented in the physical assessment but requires no further intervention.

While assessing shoulder range of motion, the nurse notes the absence of crepitation with movement. What action should the nurse take in response to this finding? Document this normal finding in the assessment. Crepitation, a grating or crunching sound heard with joint movement, is an abnormal finding. An absence of crepitation is normal and should be documented in the assessment. Ask the client about her intake of dietary calcium. Crepitation is not related to calcium intake. Record the degree of the range of motion limitation. Lack of crepitation does not indicate a loss of range of motion. Review the client's record for a history of arthritis. Lack of crepitation is not an indicator of arthritis.

Document this normal finding in the assessment. Crepitation, a grating or crunching sound heard with joint movement, is an abnormal finding. An absence of crepitation is normal and should be documented in the assessment.

What action should the nurse take in response to this finding? Stop the assessment and notify the healthcare provider (HCP) immediately of the assessment finding. This finding does not require notification of the HCP. Take the client's blood pressure and heart rate after the assessment. This finding does not require immediate assessment of the vital signs. Call another nurse to verify the finding. Abdominal vascular sounds are not normally heard, so getting another nurse to verify the finding is not necessary. Document this normal finding on the client's assessment record. Abdominal vascular sounds are not normally heard, so the only action necessary is to record this normal finding on the assessment record.

Document this normal finding on the client's assessment record. Abdominal vascular sounds are not normally heard, so the only action necessary is to record this normal finding on the assessment record.

What action should the nurse take in response to this finding? Document this normal finding on the initial assessment record. A split S2 is a normal finding that can be heard in some people as the result of the slightly asynchronous closing of the aortic and pulmonic valves. A split S2 is heard best during inspiration at the pulmonic site, the left second intercostal space. Confirm the finding on the bedside cardiac telemetry monitor. A split S2 heart sound cannot be observed on a cardiac monitor. Assess for a change in the client's oxygen saturation reading. A change in oxygen saturation would not be expected in relation to this finding. Contact the healthcare provider (HCP) to report the assessment finding. This action is not warranted in response to this finding.

Document this normal finding on the initial assessment record. A split S2 is a normal finding that can be heard in some people as the result of the slightly asynchronous closing of the aortic and pulmonic valves. A split S2 is heard best during inspiration at the pulmonic site, the left second intercostal space.

What additional finding validates the nurse's initial assessment regarding the client's fluid status? Areas of skin bruising. Bruising is an abnormal finding, but it does not validate the client's fluid volume imbalance. Rapid facial flushing. Facial flushing is not typically related to fluid volume balance unless there is an associated febrile state. Dry mucus membranes. Dry or cracked mucus membranes can be the result of inadequate hydration, which, like inelastic skin turgor, validates the initial finding of fluid volume deficit. Shiny appearance of the forehead. A shiny appearance of the forehead is not typically associated with fluid volume balance.

Dry mucus membranes. Dry or cracked mucus membranes can be the result of inadequate hydration, which, like inelastic skin turgor, validates the initial finding of fluid volume deficit.

The registered nurse (RN) is caring for an older client who recently experienced a fractured pelvis from a fall. Which assessment finding is most important for the RN to report the healthcare provider? <p> <b>The registered nurse (RN) is caring for an older client who recently experienced a fractured pelvis from a fall. Which assessment finding is most important for the RN to report the healthcare provider?</b> </p> Lower back pain. Headache of 7 on scale 1 to 10. Blood pressure of 140/98. Dyspnea.

Dyspnea. A client with a large bone fracture is at risk for intramedullary fat leaking into the blood stream and becoming embolic. Dyspnea is an indication of fat embolism to the lungs and should be reported to the healthcare provider immediately.

The registered nurse (RN) is administering haloperidol 0.5 mg IM PRN to a client for the first time. What side effects should the RN assess the client for during the initial dose? Bradykinesia. Dystonia. Somatization. Akathisia.

Dystonia. Dystonia can be a sudden adverse reaction to this psychotropic medication which should be discontinued to resolve dystonia, and the healthcare provider notified immediately.

The nurse observes Ms. Yong as she rests her lower arms on a table with her hands at a 90 degree angle to the table and the thumbs up. Ms. Yong turns her hands upward with the back of the hand flat on the table and then downward with the palm flat on the table. What action is the nurse observing? Flexion and extension of the wrist. The client's movements do not demonstrate flexion and extension of the wrist. Elbow supination and pronation. The client's movements demonstrate a normal degree of elbow supination and pronation. Lower arm adduction and abduction. The client's movements do not demonstrate lower arm adduction and abduction. Hand and finger hyperextension. The client's movements do not demonstrate hand and finger hyperextension

Elbow supination and pronation. The client's movements demonstrate a normal degree of elbow supination and pronation.

To gather data about Dev's history of chest pain, how should the nurse begin? Encourage the client to describe his chest discomfort. Because chest pain can manifest in a number of different ways, the nurse should begin by obtaining information related to any type of chest discomfort so that further responses by the client include information related to any type of chest discomfort he has experienced. Determine if the chest pain has radiated to other sites. It is important to learn if the client's pain has radiated to other sites, but another action should be taken first. Question the client about the frequency of his symptoms. It is important to obtain data related to the frequency of the client's symptoms, but the nurse should take another action first. Ask the client to rate his chest pain on a numeric scale. By focusing specifically on chest pain initially, the client may discount other feelings of chest discomfort that are important for the nurse to assess.

Encourage the client to describe his chest discomfort. Because chest pain can manifest in a number of different ways, the nurse should begin by obtaining information related to any type of chest discomfort so that further responses by the client include information related to any type of chest discomfort he has experienced.

The nurse prepares Claudine for the physical assessment of the abdomen. Before assisting her to a supine position, what action should the nurse take? Encourage the client to empty her bladder. Emptying the bladder will help promote relaxation of the abdominal wall. Ask the client to breathe deeply several times. Breathing exercises may help relax the abdominal wall but should be done just prior to and during the examination, not prior to client positioning. Dim the room lights to encourage the client to relax. The room should be brightly lit to perform an accurate assessment. Instruct the client to place her hands over her head. Placing the hands over the head can cause the abdominal muscles to tense. Discuss the sequence of steps performed during the abdominal assessment. Telling the client what to expect during a procedure helps promote relaxation.

Encourage the client to empty her bladder. Emptying the bladder will help promote relaxation of the abdominal wall. Discuss the sequence of steps performed during the abdominal assessment. Telling the client what to expect during a procedure helps promote relaxation.

To learn about the intensity of the client's pain, what action should the nurse take? Ask the client how well she normally tolerates daily aches and pains. While this assessment is important, it does not provide information about the intensity of the client's pain. Determine what actions the client has already taken to reduce her pain. While this assessment is important, it does not provide information about the intensity of the client's pain. Encourage the client to use a numeric pain scale to rate her pain. A numeric pain scale is an effective tool for measuring pain intensity. Question the client about how the pain limits her ability to function. While this assessment is important, it does not provide information about the intensity of the client's pain.

Encourage the client to use a numeric pain scale to rate her pain. A numeric pain scale is an effective tool for measuring pain intensity.

While examining the ear of an infant with an otoscope, the nurse pulls down on the ear for which of the following reasons?

Enhances visualization of the tympanic membrane by straightening the ear canal

A client is admitted for dehydration, weight loss, and a flat affect. After reviewing the client's history, the registered nurse (RN) discovers that the client's spouse died 2 weeks ago. Which nursing interventions should the RN implement to help the client begin the process of dealing with loss Establish trust by creating an safe atmosphere for sharing. Share personal stories about how other clients dealt with grief. Help the client identify ways to adapt lifestyle to accommodate loss. Assure the client that their grief will last a short period of time. Explore ways to assist the client to make new emotional investments.

Establish trust by creating an safe atmosphere for sharing. Help the client identify ways to adapt lifestyle to accommodate loss. Explore ways to assist the client to make new emotional investments. Interventions that could aid the client in maneuvering through the stages of grieving and establishing a foundation to continue life should include creating a safe atmosphere for sharing, helping the client to identify ways to adapt to a life style without their spouse and exploring ways to make new emotional investments.

After a liver biopsy is performed at the bedside, t he registered nurse (RN) is assigned the care of the client. Which nursing intervention is most important for the RN to implement? Position client on left side with pillow placed under the costal margin. Assist the client with voiding immediately after the procedure. Evaluate vital signs q10 to 20 minutes for 2 hours after procedure. Ambulate client 3 times in first hour with pillow held at abdomen.

Evaluate vital signs q10 to 20 minutes for 2 hours after procedure. Vital signs should be checked every 10 to 20 minutes to assess for bleeding after biopsy of the liver, which is highly vascular. The client should be positioned on the right side with a pillow or sandbag under the costal margin and supporting the biopsy site. The client should be maintained on bedrest for several hours to decrease the risk of bleeding from the biopsy site.

The RN observes contraction of the biceps muscle and flexion of the forearm in response to the attempt to elicit the biceps reflex. What action should the RN take in response to this finding? Repeat the test at the same location to confirm the finding. There is no need to repeat the test. Record the finding as a 4+ deep tendon biceps reflex. A 4+ response is a very brisk, hyperactive response. Document that clonus was elicited by the reflex testing. Clonus is a set of short jerking muscular contractions. This was not exhibited by the client. Explain to the client that the reflex response was normal. The client's response is normal and should be documented as a 2+ response.

Explain to the client that the reflex response was normal. The client's response is normal and should be documented as a 2+ response.

A client with progressive hearing loss appears distressed when the registered nurse (RN) asks open-ended questions about the client's health history. Which forms of communication should the RN use? Face the client so the client can see the RN's mouth. Increase one's speech volume when interacting with the client. Repeat information to the client if misunderstood. Check if the client's hearing aides are working properly. Reduce environmental noise surrounding the client.

Face the client so the client can see the RN's mouth. Check if the client's hearing aides are working properly. Reduce environmental noise surrounding the client. A client with hearing loss can develop the ability to read "lips," so facing the client during conversation allows visualization of the lips and directs the sound towards the client. Inspection of the hearing aide device's functionality is a vital step in communication. Hearing aides magnify all surrounding noise, so it is imperative to reduce outside environmental noise during the interview process. Speaking clearly with enunciation and in a regular tone is easier for a client to understand than increasing the volume of speech. If a client shows signs of confusion, rephrasing the question, instead of repeating, should be done to decrease client anxiety and facilitate understanding.

The registered nurse (RN) is assessing common complications related to a client's recent diagnosis, systemic lupus erythematosus (SLE). Which symptom should the RN instruct the client to report immediately? Fever related to infection. Weight loss and anorexia. Depressed mood. Break in tissue integrity.

Fever related to infection. Secondary infections are a major concern with SLE clients due to the use of corticosteroids and chemotherapeutic agents, which suppresses the immune system, so reporting fever and infections should be reported immediately.

A 19-year-old collage student comes to the student health center because she discovered a small, non-tender lump in her breast. The nurse recognizes that the most common cause of breast lumps in woman her age is:

Fibroadenoma

Which of the following are expected assessment findings related to the 11-year-old child?

Five-pound weight gain and beginning of a growth spurt

Ms. Yong follows the nurse's instructions to swing her arms forward and up in a wide arc, then back. This action allows the nurse to observe what shoulder range of motion? Internal and external rotation. This action does not demonstrate internal and external rotation of the shoulder. Abduction and adduction. This action does not demonstrate abduction and adduction of the shoulder. Flexion and hyperextension. Swinging the arms forward in a wide arc demonstrates forward flexion. Swinging the arms back behind the midline demonstrates hyperextension of the shoulder. Forward and reverse motion. These terms are not used to describe specific types of range of motion.

Flexion and hyperextension. Swinging the arms forward in a wide arc demonstrates forward flexion. Swinging the arms back behind the midline demonstrates hyperextension of the shoulder.

While testing a patient's bicep muscle strength, the nurse applies resistance and asks the patient to perform which motion?

Flexion of the arm

While assessing the range of motion of the patient's knee, the nurse expects the patient to be able to perform which movements?

Flexion, extension, and hyperextension

Once the client is settled in the exam room, which action by the nurse has the highest priority? Obtain more in-depth information about the client's osteoporosis management. It is important to obtain information about the client's management of her osteoporosis, but another action is of greater priority. Review the client's medical record for any history of bone or spinal fractures. Review of the client's medical record for any history related to the client's current problem is important, but another action takes priority. Gather data about the nature, location, and duration of the client's back pain. Back pain can be a symptom of a variety of health problems. To ensure basic safety and homeostasis, it is most important for the RN to obtain information related to the client's report of pain. Compare bilateral muscle strength and tone in the client's lower extremities. Since the client walked to exam room, muscle strength and tone are not of the most immediate concern, so the RN should take another action before completing this assessment.

Gather data about the nature, location, and duration of the client's back pain. Back pain can be a symptom of a variety of health problems. To ensure basic safety and homeostasis, it is most important for the RN to obtain information related to the client's report of pain.

In responding to Ms. Yong, the nurse recognizes that the client is dealing with issues related to which of Erikson's developmental stages? In responding to Ms. Yong, the nurse recognizes that the client is dealing with issues related to which of Erikson's developmental stages? Initiative vs. guilt. According to Erikson's developmental framework, this is the central task of late childhood. Identity vs. inferiority. According to Erikson's developmental framework, this is the central task of the school-aged child. Generativity vs. stagnation. According to Erikson's developmental framework, this is the central task of the adult and is supported by the client's expressed concern about continuing to work as she gets older. Integrity vs. despair. According to Erikson's developmental framework, this is the central task of the older adult (older than 65 years).

Generativity vs. stagnation. According to Erikson's developmental framework, this is the central task of the adult and is supported by the client's expressed concern about continuing to work as she gets older.

A patient has edema and redness of the skin surrounding the nail on his right index finger. Which data elicited from his history best explains this condition? 1He has a family history of liver disease. 2There has been a scabies outbreak among his family members. 3He has a new full-time position as a dishwasher at a local restaurant. 4He had several warts removed from his hands 2 years ago.

He has a new full time position as a dishwasher at a local restaurant

A client in an ambulatory clinic describes awaking in the middle of the night with difficulty breathing and shortness of breath related to paroxysmal nocturnal dyspnea. Which underlying condition should the registered nurse (RN) identify in the client's history ? Chronic bronchitis. Gastroesophageal reflux disease (GERD). Heart failure (HF). Chronic pancreatitis.

Heart failure (HF). Paroxysmal nocturnal dyspnea is classic sign of heart failure and is secondary to fluid overload associated with heart failure which causes pulmonary edema.

How should the nurse plan to continue auscultation from that site? Move the stethoscope back and forth across the sternum. This is not an effective approach to hear all of the sounds produced by the valves. Slide the stethoscope over and up in an "X" pattern. The aortic area is located at the base of the heart, at the second right intercostal space. Moving the stethoscope in an upward direction is moving away from the heart. Lift the stethoscope from one valve area to the next. Lifting the stethoscope from one valve area to the next may prevent the nurse from hearing all sounds produced by the valves. Inch the stethoscope across and down in a "Z" pattern. Inching the stethoscope across the chest and using a systematic pattern ensures that all sounds produced by the valves will be heard.

Inch the stethoscope across and down in a "Z" pattern. Inching the stethoscope across the chest and using a systematic pattern ensures that all sounds produced by the valves will be heard.

Health Promotion and Maintenance: Inspection of the PrecordiumThe nurse begins the physical assessment by inspecting the client's precordium. The nurse completes the interview and prepares to assess the client's heart and neck vessels. How should the nurse prepare the client for inspection of the precordium? Assist the client to a left side-lying position with his chest and back exposed. A left side-lying position is not the best position for inspection of the precordium. Open the back of the client's gown while he sits on the side of the bed. Inspection of the precordium may be completed with the client in a sitting position, but opening the back of the client's gown does not provide exposure of the precordium. Help the client to a supine position on the bed with his chest exposed. A supine position with the chest exposed provides the best exposure for inspection of the precordium. Loosen the client's gown and ask him to lean forward in the bedside chair. Leaning forward in a chair is not the best position for inspection of the precordium, and loosening the gown does not provide sufficient exposure of the precordium.

Help the client to a supine position on the bed with his chest exposed. A supine position with the chest exposed provides the best exposure for inspection of the precordium.

The registered nurse (RN) is caring for a client with peptic ulcer disease (PUD). What assessment should the RN identify and document that is consistent with PUD? (Select all that apply). Hematemesis. Gastric pain on an empty stomach. Colic-like pain with fatty food ingestion. Intolerance of spicy foods. Diarrhea and stearrhea.

Hematemesis. Gastric pain on an empty stomach. Intolerance of spicy foods. Manifestations of PUD include hematemesis, gastric pain, and spicy food intolerance.

After the RN asks the client to close both eyes what is the next action the RN should take? Shine a penlight into one pupil. Another action should be taken first. Move the penlight away from the pupil. Another action should be taken first. Hold a penlight to the side of one eye. The RN begins by asking the client to close both eyes. This allows the pupils to dilate. The RN next holds the penlight to the side of the eye, so it is ready as soon as the client opens her eyes. Observe the constriction of the pupil. Another action should be taken first.

Hold a penlight to the side of one eye. The RN begins by asking the client to close both eyes. This allows the pupils to dilate. The RN next holds the penlight to the side of the eye, so it is ready as soon as the client opens her eyes.

The nurse is assessing the patient's stressors. Which tool can be used to identify the degree of stressors a patient may be experiencing?

Holmes Social Readjustment Inventory Scale

Before developing the client's plan of care, what information is most important for the nurse to obtain regarding the client's spirituality? Whether the client participates in formal religious services regularly. Other information is more pertinent to the plan of care. How the client's spiritual beliefs impact his health care expectations. In planning care, the RN should try to determine how the client's spiritual and cultural beliefs impact the expectations for care in the healthcare setting. What beliefs the client holds regarding the existence of a higher power. Other information is more pertinent to the plan of care. The role played by a spiritual advisor within the client's faith tradition. This information may be important in planning the client's care, but another option offers a broader overview of the impact of the client's spiritual needs.

How the client's spiritual beliefs impact his health care expectations. In planning care, the RN should try to determine how the client's spiritual and cultural beliefs impact the expectations for care in the healthcare setting.

The registered nurse (RN) recognizes which client group is at the greatest risk for developing a urinary tract infection (UTI)? (Rank from highest risk to lowest risk.) 1. Older females.<br/> 2. School-age female.<br/> 3. Older males.<br/> 4. Adolescent males.<br/>

Hypoestrogenism and alkalotic urine are other age-related factors put older women at the highest risk for UTIs. School age girls (6 to 12 years) are at risk for UTIs due to a higher prevalence to taking baths instead of showers, but these risks can be controlled in this population as well as hypoestrogenism and alkalotic urine. Older men are at risk due to possible obstruction of the bladder due to benign prostatic hypertrophy (BPH). Adolescent males (12 to 19 years) are the lowest at risk for a UTI. All individuals regardless of gender and/or age are at risk if the following conditions exist: vesicoureteral reflux, neuromuscular conditions, like Parkinson's disease, previous brain attacks, or the use of anticholinergic medications can all cause incomplete bladder emptying which can create bacterial overgrowth. Fecal and urinary incontinence contributes to poor perineal hygiene and bacterial growth.

Which nursing diagnoses are a priority when developing Lourdes' plan of care? Impaired physical mobility. Lymphedema can make mobility difficult, especially lower extremity lymphedema. It is important to assess the client and refer to physical therapy for exercises and activity restrictions. Disturbed body image. A client with lymphedema may have a disturbed body image from such things as wearing over-sized clothes or two different sized shoes. It is important to address these things with the client. Risk for infection. Infection may be common in lymphedema; pooling of protein-rich lymph fluid increases cellulitis. Risk for impaired skin integrity. Skin on the affected arm may be more dry than normal. Good skin care is essential to prevent infection; wraps and compression stockings may retain moisture against the skin. Chronic lymphedema related to status post breast cancer surgery. This is a medical diagnosis, not a nursing diagnosis.

Impaired physical mobility. Lymphedema can make mobility difficult, especially lower extremity lymphedema. It is important to assess the client and refer to physical therapy for exercises and activity restrictions. Disturbed body image. A client with lymphedema may have a disturbed body image from such things as wearing over-sized clothes or two different sized shoes. It is important to address these things with the client. Risk for infection. Infection may be common in lymphedema; pooling of protein-rich lymph fluid increases cellulitis. Risk for impaired skin integrity. Skin on the affected arm may be more dry than normal. Good skin care is essential to prevent infection; wraps and compression stockings may retain moisture against the skin.

Recalling the client's admission diagnosis of emphysema and an acute pulmonary infection, what finding should the nurse anticipate? Diminished fremitus over areas of infection. Palpation over areas of infection does not typically result in diminished fremitus. Increased fremitus over areas of consolidation. Increased fremitus, or vibration when the client speaks, is often felt over areas of consolidated lung tissue, such as in clients with pneumonia. Absent fremitus over areas of hyperinflated alveoli. Hyperinflated alveoli, present in clients with chronic emphysema, do not cause an absence of vocal fremitus. Inability to assess for fremitus due to fatigue. Palpation for vocal fremitus does not require extensive energy expenditure by the client.

Increased fremitus over areas of consolidation. Increased fremitus, or vibration when the client speaks, is often felt over areas of consolidated lung tissue, such as in clients with pneumonia.

A client with cirrhosis of the liver asks the registered nurse (RN) to explain how varicose veins can occur in the esophagus. Which statement should the RN provide to teach the client about the physiological etiology? The enlarged liver presses on the lower half of the esophagus which weakens blood vessel walls. Abnormal vessels form as a result of liver damage that causes chronic low serum protein levels. Esophageal swelling and tissue damage causes blood to circulate blood back through the stomach. Increased portal pressure causes blood flow through liver to be shunted to the esophageal vessels.

Increased portal pressure causes blood flow through liver to be shunted to the esophageal vessels. Cirrhotic and fibrosed liver damage causes obstructed blood flow through portal vessels to the liver which increases the portal pressure causing the blood flow through the liver to be shunted to the esophageal vessels. The result of this shunting of blood causes the esophageal vessels (veins) to balloon out and weaken. As the portal hypertension increases, these esophageal varices can rupture and cause bleeding resulting in bloody emesis and black tarry stools.

When assessing a patient's abdomen, the nurse uses assessment techniques in which order?

Inspection, auscultation, jpalpation, and percussion

A complete arterial circulation assessment includes a Modified Allen's Test. Which approach is best for the nurse to use to conduct this test? Instruct Lourdes to make a fist several times for about 30 seconds. Making a fist several times causes the hand to blanch during the Modified Allen's Test. Place the client's hand in a dependent position. The hand should be elevated during the Modified Allen's Test. Obliterate the ulnar and radial pulses. The ulnar and radial arteries are obliterated by holding pressure for a few seconds until pallor in the hand occurs. Instruct Lourdes to flex all of her fingers. This is not the correct instruction. Document that the test results show inadequate circulation to the hand if pinkness fails to return within 6 seconds. A blanched hand or pallor in the fingernails greater than 6 seconds after releasing ulnar pressure indicates inadequate circulation.

Instruct Lourdes to make a fist several times for about 30 seconds. Making a fist several times causes the hand to blanch during the Modified Allen's Test. Obliterate the ulnar and radial pulses. The ulnar and radial arteries are obliterated by holding pressure for a few seconds until pallor in the hand occurs Document that the test results show inadequate circulation to the hand if pinkness fails to return within 6 seconds. A blanched hand or pallor in the fingernails greater than 6 seconds after releasing ulnar pressure indicates inadequate circulation.

When a nurse asks a patient to place the right arm behind the back, so that the back of the hand is touching the lower spine, the nurse is testing for which range of motion?

Internal rotation and adduction of the shoulder

Which instruction is most important for the nurse to give a client who has lymphedema, when teaching about the importance of the lymphatic system? It circulates blood throughout the body. The lymphatic system contains fluid, not blood. It plays a major role in the body's defense against diseases. The lymph system circulates lymphocytes, such as T, B and NK cells that help prevent disease. It helps to maintain the byproducts of cellular feeding, such as carbon dioxide and sodium in your body. The lymph system works to eliminate those byproducts through perspiration, urination, and bowel movements, not to hold on to them. It moves cancer cells away from the lymph nodes. If cancer cells break away from a tumor, they can attach to the nearby lymph nodes. This is the reason doctors will check the lymph nodes first when determining how far a cancer has spread.

It plays a major role in the body's defense against diseases. The lymph system circulates lymphocytes, such as T, B and NK cells that help prevent disease.

The nurse should observe the force of the impulse at what location? Left midclavicular line, 2nd intercostal space. A left ventricular heave is not observed at this site. Left sternal border, 4th intercostal space. A left ventricular heave is not observed at this site. Right sternal border, 2nd intercostal space. A left ventricular heave is not observed at this site. Left midclavicular line, 5th intercostal space. A left ventricular heave is seen at the apex, located at the left midclavicular line, 5th intercostal space. This forceful thrusting of the ventricle occurs with hypertrophy of the left ventricle.

Left midclavicular line, 5th intercostal space. A left ventricular heave is seen at the apex, located at the left midclavicular line, 5th intercostal space. This forceful thrusting of the ventricle occurs with hypertrophy of the left ventricle.

Twenty four hours after a client returns from surgical gastric bypass, the registered nurse (RN) observes large amounts of blood in the nasogastric tube (NGT) cannister. Which assessment finding should the RN report as early signs of hypovolemic shock? Faint pedal pulses. Decrease in blood pressure. Lethargy. Slow breathing.

Lethargy One of the early signs of hypovolemic shock is changes in the client's level of consciousness due to the decrease perfusion to the brain which can manifests as lethargy or confusion.

The nurse's goal in palpating the client's abdomen is to screen for any masses or tenderness. To achieve this goal, what action should the nurse take first? Deeply palpate each abdominal organ. Deep palpation of the organs is not the first step when palpating the abdomen. Carefully palpate areas of tenderness. Palpation of any areas of tenderness should be saved for last to prevent resulting discomfort or muscle rigidity. Lightly palpate the abdominal surface. Light palpation allows the nurse to screen the abdomen for any obvious masses or tenderness before applying deeper palpation that may cause pain or rigidity. Gently palpate the edges of the liver. Deep palpation used to palpate the liver is not the first step when screening the abdomen.

Lightly palpate the abdominal surface. Light palpation allows the nurse to screen the abdomen for any obvious masses or tenderness before applying deeper palpation that may cause pain or rigidity.

the nurse palpates the abdomen to gather data about which organs located in the right upper quadrant?

Liver and gallbladder

In order to percuss the client's thorax posteriorly beginning at the apex of the right lung, how should the nurse begin? Palpate the space directly below the clavicle. The clavicles are located anteriorly and are not a useful landmark for posterior thoracic assessment. Place one finger pad over the first rib. Percussion over boney areas, such as the ribs, will not provide useful assessment data because the elicited sound will always be dull. Locate the client's first intercostal space. Percussion should be performed systematically, percussing in the intercostal spaces to avoid the ribs and scapulae. Find the space directly above the diaphragm. The diaphragm is not a useful landmark to begin this assessment.

Locate the client's first intercostal space. Percussion should be performed systematically, percussing in the intercostal spaces to avoid the ribs and scapulae.

With the patient lying supine, a nurse raises the patient's leg to flex the hip. The patient complains of pain when the leg is raised to 40 degrees. The nurse correlates this finding with which disorder?

Lumbar nerve compression

Which action should the registered nurse (RN) implement to complete an assessment for a client while using an interpreter? Ask closed-ended questions with the assistance of the interpreter. Maintain eye contact with the client while listening to the translation. Instruct interpreter to answer questions from interpreter's point of view. Protect the client's privacy by asking a limited number of questions.

Maintain eye contact with the client while listening to the translation. When completing an assessment, the RN should maintain eye contact with the client to gather additional information from the client's nonverbal cues.

The registered nurse (RN) is caring for a client with aplastic anemia who is hospitalized for weight loss and generalized weakness. Laboratory values show a white blood count (WBC) of 2,500/ mm3 and a platelet countof 160,000/mm3. Which intervention is the primary focus in the client's plan of care for the RN to implement? Assist with frequent ambulation. Encourage visitors to visit. Maintain strict protective precautions. Avoid peripheral injections.

Maintain strict protective precautions. The client should be under strict protective transmission precautions because the WBC values are low and normal WBC levels are 4,000-10,000/mm3, so the client is an increased high risk for infection.

The nurse notes dark brown pigmentation on the abdominal area. Which action is most important for the nurse to perform next? Question the client to obtain more information. This is a normal finding in the African American client. There is no need for further questions. Make a note to consult a dermatologist. This is a normal finding in the African American client. There is no need for further consultation. Document the finding as normal. This is a normal finding in the African American client. The nurse should document the finding. Document the finding as abnormal. This is a normal finding in the African American client.

Make a note to consult a dermatologist. This is a normal finding in the African American client. There is no need for further consultation.

The nurse is comparing the right and left legs of a patient and notices that they are asymmetric. Which additional data does the nurse collect at this time?

Measures the length of each leg and compares the findings

While caring for a client who has esophageal varices, which nursing intervention is most important for the registered nurse (RN) to implement? Monitor infusing IV fluids and any replacement blood products. Prepare for esophagogastroduodenoscopy (EGD). Maintain the client on strict bedrest. Insert a nasogastric tube (NGT) for intermittent suction.

Monitor infusing IV fluids and any replacement blood products. Maintaining hemodynamic stability in a client with esophageal varicescan precipitatea life-threatening crisis if esophageal varies leak or rupture and can result in hemorrhage. The priority is assessing and monitoring infusions of IV fluids and any replacement blood products.

The nurse does not hear a bruit. What should the nurse do next? Reassure the client that his right artery sounds "clear" and listen on the left side. The absence of a bruit does not always indicate absence of carotid occlusion. Listen at the base of the neck again, this time using the diaphragm of the stethoscope. The bell of the stethoscope is used to assess for a carotid bruit. Move the bell of the stethoscope up the right side of the neck to the mid-cervical area. The RN should auscultate each carotid artery systematically, including the base of the neck, the mid-cervical area, and the angle of the jaw. Press the bell of the stethoscope more firmly against the base of the neck and listen again. Placing the bell of the stethoscope more firmly may result in an artificial bruit.

Move the bell of the stethoscope up the right side of the neck to the mid-cervical area. The RN should auscultate each carotid artery systematically, including the base of the neck, the mid-cervical area, and the angle of the jaw.

After listening in this area, how should the nurse proceed? Move the diaphragm across to the apex of the right lung posteriorly. Moving the diaphragm of the stethoscope across the posterior thorax provides the most systematic approach to comparing the lung sounds bilaterally. Listen again at the same location using the bell of the stethoscope. The bell of the stethoscope is used to listen for soft, low-pitched sounds such as extra heart sounds and is less effective for hearing high-pitched breath sounds. Stand in front of the client and listen to the left lung apex anteriorly. Moving from front to back when auscultating each area of the lung is not the most efficient, systematic approach to lung auscultation. Inch down the left side posteriorly to listen to the left middle lobe. This approach is not the best method to ensure effective comparison of the breath sounds bilaterally.

Move the diaphragm across to the apex of the right lung posteriorly. Moving the diaphragm of the stethoscope across the posterior thorax provides the most systematic approach to comparing the lung sounds bilaterally.

After assessing the femoral artery, the nurse palpates the inguinal lymph nodes. What technique should be used? Lightly press the palmar surface of one hand over the inguinal area. This is not the most effective technique to palpate the lymph nodes. Move the finger pads over the area using a gentle circular motion. This technique allows effective palpation of the lymph nodes. Firmly compress the area until blanching occurs and then release. This technique is not used to palpate the lymph nodes. Gently press downward with the fingertips until the node is felt. This is not the most effective technique to palpate the lymph nodes.

Move the finger pads over the area using a gentle circular motion. This technique allows effective palpation of the lymph nodes.

A nurse makes observations about a toddler's motor development. Which behavior is an example of fine motor behavior? a. Sitting up in a chair b. Walking while holding on to the edge of a table c. Creeping up the stairs d. Stacking blocks to make a tower

NS: D Feedback A Sitting up in a chair is an example of gross motor skills. B Walking while holding on to the edge of a table is an example of gross motor skills. C Creeping up the stairs is an example of gross motor skills. D Stacking blocks to make a tower is an example of fine motor skills with hands and fingers.

The registered nurse (RN) is teaching a client who is being discharged after treatment of tuberculosis (TB). Which cultural issues should the RN assess when preparing the client for discharge? (Select all that apply.) Native language. Education level. Type of lifestyle. Financial resources. Previous medical history

Native language. Education level. Type of lifestyle. Financial resources. To ensure compliance the client's native language, education level, lifestyle, and financial resources should be considered when preparing the client's discharge instructions about the continuation of treatment for TB.

How should the nurse document the assessment? Normal bowel sounds. Normal bowel sounds occur irregularly, approximately 5 to 30 times per minute. Hypoactive bowel sounds. A frequency of 8 to 20 sounds per minute is not considered hypoactive. Hyperactive bowel sounds. A frequency of 8 to 20 sounds per minute is not considered hyperactive. Borborygmus present. This term describes a very hyperactive frequency of bowel sounds; a frequency of 8 to 20 sounds per minute is not considered hyperactive

Normal bowel sounds. Normal bowel sounds occur irregularly, approximately 5 to 30 times per minute.

The nurse observes that there are numerous blackheads around Amanda's chin and nose. What action should the nurse take in response to this finding? Note any pustules or nodules. Blackheads are a form of acne, common in the adolescent when sebaceous gland activity increases. The nurse should look for signs of severe acne which may be manifested as pustules or nodules on other parts of the client's body (such as the back or chest). Ask about a history of eczema. Blackheads are not related to a history of eczema. Measure for pitting edema. Blackheads do not cause edema. Palpate the areas for tenderness. Blackheads do not cause tenderness.

Note any pustules or nodules. Blackheads are a form of acne, common in the adolescent when sebaceous gland activity increases. The nurse should look for signs of severe acne which may be manifested as pustules or nodules on other parts of the client's body (such as the back or chest).

To determine the grade of the murmur, what action should the nurse take? Listen in surrounding areas for the extent of radiation of the sound. The grade of a murmur is not determined by the extent of sound radiation. Assess for a change in the murmur during a change in the client's position. The grade of a murmur is not determined by a change based on the client's position. Determine the location on the client's chest where the murmur is best heard. The location where a murmur is best heard does not determine the grade of the murmur. Note how easily the murmur is heard by gradually lifting the stethoscope. Murmurs are graded based on the intensity of the sound, ranging from a grade 1 murmur, which is barely audible, to a grade 6 murmur, which can be heard with the stethoscope lifted off the chest wall.

Note how easily the murmur is heard by gradually lifting the stethoscope. Murmurs are graded based on the intensity of the sound, ranging from a grade 1 murmur, which is barely audible, to a grade 6 murmur, which can be heard with the stethoscope lifted off the chest wall.

After answering a few questions, the client begins to cough. What assessment should the nurse perform? Assess for the presence of nail bed clubbing. Nail bed clubbing indicates chronic hypoxemia, but it will not provide information immediately relevant to the client's cough. Review oral fluid intake for the last 24 hours. Oral fluid intake is an important assessment, but it does not provide information immediately relevant to the client's cough. Observe for dryness of the oral mucosa. Moistness of the oral mucosa provides information about the client's fluid status, but does not provide information immediately relevant to the client's cough. Note the amount and appearance of any sputum. The amount and appearance of any sputum when the client coughs provides useful data related to the cause of the client's cough and any underlying problems.

Note the amount and appearance of any sputum. The amount and appearance of any sputum when the client coughs provides useful data related to the cause of the client's cough and any underlying problems.

To assess the client for signs of protein malnutrition, what action should the nurse take? Compress the client's nail beds. Nail bed compression and release measures capillary refill, providing data related to the client's circulation. Observe the color of the conjunctiva. Pale conjunctiva may be an indication of anemia. Note the texture of the client's hair. Dull, dry, sparse hair may be an indication of nutritional deficiencies, including protein deficiency. Measure the client's deep tendon reflexes. Hyporeflexia may be an indication of vitamin deficiency.

Note the texture of the client's hair. Dull, dry, sparse hair may be an indication of nutritional deficiencies, including protein deficiency.

Which is the most appropriate follow up action the nurse should implement? Note this location as the border of the liver. Dullness upon percussion is generally heard over organs, such as the liver. The right costal margin is the location at which abdominal tympany should change to dullness over the liver border. This location is useful in determining liver span. Ask the client if she is constipated. This finding does not indicate constipation. Document the presence of splenic dullness. The spleen is percussed on the left side of the abdomen. Document the finding as normal. The right costal margin is the location at which abdominal tympany should change to dullness over the liver border. Make a note to notify the HCP of the findings. This is a normal finding and does not warrant notifying the HCP.

Note this location as the border of the liver. Dullness upon percussion is generally heard over organs, such as the liver. The right costal margin is the location at which abdominal tympany should change to dullness over the liver border. This location is useful in determining liver span. Document the finding as normal. The right costal margin is the location at which abdominal tympany should change to dullness over the liver border.

After observing the presence of rebound tenderness, the nurse notes the onset of involuntary rigidity of the client's abdomen. Which action should the nurse implement? Notify the HCP of the findings. Rebound tenderness and involuntary rigidity (guarding) are abnormal findings associated with peritoneal irritation and are signs that should be reported to the HCP immediately for further diagnostic evaluation. Assist the client to a semi-Fowler's position. This action is not useful following the onset of involuntary rigidity (guarding). Administer a pain medication. More assessment is needed prior to the administration of a pain medication, which could mask other symptoms. Place a warm moist pack on the client's abdomen. This action is not useful following the onset of involuntary rigidity (guarding).

Notify the HCP of the findings. Rebound tenderness and involuntary rigidity (guarding) are abnormal findings associated with peritoneal irritation and are signs that should be reported to the HCP immediately for further diagnostic evaluation.

Amanda also points out a small (1 mm), smooth, slightly raised bright red dot on her abdomen and asks the nurse to examine that spot as well. How should the nurse proceed? Apply pressure over the lesion and observe for blanching. This action is not indicated for a lesion with that description. Advise the client to be examined by a healthcare provider. A lesion of this description does not warrant examination by a healthcare provider. Offer assurance that this lesion is not an abnormal finding. Cherry angiomas are commonly seen on the abdomen, particularly in persons over the age of 30. Angiomas typically increase in number and size with aging and are not a cause for concern. Determine if the client experienced any trauma at the site. This description does not indicate trauma to the site.

Offer assurance that this lesion is not an abnormal finding. Cherry angiomas are commonly seen on the abdomen, particularly in persons over the age of 30. Angiomas typically increase in number and size with aging and are not a cause for concern.

During the health history, Lourdes reported that her feet and ankles swell occasionally. To assess for edema, what action the nurse take first? Ask the client to lie down and elevate her feet and legs. If swelling is present, the RN may advise a client to elevate the extremities to help reduce the swelling, but this is not the first action when assessing for the presence of edema. Observe and compare the client's lower extremities. The RN should first assess for edema by observing a client's legs for any obvious swelling and by comparing the two extremities for differences in size. Gently compress the tissue on the top of the client's feet. The RN may choose to compress any obvious areas of swelling after first taking another action. Ask the client to gently dorsiflex each of her feet. This may be done later in the assessment, if unilateral swelling is observed and the RN is concerned about a possible deep vein thrombosis. Assessment of Homan's sign (pain upon dorsiflexion) is not always recommended because of the risk of dislodging an existing clot.

Observe and compare the client's lower extremities. The RN should first assess for edema by observing a client's legs for any obvious swelling and by comparing the two extremities for differences in size.

Mrs. McElroy continues to feel nauseated and Mr. McElroy remains with his wife while the nurse leaves the room to prepare a PRN dose of a prescribed antiemetic. Shortly after the nurse administers the antiemetic, Mrs. McElroy states she feels better. The nurse offers to provide oral care with a mint-flavored foam swab and cool water. Which assessment takes priority while the nurse provides oral care? Assess for enlarged cervical lymph nodes. This information may be important, but another assessment takes priority at this time. Observe for excessive dryness of the mucus membranes. Because the client has a recent history of nausea, vomiting, and weight loss, the RN should assess the client for signs of fluid volume deficit, including observing the mucus membranes for excessive dryness. Palpate the thyroid gland for tenderness or swelling. Considering the client's recent history of nausea and vomiting, another assessment takes priority at this time. Check for the presence of cavities. Considering the client's recent history of nausea and vomiting, another assessment takes priority at this time

Observe for excessive dryness of the mucus membranes. Because the client has a recent history of nausea, vomiting, and weight loss, the RN should assess the client for signs of fluid volume deficit, including observing the mucus membranes for excessive dryness.

To continue the cranial nerve assessment, the RN asks Mrs. Green to first smile, then frown, and then show her teeth. While the client performs these tasks, what should the RN do? Apply light pressure over the facial nerve. This action is not useful in assessing cranial nerve VII. Observe for symmetric facial movement. The RN observes for symmetric movement when the client smiles, frowns, or shows her teeth. This assessment provides data related to the function of the facial nerve, cranial nerve VII. Gently palpate for swelling over the cheeks. This action is not useful in assessing cranial nerve VII. Note how quickly Mrs. Green completes each task. This action is not useful in assessing cranial nerve VII.

Observe for symmetric facial movement. The RN observes for symmetric movement when the client smiles, frowns, or shows her teeth. This assessment provides data related to the function of the facial nerve, cranial nerve VII.

To assess the symmetry of the abdomen, what action should the nurse take? Have the client breathe in and out slowly. Breathing in and out does not provide information related to the symmetry of the abdomen. Ask the client to cough. Coughing does not provide information related to the symmetry of the abdomen. Observe for any visible peristalsis. The presence of visible peristalsis does not provide information related to the symmetry of the abdomen. Observe the abdomen from two different angles. To evaluate symmetry, the nurse should stand behind the client's head and squat at the side to view the abdomen at eye level.

Observe the abdomen from two different angles. To evaluate symmetry, the nurse should stand behind the client's head and squat at the side to view the abdomen at eye level.

What additional assessment is most important for the nurse to implement? Observe the appearance of the left hand. A 1+ pulse volume indicates diminished circulation, so further assessment of the left hand is a priority. The RN should assess the color, warmth, and capillary refill. Palpate the rate of the ulnar pulses bilaterally. Ulnar pulse assessment is typically only performed prior to a procedure that may traumatize and damage the radial artery. Measure the fingernail capillary refill of the right hand. A 3+ pulse volume indicates a normal, or full pulse volume. Further assessment of circulation to the right hand is not a priority. Compare hand grip strength bilaterally. This is not a priority assessment at this time.

Observe the appearance of the left hand. A 1+ pulse volume indicates diminished circulation, so further assessment of the left hand is a priority. The RN should assess the color, warmth, and capillary refill.

Health Promotion and MaintenanceAfter completing auscultation of the client's abdomen, the nurse prepares to percuss Claudine's abdomen.A dull sound is heard when the nurse percusses over the suprapubic area. What action should the nurse take in response to this finding? Reposition the client to her right side. Repositioning the client will not change the sound heard upon percussion of the lower abdomen. Observe the area for bladder distention. A dull sound upon percussion may be heard over a distended bladder. Determine if the client feels bloated or gaseous. Gaseous distention may cause a hyperresonant sound. Assist the client to a sitting position immediately. This action is not warranted in response to this finding.

Observe the area for bladder distention. A dull sound upon percussion may be heard over a distended bladder.

Amanda tells the nurse that her scalp itches sometimes. The nurse observes white flecks on the client's shoulders. What action should the nurse take first? Instruct the client about dandruff treatments. Another action should be taken before providing client teaching. Observe the client's hair shafts and scalp. Loose white flecks typically indicate dandruff. However, since itching may also be the result of head lice, the nurse should observe the scalp and hair shafts for the presence of nits, which adhere to the hair shaft.Further assessment does not reveal the presence of any head lice or nits. Remind the client not to share hairbrushes. Another action should be taken before providing client teaching. Explain how to treat the hair and scalp for head lice. The observation of loose white flecks does not necessarily indicate the presence of head lice. Another action should be taken before assuming the client has head lice.

Observe the client's hair shafts and scalp. Loose white flecks typically indicate dandruff. However, since itching may also be the result of head lice, the nurse should observe the scalp and hair shafts for the presence of nits, which adhere to the hair shaft.Further assessment does not reveal the presence of any head lice or nits.

Management of CareMrs. McElroy is admitted to her room accompanied by her husband. Before the nurse can begin the admission assessment, Mrs. McElroy states that she needs to throw up. The nurse helps Mrs. McElroy sit up and provides an emesis basin.Mrs. McElroy vomits into the emesis basin and then remains sitting on the side of the bed, stating that she may need to throw up again. Which assessment should the nurse complete first? Check her pulse. Another assessment should be completed before assessing the client's pulse rate, which might be elevated secondary to vomiting. Listen to breath sounds. Another assessment should be completed before assessing for breath sounds. Observe the color of the emesis. Since the client is vomiting, the nurse should first observe the color and appearance of the emesis for any obvious bleeding or other indications of risk to the client's homeostasis. Obtain a STAT blood pressure. The nurse will need to obtain a blood pressure, but that is not a priority at this time, as it might be elevated secondary to the vomiting.

Observe the color of the emesis. Since the client is vomiting, the nurse should first observe the color and appearance of the emesis for any obvious bleeding or other indications of risk to the client's homeostasis.

What action should the nurse take? Switch to using the heel of the hand to palpate. The heel of the hand should not be used. Observe the muscles while the client exhales. Bilateral tensing is often an indication of voluntary guarding by the client. To help distinguish between voluntary and involuntary guarding, the nurse should observe the muscles during exhalation because the client usually does not demonstrate voluntary guarding during exhalation. Obtain an order for a muscle relaxant. The nurse is using the correct amount of pressure and does not need to decrease the amount of pressure applied. Stop any further palpation immediately. Discontinuing further palpation is not warranted.

Observe the muscles while the client exhales. Bilateral tensing is often an indication of voluntary guarding by the client. To help distinguish between voluntary and involuntary guarding, the nurse should observe the muscles during exhalation because the client usually does not demonstrate voluntary guarding during exhalation.

Ms. Yong states that she is uncomfortable lying on the exam table, so the nurse assists her to a sitting position before completing the assessment of her knees. The nurse begins by observing the anterior thighs and knees. How should the nurse assess for the presence of muscle atrophy? Gently apply pressure around the patella. This action will not provide data related to muscle atrophy. Observe the size of the muscle. Atrophy, decrease in size, may first be observed in the medial portion of the anterior thigh muscle and is a sign of muscle disuse. Palpate the tissues for edema. Palpation for edema does not provide information about muscular atrophy. Measure the muscle with a goniometer. A goniometer is used to measure joint angles when assessing range of motion.

Observe the size of the muscle. Atrophy, decrease in size, may first be observed in the medial portion of the anterior thigh muscle and is a sign of muscle disuse.

Which assessment is most important for the nurse to complete? Ask the client how long she has colored her hair. Excessive use of artificial hair coloring may cause damage to the hair, but this is of less importance than another finding. Check the client's hair for split ends. Split ends indicate damage due to overheating or drying. Another finding is more significant as an indicator of the client's overall health. Observe the texture and distribution of hair growth on the scalp. Dull, dry, sparse hair may be the result of a nutrient deficiency, such as insufficient protein or zinc. These findings would support the nurse's concerns regarding Amanda's overall nutritional status. Note the pattern of hair growth around the client's forehead. The pattern of hair growth around the forehead is often hereditary and is unlikely to be a significant finding.

Observe the texture and distribution of hair growth on the scalp. Dull, dry, sparse hair may be the result of a nutrient deficiency, such as insufficient protein or zinc. These findings would support the nurse's concerns regarding Amanda's overall nutritional status.

The nurse observes raised, pink wheals on Amanda's neck. How should the nurse respond to this observation? Ask if the client has been stung by an insect recently. Hives (urticaria) are not caused by insect bites. Explain that antibiotics will need to be prescribed. Antibiotics are not useful in the treatment of hives, an inflammatory response to a number of different stimuli. Antihistamines may be useful to manage the symptoms of urticaria. Apply a warm compress directly over the wheals. Warmth will increase vasodilation and worsen the symptoms of this inflammatory response. Offer assurance that this is a temporary response. Urticaria, or hives, is an inflammatory response that is generally transient. Amanda states she often gets hives when she is stressed. Amanda's chart indicates parental consent has been given to administer a prn antihistamine medication.

Offer assurance that this is a temporary response. Urticaria, or hives, is an inflammatory response that is generally transient. Amanda states she often gets hives when she is stressed. Amanda's chart indicates parental consent has been given to administer a prn antihistamine medication.

How should Cilostazole (Pletal) be taken? With 2 full glass of water or juice. Consumption of large amounts of water and consumption of alcoholic beverages should be avoided. One hour before, or two hours after a meal. The medication is most effective if taken on an empty stomach. Prior to driving. The medication can cause dizziness, so it is not recommended that you operate a vehicle or machinery until you are sure you can perform such activities safely. In the morning and before bed. The medication should be taken before breakfast and dinner

One hour before, or two hours after a meal. The medication is most effective if taken on an empty stomach.

Which assessment data are important for the nurse to report to the client's HCP? Presence of S1 and S2 heart sounds. This is a normal finding that does not need to be reported. Onset of an S3 heart sound. This is an abnormal finding, indicative of a change in the client's status, and should be included in the nurse's report. Observed jugular vein distention. This is an abnormal finding, indicative of a change in the client's status, and should be included in the nurse's report. Noted absence of a carotid bruit. This is typically a normal finding, and, in conjunction with the carotid artery pulse volume, indicates no acute problems involving the carotid arteries. Client's subjective report of dyspnea. This is an abnormal finding, indicative of a change in the client's status, and should be included in the nurse's report.

Onset of an S3 heart sound. This is an abnormal finding, indicative of a change in the client's status, and should be included in the nurse's report. Observed jugular vein distention. This is an abnormal finding, indicative of a change in the client's status, and should be included in the nurse's report. Client's subjective report of dyspnea. This is an abnormal finding, indicative of a change in the client's status, and should be included in the nurse's report.

What additional information related to end of life wishes is most important for the RN to assess? Wishes of other children. While the wishes of family members are important to assess, the client's wishes as designated in the living will are more important. Therefore, other information is of higher priority. If the client prepared a will. This information is unlikely to impact nursing care and is not of high priority for the RN to ascertain. Desired funeral home. It is important to obtain information regarding the client's desired mortuary, but other information is of higher priority. Organ donor status. It is essential for the RN to assess the client's wishes regarding organ donation so that any necessary arrangements to preserve organs can be made prior to the client's death.

Organ donor status. It is essential for the RN to assess the client's wishes regarding organ donation so that any necessary arrangements to preserve organs can be made prior to the client's death.

Which assessment by the RN accurately reflects the client's statement? Disoriented to place. The fact that the client does not know her hospital room number does not indicate that she is disoriented. Oriented to situation. The client's statement that she needs to notify her daughter that she is in the hospital indicates she is oriented to her situation. Lack of knowledge of her room number does not reflect disorientation or memory loss. Loss of recent memory. The fact that the client does not know her hospital room number does not indicate that she is experiencing recent memory loss. Loss of immediate memory. The fact that the client does not know her hospital room number does not indicate that she is experiencing immediate memory loss

Oriented to situation. The client's statement that she needs to notify her daughter that she is in the hospital indicates she is oriented to her situation. Lack of knowledge of her room number does not reflect disorientation or memory loss.

An older client is admitted to the hospital with severe diarrhea. The registered nurse (RN) is completing an assessment and notes the client has dry mucous membranes and poor skin turgor. Which assessment data should the RN gather to determine if the client has a fluid volume deficit? Lower extremity edema. Orthostatic hypotension. Elevated blood pressure. Cheyne-Stokes respirations.

Orthostatic hypotension. Orthostatic hypotension can be a sign of fluid volume deficit in an older client who has experienced severe diarrhea.

To ensure that the client's respiratory status is stable upon his arrival on the medical unit, the nurse should complete which assessment first? Breath sounds. The presence of clear versus adventitious breath sounds does not provide the best data to ensure a stable respiratory status. Oxygen saturation. Measurement of the client's oxygen saturation provides information about the effectiveness of gas exchange. A low oxygen saturation level requires immediate nursing intervention. Level of fatigue. Increasing fatigue may be a reflection of poor oxygenation, but this is not the best data to use to ensure physiologic homeostasis. Chest excursion. Assessment of the client's chest excursion does not provide the best data to ensure physiologic homeostasis.

Oxygen saturation. Measurement of the client's oxygen saturation provides information about the effectiveness of gas exchange. A low oxygen saturation level requires immediate nursing intervention.

A patient had a knee replaced because of arthritis. He reports that he has not slept well in several nights. He states that he can't ge comfortable. Today he is asking for pain medication more often. What might be a reason for this increase in pain?

Pain tolerance decreases with sleep deprivation

The registered nurse (RN) is interviewing a female client who states she has a persistent productive cough during the winter caused by bronchitis. Which additional finding should the RN assess for bronchitis? Phlegm production and wheezing. Smoking history. Hemoptysis. Night sweats.

Phlegm production and wheezing. A chronic seasonal cough related to bronchitis is likely accompanied withphlegm production and wheezing. Although smoking can contribute to a chronic cough, the typical seasonal cough is an inflammatory reaction to seasonal changes.

The nurse palpates the dorsalis pedis pulses bilaterally and determines that both pulses are weak and thready. What additional assessment information will validate this finding? Pale, cool skin. Weak, thready pulses indicate diminished arterial circulation. Pale, cool skin is also likely to be present when arterial circulation is diminished, validating the finding of weak, thready pulses. Flushed, moist skin. This finding is not consistent with weak, thready pulses. Inflamed, hot skin. This finding is not consistent with weak, thready pulses. Dry, inelastic skin. This finding may be present, but it does not validate the presence of weak, thready pulses.

Pale, cool skin. Weak, thready pulses indicate diminished arterial circulation. Pale, cool skin is also likely to be present when arterial circulation is diminished, validating the finding of weak, thready pulses.

How should the nurse begin the carotid artery assessment? Palpate one artery while listening to the other side with a stethoscope. The nurse should only assess one artery at a time. Palpate one artery and then palpate the artery on the opposite side. This technique allows the nurse to effectively and thoroughly assess each artery. Gently compress both arteries simultaneously to compare the volume Simultaneous compression of both carotid arteries may impede blood flow to the brain. Avoid palpation and only use a stethoscope to listen to each artery. Palpation should be included in the assessment of the carotid arteries

Palpate one artery and then palpate the artery on the opposite side. This technique allows the nurse to effectively and thoroughly assess each artery.

While continuing to listen at the aortic site, what action should the RN take? Observe the P wave on the telemetry monitor. S1 occurs at the same time as the R wave, the upstroke of the QRS complex. S1 does not coincide with the P wave. Watch the client's inhalation and exhalation. The client's pattern of inhalation and exhalation does not coincide with S1 or S2. Palpate the carotid artery pulse. S1 occurs simultaneously with the carotid artery pulsation. By gently palpating the carotid artery, the nurse can distinguish S1 as the sound that occurs with each pulsation. Check for a pulse deficit. A pulse deficit is assessed by comparing the apical heart rate with the radial pulse and is not useful in distinguishing S1 and S2.

Palpate the carotid artery pulse. S1 occurs simultaneously with the carotid artery pulsation. By gently palpating the carotid artery, the nurse can distinguish S1 as the sound that occurs with each pulsation.

The nurse reviews Lourdes' initial complaint that her feet feel numb. Which assessment should the nurse perform first? Locate the inguinal lymph nodes. Palpation of lymph nodes should be included in the physical assessment, but it is not the priority assessment at this time. Measure toenail capillary refill. Length of capillary refill provides useful information about arterial circulation, but this is not the best assessment to complete at this time. Compare calf circumferences. Calf circumference provides useful data related to possible deep vein thrombosis, but this is not the most important assessment at this time. Palpate the dorsalis pedis pulses. Because the client has complained of numbness, it is important to assess for the presence and strength of the pedal pulses, a measure of the arterial circulation to the feet. The acute absence of arterial circulation would require immediate intervention.

Palpate the dorsalis pedis pulses. Because the client has complained of numbness, it is important to assess for the presence and strength of the pedal pulses, a measure of the arterial circulation to the feet. The acute absence of arterial circulation would require immediate intervention.

The nurse suspects that the patient has appendicitis. Which assessment techniques can the nurse use to confirm his or her suspicion?

Palpate the left lower quadrant at a 90-degree angle and quickly release his or her hand

While inspecting Claudine's abdomen, the nurse observes silvery white striae on the lower abdomen. In response to this finding, what information should the nurse obtain? Recent history of any rashes. This information is not related to the development of striae. Past medical history of ascites. Striae are the result of a change in skin pigmentation that occurs following significant stretching of the elastic fibers of the skin on the abdomen. Causes can include ascites (fluid collection in the peritoneal cavity). Any recent exposure to sunlight. Exposure to sunlight does not cause striae. Previous trauma or injury to the area. Striae are not related to tissue trauma or injury. Change in body mass index (BMI). Striae are the result of a change in skin pigmentation that occurs following significant stretching of the elastic fibers of the skin on the abdomen. Causes can include obesity or pregnancy.

Past medical history of ascites. Striae are the result of a change in skin pigmentation that occurs following significant stretching of the elastic fibers of the skin on the abdomen. Causes can include ascites (fluid collection in the peritoneal cavity). Change in body mass index (BMI). Striae are the result of a change in skin pigmentation that occurs following significant stretching of the elastic fibers of the skin on the abdomen. Causes can include obesity or pregnancy.

Researchers have found that it is the ______ of a recent life event that determines a person's emotional or psychological reaction to it.

Perceptions Each culture influences how a stressful event is perceived and the acceptable ways that people of that culture are expected to respond.

The nurse is interviewing a patient with a history of flack pain, fever, chills, and pain radiating to the groin. Which examination technique is most appropriate for this patient?

Percussion of the costrovertebral angle

What additional assessment should the RN perform to validate the finding of unilateral upper extremity weakness? Perform a palmar drift test. A palmar drift test is used to assess upper extremity weakness. The client is asked to hold up both arms with the palms up and the eyes closed for 10 to 20 seconds. The weak arm will "drift" downward. Complete a Romberg test. A Romberg test assesses the client's balance when standing. Check for a placing reflex. A placing reflex is found in infants when the infant is held upright under the arms and the infant's hip and leg move to "place" the foot. Observe for decorticate posturing. Decorticate posturing, abnormal flexion, is observed when a client has a lesion of the cerebral cortex; it is not a reflection of upper extremity weakness.

Perform a palmar drift test. A palmar drift test is used to assess upper extremity weakness. The client is asked to hold up both arms with the palms up and the eyes closed for 10 to 20 seconds. The weak arm will "drift" downward.

Next, the RN asks Mrs. Green to close her eyes. The RN places the tuning fork in the palm of Mrs. Green's left hand and asks her to identify what she is holding. Mrs. Green is unable to identify the tuning fork. What action should the RN take in response to this finding? Hold the tuning fork on the back of her hand while she tries to identify it. The back of the hand is not used to test for stereognosis, the ability to recognize objects by touch. Document that the client is exhibiting left-sided astereognosis. The RN does not have sufficient data to support this documentation. Place a comb in the client's left hand and ask her to identify the object. Stereognosis, the ability to recognize objects by touch, should be assessed by placing a familiar object in the client's hand. A tuning fork in not a familiar object to many people, so the RN should replace the fork with a more familiar object, such as a comb. Ask the client to open her eyes and identify the object she is holding. Stereognosis, the ability to recognize objects by touch, cannot be assessed if the client is able to see the object.

Place a comb in the client's left hand and ask her to identify the object. Stereognosis, the ability to recognize objects by touch, should be assessed by placing a familiar object in the client's hand. A tuning fork in not a familiar object to many people, so the RN should replace the fork with a more familiar object, such as a comb.

The registered nurse (RN) is caring for a client who has a closed head injury from a motor vehicle collision. Which finding should the RN assess the client for the risk of diabetes insipidus (DI)? High fever. Low blood pressure. Muscle rigidity. Polydipsia.

Polydipsia. A characteristic finding of DI is excretion of large quantities of urine (5 to 20L/day), and most clients compensate for fluid loss by drinking large amounts of water (polydipsia). DI can occur when there has been damage or injury to the pituitary gland or hypothalamus as a result of head trauma, tumor or an illness such as meningitis. This damage interrupts the ADH production, storage and release causing the excessive urination and thirst.

Which disorder is an example of a vascular lesion? 1Dermatofibroma 2Vitiligo 3Sebaceous cyst 4Port wine stain

Port wine stain

During a health history a patient says, "Stressors? Oh, yeah, I have stressors. I got a promotion at work; and, with the extra income I'm going to move into a new house, but that has been delayed because my mother is in the hospital and my son is going off to college. To get through this time I just keep using my support systems, exercising, and medicating." How does a nurse interpret these comments by this patient?

Positive coping strategies

The nurse begins the assessment as the client ambulates in the hallway. What observations should the nurse make while the client is walking to the exam room? Fine motor function. Fine motor function involves movement of the smaller muscles and is not typically observed while the client is ambulating. Posture. Observation of the client's posture can be completed while the client is ambulating. This assessment provides useful data related to musculoskeletal function. Gait. Observation of the client's gait can be completed while the client is ambulating. This assessment provides useful data related to musculoskeletal function. Bone density. Bone density is measured via diagnostic testing and is not directly observable upon examination. Balance. Observation of the client's balance can be completed while the client is ambulating. This assessment provides useful data related to musculoskeletal function.

Posture. Observation of the client's posture can be completed while the client is ambulating. This assessment provides useful data related to musculoskeletal function. Gait. Observation of the client's gait can be completed while the client is ambulating. This assessment provides useful data related to musculoskeletal function. Balance. Observation of the client's balance can be completed while the client is ambulating. This assessment provides useful data related to musculoskeletal function

Which strategy should the nurse use to document the extent of the physical injuries? Complete the Abuse Assessment Screen. The Abuse Assessment Screen is useful to screen clients for possible abuse, but is not used to document the extent of physical injury. Prepare a detailed injury map. An injury map provides is a useful visual documentation of the locations of observable injuries along with descriptive progress notes and photographs of the injuries if the client gives consent to photograph the injuries. Include X-Rays in the client record. Including X-Rays will give a visual account of damage to bones but will not show the extent of injury to soft tissue. Use the numeric Braden scale. The Braden scale is an effective scale used to determine pressure ulcer risk. It is not useful in documenting physical injuries.

Prepare a detailed injury map. An injury map provides is a useful visual documentation of the locations of observable injuries along with descriptive progress notes and photographs of the injuries if the client gives consent to photograph the injuries.

The registered nurse (RN) is assisting the healthcare provider (HCP) with the removal of a chest tube. Which intervention has the highest priority and should be anticipated by the RN after the removal of the chest tube? Prepare the client for chest x-ray at the bedside. Review arterial blood gases after removal. Elevate the head of bed to 45 degrees. Assist with disassembling the drainage system.

Prepare the client for chest x-ray at the bedside. A chest x-ray should be performed immediately after the removal of a chest tube to ensure lung expansion has been maintained after its removal.

The registered nurse (RN) is teaching a client who is newly diagnosed with emphysema how to perform pursed lip breathing . What is the primary reason for teaching the client this method of breathing? Decreases respiratory rate. Increases O2 saturation throughout the body. Conserves energy while ambulating. Promotes CO2 elimination.

Promotes CO2 elimination. Pursed lip breathing helps eliminate CO2 by increasing positive pressure within the alveoli increasing the surface area of the alveoli making it easier for the O2 and CO2 gas exchange to occur .

To gather data related to the frequency of abuse by Amanda's boyfriend, what action should the nurse take first? Instruct the client that there is no point in denying the pattern of abuse because of the varying colors of the bruises. Since this action may, or may not be necessary depending on the client's willingness to provide the information related to abuse frequency, another action should be taken first. Determine if the client has ever tried to break up with her boyfriend and how she attempted to end the relationship. This is additional information the nurse should obtain to help determine the possible risk for danger to the client, but another action should be taken first. Ask the client about her boyfriend's use of any illegal drugs or frequency of alcohol abuse since they have been dating. This is additional information the nurse should obtain to help determine the possible risk for danger to the client, but another action should be taken first. Provide a calendar for the client to mark the dates when any violent and abusive behavior by her boyfriend occurred. A calendar is a useful visual aid in that it can help the client "see" the frequency of the abuse, and it can help the nurse determine if there is an escalation of violence toward the client. This is the first step when implementing a danger assessment for the client. The client may also be requested to complete a scale of violence to help the nurse assess the magnitude of the abuse. Filling in the dates on the calendar helps Amanda recognize the extent of the abuse she has experienced

Provide a calendar for the client to mark the dates when any violent and abusive behavior by her boyfriend occurred. A calendar is a useful visual aid in that it can help the client "see" the frequency of the abuse, and it can help the nurse determine if there is an escalation of violence toward the client. This is the first step when implementing a danger assessment for the client. The client may also be requested to complete a scale of violence to help the nurse assess the magnitude of the abuse. Filling in the dates on the calendar helps Amanda recognize the extent of the abuse she has experienced

When completing the pain assessment, how should the nurse assess for rebound tenderness? Position the client on her right side. It is not necessary to position the client on her side. Lightly palpate over the painful area. When assessing for rebound tenderness, palpation should not be performed over the painful area. Ask the client to describe the pain. A description of the client's pain is not part of the assessment for rebound tenderness. Push down on the left side of the abdomen. After applying pressure at a site away from the area of pain, the nurse quickly lifts and removes the hand from the client's abdomen. Pain upon release of the pressure is referred to as rebound tenderness.

Push down on the left side of the abdomen. After applying pressure at a site away from the area of pain, the nurse quickly lifts and removes the hand from the client's abdomen. Pain upon release of the pressure is referred to as rebound tenderness.

To assess Mrs. Green's recent memory more completely, what action should the RN take? Question her about how she arrived at the hospital today. This action provides information related to the client's recent memory. The RN should ask questions with verifiable answers to ensure the client does not make up responses. Observe her cooperation in answering interview questions. This action provides information related to the client's mood and thought processes. Encourage her to reminisce about the birth of her daughter. This action will help the RN assess the client's remote memory, not her recent memory. List four words and ask her to repeat them back to the RN. This action will help the RN evaluate the client's immediate memory rather than her recent memory.

Question her about how she arrived at the hospital today. This action provides information related to the client's recent memory. The RN should ask questions with verifiable answers to ensure the client does not make up responses.

How should the nurse document the information obtained when charting Amanda's abuse assessment? Delete any expletives the client used when describing the abuser. It is not necessary to delete any specific words or comments made by the client. Quote the client's responses to the questions as verbatim as possible. Documentation should be as verbatim as possible to provide the most detailed, accurate information. Summarize the abusive events without directly quoting the client. This does not provide the most accurate documentation. Refrain from including information that might identify the alleged abuser. Information that identifies the alleged abuser is important to include in the documentation.

Quote the client's responses to the questions as verbatim as possible. Documentation should be as verbatim as possible to provide the most detailed, accurate information.

What movement from the patient does a nurse request to assess for hyperextension of the hip?

Raise one leg at a time while lying prone

The registered nurse (RN) is making early morning rounds on a group of clients when a client begins exhibiting symptoms of an acute asthma attack. The RN administers a PRN prescription for a Beta 2 receptor agonist agent. Which client response should the RN expect? Tachycardia. Increased blood pressure. Rapid resolution of wheezing. Improved pulse oximetry values. Reduce fever airway inflammation.

Rapid resolution of wheezing. Improved pulse oximetry values Beta 2 receptor agonist agents should provide immediate return of airflow and resolve wheezing and improve oxygenation.

The registered nurse (RN) is evaluating a client who presents with symptoms of viral gastroenteritis. Which assessment finding should the RN report to the healthcare provider? Dry mucous membranes and lips. Rebound abdominal tenderness over right lower quadrant. Dizziness when client ambulates from a sitting position. Poor skin turgor over client's wrist.

Rebound abdominal tenderness over right lower quadrant. RLQ rebound abdominal tenderness may be related to acute appendicitis and should be reported to the healthcare provider.

The registered nurse (RN) places an ice pack on a middle school student who comes to the school clinic complaining of a sprained ankle. Which therapeutic response should the RN anticipate? Reduced pain and minimized brusing. Lowering of body core temperature. Increased circulation around injury. Reabsorption of edema at injury.

Reduced pain and minimized brusing. Cold applications produce a topical anesthetic effect to reduce pain as well as constricts blood vessels to minimize bruising.

A 50 year old patient asks how he can reduce his risk of colon cancer. What is the most appropriate response by the nurse?

Regular exercise to reduce body fat helps prevent colon cancer

To inspect for jugular vein distention, what actions should the nurse take? Place the client in a Fowler's position with his head straight. It is not necessary to place the client in a Fowler's position. Lower the head of the bed while observing the neck veins. The client should not be lying flat when the nurse is assessing for distention. Remove the client's pillow and turn his head away slightly. Turning the client's head slightly away allows the nurse to best measure the height of any jugular vein pulsations. Assist the client to lean forward at a 30 to 45° angle. This position will not allow accurate assessment for jugular vein distention. Place the client in a Semi-Fowler's position. Raising the head of the bed to a 30 to 45° angle is the first step when assessing jugular vein distention.

Remove the client's pillow and turn his head away slightly. Turning the client's head slightly away allows the nurse to best measure the height of any jugular vein pulsations. Place the client in a Semi-Fowler's position. Raising the head of the bed to a 30 to 45° angle is the first step when assessing jugular vein distention.

What action should the nurse take next? Document the findings and report the murmur to the charge nurse. Another action should be taken before documenting or reporting the findings. Repeat auscultation across the chest using the bell of the stethoscope. After completing assessment with the diaphragm of the stethoscope, the nurse should repeat the sequence using the bell of the stethoscope. The bell of the stethoscope is used to listen for relatively lower pitched sounds than the diaphragm. Continue assessment of heart sounds across the client's posterior thorax. Heart sounds are not assessed across the posterior thorax. Plan to repeat the assessment in 1 hour, after the client rests. There is no indication of a need to stop the assessment to allow the client to rest.

Repeat auscultation across the chest using the bell of the stethoscope. After completing assessment with the diaphragm of the stethoscope, the nurse should repeat the sequence using the bell of the stethoscope. The bell of the stethoscope is used to listen for relatively lower pitched sounds than the diaphragm.

The nurse observes several bruises of various colors across Amanda's lower abdomen. What concern should the nurse address? Repeated injury over a period of time. New bruises are generally red in color and change color over time. Bruises typically progress from purple-blue to blue-green to green-brown and finally to a brownish-yellow color before disappearing. Recent injury with different sized objects. Different sized objects do not typically cause different bruise coloration. Painful injury with hot and cold objects. Although bruising could occur as a result of a thermal injury due to the force of the trauma, it does not account for the varying coloration of the bruises. Self-inflicted injury to obtain attention. Bruises of varying colors may be the result of self-inflicted injuries, but they are not a specific indicator of self-inflicted injury.

Repeated injury over a period of time. New bruises are generally red in color and change color over time. Bruises typically progress from purple-blue to blue-green to green-brown and finally to a brownish-yellow color before disappearing.

The nurse performs McMurray's test and hears an audible click while maneuvering Ms. Yong's left leg. In response to this finding, what action should the nurse implement? Observe the client's gait as she walks across the room. Observation of the gait is not indicated in response to the results of the McMurray's test. Explain to the client that her knee dislocation has resolved. The assessment and test result do not indicate that the client's knee was dislocated. Plan to instruct the client about knee strengthening exercises. The test results do not indicate that knee strengthening exercises should be started at this time. Report the assessment to the clinic healthcare provider (HCP). An audible click during McMurray's test indicates that the client may have a torn meniscus. This finding should be reported promptly to the HCP for further evaluation and treatment.

Report the assessment to the clinic healthcare provider (HCP). An audible click during McMurray's test indicates that the client may have a torn meniscus. This finding should be reported promptly to the HCP for further evaluation and treatment.

A Muslim male client refuses to let the female registered nurse (RN) listen to his breath sounds during the examination. How should the RN respond? Explain how the nursing skill will be performed before proceeding. Examine client with an additional healthcare provider for support. Request a male nurse or healthcare provider to perform the exam. Avoid any skills that involve touching the client during the exam.

Request a male nurse or healthcare provider to perform the exam. Modesty is an important value in the Muslim community, and Muslims are reluctant to expose any part of their body to healthcare members. Muslim clients are accustomed to examination by "same sex" healthcare providers, so is the best solution for the client.

Which data is most important for the nurse to obtain before contacting the HCP? Pedal pulse volume. The volume of the pedal pulses is not the highest priority assessment at this time. Orientation to situation. The client may be assessed for orientation to person, place, time, and situation. However, another assessment takes priority at this time. White blood cell count. Since there has been a change in the client's breath sounds, indicating a worsening in the client's pneumonia, it is important to note any increase in the client's white blood cell count. However, another assessment takes priority at this time. Respiratory effort. Confusion may be an indicator of decreasing oxygenation, especially in the older person. Based on the client's signs of worsening pneumonia coupled with the confusion, his respiratory rate and effort along with his oxygen saturation level should be obtained before the nurse contacts the HCP.

Respiratory effort. Confusion may be an indicator of decreasing oxygenation, especially in the older person. Based on the client's signs of worsening pneumonia coupled with the confusion, his respiratory rate and effort along with his oxygen saturation level should be obtained before the nurse contacts the HCP.

To begin palpation at the base of the heart, where should the nurse palpate first? Right sternal border, 2nd intercostal space. This is the location of the aortic site. The aortic and pulmonic sites are found at the base of the heart. Right sternal border, 4th intercostal space. This is not the location of the base of the heart. Left sternal border, 5th intercostal space. This is not the location of the base of the heart. Left midclavicular line, 5th intercostal space. This is not the location of the base of the heart.

Right sternal border, 2nd intercostal space. This is the location of the aortic site. The aortic and pulmonic sites are found at the base of the heart.

The registered nurse (RN) is caring for a client who has taken atenolol for 2 years. The healthcare provider recently changed the medication to enalapril to manage the client's blood pressure. Which instruction should the RN provide the client regarding the new medication? Take the medication at bedtime. Report presence of increased bruising. Check pulse before taking medication. Rise slowly when getting out of bed or chair.

Rise slowly when getting out of bed or chair. The client's new medication is an angiotensin-converting enzyme (ACE) inhibitor, which has the side effect oforthostatic hypotension. Instructing the client to rise slowly from a sitting or lying down position is important to teach the client to avoid dizziness and potentially falling.

Before attempting to palpate again, the nurse should give the client what instruction? Lift his left arm above his head. This instruction will not enable the nurse to palpate the apical impulse. Turn onto his right side. This instruction will not enable the nurse to palpate the apical impulse. Externally rotate his right shoulder. This instruction will not enable the nurse to palpate the apical impulse. Roll half-way to his left side. Turning half-way to the left side moves the apex of the heart closer to the chest wall, so it is easier to palpate.

Roll half-way to his left side. Turning half-way to the left side moves the apex of the heart closer to the chest wall, so it is easier to palpate

In listening at this site, what should the nurse attempt to distinguish first? S1 and S2 heart sounds. The nurse should begin by listening for the normal heart sounds, S1 and S2, before attempting to distinguish abnormal heart sounds, such as S3 and S4 or heart murmurs. Diastolic heart murmur. The nurse should listen for other sounds before listening for heart murmurs. S3 and S4 heart sounds. The nurse should listen for other sounds before listening for these abnormal heart sounds. Systolic heart murmur. The nurse should listen for other sounds before listening for heart murmurs.

S1 and S2 heart sounds. The nurse should begin by listening for the normal heart sounds, S1 and S2, before attempting to distinguish abnormal heart sounds, such as S3 and S4 or heart murmurs.

While palpating the client's hands, the nurse asks Ms. Yong if she has any tenderness in her fingers. Ms. Yong reports that sometimes her fingers get stiff after several hours of computer work but states that she is not currently experiencing any tenderness. When using the SOAP format of charting, how should the nurse document this finding? O: Client reports fingers are stiff. The client's report of finger stiffness is not an objective (O) finding. S: History of finger discomfort. The client's history of finger stiffness is a subjective (S) finding, but this notation is too vague. S: Fingers get stiff after computer work. This notation correctly identifies the client's report of finger stiffness as a subjective (S) finding and accurately and concisely describes what the client reported. O: Denies finger tenderness at present. This notation is not an objective (O) finding.

S: Fingers get stiff after computer work. This notation correctly identifies the client's report of finger stiffness as a subjective (S) finding and accurately and concisely describes what the client reported

Which information in Ms. Yong's history reflects a high risk for low back pain? Frequently travels with her husband to Korea by air to visit relatives. Traveling by air does not increase the risk for low back pain. Spends her evenings working in her large vegetable and flower garden. Gardening activities such as bending and pulling, lifting, and moving heavy objects increases the risk for low back pain. Often rides a bicycle to her job as a history professor at a local college. Riding a bicycle and teaching are not activities that increase the risk for low back pain. Volunteers on the weekend as a tour guide at a historical city mansion. Serving as a tour guide involves walking and is unlikely to increase the client's risk for low back pain.

Spends her evenings working in her large vegetable and flower garden. Gardening activities such as bending and pulling, lifting, and moving heavy objects increases the risk for low back pain.

A 60-year-old male patient states that he has a sore above his lip that has not healed and is getting bigger. The nurse observes a red scaly patch with an ulcerated center and sharp margins. These findings are commonly associated with which malignancy? 1Kaposi's sarcoma. 2Malignant melanoma. 3Basal cell carcinoma. 4Squamous cell carcinoma.

Squamous cell carcinoma

What instruction should the RN provide next? Push my fingers back, using both hands at the same time. Another action more effectively assesses the client's muscle strength. Squeeze my fingers with one hand, then with the other. Another action more effectively assesses the client's muscle strength. Pull my fingers forward toward you, one hand at a time. This action will not allow effective evaluation of the client's muscle strength. Squeeze my fingers with both hands at the same time. When performing a hand grip test, the RN asks the client to squeeze the RN's fingers with both hands simultaneously, so that the RN can compare muscle strength bilaterally.

Squeeze my fingers with both hands at the same time. When performing a hand grip test, the RN asks the client to squeeze the RN's fingers with both hands simultaneously, so that the RN can compare muscle strength bilaterally.

It is most important to report which finding to the supervisor? Lack of dependent rubor. This is a normal finding and does not take precedence when reporting to the supervisor. Location of varicose veins. The presence of varicose veins is a chronic problem and is of less importance than another finding. Stage 2 pressure ulcer. A stage 2 pressure ulcer in a client with diminished sensation and circulation requires intervention and should be immediately reported to the supervisor. Bilateral cyanosis in both legs. Bilateral cyanosis with diminished sensation and circulation requires intervention and should be immediately reported to the supervisor. Ankle brachial index of .94. An ABI value of .90 (90%) or less indicates the presence of peripheral vascular disease. A value of .94 is important to report but of less priority than other findings.

Stage 2 pressure ulcer. A stage 2 pressure ulcer in a client with diminished sensation and circulation requires intervention and should be immediately reported to the supervisor. Bilateral cyanosis in both legs. Bilateral cyanosis with diminished sensation and circulation requires intervention and should be immediately reported to the supervisor.

To check for scoliosis, the nurse provides which client instruction? Stand with arms straight at your sides and your feet together. With the client standing upright, the nurse first observes for a difference in the elevation of the shoulders or scapulae before testing further for scoliosis by asking the client to touch her toes. Place hands on hips and lean to one side and then the other. This instruction helps the nurse assess the client's range of motion but is not useful in assessing for scoliosis. Twist from one side to the other with your hands on your hips. This action helps the nurse assess the client's range of motion but is not useful in assessing for scoliosis. Place feet apart and slowly raise both arms above your head. This action is not helpful in assessing for scoliosis.

Stand with arms straight at your sides and your feet together. With the client standing upright, the nurse first observes for a difference in the elevation of the shoulders or scapulae before testing further for scoliosis by asking the client to touch her toes

The registered nurse (RN) is caring for an older client who has been bedridden for two weeks. Which assessment findings indicate to the RN that the client is developing a complication related to immobility? Decreased pedal pulses. Edema in upper extremities. Loss of appetite for food. Stiffness in right ankle joint.

Stiffness in right ankle joint. Stiffness in joints is an early sign of contractures and muscle atrophy related to inactivity and immobility.

Ms. Yong shares with the nurse that she often experiences knee pain. The nurse asks Ms. Yong about other common joint symptoms. On which symptoms should the nurse focus? Stiffness. Stiffness is a common joint symptom related to acute inflammation or arthritis. Swelling. Swelling is a common joint symptom related to acute inflammation or arthritis. Cramping. Cramping is typically associated with muscles rather than joints. Numbness. Numbness is typically associated with nerve or circulation problems rather than joints. Warmth. Warmth is a common joint symptom related to acute inflammation or arthritis.

Stiffness. Stiffness is a common joint symptom related to acute inflammation or arthritis. Swelling. Swelling is a common joint symptom related to acute inflammation or arthritis. Warmth. Warmth is a common joint symptom related to acute inflammation or arthritis.

The RN begins by testing the client's biceps reflex. With the client's forearm resting on the RN's forearm and the RN's thumb over the biceps tendon, what action should the RN take next to test the client's biceps reflex? Ask the client to contract the biceps muscle. Contraction of the muscle should be avoided when preparing to test DTRs. Strike the thumb with the reflex hammer. With the client's forearm slightly flexed and relaxed, the RN should strike the thumb with the pointed end of the reflex hammer to elicit a response. Extend and externally rotate the client's forearm. The arm should be lightly flexed and relaxed rather than extended when testing DTRs. Instruct the client to repeatedly clench her fist. Repeatedly clenching the fist will cause muscular contraction. This should be avoided when testing DTRs.

Strike the thumb with the reflex hammer. With the client's forearm slightly flexed and relaxed, the RN should strike the thumb with the pointed end of the reflex hammer to elicit a response.

Ms. Yong continues the conversation, stating, "I guess I may have to change the way I do certain things in order to continue to do the things I love." How should the nurse respond to this statement? Recognize that the client has regressed to an earlier developmental stage because of her worry about her current physical problems. Erikson supports the understanding that individuals can regress to earlier developmental stages under stressful circumstances. However, the client's statement does not reflect a regression to an earlier developmental task. Offer encouragement to the client as she struggles to find meaning in her life despite her current physical problems. The client's statement does not indicate that she is struggling to find meaning and purpose in her life, the developmental task of the older adult. Document on the assessment form that the client seems to be overly fixated on her current physical problems. Excessive self-concern indicates a negative resolution to the client's developmental task of generativity vs. stagnation. The client's statement does not reflect inappropriate fixation on the changes occurring in her body. Support the client as she considers strategies to adapt to the physiologic changes contributing to her current physical problems. Changing and adapting behavior to maintain control is emphasized by Erikson. A positive resolution to the client's developmental task of generativity vs. stagnation is the ability to be creative and productive, and the RN should support the client in her efforts to adapt creatively to the physiologic changes causing her current physical problems. Explore with the client what strategies she has used in the past to successfully adjust to changes in her life. Changing and adapting behavior to maintain control is emphasized by Erikson. A positive resolution to the client's developmental task of generativity vs. stagnation is the ability to be creative and productive, and the RN should support the client in her efforts to adapt creatively to the physiologic changes causing her current physical problems.

Support the client as she considers strategies to adapt to the physiologic changes contributing to her current physical problems. Changing and adapting behavior to maintain control is emphasized by Erikson. A positive resolution to the client's developmental task of generativity vs. stagnation is the ability to be creative and productive, and the RN should support the client in her efforts to adapt creatively to the physiologic changes causing her current physical problems. Explore with the client what strategies she has used in the past to successfully adjust to changes in her life. Changing and adapting behavior to maintain control is emphasized by Erikson. A positive resolution to the client's developmental task of generativity vs. stagnation is the ability to be creative and productive, and the RN should support the client in her efforts to adapt creatively to the physiologic changes causing her current physical problems.

Ms. Yong lies down on the exam table in a supine position. The nurse assesses adduction and abduction of the hip by instructing the client to take what action? Bend the knee so the foot is flat on the table and allow the knee to drop inward then outward. This action demonstrates internal and external rotation. Swing the entire leg laterally and then medially, keeping the knee straight while moving. Lateral movement demonstrates abduction, and medial movement demonstrates adduction. Lift each leg straight above the body to a 90-degree angle. This action demonstrates flexion and extension. Turn both legs so the toes are pointed inward and then outward. This action does not demonstrate a specific type of range of motion.

Swing the entire leg laterally and then medially, keeping the knee straight while moving. Lateral movement demonstrates abduction, and medial movement demonstrates adduction.

What terminology should accurately be included in the RN's documentation? Dysphagia. Dysphagia, which refers to difficulty swallowing, is not the correct term to use. Tachycardia. Tachycardia refers to a rapid pulse rate, typically greater than 100 in an adult. Syncope. Syncope is a sudden loss of strength or temporary loss of consciousness, which the client described as "passing out." Paresis. Paresis refers to partial or incomplete paralysis

Syncope. Syncope is a sudden loss of strength or temporary loss of consciousness, which the client described as "passing out."

Which immunizations does the nurse ask about when interviewing a 75-year-old client?

Tetanus and influenza

Which of the following findings on a 2-month-old baby is considered abnormal and requires further follow up?

The anterior fontanel is not palpable

What additional observation is important in assessing the mole? The border of the mole is smooth. Border regularity is an important finding because border irregularity may be a cancer danger sign. The nurse assures Amanda that there are no abnormal findings. The mole is surrounded by freckles. This is not a significant finding. There is no inflammation around the mole. Moles do not generally become inflamed, so this is not a significant finding. The mole does not blanche when compressed. Because this is a characteristic of all moles, it is not a significant finding.

The border of the mole is smooth. Border regularity is an important finding because border irregularity may be a cancer danger sign. The nurse assures Amanda that there are no abnormal findings.

Which interview data provides the RN with information related to the client's judgment? Repeating back a list of four words made the client anxious and uncomfortable. Anxiety over the need to perform a test is not a reflection of the client's judgment. The client was cooperative but vague in describing how her neighbor found her. This behavior is appropriate to the situation but is not a reflection of the client's judgment. Reminiscing about the birth of her daughter caused the client to cry gently. This behavior is appropriate to the situation but is not a reflection of the client's judgment. The client indicated the need to notify her daughter that she is in the hospital. The client's recognition of the need to notify her daughter that she is in the hospital is an indication of good judgment.

The client indicated the need to notify her daughter that she is in the hospital. The client's recognition of the need to notify her daughter that she is in the hospital is an indication of good judgment.

The registered nurse (RN) is caring for an Asian client who refuses to make eye contact during conversations. How should the RN assess this client's response? The client cannot understand the nurse. The client is uncomfortable with the nurse. The client is treating the nurse with respect. The client is purposefully disrespecting the nurse.

The client is treating the nurse with respect. In some Asian cultures, it is not appropriate to look a person of authority in the eye, so the client is being respectful by looking down while speaking with the nurse.

The registered nurse (RN) is developing the plan of care for a client who is admitted for alcohol detoxification. Which goal should be most important for the RN to primarily focus the client's care? The client maintains optimal nutritional status. The client will remain alert and oriented. The client will remain free from injury. The client will remain alcohol free during hospitalization.

The client will remain free from injury. The client is at highest risk for injury due to altered cognitive and sensory disturbances as well as delirium tremors during withdrawal. Remaining free from injury is the most important goal for the acute phase of alcohol withdrawal.

In documenting the client's difficulty speaking, the RN recalls that Mrs. Green had difficulty forming some of her words and phrases. Before describing this finding on the assessment form, what additional data should the RN consider? The client's ability to comprehend what she is being asked. Aphasia should be assessed to determine if the client has lost the ability to comprehend information (receptive aphasia) or the ability to express herself (expressive aphasia). Most commonly, the client experiences both, referred to as global aphasia. How many words per minute the client is able to speak. This data is not useful in describing the nature of the client's dysphasia. If any mouth drooping is observed when the client spoke. The presence of facial drooping is not relevant in describing the nature of the client's dysphasia. Whether the client is able to read the RN's lips accurately. The client's ability to read the RN's lips is not relevant in assessing her aphasia.

The client's ability to comprehend what she is being asked. Aphasia should be assessed to determine if the client has lost the ability to comprehend information (receptive aphasia) or the ability to express herself (expressive aphasia). Most commonly, the client experiences both, referred to as global aphasia.

In assessing the client's end-of-life wishes, the RN remembers that Mrs. Green's husband is deceased. It is most important for the RN to communicate with which person? The client's daughter Nakida, her oldest child. While the wishes of a client's children are important, another person is more important in assessing a client's end of life wishes. The client's designated power of attorney for health care. The person designated as a client's power of attorney for health care has been designated by the client to make health care decisions for the client if the client is unable to do so. The client's physician, with whom she has discussed her wishes. The client's physician is an important member of the health care team, but is not the most important person to consult regarding a client's end of life wishes. The client's minister from the Baptist congregation where she is a member. A client's spiritual leader, such as a rabbi, minister, priest, or pastor, may be contacted at the request of the client or family, but this is not the most important person to consult when assessing a client's end-of-life wishes.

The client's designated power of attorney for health care. The person designated as a client's power of attorney for health care has been designated by the client to make health care decisions for the client if the client is unable to do so

Thirty minutes later, the nurse returns to assess Claudine's response to the medication. Which findings provide the best data about the effectiveness of the medication? The client's vital signs are within normal limits. The client's vital signs (within normal limits) provide useful data about the client's response to pain. The client is holding a pillow over her abdomen. The client's actions may provide useful data about her response to pain, but they are not the most useful source of information about the effectiveness of an analgesic. The client's facial expression is calm and relaxed. The client's nonverbal behavior can provide valuable data about her response to pain, and it is a useful source of information about the effectiveness of an analgesic. The client states a lessening of her pain. The client's subjective report regarding her pain is important information for the nurse to assess when evaluating the effectiveness of analgesic administration. The husband reports that his wife looks like her pain has improved. This input is not the most useful data for the nurse as it is not the patient's own description of the effectiveness of the medication.

The client's vital signs are within normal limits. The client's vital signs (within normal limits) provide useful data about the client's response to pain. The client's facial expression is calm and relaxed. The client's nonverbal behavior can provide valuable data about her response to pain, and it is a useful source of information about the effectiveness of an analgesic. The client states a lessening of her pain. The client's subjective report regarding her pain is important information for the nurse to assess when evaluating the effectiveness of analgesic administration.

The registered nurse (RN) is caring for a client with tuberculosis (TB) who is taking a combination drug regimen. The client complains about taking "so many pills." W hat information should the RN provide to the client about the prescribed treatement? The development of resistant strains of TB are decreased with a combination of drugs. Compliance to the medication regimen is challenging but should be maintained. Side effects are minimized with the use of a single medication but is less effective. The treatment time is decreased from 6 months to 3 months with this standard regimen.

The development of resistant strains of TB are decreased with a combination of drugs. Combination therapy is necessary to decrease the development of resistant strains of TB and ensure treatment efficacy.

Which is the best answer the nurse can give in response to how the lymphatic system works? The lymphatic system provides immunity by collecting red blood cells that destroy foreign cells. The lymphatic system collects interstitial fluid not red blood cells. Interstitial fluid is extracellular. Red blood cells are intracellular, and cannot pass through the walls of the capillaries Lymph nodes filter useful substances and contain lymphocytes that activate the immune system. The lymph nodes filter harmful substances. The lymph system carries the lymphocytes throughout the body. They respond to foreign and abnormal substances, and communicate responses to other parts of the body. This summarizes the function of the lymphatic system most concisely and in the most understandable way. Lymph travels through multiple lymphatic channels and nodes before returning to the venous system by the hepatic duct. Lymph returns to the venous system via the thoracic duct.

The lymph system carries the lymphocytes throughout the body. They respond to foreign and abnormal substances, and communicate responses to other parts of the body. This summarizes the function of the lymphatic system most concisely and in the most understandable way.

A nurse is caring for a patient who has just had major abdominal surgery to resect a portion of his colon. What is the most reliable sign that the patient has significant postoperative pain? A. The patient rates his pain a 7 on a scale of 0 to 10. B. The patient winces and guards the area as the nurse gently palpates the abdomen. C. The patient is having trouble sleeping and has become irritable. D. The patient is moaning softly and frowning, with a pinched expression on his face.

The patient rates his pain a 7 on a scale of 0 to 10 Rationale: There is no objective test by which pain can be measured. Pain is a subjective experience and is highly individualized. The patient's self-report is the strongest evidence that he has significant postoperative pain. The patient is the only one who knows whether pain is present and what the experience is like. Wincing and guarding may be associated with pain but are observations by the nurse, not the patient's self-report of pain. Insomnia and irritability may be associated with pain but are observations by the nurse, not the patient's self-report of pain. Vocalization and grimacing may be associated with pain but are observations by the nurse, not the patient's self-report of pain.

The RN observes the client moving her eyes through the six cardinal fields of gaze by following an object or fingers without moving her head. Which cranial nerves are tested when the RN is evaluating the client's extra ocular movements? Trochlear (CN IV). Oculomotor (CN III). Facial (CN VII). Trigeminal (CN V). Abducens (CN VI).

Trochlear (CN IV). Oculomotor (CN III). Abducens (CN VI).

When assessing the skin of a client with bulimia, which data is important for the nurse to obtain? Thickness. Skin thickness does not provide data related to fluid volume balance. Turgor. Assessment of skin turgor, or the degree of skin elasticity, provides data related to fluid volume balance. Texture. Skin texture does not provide data related to fluid volume balance. Pigmentation. Skin pigmentation does not provide data related to fluid volume balance.

Turgor. Assessment of skin turgor, or the degree of skin elasticity, provides data related to fluid volume balance.

The registered nurse (RN) is assessing a male client who arrives at the clinic with severe abdominal cramping, pain, tenesmus, and dehydration. The RN discovers that the client has had 14 to 20 loose stools with rectal bleeding. Which condition should the RN ask the client about his medical history? Irritable bowel syndrome. Diverticulitis. Crohn's disease. Ulcerative colitis.

Ulcerative colitis. The RN should ask the client if he has a history of ulcerative colitis, which is characterized by severe abdominal cramping, pain, tenesmus, and dehydration .

The registered nurse (RN) is caring for a client who developed oliguria and was diagnosed with sepsis and dehydration 48 hours ago. Which assessment finding indicates to the RN that the client is stabilizing? Urine output of 40 mL/hour. Apical pulse 100 and blood pressure 76/42. Urine specific gravity 1.001. Tented skin on dorsal surface of hands.

Urine output of 40 mL/hour. decrease in urinary output is a sign of dehydration. When the urine output returns to a normal range, 40 mL/hour, the client's kidneys are perfusing adequately and indicates the client's status is stablizing

What finding should the nurse expect in response to Amanda's itching? Purpura. Purpura, or bleeding under the skin, does not appear in response to a stressful situation. Hirsutism. Hirsutism, or excessive growth of hair, does not occur in response to a stressful situation. Urticaria. Urticaria, or hives, are highly pruritic and can appear in response to many stimuli, including emotional stress. Pustules. Pustules, or lesions that contain pus in the cavity of the lesion, do not appear in response to a stressful situation.

Urticaria. Urticaria, or hives, are highly pruritic and can appear in response to many stimuli, including emotional stress.

The nurse prepares to palpate the joints in Ms. Yong's wrist and hands. First, the nurse supports the client's hands. What action should the nurse take next? Use both thumbs to apply gentle pressure. Both thumbs are used to apply gentle but firm pressure over the joints to palpate for swelling, thickening, nodules, or tenderness. Use the index fingers to lightly compress the pulses. Compressing the pulses is not helpful in palpating the joints of the hand. Ask the client to spread her fingers apart. Asking the client to spread her fingers apart provides information about the client's range of motion but is not useful or necessary in palpating the joints of the hand. Instruct the client to make a fist with both hands. Instructing the client to make a fist provides information about the client's range of motion and fine motor skills but is not useful or necessary in joint palpation.

Use both thumbs to apply gentle pressure. Both thumbs are used to apply gentle but firm pressure over the joints to palpate for swelling, thickening, nodules, or tenderness.

What follow-up assessment data should the nurse obtain first? Cigarette smoking history. This is important data to collect during the health history, but it is not the first data the nurse should obtain when a client has an increased respiratory rate. Use of accessory muscles. Use of accessory muscles indicates an increased respiratory effort by the client and indicates that the client may be experiencing respiratory distress. Cultural health beliefs. While this data is important to gather as part of the overall client assessment, it does not have the highest priority at this time because it does not reveal further information about the client's respiratory problems. Color of sputum. Although this is important data to obtain, it does not provide additional details about the client's increased respiratory rate.

Use of accessory muscles. Use of accessory muscles indicates an increased respiratory effort by the client and indicates that the client may be experiencing respiratory distress.

A registered nurse (RN) is performing a mini-mental state examination (MMSE) for a client who is being admitted to an assisted living community. Which communication techniques should the RN implement to decrease anxiety in the client? (Select all that apply.) Use simple sentences during the examination. Move to another question if the client seems confused. Reduce environmental detractors during the examination. Allow family to answer for the client to decrease frustration. Ask questions one at a time to decrease confusion.

Use simple sentences during the examination. Reduce environmental detractors during the examination. Ask questions one at a time to decrease confusion ommunication techniques for clients with cognitive impairments should be simple, withoutenvironmental distractions, and direct.

What action will help the nurse confirm the presence of this sound? Move the diaphragm of the stethoscope to the base of the heart. This action will not enable the nurse to hear the extra heart sound more effectively. Use the bell of the stethoscope to continue listening at the apical site. A soft dull sound heard after S2 is an abnormal heart sound. This S3 heart sound is low pitched and is heard best at the apex with the bell of the stethoscope. Palpate the apical impulse while listening at the base of the heart. This action will not enable the nurse to hear the extra heart sound more effectively. Place the bell of the stethoscope at the right sternal border at the third interspace. This action will not enable the nurse to hear the extra heart sound more effectively.

Use the bell of the stethoscope to continue listening at the apical site. A soft dull sound heard after S2 is an abnormal heart sound. This S3 heart sound is low pitched and is heard best at the apex with the bell of the stethoscope.

What data should the RN obtain to complete the client's GCS rating? Verbal response. Best verbal response is assessed using the GCS and is based on a 5 point scale ranging from none to oriented. Pupillary response. The pupillary response to light is an important neurologic assessment, but it is not included in the GCS rating. Eye opening response. Best eye opening response is assessed using the GCS and is based on a 4 point scale ranging from none to opens eyes spontaneously. Babinski reflex. The Babinski reflex is not a component of the GCS rating. Motor response. Best motor response is assessed using the GCS and is based on a 6 point scale ranging from none to obeys commands for movements.

Verbal response. Best verbal response is assessed using the GCS and is based on a 5 point scale ranging from none to oriented. Eye opening response. Best eye opening response is assessed using the GCS and is based on a 4 point scale ranging from none to opens eyes spontaneously. Motor response. Best motor response is assessed using the GCS and is based on a 6 point scale ranging from none to obeys commands for movements.

The RN uses a tuning fork to evaluate what sensory function? Pain. The client's ability to sense pain is assessed by gently pricking the client with a sharp object, such as a needle, rather than with a tuning fork. Vibration. The client's ability to sense vibration is assessed by placing a vibrating tuning fork on a bony surface. Two point discrimination. The client's ability to discriminate between two points is assessed by touching the client at two distinct points and noting the client's ability to distinguish the two separate points. A tuning fork is not used for this assessment. Passive motion. The client's ability to sense passive motion is assessed by moving the client's fingers. A tuning fork is not used for this assessment.

Vibration. The client's ability to sense vibration is assessed by placing a vibrating tuning fork on a bony surface.

After Mr. Jackson stops coughing, the nurse continues the interview. To assess the client's history related to dyspnea on exertion (DOE), what question should the nurse ask Mr. Jackson? "Do you become short of breath while lying flat?" This question will elicit information about orthopnea, i.e. difficulty breathing while recumbent. "What activities cause you to feel short of breath?" The type of activity and the amount of physical effort should elicit information about the client's DOE. "How frequently do you experience difficulty breathing?" This is an important question to include in the interview, but it will not elicit information specific to DOE. "Are you having trouble catching your breath right now?" This question does not specifically address any dyspnea that may occur with varying levels of exertion.

What activities cause you to feel short of breath?" The type of activity and the amount of physical effort should elicit information about the client's DOE.

Which information is most important to report to the nurse assuming responsibility for Claudine's care? The presence of brown spots on the client's abdomen. This information will not directly impact the client's immediate care and is of less importance than other data. The presence of dullness in the suprapubic area. This information will not directly impact the client's immediate care and is of less importance than other data. What time the client received an antiemetic. This information is essential to report to the nurse assuming responsibility for the client to ensure client safety after receiving a sedating medication. The absence of a venous hum over the epigastric area. This is a normal finding and it does not impact the client's care.

What time the client received an antiemetic. This information is essential to report to the nurse assuming responsibility for the client to ensure client safety after receiving a sedating medication.

A client who is uses ipratropium reports having nausea, blurred vision, headaches, and insomnia after using the inhaler. Which action should the registered nurse (RN) implement first? Withhold medication and report symptoms and vital signs to healthcare provider. Give PRN medication for nausea and vomiting and evaluate client in 30 minutes. Reassure client that the ipratropium given will alleviate the symptoms. Delay administration of ipratropium until next maintenance medication is scheduled.

Withhold medication and report symptoms and vital signs to healthcare provider. Headache, nausea, blurred vision and insomnia are symptoms of excessive use of ipratropium, so withholding the medication until the healthcare provider is notified should be initiated to maintain client safety.

As a patient is walking into the exam room, the nurse notices his unsteady gait. What findings does the nurse anticipate during the neurologic exam

a tremor is observed in his hands while he touches his finger to his thumb on the same hand

Which questions are pertinent to ask when obtaining a symptom analysis from a patient who reports headache? Select all that apply. a. "Describe what the headache feels like?" b. "When was your last eye examination?" c. "What makes the headaches worse?" d. "How do you rate the headaches on a scale of 0 (meaning no pain) to 10 (meaning the worse pain ever)?" e. "Do you have any symptoms with the headaches, such as nausea?" f. "When did you first notice the headaches?"

a. "Describe what the headache feels like?" c. "What makes the headaches worse?" d. "How do you rate the headaches on a scale of 0 (meaning no pain) to 10 (meaning the worse pain ever)?" e. "Do you have any symptoms with the headaches, such as nausea?" f. "When did you first notice the headaches?" Correct: "Describe what the headache feels like?", "What makes the headaches worse?", "How do you rate the headaches on a scale of 0 (meaning no pain) to 10 (meaning the worse pain ever)?", "Do you have any symptoms with the headaches, such as nausea?", and "When did you first notice the headaches?" are questions asked in a symptom analysis. Use the mnemonic of OLD CARTS (e.g., onset of symptoms, location and duration of symptoms, characteristics, aggravating factors, related symptoms, treatment used, and severity of symptoms). Incorrect: "When was your last eye examination?" assumes that the headaches are related to a vision problem. Last eye examination is documented in the history under the heading of Past Health History.

While conducting a health history, the nurse asks which questions to assess for risk factors associated with depression? Select all that apply. a. "Has anyone in your family ever been diagnosed with depression?" b. "Have you noticed a change in how much energy you have?" c. "Do you have crying spells?" d. "Do your muscles seem tense?" e. "Do you feel that something bad is about to happen to you?" f. "Do you have difficulty making decisions?"

a. "Has anyone in your family ever been diagnosed with depression?" b. "Have you noticed a change in how much energy you have?" c. "Do you have crying spells?" f. "Do you have difficulty making decisions?" Correct: These questions are related to risk factors for depression. Incorrect: Tense muscles are associated with stress and anxiety rather than depression. Feeling that something bad is about to happen relates to paranoia rather than depression

The nurse is performing a symptom analysis of a patient with pain. Which questions below are appropriate for a symptom analysis? Select all that apply. a. "Have you had any other symptoms such as nausea, vomiting, and sweating?" b. "Where is the pain located?" c. "Have you had a pain like this before?" d. "What does the pain feel like?" e. "What do you do to make your pain better?" f. "In your culture, how are you encouraged to express your pain?"

a. "Have you had any other symptoms such as nausea, vomiting, and sweating?" b. "Where is the pain located?" d. "What does the pain feel like?" e. "What do you do to make your pain better?" Correct: "Have you had any other symptoms such as nausea, vomiting, and sweating?" "Where is the pain located?" "What does the pain feel like?" "What do you do to make your pain better?" These four questions are asked in a symptom analysis that includes the following variables: onset of symptoms, location and duration of symptoms, characteristics, aggravating factors, related symptoms, self-treatment, and severity. Incorrect: "Have you had a pain like this before?" This question relates to the patient's health history. "In your culture, how are you encouraged to express your pain?" This question relates to the patient's culture and does not help analyze the patient's pain experience.

Which questions are pertinent to ask when obtaining a symptom analysis from a patient who reports breathing problems? Select all that apply. a. "How long have you had this problem with your breathing?" b. "Do you have a family history of breathing problems?" c. "Does this breathing problem come and go or is it constant?" d. "What do you do to make your breathing better?" e. "How does this breathing problem affect your work or daily activities?" f. "How many packs of cigarettes do you smoke a day?"

a. "How long have you had this problem with your breathing?" c. "Does this breathing problem come and go or is it constant?" d. "What do you do to make your breathing better?" e. "How does this breathing problem affect your work or daily activities?" Correct: "How long have you had this problem with your breathing?", "Does this breathing problem come and go or is it constant?", "What do you do to make your breathing better?", and "How does this breathing problem affect your work or daily activities?" are questions asked in a symptom analysis. Use the mnemonic of OLD CARTS (e.g., onset of symptoms, location and duration of symptoms, characteristics, aggravating factors, related symptoms, treatment used, and severity of symptoms). Incorrect: "Do you have a family history of breathing problems?" This question relates to the patient's history; "How many packs of cigarettes do you smoke a day?" This question relates to the patient's history.

During symptom analysis, the nurse helps the patient distinguish between dizziness and vertigo. Which description by the patient indicates dizziness? a. "I felt faint, like I was going to pass out." b. "It felt like I was on a merry-go-round." c. "The room seemed to be spinning around." d. "My body felt like it was revolving and could not stop."

a. "I felt faint, like I was going to pass out." A This is a description of lightheadedness, a form of dizziness. B This is consistent with objective vertigo because it includes a sensation of motion. C This is consistent with objective vertigo because it includes a sensation of motion. D This is consistent with subjective vertigo because it includes a sensation of one's body rotating in space.

1. Which statement or question does the nurse use during the introduction phase of the interview? a. "I'm here to learn more about the pain you're experiencing." b. "Can you describe the pain that you're experiencing?" c. "I heard you say that the pain is 'all over' your body." d. "What relieves the pain you are having?"

a. "I'm here to learn more about the pain you're experiencing." A "I'm here to learn more about the pain you're experiencing" is an example of the introduction phase when the nurse tells the patient the purpose of the interview. B "Can you describe the pain that you're experiencing?" is an example of part of a symptom analysis that occurs in the discussion phase. C "I heard you say that the pain is 'all over' your body" is an example of a summary statement by the nurse that occurs in the summary phase. D "What relieves the pain you are having?" is an example of part of a symptom analysis that occurs in the discussion phase.

During a sports physical for a 16-year-old girl, the nurse asks which question to collect data about drug use? a. "Many teenagers have tried street drugs. Have you tried these drugs? " b. "Tell me which street drugs your friends have offered to you?" c. "Do most of your friends drink alcohol or do street drugs?" d. "Your high school has a reputation for drug use. Do you use drugs?"

a. "Many teenagers have tried street drugs. Have you tried these drugs? " A This uses a questioning technique called "permission giving" in which the nurse "gives permission" to the patient to discuss drug use. Questions like this help identify a pattern of drug use and screen for drug abuse. B This is not an appropriate question because the nurse does not need this information to assess the patient. C This is not an appropriate question because the nurse does not need this information to assess the patient. D This is not an appropriate question because the nurse is associating the school's reputation with the patient's behaviors.

What instruction does a nurse give a patient to facilitate palpation of the right lobe of the thyroid gland? a. "Swallow for me one time." b. "Flex your head down and to the left." c. "Rotate your head to the right for me." d. "Hold your breath for a few seconds."

a. "Swallow for me one time." A The patient is asked to swallow to make the thyroid lobe easier to palpate. B This is incorrect to palpate the right lobe. The patient flexes the neck toward the side being palpated. C This is incorrect to palpate the right lobe. The patient flexes the neck toward the side being palpated, but does not rotate the head. D This is not part of the thyroid palpation.

During the history, the patient states that she does not use many drugs. What is the nurse's appropriate response to this statement? a. "Tell me about the drugs you are using currently." b. "To some people six or seven is not many." c. "Do you mean prescription drugs or illicit drugs?" d. "How often are you using these drugs?"

a. "Tell me about the drugs you are using currently." A "Tell me about the drugs you are using currently" is an open-ended question that allows patients to provide further data. B "To some people six or seven is not many" is a comment that does not ask a question or obtain useful data. C "Do you mean prescription drugs or illicit street drugs?" is a closed-ended question that yields data about the type of drugs used only. D "How often are you using these drugs?" asks about frequency of drug use, which is not useful until the drugs are known.

A nurse is interviewing a patient who was diagnosed with type 2 diabetes mellitus 6 months ago. Since that time, the patient has gained weight and her blood glucose levels remain high. The nurse suspects that the patient is noncompliant with her diet. Which response by the nurse enhances data collection in this situation? a. "Tell me about what foods you eat and the frequency of your meals" b. "What symptoms do you notice when your blood sugar levels are high?" c. "You need to follow what the doctor has prescribed to manage your disease" d. "Tell me what you know about the cause of type 2 diabetes."

a. "Tell me about what foods you eat and the frequency of your meals" A "Tell me about what foods you eat and the frequency of your meals" gathers more data from the patient to help the nurse confirm if noncompliance is the reason for the weight gain and high glucose levels. B "What symptoms do you notice when your blood sugar levels are high?" does not help the nurse determine if the patient is noncompliant. It may be useful later when teaching the patient about her disease. C "You need to follow what the doctor has prescribed to manage your disease" does not provide additional data for the nurse and may be viewed as authoritarian and paternalistic. D "Tell me what you know about the cause of type 2 diabetes" assumes that the reason for the weight gain and high glucose levels is a lack of knowledge. A more therapeutic approach is to gather more data from the patient about how the diabetes has been managed.

During an interview, a patient begins to cry and appears angry. Which response by the nurse is most therapeutic? a. "This topic prompted an emotional response, tell me what you are feeling." b. "This topic does not usually cause such an emotional response." c. "Calm down and tell me what is wrong." d. "I will leave you alone for a few minutes so you can pull yourself together."

a. "This topic prompted an emotional response, tell me what you are feeling." A Acknowledging the patient's feelings and encouraging their expression communicates acceptance of the emotion. Crying is a natural behavior and should be permitted. B "This topic does not usually cause such an emotional response" may be perceived by the patient as judgmental and it does not help the patient meet the current need. C Encouraging the patient to stop crying so that the nurse can help is not supportive of the patient's current need. The therapeutic action is to postpone further questioning until the patient is ready to proceed. D Leaving the room so that the patient can be alone is not supportive of the patient.

Which question is most effective in assessing a patient's personal beliefs about health and illness? a. "What or who do you believe controls your health?" b. "Do you see your health care provider annually?" c. "Do you have specific beliefs about health and illness?" d. "Who makes the health decisions in your family?"

a. "What or who do you believe controls your health?" A What or who do you believe controls your health?" is a broad question that allows the patient to describe his or her beliefs. B Asking if the patient visits his or her health care provider annually assumes the patient has a health care provider, and it is a closed-ended question that yields little data. C "Do you have specific beliefs about health and illness?" will yield a "yes" or "no" answer, but does not provide information needed. D "Who makes the health decisions in your family?" does not provide information about the beliefs of the patient

Which questions are appropriate to ask a patient when performing a symptom analysis for a rash? Select all that apply. a. "When did the rash first start?" b. "Do you have a family history of rashes?" c. "What makes the rash worse?" d. "What do you do to make your rash better?" e. "Describe the sensation from the rash, does it burn or itch?" f. "Describe what the rash looked like initially."

a. "When did the rash first start?" c. "What makes the rash worse?" d. "What do you do to make your rash better?" e. "Describe the sensation from the rash, does it burn or itch?" f. "Describe what the rash looked like initially." Correct: These are questions asked in a symptom analysis that includes the following variables: onset of symptoms, location and duration of symptoms, characteristics, severity of symptoms, related symptoms, alleviating factors, aggravating factors, and attempts at self-treatment. Incorrect: This question relates to the patient's history.

An Asian woman comes to the clinic with a complaint of back pain. During the history, she tells the nurse that she usually uses acupuncture for her pain. What is the nurse's best response? a. "When have you used acupuncture, and what effects did it have?" b. "Acupuncture is good for some problems, but for major illnesses it's best to use medications." c. "Why did you use acupuncture?" d. "I have heard that many Asian people use acupuncture."

a. "When have you used acupuncture, and what effects did it have?" A Asking about the use and effectiveness of acupuncture acknowledges the patient's choice of treatment and allows the nurse to gather more data about its effectiveness for this patient. B Asking "Why did you use acupuncture?" is a nontherapeutic response and questions the patient's reason for her health care decisions. C Asking "Why did you use acupuncture?" is a nontherapeutic response and questions the patient's reason for her health care decisions. D Commenting that, "I have heard that many Asian people use acupuncture" is an example of stereotyping.

A temperature of 99.8° F taken in the axilla is equivalent to which temperature value taken orally? a. 100.8° F b. 99.8° F c. 98.8° F d. 97.8° F

a. 100.8° F A Normal temperature readings from the axilla are about 1° F below the normal oral temperature. B Normal temperature readings from the axilla are about 1° F below the normal oral temperature. C Normal temperature readings from the axilla are about 1° F below the normal oral temperature. D Normal temperature readings from the axilla are about 1° F below the normal oral temperature.

Which patient would be expected to experience acute pain? a. A patient who had abdominal surgery 8 hours ago b. A patient who has cancer and has been receiving treatment for 4 months c. A patient who states that he or she has lived with severe pain for many years d. A patient who has been treated unsuccessfully over the past year for back pain

a. A patient who had abdominal surgery 8 hours ago A Acute pain has a recent onset and results from tissue damage; is usually self-limiting; and ends when the tissue heals. B Acute pain has a recent onset and results from tissue damage; is usually self-limiting; and ends when the tissue heals. C This patient has experienced chronic pain for years. Acute pain has a recent onset and results from tissue damage; is usually self-limiting; and ends when the tissue heals. D This patient has experienced chronic pain for one year. Acute pain has a recent onset and results from tissue damage; is usually self-limiting; and ends when the tissue heals.

While assessing a man during a physical examination for work, the nurse suspects alcohol use. Which assessment tool is appropriate in this situation? a. AUDIT screening tool b. Rapid eye test c. Mental status examination d. Holmes Social Readjustment Rating Scale

a. AUDIT screening tool A AUDIT, an abbreviation for Alcohol Use Disorders Identification Test, is the correct assessment tool in this situation. B The rapid eye test is used when there is suspicion of drug intoxication. C Mental status examination is not indicated in this case because there are no data to suggest an alteration in mental status. D The Holmes Social Readjustment Rating Scale is used to assess stress and predict the occurrence of a serious illness over the next 2 years based on stress alone.

After the death of a Native American man, the nurse opened a window to allow spirits to leave. This action is an example of which attribute of the concept of cultural competence? a. Adapting interventions based on cultural practices (Tailoring) b. Gaining information about cultural differences (Knowledge) c. Considering the effects of another's values and experiences (Understanding) d. Showing appreciation for cultural differences (Respect)

a. Adapting interventions based on cultural practices (Tailoring) A The nurse changed (adapted) the usual procedure after death by opening the window, which was a practice of this Native American family. B Gaining information is the first step, but is insufficient to adapt an intervention as described in this example. C Understanding develops after knowledge, but does not involve taking action. D Respect is the step before Adaptation, when nurses comprehend the cultural differences, but do not adapt nursing care.

During a report, a nurse hears about a patient who was admitted at 8 PM after an automobile accident. He had a blood alcohol level of 100 mg/dl at the time of admission. During the 8 AM assessment, the nurse notes that the patient is having hand tremors, is sweaty, is slightly agitated, and complains of nausea. The nurse recognizes that the patient may be exhibiting signs of which disorder? a. Alcohol withdrawal syndrome b. Delirium tremens c. Panic d. Delirium

a. Alcohol withdrawal syndrome A The history and laboratory data reflect alcohol withdrawal syndrome. Early manifestations of alcohol withdrawal include hand tremors, sweating, nausea and vomiting, anxiety, and agitation. These manifestations begin 6 to 24 hours after the patient's last drink, peak in 24 to 36 hours, and end after 48 hours of abstinence. B The time sequence is not consistent with delirium tremens. During delirium tremens, a patient experiences cardiac dysrhythmias, hypertension, increased respirations, profuse sweating, delusion, and hallucinations. C The history is not consistent with panic disorder. In the panic level of anxiety, a person experiences manifestations of anxiety that represent sympathetic nervous system stimulation, as well as muscle tension, tachycardia, dyspnea, hypertension, increased respiration, and profuse perspiration. D Manifestations of delirium include attention deficits, disorganized thinking, confusion, disorientation, restlessness, incoherence, anxiety, excitement, and, at times, illusion

According to research findings, which site is preferred for measuring blood pressure when the nurse is unable to use the patient's upper arms? a. Ankle b. Thigh c. Calf d. Wrist

a. Ankle A A study comparing accuracy among sites recommended the ankle as an alternative site for blood pressure measurement. B The thigh is an alternative site, but the ankle is the preferred site. C A study comparing accuracy among sites recommended the ankle site in preference to the calf as an alternative site for blood pressure measurement if the upper arm is unavailable. D Approaches to measuring blood pressure using the wrist and finger sites have been developed, but these lack acceptable accuracy and cost efficiency to be recommended for clinical practice.

Which activity illustrates the concept of secondary prevention? a. Annual mammogram b. Nutrition classes on low-fat cooking c. Education on living with diabetes mellitus d. Cardiac rehabilitation after coronary artery bypass surgery

a. Annual mammogram A A mammogram screens for breast cancer and is an example of secondary prevention to promote early detection of disease. B Nutrition classes are an example of primary prevention to prevent a disease from developing by promoting a healthy lifestyle. C Education about diabetes mellitus is an example of tertiary prevention directed toward minimizing the disability from chronic disease and helping the patient maximize his or her health. D Cardiac rehabilitation after coronary artery bypass surgery is an example of tertiary prevention directed toward minimizing the disability from chronic disease and helping the patient maximize his or her health.

What signs of cyanosis does a nurse inspect for in a dark-skinned patient? a. Ashen-gray color of the oral mucous membranes b. Blue color in the nail beds c. Ashen-blue color in the palms and soles d. Blue-gray color in the ear lobes and lips

a. Ashen-gray color of the oral mucous membranes A Cyanosis is manifested by ashen-gray color of the oral mucous membranes and nail beds in a dark-skinned patient. B An ashen-gray color of the nail beds is expected in a dark-skinned patient, rather than blue. C An ashen-gray color of the oral mucous membranes and nail beds are expected in a dark-skinned patient. D An ashen-gray color of the oral mucous membranes and nail beds are expected in a dark-skinned patient.

How do nurses assess pain of neonates or of adults with dementia or decreased level of consciousness? Select all that apply. a. Ask family or caregivers what indicators they think may indicate the patient's pain. b. Review results of blood tests for signs of pain. c. Administer the ordered analgesic to the patient. d. Identify any physiologic signs of pain. e. Examine the patient for possible causes of pain.

a. Ask family or caregivers what indicators they think may indicate the patient's pain. c. Administer the ordered analgesic to the patient. d. Identify any physiologic signs of pain. e. Examine the patient for possible causes of pain. Correct: These four answers are the clinical practice recommendations of Herr and colleagues. Incorrect: Pain cannot be detected with laboratory tests.

A nurse can improve cultural awareness with which behavior? a. Being sensitive to differences between the cultures of the nurse and patient b. Making generalizations about various ethnic and cultural groups c. Learning everything about the various cultural groups in the nurse's city d. Taking a foreign language class

a. Being sensitive to differences between the cultures of the nurse and patient A Being sensitive to differences between the cultures of the nurse and patient. This allows the nurse to be open and to collect data about each unique patient. B Making generalizations about various ethnic and cultural groups leads to stereotyping. C Learning everything about the various cultural groups in the nurse's city provides useful information about a specific city, but does not necessarily improve cultural awareness of cultures in other cities. D Learning a foreign language may help a nurse learn information about one culture, but does not improve cultural awareness of other cultures.

A nurse is assessing an 80-year-old patient who is cared for at home by his 79-year-old wife. Which data indicate this patient has malnutrition? Select all that apply. a. Body mass index (BMI) of 17 b. Waist-to-hip ratio of 1.0 c. Weight loss of 6% since last month's visit d. Prealbumin level of 16 mg/dl e. Hematocrit level of 50% f. Hemoglobin level of 20 g/dl

a. Body mass index (BMI) of 17 c. Weight loss of 6% since last month's visit f. Hemoglobin level of 20 g/dl Correct: A BMI of 18.5 to 24.9 is the normal range, and this patient is below normal. Severe weight loss is a more than 2% weight change over 1 week. The expected hemoglobin for a man is 14 to 18 g/dl. These values may also indicate dehydration. Incorrect: Waist-to-hip ratio of 1.0 is an expected value for men. The expected level for prealbumin is 15 to 36 mg/dl. This hematocrit level is within normal limits.

A nurse is assessing the eyes of a healthy 72-year-old adult. What findings does the nurse expect? Select all that apply. a. Bulbar conjunctiva pink and clear, with small red vessels noted b. Sclera yellow and moist, cornea transparent c. Extraocular movement symmetric with peripheral vision noted d. Newspaper held at 18 inches to see clearly e. Sclera visible between upper lid and iris f. Gray to white circle noted where the sclera merges with the cornea g. Light reflects on the cornea at 12 o'clock in each eye

a. Bulbar conjunctiva pink and clear, with small red vessels noted c. Extraocular movement symmetric with peripheral vision noted g. Light reflects on the cornea at 12 o'clock in each eye Correct: These are expected findings from an assessment of the eyes of a healthy adult. Incorrect: Sclera should be white and moist. Newspaper held at 18 inches to see clearly is due to presbyopia due to the patient's age. Patient must hold paper further away to see clearly. The upper lid should cover the upper part of the iris. Sclera is visible in hyperthyroidism. A gray to white circle is arcus senilis, which is an abnormal finding in older adults.

What findings does the nurse expect when assessing the ears of a healthy adult? Select all that apply. a. Cerumen noted in the outer ear canal b. Pinna located below the external corner of the eye c. Cone of light located in the 5 o'clock position in the left ear d. Ratio of air conduction to bone conduction 2:1 e. Tympanic membrane pearly gray f. Whispered words repeated accurately

a. Cerumen noted in the outer ear canal d. Ratio of air conduction to bone conduction 2:1 e. Tympanic membrane pearly gray f. Whispered words repeated accurately Correct: These are all expected findings from an assessment of the ears of a healthy adult. Incorrect: The pinna should align with the outer canthus of the eye. Cone of light should be located in the 7 o'clock position in the left ear and the 5 o'clock position in the right ear.

A patient complains of itching, swelling, and drainage from the eyes with a postnasal drip and sneezing. What type of nasal drainage does the nurse anticipate seeing during inspection of this patient's nares? a. Clear b. Malodorous c. Yellow d. Green

a. Clear A The patient has allergic rhinitis, which produces clear drainage. B Malodorous drainage is associated with bacterial infection, which is not consistent with the history given by this patient. C Yellow drainage is associated with bacterial infection, which is not consistent with the history given by this patient. D Green drainage is associated with bacterial infection, which is not consistent with the history given by this patient.

After collecting the data, the nurse begins data analysis with which action? a. Clustering data b. Documenting subjective data c. Reporting information to other health team members d. Documenting objective information

a. Clustering data A After collecting data, the nurse organizes or clusters the data so that the problems appear more clearly. To cluster data, the nurse interprets the assessment data collected. B Documenting subjective data is necessary for the medical record, but does not provide analysis. C Before reporting data to health team members, the nurse clusters and interprets data. D Documenting objective data is necessary for the medical record, but does not provide analysis

When inspecting a patient's nasal mucous membrane, which finding does the nurse expect to see? a. Deep pink turbinates b. Red, edematous mucous membranes c. Septum that angles to the left d. Clear exudate

a. Deep pink turbinates A These are expected for a nasal inspection. B These indicate a local infection within the nose. C This is abnormal. D This occurs with nasal allergies.

A 24-year-old male patient tells the nurse he has had no energy for 2 weeks. He has no trouble falling asleep; in fact, he sleeps deeply about 12 hours every night. He states that he has gained 10 lb in the past 2 months and has no friends. The nurse associates these manifestations with which mental health disorder? a. Depression b. Schizophrenia c. Bipolar disorder d. Anxiety disorder

a. Depression A These are symptoms of depression. B Clinical manifestations of schizophrenia include apathy and confusion, delusions and hallucinations, and rambling or stylized patterns of speech. C Characteristics of the manic phase are excessive emotional displays, excitement, euphoria, and hyperactivity. In contrast, characteristics of the depressive phase are marked apathy and feelings of profound sadness, loneliness, guilt, and lowered self-esteem. D Anxiety is a feeling of uneasiness or discomfort experienced in varying degrees, from mild anxiety to panic. The energy that anxiety provides may mobilize a person to take constructive action such as solving a major problem or filling an unmet need.

What function do neurotransmitters have in mental health disorders? a. Dopamine levels are increased in schizophrenia. b. Increased levels of gamma aminobutyric acid (GABA) contribute to anxiety. c. Serotonin is decreased in a state of anxiety. d. Norepinephrine is increased in depression.

a. Dopamine levels are increased in schizophrenia. A Dopamine levels are increased in schizophrenia. B Insufficient GABA may contribute to anxiety. GABA is an inhibitory neurotransmitter. C Serotonin is increased in anxiety states. D Norepinephrine is decreased in depression.

A nurse notices several reddish purple, nonblanchable spots of different sizes on the arms and legs of a patient with a low platelet count. How does the nurse distinguish ecchymosis from purpura? a. Ecchymosis is variable in size and a purpura is greater than 0.5 cm in diameter. b. Ecchymosis does not blanch and purpura does blanch. c. Ecchymosis has raised lesions and purpura has flat lesions. d. Ecchymosis is irregularly shaped and purpura is round.

a. Ecchymosis is variable in size and a purpura is greater than 0.5 cm in diameter. A Ecchymosis is variable in size and a purpura is greater than 0.5 cm in diameter. This is an accurate statement. B Ecchymosis does not blanch and purpura does blanch. Both of these lesions are nonblanchable. C Ecchymosis has raised lesions and purpura has flat lesions. Both of these lesions are flat. D Ecchymosis is irregularly shaped and purpura is round. There is no specified shape for either type of lesion.

While giving a history, a patient reports itching arms, legs, and chest after using a new soap. What manifestations does the nurse expect to find on the arms, legs, and chest when inspecting this patient's skin? a. Elevated irregularly shaped areas of edema of variable diameter b. Elevated, firm, and rough lesions with flat surface greater than 1 cm in diameter c. Elevated circumscribed superficial lesions less than 1 cm in diameter filled with serous fluid d. Elevated, firm circumscribed areas less than 1 cm in diameter

a. Elevated irregularly shaped areas of edema of variable diameter A This is a description of wheals, which occur as a result of allergic reactions. B This is a description of plaque. C This is a description of a vesicle. D This is a description of a papule.

During the history, a patient reports watery nasal drainage from allergies. Based on this information, what does the nurse expect to find on inspection of the nares? a. Enlarged and pale turbinates b. Polyps within the nares c. High vascularity of the turbinates d. Dry and dull turbinates

a. Enlarged and pale turbinates A Enlarged and pale turbinates are expected findings for allergic rhinitis. B Polyps within the nares is not an expected finding. C High vascularity of the turbinates is not an expected finding. D Dry and dull turbinates is not an expected finding.

During a physical examination, the nurse notes that the patient's skin is dry and flaking, with patches of eczema, and suspects a nutritional deficiency. What additional data should the nurse expect to find to confirm the suspicion? a. Hair loss and hair that is easily removed from the scalp b. Inflammation of the tongue and fissured tongue c. Inflammation of peripheral nerves, and numbness and tingling in extremities d. Fissures and inflammation of the mouth

a. Hair loss and hair that is easily removed from the scalp A Hair loss (alopecia) and hair that is easily removed from the scalp (easy pluck ability), such as dry, flaking skin, is caused by essential fatty acid deficiency. B Inflammation of the tongue (glossitis) and fissured tongue are manifestations of a niacin deficiency. C Inflammation of peripheral nerves (neuropathy) and numbness and tingling in extremities (paresthesia) are manifestations of a thiamine deficiency. D Fissures of the mouth (cheilosis) and inflammation of the mouth (stomatitis) are manifestations of a pyridoxine deficiency.

Which findings does the nurse expect when assessing the mouth of a healthy adult? Select all that apply. a. Lips appear pink, smooth, moist, and symmetric b. Teeth are white, yellow, or gray, with smooth edges c. Exposed tooth neck and brown spots between teeth d. Slight roughness on the dorsum of the tongue e. Hard palate appears smooth, pale, and immovable f. Mucous membranes are dry and intact

a. Lips appear pink, smooth, moist, and symmetric b. Teeth are white, yellow, or gray, with smooth edges d. Slight roughness on the dorsum of the tongue e. Hard palate appears smooth, pale, and immovable Correct: These are all expected findings from a mouth assessment of a healthy adult. Incorrect: Receding gums expose tooth neck and may indicate gingival disease. Brown spots may indicate caries. Dry and intact mucous membranes may indicate dehydration.

Each patient has had consistent blood pressure readings during the last three clinical visits. Which patient has a blood pressure consistent with expected findings? a. Mr. P, whose blood pressure has been 110/78 b. Ms. J, whose blood pressure has been 140/90 c. Mr. Q, whose blood pressure has been 130/76 d. Ms. Y, whose blood pressure has been 120/80

a. Mr. P, whose blood pressure has been 110/78

After assessment of the nose and paranasal sinuses, which finding requires further investigation by the nurse? a. Nasal septum off the midline b. Nose in the midline of the face c. Middle turbinates deep pink in color d. Noiseless exchange of air from each naris

a. Nasal septum off the midline A A deviated septum is an abnormal finding that needs further investigation. B This is an expected finding. C This is a normal finding. D This is a normal finding.

A patient reports having migraine headaches on one side of the head that often start with an aura and last 1 to 3 days. As a part of the symptom analysis, the patient reports which associated symptoms of migraine headaches? a. Nausea, vomiting, or visual disturbances b. Nasal stuffiness or discharge c. Ringing in the ears or dizziness d. Red, watery eyes or drooping eyelids

a. Nausea, vomiting, or visual disturbances A These are symptoms associated with migraine headaches. B This is a symptom associated with cluster headaches rather than migraine headaches. C These symptoms are not associated with migraine headaches. D These are symptoms associated with cluster headaches rather than migraine headaches.

Which nurse demonstrates culturally competent care for a female patient from Russia? a. Nurse A who asks the patient about cultural factors that influence health care b. Nurse B who interacts with every patient from Russia in the same manner c. Nurse C who learns the cultural variables of every culture, including Russia d. Nurse D who relies on her previous experience with patients from Russia

a. Nurse A who asks the patient about cultural factors that influence health care A Asking the patient about cultural factors that influence health care is demonstrating culturally competent care, along with interacting with each patient as a unique person who is a product of past experiences, beliefs, and values. B Interacting with every patient from Russia in the same manner does not allow for the uniqueness of each person within the same culture. C Learning the cultural variables of every group encountered can be valuable but it is impractical to learn about all cultures because each patient is unique. A better approach is to ask patients about their beliefs. D Relying on previous experience with patients from Russia does not allow for the uniqueness of each person within the same culture.

A nurse notes that a 2-year-old child has multiple bruises over his body at different stages of healing. What is the most appropriate action for the nurse at this time? a. Obtain further data now to rule out abuse. b. Remind parents that toddlers are clumsy and may fall, causing bruising. c. Determine if this toddler has a coagulation disorder. d. Recommend further observation at future visits.

a. Obtain further data now to rule out abuse. A Further investigation is needed to rule out abuse. The important clue is bruises in different stages in healing. Injuries to the skin are generally recognized in three forms: bruises, bites, and burns. B Remind parents that toddlers are clumsy and may fall, causing bruising. The important clue is bruises in different stages of healing. C A coagulation disorder can be ruled out by a laboratory test for platelets. D Recommend further observation at future visits. Action must be taken during this visit if abuse is suspected.

Which cranial nerve is assessed by using the Snellen visual acuity chart? a. Optic cranial nerve (CN II) b. Oculomotor cranial nerve (CN III) c. Abducens cranial nerve (CN IV) d. Trochlear cranial nerve (CN VI)

a. Optic cranial nerve (CN II) A The optic cranial nerve (CN II) provides vision tested by the Snellen visual acuity chart. B CN III controls pupillary constriction, eyelid movement, and eyeball movement. C CN IV controls eyeball movement. D CN VI controls eyeball movement.

In the labor and delivery department, the nurse notices that two women who are in labor are responding differently to their contractions. The first woman, who is having her first baby, has rated her pain as a "7," seems agitated, and has asked for pain medication. The second woman, who is having her third baby, has also rated her pain as a "7," but is calmer and says she does not need anything for pain at this time. What explains the differences in the outward responses to pain between these women? a. Pain tolerance b. Pain threshold c. Nociception d. Physiologic stress

a. Pain tolerance A Pain tolerance is the duration or intensity of pain a person will endure before outwardly responding. A person's culture, pain experience, expectations, role behaviors, and physical and emotional health influence pain tolerance. The second woman had experienced the birth process before and had different expectations than the first woman, who was having her first baby. B Pain threshold is the point at which a stimulus is perceived as pain. This threshold does not vary significantly among people or in the same person over time. C Nociception is the process of pain perception and involves transduction, transmission, perception, and modulation. D Physiologic stress stimulates the sympathetic nervous system causing tachycardia, increased respiratory rate, and dilated pupils, but does not necessarily affect pain response.

A nurse notices a patient's nails are thin and depressed with the edges turned up. What additional abnormal data should the nurse expect to find on this patient? a. Pale conjunctiva b. Jaundice c. Ecchymosis d. Rashes

a. Pale conjunctiva A The abnormal nail finding was koilonychia, which occurs in patients with anemia who frequently have pale conjunctiva. B Jaundice is due to increased serum bilirubin, indicating liver or gallbladder disease, and does not create changes in nail structure. C Ecchymosis occurs after trauma to the blood vessel resulting in bleeding under the tissue and does not cause changes in nail structure. D Rashes indicate an inflammation or allergic reaction that does cause changes in the nails.

A patient admitted to the emergency department with "excruciating chest pain, above the rating of 10," has a heart rate of 55, rapid, irregular respirations, complains of nausea, and is too weak to move to the stretcher without aid. The nurse recognizes that this response to severe pain is due to the response of the _____ nervous system. a. Parasympathetic b. Sympathetic c. Central d. Peripheral

a. Parasympathetic A During severe or deep pain the parasympathetic nervous system may cause pallor; rapid, irregular breathing; nausea; and vomiting. B The sympathetic nervous system responds to acute pain by increasing heart rate, increasing blood pressure, causing diaphoresis, increasing respiratory rate, increasing muscle tension, dilating pupils, and decreasing gastrointestinal motility. C The central nervous system includes the brain and spinal cord. The manifestations described in the case are due to parasympathetic nervous system stimulation. D The manifestations described in the case are due to parasympathetic nervous system stimulation.

What does the nurse teach to parents to prevent sudden infant death syndrome (SIDS)? a. Place the baby on back to sleep. b. Place the baby on side to sleep. c. Not to feed the baby for 3 hours before sleep. d. Place the baby on her stomach to sleep.

a. Place the baby on back to sleep. A The American Academy of Pediatrics recommends positioning infants on their backs; the slogan to help people remember is "Back to Sleep." B The side-lying position is not recommended for sleep because of the risk of aspiration. C Not feeding the baby for 3 hours before sleep is not a prevention for SIDS. D The prone position is not recommended for sleep due to the risk of aspiration.

An adult patient comes to the clinic complaining of right ear pain. What technique does the nurse use to inspect this patient's auditory canal? a. Position the otoscope speculum 1.0 to 1.5 cm (about 0.5 inches) into the ear canal. b. Remove cerumen from each canal before inserting otoscope. c. Choose the smallest otoscope speculum that will fit the patient's ear comfortably. d. Pull the pinna slightly downward and backward before inserting the otoscope speculum.

a. Position the otoscope speculum 1.0 to 1.5 cm (about 0.5 inches) into the ear canal. A This is the correct technique. B Removing cerumen is not necessary. C The largest speculum that comfortably fits in the ear canal is the one that should be chosen. D For adults, the pinna is pulled up and backward to straighten the ear canal.

A patient who has anorexia nervosa reports a healthy diet and no protein calorie malnutrition. Which lab value best confirms this patient's report? a. Prealbumin b. Serum albumin c. Blood glucose d. Serum cholesterol

a. Prealbumin A Prealbumin is a reflection of protein and calorie intake for the previous 2 to 3 days. B Serum albumin measures circulating protein. Albumin can be affected by a number of factors, including fluid status, blood loss, liver function, and stress. Fluctuation of albumin levels occurs over 3 to 4 weeks. C Blood glucose measures can be affected by many factors such as stress, which raises blood glucose levels, and is not a measure of protein calorie malnutrition. D Serum cholesterol is part of the lipid profile, which provides data on fat metabolism only, and is not a measure of protein calorie malnutrition.

When inspecting a patient's eyes, the nurse assesses the presence of cranial nerve III (oculomotor nerve) by observing the eyelids open and close bilaterally. What other technique does a nurse use to test the function of this cranial nerve? a. Pupillary constriction to light b. Visual acuity c. Peripheral vision d. Presence of the red reflex

a. Pupillary constriction to light A Cranial nerve III (oculomotor) controls pupillary dilation and constriction, as well as eyelid movement. Pupil dilation and ptosis may occur when CN III is impaired. B Cranial nerve II (optic) provides vision. C Cranial nerve II (optic) provides peripheral vision. D The red reflex is not controlled by cranial nerve III, but is created by a light illuminating the retina.

When the patient's chart includes a notation that petechiae are present, what finding does a nurse expect during inspection? a. Purplish-red pinpoint lesions b. Deep purplish or red patches of skin c. Small raised fluid-filled pinkish nodules d. Generalized reddish discoloration of an area of skin

a. Purplish-red pinpoint lesions A Purplish-red pinpoint lesions describes the appearance of petechiae. B Petechiae are pinpoints, not as large as a patch. C Petechiae are pinpoints, not raised as a nodule. D Petechiae are pinpoints, not generalized.

On palpation the nurse determines that the patient's left thyroid lobe is larger than the right thyroid lobe. What is the nurse's most appropriate action at this time? a. Refer the patient to the health care provider for further evaluation. b. Document that the patient's thyroid is normal on palpation. c. Palpate the left thyroid lobe again using very firm pressure. d. Ask the patient to flex the chin toward his chest and palpate again.

a. Refer the patient to the health care provider for further evaluation. A The nurse found an abnormality that needs referral for follow-up. B This is not an appropriate action because the nurse found an abnormality. C Repeating the examination will yield the same abnormal finding. D Repeating the examination will yield the same abnormal finding.

What are characteristics of an audioscope? a. Screens for hearing ability b. Allows visualization into the ear canal c. Must be calibrated before use d. Uses vibration to estimate hearing loss

a. Screens for hearing ability A An audioscope screens for hearing ability. B The otoscope allows inspection of the ear canal. C Calibration is unnecessary. An audioscope needs batteries that are charged. D The tuning fork is the tool that uses vibration to detect hearing loss.

Which patient information does the nurse document in the patient's physical assessment? a. Slurred speech b. Immunizations c. Smoking habit d. Allergies

a. Slurred speech A Slurred speech should be noticed by the nurse and documented as objective data in the physical assessment. B Data on immunizations are collected from the patient, are subjective, and documented in the history. C A smoking habit is information that comes from the patient, making it subjective data that is documented in the history. D Allergies are information that come from the patient, making it subjective data that is documented in the history.

Nurses inquire about lifestyle behaviors in those patients with specific risk factors for cataracts. Which characteristics are associated with risk factors for cataracts? Select all that apply. a. Smoking more than 20 cigarettes a day b. Having parents with cataracts c. Chronic consumption of alcohol d. Having a chronic disease, such as diabetes mellitus e. Being Asian f. Being a man

a. Smoking more than 20 cigarettes a day c. Chronic consumption of alcohol d. Having a chronic disease, such as diabetes mellitus Correct: These are all risk factors for cataracts. Incorrect: Having parents with cataracts is not a genetic or familial disorder. Being Asian or a man are not risk factors, but being an African-American or being a woman are risk factors.

While giving a history, a male patient describes several events out of order that occurred in different decades in his life. What technique does the nurse use to understand the timeline of these events? a. State the order of events as understood and ask the patient to verify the order. b. Draw conclusions about the order of events from data given. c. Ask the patient to elaborate about these events. d. Ask the patient to repeat what he said about these events.

a. State the order of events as understood and ask the patient to verify the order. A State the order of events as understood and ask patient to verify the order is correct. This technique is useful when interviewing a patient who rambles or does not provide sequential data. B Drawing conclusions about the order of events is interpretation. In this example, the sequence of events is more relevant than an interpretation. The nurse may have difficulty interpreting an unclear sequence of events. C Asking the patient to elaborate about these events will not provide order to the sequence of events. D Asking the patient to repeat what he said about these events will not necessarily provide a sequence of events.

A nurse is assessing a patient who complains of "awful" abdominal pain and rates it as a 9 on a scale of 0 to 10. Which of the following physiologic signs may accompany acute pain? Select all that apply. a. Tachycardia b. Irritability c. Increased blood pressure d. Depression e. Insomnia f. Sweating

a. Tachycardia c. Increased blood pressure f. Sweating Correct: The sympathetic nervous system responds to acute pain by increasing heart rate, increasing blood pressure, causing diaphoresis, increasing respiratory rate, increasing muscle tension, dilating pupils, and decreasing gastrointestinal motility. Incorrect: Irritability, depression, and insomnia are manifestations of chronic rather than acute pain.

When using an ophthalmoscope to examine the internal eye, how does the nurse distinguish the retinal arteries from the retinal veins? a. The arteries are narrower than veins. b. The arteries are a darker red than veins. c. The arteries have no light reflex and the veins have a narrow band of light in the center. d. The arteries have prominent pulsations and veins have no pulsations.

a. The arteries are narrower than veins. A The artery-to-vein width should be 2:3 to 4:5. B Arteries are lighter red than veins. C Arteries have a narrow band of light in the center and veins have no light reflex. D Arteries show little to no pulsations and venous pulsations may be visible.

An example of a health promotion question included in the health history is: a. "Do you have any allergies?" b. "How often are you exercising?" c. "What are you doing to relieve your leg pain?" d. "What kind of herbs are you using?"

b. "How often are you exercising?" A "Do you have any allergies?" is a question for the present health status rather than health promotion. B "How often are you exercising?" is a question about activities patients regularly perform to maintain health. C "What are you doing to relieve your leg pain?" is a question that is part of the symptom analysis. D "What kind of herbs are you using?" is a question for the present health status rather than health promotion.

A nurse shines a light toward the bridge of the patient's nose and notices that the light reflection in the right cornea is at the 9 o'clock position and in the left cornea at the 9 o'clock position. What is the interpretation of this finding? a. The extraocular muscles of both eyes are intact. b. The cornea of each eye is transparent. c. The sclera of each eye is clear. d. The consensual reaction of both eyes is intact.

a. The extraocular muscles of both eyes are intact A The reflection of the light in both eyes in the same location indicates muscles holding the eyes are symmetric. B The reflection of the light in both eyes in the same location indicates muscles holding the eyes are symmetric. C The reflection of the light in both eyes in the same location indicates muscles holding the eyes are symmetric. D Consensual reaction involves constriction of pupils.

The temperature of a patient is measured every 6 hours at 6 AM, 12 PM, 6 PM, and 12 AM. Which temperature reading is expected to be low due to a normal variation? a. The measurement at 6 AM b. The measurement at 12 PM c. The measurement at 6 PM d. The measurement at 12 AM

a. The measurement at 6 AM A Early in the morning is the time of the lowest temperature of the day due to circadian rhythms. B A low temperature due to circadian rhythms is not expected at this time. C The highest temperature occurs in the late afternoon and early evening due to circadian rhythms. D A low temperature due to circadian rhythms is not expected at this time.

Which finding indicates that this patient has a sensorineural hearing loss? a. The patient hears sound by air conduction longer than by bone conduction. b. The patient hears sound from a vibrating tuning fork in the affected ear only. c. The patient hears normal conversation at 40 dB and a whisper at 20 dB. d. The patient hears the rubbing of fingers together from a distance of 4 inches from each ear.

a. The patient hears sound by air conduction longer than by bone conduction. A In the Rinne test, hearing sound from a vibrating tuning fork longer by air conduction than by bone conduction is consistent with a sensorineural hearing loss. B This finding from the Weber test is consistent with a conduction hearing loss. C This is an expected finding using audiometry. D This is an expected finding using the finger rubbing screening hearing test.

The nurse taking a patient's blood pressure recognizes that several factors may cause an increased blood pressure reading. Which factors below can increase blood pressure? Select all that apply. a. The patient rates pain at a level of 7 on a scale of 0 to 10. b. The cuff was reinflated before being completely deflated. c. The patient drank cold milk just before the reading. d. The time of day is late afternoon. e. The cuff is too wide for the extremity.

a. The patient rates pain at a level of 7 on a scale of 0 to 10. b. The cuff was reinflated before being completely deflated. d. The time of day is late afternoon Correct: Rating pain at a level of 7 on a scale of 0 to 10, reinflating the cuff before being completely deflated, and taking the reading in late afternoon are all factors that can increase blood pressure. Incorrect: Drinking cold milk just before the reading will not affect blood pressure, but drinking caffeine such as coffee or cola may increase blood pressure. A wide cuff makes the reading lower than it actually is rather than higher.

Which is an example of data a nurse collects during a physical examination? a. The patient's lack of hair and shiny skin over both shins b. The patient's stated concern about lack of money for prescriptions c. The patient's complaints of tingling sensations in the feet d. The patient's mother's statements that the patient is very nervous lately

a. The patient's lack of hair and shiny skin over both shins A The lack of hair and shiny skin over both shins are objective data or signs that are part of the physical examination B A patient's concerns about lack of money are subjective data and are part of the health history. C A patient's complaints of tingling sensations in the feet are subjective data and are part of the health history. D A patient's family statements are considered secondary data, are subjective data, and are part of the health history.

A nurse is taking vital signs of an adult patient whose oxygen saturation is 96%. The patient's temperature is 102° F, blood pressure is 130/86, pulse is 100 beats/min, and respiratory rate is 26 breaths/min. Which factor may be contributing to the elevated respiratory rate? a. The patient's temperature b. The patient's oxygen saturation c. The patient's pulse rate d. The patient's blood pressure

a. The patient's temperature A Fever is a factor that may increase respiratory rate, and this patient's temperature is 102° F. B The patient's oxygen saturation is a measure of the oxygen carried by hemoglobin and it is within expected limits—above 90%. C The patient's pulse rate may be due to the high temperature, but a pulse of 100 does not contribute to an elevated respiratory rate in this case. D The patient's blood pressure is higher than normal, but does not contribute to an elevated respiratory rate in this case.

For which person is a shift assessment indicated? a. The person who had abdominal surgery yesterday b. The person who is unaware of his high serum glucose levels. c. The person who is being admitted to a long-term care facility. d. The person who is beginning rehabilitation after a knee replacement.

a. The person who had abdominal surgery yesterday A A shift assessment is most appropriate for the person who is recovering in the hospital from surgery. B A screening assessment is performed for the purpose of disease detection, in this case diabetes mellitus. C A comprehensive assessment is performed during admission to a facility to obtain a detailed history and complete physical examination. D An episodic or follow-up assessment is performed after knee replacement to evaluate the outcome of the procedure.

The nurse is using the Snellen chart to assess a patient's vision. The patient states that the green line on the chart is shorter than the red line. What is the interpretation of this finding? a. This patient has normal color perception and abnormal field perception. b. This patient is color blind but has normal field perception. c. This patient's color perception and field perception are normal. d. This patient is color blind and has abnormal field perception.

a. This patient has normal color perception and abnormal field perception. A Naming the colors of the horizontal lines is a screening for color perception. The top line is green, and the bottom line is red. Asking which line is longer is a screening for field perception measurement. The green line is longer. B This is an incorrect interpretation of the data. C This is an incorrect interpretation of the data. D This is an incorrect interpretation of the data.

A patient has come to the clinic complaining of a "bump" behind his right ear. Upon inspection, the nurse notes a lesion that is elevated, solid, and 4 cm in diameter. What does the nurse call this lesion when she reports her findings to the health care provider? a. Tumor b. Nodule c. Keloid d. Papule

a. Tumor A A tumor is an elevated and solid lesion, may or may not be clearly demarcated, extends deeper in the dermis, and greater than 2 cm in diameter. B A nodule is an elevated, firm, circumscribed lesion that extends deeper into the dermis than a papule and is 1 to 2 cm in diameter. C A keloid is an irregularly-shaped, elevated, progressively-enlarging scar that grows beyond the boundaries of the wound. D A papule is an elevated, firm, circumscribed area less than 1 cm in diameter.

A patient is being seen in the clinic for suspected nasal obstruction from a foreign body. The nurse recognizes which finding as most consistent with this diagnosis? a. Unilateral foul-smelling drainage b. Bilateral purulent green-yellow discharge c. Bilateral bloody discharge d. Unilateral watery discharge

a. Unilateral foul-smelling drainage A This is consistent with presence of a foreign object in one side of the nose. B This is consistent with a nasal or sinus infection. C This is consistent with localized trauma, such as a nasal fracture. D This is consistent with a history of head injury and may indicate skull fracture.

Which action by the nurse results in the patient's blood pressure measurement being falsely high? Select all that apply. a. Using a blood pressure cuff that is too narrow for the patient's upper arm b. Deflating the blood pressure cuff too rapidly c. Wrapping the blood pressure cuff too loosely d. Reinflating the blood pressure cuff before it completely deflates e. Positioning the patient's arm above the level of the heart

a. Using a blood pressure cuff that is too narrow for the patient's upper arm c. Wrapping the blood pressure cuff too loosely d. Reinflating the blood pressure cuff before it completely deflates e. Positioning the patient's arm above the level of the heart Correct: Using a blood pressure cuff that is too narrow for the patient's upper arm, wrapping the cuff too loosely, reinflating the cuff before it completely deflates, and positioning the patient's arm above the level of the heart all result in readings that are falsely high. Incorrect: Deflating the blood pressure cuff too rapidly causes the blood pressure reading to be falsely low.

Which action by the nurse results in the patient's blood pressure measurement being falsely low? Select all that apply. a. Using a blood pressure cuff that is too wide for the patient's arm b. Not inflating the blood pressure cuff enough c. Positioning the patient's arm above the level of the heart d. Wrapping the cuff too loosely around the arm e. Deflating the cuff too rapidly

a. Using a blood pressure cuff that is too wide for the patient's arm b. Not inflating the blood pressure cuff enough e. Deflating the cuff too rapidly Correct: Using a blood pressure cuff that is too wide for the patient's arm, not inflating the blood pressure cuff enough, and deflating the cuff too rapidly could result in a false low reading. Incorrect: Positioning the patient's arm above the level of the heart and wrapping the cuff too loosely around the arm causes the blood pressure to be falsely high

While assessing a patient's lower extremities, the nurse suspects the lower extremities feel cooler than the upper extremities. To confirm this suspicion, how does the nurse compare the temperatures of the lower extremities with the upper extremities? a. Using the backs (dorsum) of the hands to detect differences b. Using the ulnar surface of the hands to detect differences c. Using the pads of the fingers to detect differences d. Using the palmar surface (underside) of the hands to detect differences

a. Using the backs (dorsum) of the hands to detect differences A The dorsal surfaces of the hands detect temperature best. B The ulnar surfaces of the hands are the most sensitive to vibration. C The pads of the fingers are used in palpation. D The palmar surfaces (underside) of the fingers and finger pads are better for determining position, texture, size, consistency, masses, fluid, and crepitus.

How does the nurse detect an extra heart sound in an adult? a. Using the bell of a stethoscope b. With a pulse oximeter c. Using the diaphragm of a stethoscope d. With a Doppler ultrasound probe

a. Using the bell of a stethoscope A The bell of the stethoscope is used to hear soft, low-pitched sounds such as extra heart sounds or vascular sounds (bruit). B Pulse oximetry is a noninvasive measurement of arterial oxygen saturation in the blood. C The diaphragm is used to hear high-pitched sounds such as breath sounds, bowel sounds, and normal heart sounds. D A Doppler ultrasound probe is used to detect difficult-to-hear vascular sounds such as fetal heart tones or peripheral pulses

Which data do nurses document under the heading of Personal and Psychosocial History? Select all that apply. a. Walks for 45 minutes each day b. Eats meats, vegetables, and fruit at two meals daily c. Is allergic to milk and milk products d. Is married and has two daughters whom is he close to e. Smokes marijuana once a week f. Grandfather died from prostate cancer

a. Walks for 45 minutes each day b. Eats meats, vegetables, and fruit at two meals daily d. Is married and has two daughters whom is he close to e. Smokes marijuana once a week Correct: Walks for 45 minutes each day is documented under health promotion activity in Personal and Psychosocial History; eats meats, vegetables, and fruit at two meals daily is documented about diet activity in Personal and Psychosocial History; is married and has two daughters whom is he close to is documented under family and social relationship activity in Personal and Psychosocial History; smokes marijuana once a week is documented under personal habits activity in Personal and Psychosocial History. Incorrect: Allergic to milk and milk products is an allergy, which is documented under the heading Present Health Status; Grandfather died from prostate cancer is documented under the heading Family History.

A nurse notices that the patient has gained 11 lb. If this increase in weight is related to fluid retention, the patient is retaining approximately how many liters of fluid? a. 1 L b. 5 L c. 11 L d. 24 L

b. 5 L A Every kg equals a liter of fluid. Thus, 11 lb ÷ 2.2 = 5 L. B Every kilogram (kg) equals a liter of fluid. Thus, 11 lb ÷ 2.2 = 5 L. C Every kg equals a liter of fluid. Thus, 11 lb ÷ 2.2 = 5 L. D Every kg equals a liter of fluid. Thus, 11 lb ÷ 2.2 = 5 L. This answer is obtained by multiplying 11 by 2.2 instead of dividing.

A nurse is caring for a woman who has given birth to a healthy baby. The woman's husband and mother are in the room, and more family members are in the lobby. Which comment by the nurse demonstrates culturally competent care? a. "We need to take your baby to the nursery now for a physical examination." b. "Are there any ceremonies or other practices that are important to you at this time?" c. "We can only allow immediate family in the room with you at this time." d. "Because breastfeeding is the best way to feed your baby, we'll bring your baby to you when she is hungry."

b. "Are there any ceremonies or other practices that are important to you at this time?" A Although a physical examination is important, it is not necessary for culturally competent care and can be performed at another time. B Asking about ceremonies or other practices collects data about culturally-based behaviors of this patient and family surrounding childbirth. C Limiting visitors based on an arbitrary or subjective definition of "immediate family" may interfere with culturally competent care. D Breastfeeding is a choice the mother makes, not the nurse.

During a mental health history, the nurse suspects altered mental status for a patient. Which questions are appropriate to ask when assessing mental status? Select all that apply. a. "Do you have difficulty making decisions?" b. "Do you know where you are?" c. "Are there times when you wanted to escape?" d. "If you bought a hat for $5.75 and gave the sales person $10.00, how much change do you expect back?" e. "What would you do if a fire started in your home?" f. "What does this phrase 'A rolling stone gathers no moss' mean?"

b. "Do you know where you are?" d. "If you bought a hat for $5.75 and gave the sales person $10.00, how much change do you expect back?" e. "What would you do if a fire started in your home?" f. "What does this phrase 'A rolling stone gathers no moss' mean?" Correct: "Do you know where you are?" assesses orientation. "If you bought a hat for $5.75 and gave the sales person $10.00, how much change do you expect back?" assesses calculation ability. "What would you do if a fire started in your home?" assesses judgment. "What does this phrase 'A rolling stone gathers no moss' mean?" assesses abstract reasoning. Incorrect: For the nurse to assess mental status, the patient needs to demonstrate abilities such as calculation, judgment, and abstract reasoning.

A patient complains of pain in the calf when walking. Which question should the nurse ask for further data? a. "Does your calf also swell when this pain occurs?" b. "Does the pain go away when you stop walking?" c. "Do you become short of breath when you're walking?" d. "Do you feel dizzy when the pain occurs?"

b. "Does the pain go away when you stop walking?"

What does the nurse say to obtain more data about a patient's vague statement about diet such as, "My diet's okay"? a. "Eating a variety of meats, fruits, and vegetables each day is important." b. "Give me an example of the foods you eat in a typical day." c. "Go on." d. "Does your diet meet your needs or does it need improvement

b. "Give me an example of the foods you eat in a typical day." A "Eating a variety of meats, fruits, and vegetables each day is important." While this statement is true, it does not obtain data about what foods the patient consumes. B "Give me an example of the foods you eat in a typical day." This statement asks the patient to clarify the vague statement, "My diet is okay." C "Go on" encourages patients to continue talking, but does not help clarify what foods are consumed. D "Does your diet meet your needs or does it need improvement?" This response does not help clarify what foods the patient eats. Also it contains two questions rather than asking one question at a time.

Which questions are pertinent for a nurse to ask a patient while performing a review of the cardiovascular system? Select all that apply. a. "Do you remember what your last cholesterol value was?" b. "Have you had chest pain or shortness of breath?" c. "Do you have trouble breathing when you lie down?" d. "Are your feet cold, numb, or do they change color?" e. "How much do you weigh?" f. "Have you noticed edema in your ankles at the end of the day?"

b. "Have you had chest pain or shortness of breath?" c. "Do you have trouble breathing when you lie down?" d. "Are your feet cold, numb, or do they change color?" f. "Have you noticed edema in your ankles at the end of the day?" Correct: "Have you had chest pain or shortness of breath?", "Do you have trouble breathing when you lie down?", "Are your feet cold, numb, or do they change color?", and "Have you noticed edema in your ankles at the end of the day?" are questions asked to give the patient an opportunity to report symptoms of the cardiovascular system. Incorrect: "Do you remember what your last cholesterol value was?" relates to a lab value, which is objective data not documented in the history; "How much do you weigh?" is objective data not documented in the history.

A nurse is getting a history from a patient who is disabled from rheumatoid arthritis. Which question will provide data about this patient's functional ability? a. "When did your arthritis symptoms begin?" b. "How has your arthritis affected your daily life?" c. "Why did you come to the clinic today?" d. "How do you feel about your diagnosis of rheumatoid arthritis?"

b. "How has your arthritis affected your daily life?" A "When did your arthritis symptoms begin?" is a question asked as part of the history, but does not collect data about functional ability. B "How has your arthritis affected your daily life?" is a question that leads to data about the patient's ability to perform self-care activities or functional abilities. C "Why did you come to the clinic today?" is a question asked to obtain the chief complaint about a current problem, but does not focus directly on the functional assessment. D "How do you feel about your diagnosis of rheumatoid arthritis?" is a question to ask in the psychosocial history, but does not focus directly on the functional assessment.

Which explanation is most appropriate for a nurse preparing to palpate a patient's neck? a. "I need to feel for tumors in your neck." b. "I'm going to feel your neck for any abnormalities." c. "I need to press deeply on your neck so please hold still." d. "Is there any tenderness in your neck?"

b. "I'm going to feel your neck for any abnormalities." A I need to feel for tumors in your neck" uses the term "tumors" and may alarm the patient unnecessarily. B Palpating the neck enters the patient's personal space and may have cultural significance. Thus it is important to inform patients of the impending action and its purpose. C "I need to press deeply on your neck so please hold still" may alarm the patient and is not accurate. To palpate the neck, light palpation is used to detect abnormalities such as enlarged nodes. Deep palpation is used on the abdomen. D "Is there any tenderness in your neck?" obtains subjective data, but does not tell the patient what the nurse is planning to do.

A patient asks the nurse if it is possible to grow new skin. What is the nurse's most appropriate response? a. "Even if new skin growth is required, the melanocytes do not regenerate." b. "The avascular epidermis sheds slowly and is replaced completely every 4 weeks." c. "The outer layer of skin remains the same over the lifetime except for repairing injuries." d. "Epidermal regeneration is impossible because it is avascular."

b. "The avascular epidermis sheds slowly and is replaced completely every 4 weeks." A Melanocytes are not involved in regeneration. They secrete melanin, which provides pigment for the skin and hair and serves as a shield against ultraviolet radiation. B Within this deepest layer of epidermis, active cell generation takes place. As cells are produced, they push up the older cells toward the skin surface. The entire process takes about 30 days. C The dead cells are continuously sloughed off and replaced by new cells moving up from the underlying epidermal layers. D Within this deepest layer of epidermis, active cell generation takes place.

When a patient complains of chest pain, which question is pertinent to ask to gain additional data? a. "What were you doing when the pain first occurred?" b. "What does the pain feel like?" c. "Do you have episodes of shortness of breath?" d. "Has anyone in your family ever had a similar pain?

b. "What does the pain feel like?"

During the first prenatal visit for a 20-year-old Hispanic woman, the nurse assesses the patient's health beliefs and practices. Which questions are appropriate as part of this assessment? Select all that apply. a. "You are Hispanic, do you need me to find an interpreter?" b. "What is the language that is usually spoken in your home?" c. "How do you define health and illness?" d. "Which Catholic church do you attend?" e. "Do you have specific beliefs or preferences concerning food or food preparation?" f. "Do you or the members of your family have certain beliefs and practices surrounding pregnancy and childbirth?"

b. "What is the language that is usually spoken in your home?" c. "How do you define health and illness?" e. "Do you have specific beliefs or preferences concerning food or food preparation?" f. "Do you or the members of your family have certain beliefs and practices surrounding pregnancy and childbirth?" Correct: "What is the language that is usually spoken in your home?" "How do you define health and illness?" "Do you have specific beliefs or preferences concerning food or food preparation?" "Do you or the members of your family have certain beliefs and practices surrounding pregnancy and childbirth?" These four questions support culturally sensitive nursing care. Incorrect: "You are Hispanic, do you need me to find an interpreter?" assumes that all Hispanic persons speak Spanish only and is an example of stereotyping. "Which Catholic church do you attend?" assumes that all Hispanics are Catholics and is an example of stereotyping.

Which statement is appropriate to use when beginning an interview with a new patient? a. "Have you ever been a patient in this clinic before?" b. "What is your purpose for coming to the clinic today?" c. "Tell me a little about yourself and your family." d. "Did you have any difficulty finding the clinic?"

b. "What is your purpose for coming to the clinic today?" A "Have you ever been a patient in this clinic before?" is a close-ended question that yields a "yes" or "no" response. This question may be asked on the first visit, but not as an opening question for a health interview. B "What is your purpose for coming to the clinic today?" is an open-ended question that focuses on the patient's reason for seeking care. C "Tell me a little about yourself and your family" is an open-ended question, but it is too general, and it is at least two questions: one about the patient and another about the family. D "Did you have any difficulty finding the clinic?" is a social question and does not focus on the patient's purpose for the visit.

A female patient states that she has had problems with depression in the past and thinks she is depressed again. Which response by the nurse is most appropriate? a. "What do you think is causing your depression this time?" b. "What therapies have worked for you in the past?" c. "Did you stop taking your medication?'" d. "Do you think this is a situational depression?"

b. "What therapies have worked for you in the past?" A This question provides information but does not direct the patient toward identifying a treatment. B This question is a therapeutic response to determine if the same or similar therapy can be used again for this depression. It is an open-ended question to collect more data. Also treatment is information collected in a symptom analysis that is useful in this situation. C This question sounds accusatory, and the nurse is guessing the cause of this episode of depression without collecting data from the patient. This is a closed-ended question asking for a "yes" or "no" response. D This is a closed-ended question and does not collect data to determine if the patient has depression again.

A patient is put on an 1800-calorie a day diet plan. During discharge teaching, the nurse explains to this patient how to use nutrition labels to determine the amount of carbohydrates in the product. The nurse explains, however, that the label is based on 2000 calories. Which is the appropriate formula to teach this patient of the maximum grams of carbohydrates she can eat on her prescribed diet? a. 1800 calories ´ 0.45 = 810/4 calories/gram = 202.5 g b. 1800 calories ´ 0.60 = 1080/4 calories/gram = 270 g c. 1800 calories ´ 0.55 = 990/9 calories/gram = 110 g d. 1800 calories ´ 0.50 = 900/9 calories/gram = 100 g

b. 1800 calories ´ 0.60 = 1080/4 calories/gram = 270 g A Carbohydrates should account for a maximum of 60% of total calories rather than 45%. B Carbohydrates should account for 55% to 60% of total calories. Each gram of carbohydrates yields 4 calories. C Each gram of carbohydrates yields 4 calories rather than 9. D Carbohydrates should account for a maximum of 60% of total calories, rather than 50%. Each gram of carbohydrates yields 4 calories rather than 9.

Which finding on assessment of a patient's eyes should the nurse document as abnormal? a. An Asian American patient with an upward slant to the palpebral fissure b. A Caucasian American patient whose sclerae are visible between the upper and lower lids and the iris c. An African American patient who has off-white sclerae with tiny black dots of pigmentation near the limbus d. An American Indian patient whose pupillary diameters are 5 mm bilaterally

b. A Caucasian American patient whose sclerae are visible between the upper and lower lids and the iris A An Asian American patient with an upward slant to the palpebral fissure has an expected racial variation. B A Caucasian American patient whose sclerae are visible between the upper and lower lids and the iris has eyeball protrusion beyond the supraorbital ridge, which indicates exophthalmos caused by hyperthyroidism. C An African American patient who has off-white sclerae with tiny black dots of pigmentation near the limbus has an expected racial variation. D An American Indian whose pupils are 5 mm bilaterally is an expected finding.

Which situation illustrates a screening assessment? a. A patient visits an obstetric clinic for the first time and the nurse conducts a detailed history and physical examination. b. A hospital sponsors a health fair at a local mall and provides cholesterol and blood pressure checks to mall patrons. c. The nurse in an urgent care center checks the vital signs of a patient who is complaining of leg pain. d. A patient newly diagnosed with diabetes mellitus comes to test his fasting blood glucose level.

b. A hospital sponsors a health fair at a local mall and provides cholesterol and blood pressure checks to mall patrons A A detailed history and physical examination conducted during a first-time visit to an obstetric clinic is an example of a comprehensive assessment. B A health fair at a local mall that provides cholesterol and blood pressure checks is an example of a screening assessment focused on disease detection. C Assessing a patient complaining of leg pain in the triage area of an urgent care center is an example of a problem-based/focused assessment. D A patient's return appointment 1 month after today's office visit to report fasting blood glucose levels is an example of an episodic or follow-up assessment.

For which patient is a focused health history most appropriate? a. A new patient at the health clinic for an annual examination b. A patient admitted to the hospital with vomiting and abdominal pain c. A patient at the health care provider's office for a sport physical d. A patient discharged 11 months ago who is being readmitted today

b. A patient admitted to the hospital with vomiting and abdominal pain A A new patient at the health clinic for an annual examination needs a comprehensive history that includes biographic data, reason for seeking care, present health status, past medical history, family history, personal and psychosocial history, and a review of all body. B A patient admitted to the hospital with vomiting and abdominal pain benefits from a focused health history that limits data to the immediate problem. C A patient with a specific need, such as a sport physical, needs a history for an episodic assessment. D A patient discharged months ago who is being readmitted needs a history for a follow-up assessment that generally focuses on the specific problem or problems that caused the patient to be readmitted.

Which patient may require additional nutritional assessment? a. A male patient with a blood glucose level of 100 mg/dl b. A pregnant patient with a hemoglobin level of 10.5 g/dl c. A female patient with a prealbumin level of 25 mg/dl d. A male patient with a serum triglyceride level of 100 mg/dl

b. A pregnant patient with a hemoglobin level of 10.5 g/dl A This patient's glucose level is within normal limits. B The expected hemoglobin level for a pregnant patient is 12 to 16 g/dl. C This patient's prealbumin level is within normal limits. D This patient's triglyceride level is within normal limits.

Using an ophthalmoscope, how does the nurse bring a patient's interior eye structures into focus? a. Using the red filter b. Adjusting the diopters c. Dilating the patient's pupils d. Using the wide-beam light

b. Adjusting the diopters A The red filter facilitates the identification of pallor of the disc and permits the recognition of retinal hemorrhages by making the blood appear black. B The lens selector dial (diopter) allows the nurse to adjust a set of lenses that controls focus. C When the patient's pupils are dilated, a larger light may be used for the internal eye examination. D The wide beam light can be used when the patient's pupils are dilated for better visualization of internal structures.

A nurse assessing the hearing of a patient with presbycusis expects which finding on a test for hearing? a. Bone conduction will be longer than air conduction on the Rinne test (BC > AC). b. Air conduction will be longer than bone conduction on the Rinne test (AC > BC). c. Sound lateralizes to the affected ear on the Weber test. d. Sound lateralizes to both ears equally on the Weber test.

b. Air conduction will be longer than bone conduction on the Rinne test (AC > BC). A This finding indicates a conduction hearing loss, rather than a sensorineural hearing loss. B This finding indicates a sensorineural health loss, the most common cause of presbycusis. C This finding indicates a conduction hearing loss, rather than a sensorineural hearing loss. D This is a normal finding on this test.

A nurse notices multiple lesions on the back of a patient's left hand that are 0.5 cm in width, elevated, circumscribed, and filled with serous fluid. How does the nurse document these lesions? a. As multiple macules on the dorsum of the left hand b. As multiple vesicles on the dorsum of the left hand c. As several patches on the left hand d. As several bullae on the dorsum of the left hand

b. As multiple vesicles on the dorsum of the left hand A Macules are flat, circumscribed areas that are a change in the color of the skin and are less than 1 cm in diameter. B Vesicles are elevated, circumscribed, superficial (do not extend into dermis), filled with serous fluid, and less than 1 cm in diameter. This documentation tells the number and location of the lesions. C Patches are flat, nonpalpable, irregular-shaped macules greater than 1 cm in diameter. This documentation does not include location of lesions. D Bullae are large vesicles greater than 1 cm in diameter.

Which data do nurses document under the heading of Past Health History? Select all that apply. a. Father has Alzheimer disease b. Last tetanus in 2009 c. Had chicken pox as a child d. Drinks three to four beers each day e. Had a dental examination 6 months ago

b. Last tetanus in 2009 c. Had chicken pox as a child e. Had a dental examination 6 months ago Correct: Last tetanus is an immunization, chicken pox as a child is a childhood illness, and last examinations, including dental, are documented under the heading of Past Health History. Incorrect: Family History documents father's Alzheimer disease; patient drinking three to four beers each day refers to alcohol use, which is documented under the heading Personal and Psychosocial History.

The nurse is interviewing a woman with her husband present. The husband answers the questions for the wife most of the time. What is the most appropriate therapeutic nursing action to hear the patient's viewpoint? a. Continue the interview. b. Ask the husband to step out of the room. c. Ask another nurse to complete the interview. d. Tell the woman to speak up for herself.

b. Ask the husband to step out of the room A Continuing the interview is not a therapeutic action because the nurse needs to obtain the patient's answers to the questions. B Asking the husband to step out of the room will allow the patient to answer questions in her own way. C Asking another nurse to complete the interview does not solve the problem that the husband is answering questions for his wife. D Telling the woman to speak up for herself does not solve the problem and may interfere with the therapeutic relationship between the patient and the nurse.

How does the nurse test the function of the patient's spinal accessory nerve (CN XI)? a. Ask the patient to stick out the tongue and move it side to side. b. Ask the patient to shrug the shoulders against the resistance of the nurse's hands. c. Ask the patient to open the mouth and observe the uvula rise when he says "ah." d. Ask the patient to move the chin to the chest and then up toward the ceiling.

b. Ask the patient to shrug the shoulders against the resistance of the nurse's hands. A This is a test of the hypoglossal cranial nerve (XII). B This is the correct technique for assessing the spinal accessory cranial nerve (XI). C This is a test for cranial nerves IX (glossopharyngeal) and X (vagus). D This technique assesses the range motion of the neck.

After seeing the red reflex and retinal vessels through the ophthalmoscope, how does the nurse locate the optic disc? a. By rotating the diopter to the block (positive) numbers until the optic disc comes into focus b. By following the retinal vessels inward toward the nose until optic disc is seen c. By using the green beam light while looking outward toward the ear until the disc is seen d. By locating the macula and then looking temporally (toward the ear) until the disc is seen

b. By following the retinal vessels inward toward the nose until optic disc is seen A This procedure is used for patients who are myopic. B This procedure locates the optic disc. C The green beam is used to identify retinal hemorrhages. D The macula lies temporal to the optic disc; thus the optic disc is in the opposite direction.

A nurse is assessing a patient who was hit at the base of the skull with a blunt instrument causing a skull fracture. What assessment finding does this nurse anticipate during the inspection? a. Tinnitus, vertigo, and dizziness b. Clear drainage from the ear and nose c. Loss of hearing and smell d. Purulent drainage from the ear and bloody drainage from the nose

b. Clear drainage from the ear and nose A These are subjective and gathered during the history rather than inspection. Although the patient may report having dizziness or vertigo, the finding of tinnitus is inconsistent with a basilar skull fracture. B This may occur after a basilar skull fracture. The clear drainage may be cerebrospinal fluid. C This is inconsistent with a basilar skull fracture. D Purulent drainage is inconsistent with a basilar skull fracture, and bloody drainage usually does not come from the nose, but may be seen from the ear.

During an eye assessment, the nurse asks the patient to keep the head stationary and by moving the eyes only follow the nurse's finger as it moves side to side, up and down, and obliquely. This assessment technique collects what data about which cranial nerves? Select all that apply. a. Cranial nerve II (optic) b. Cranial nerve III (oculomotor) c. Cranial nerve IV (trochlear) d. Cranial nerve VI (abducens) e. Cranial nerve V (trigeminal)

b. Cranial nerve III (oculomotor) c. Cranial nerve IV (trochlear) d. Cranial nerve VI (abducens) Correct: Cranial nerve III (oculomotor), cranial nerve IV (trochlear), and cranial nerve VI (abducens) provide muscle movement for the eyes. Incorrect: Cranial nerve II (optic) provides vision. Cranial nerve V (trigeminal) provides movement for the jaw and sensation for cornea, conjunctive, eyelids, teeth, tongue, and mouth.

An elderly patient was admitted with pneumonia and a fever of 104.5° F. At the time of admission he was confused, disoriented, restless, and tried to slap the nurse who started an intravenous line. His daughter stated, "Just yesterday he was perfectly fine, except for a cold. I can't believe he is acting this way now." Within a few days, his erratic behavior subsided and his daughter was relieved that he was "back to normal." The nurse recognizes that this patient was exhibiting signs of which disorder? a. Dementia b. Delirium c. Panic attack d. Alcohol withdrawal

b. Delirium A Dementia is a syndrome of acquired, progressive, intellectual impairment that compromises function, such as memory, language, visual-spatial skills, emotion, personality, and cognition. B Delirium is characterized by a disturbance of consciousness and a change in cognition that develops rapidly over a short period of time. C A panic attack is characterized by complete disruption of the perceptual field. The person experiences intense terror and is unable to think logically or make decisions. The person experiences muscle tension, tachycardia, dyspnea, hypertension, increased respiration, and profuse perspiration. D Early manifestations of alcohol withdrawal include hand tremors, sweating, nausea and vomiting, anxiety, and agitation.

What technique does a nurse use when palpating the right lobe of a patient's thyroid gland using the anterior approach? a. Pushes the cricoid process to the left with the right thumb b. Displaces the trachea to the right with the left thumb c. Manipulates the thyroid between the thumb and index finger d. Moves the sternocleidomastoid muscle to the right with the left thumb

b. Displaces the trachea to the right with the left thumb A This is not a correct technique. B This is the correct technique for palpating the thyroid gland using the anterior approach. C This is not a correct technique. D This is not a correct technique.

The nurse is unable to hear the patient's breath sounds. What checks does the nurse make of the stethoscope to determine the cause of this problem? a. Ensure the stethoscope tubing is at least 20 inches long. b. Ensure the valve is open to the diaphragm on the head of the stethoscope. c. Ensure the earpieces are pointed toward the back of the ears. d. Ensure the bell is placed firmly against the patient's skin.

b. Ensure the valve is open to the diaphragm on the head of the stethoscope A Tubing should be no longer than 12 to 18 inches. If the tubing is longer than 18 inches, the sounds may become distorted. B The diaphragm is used to hear high-pitched sounds, such as breath sounds, bowel sounds, and normal heart sounds. Its structure screens out low-pitched sounds. C Earpieces are angled toward the nose so that sound is projected toward the tympanic membrane. D The bell of the stethoscope is used to hear soft, low-pitched sounds such as extra heart sounds or vascular sounds (bruit).

A male nurse is assigned to the care of a gay male with alcoholism. This sexual orientation is inconsistent with the beliefs of the nurse. What actions, if any, can the nurse take to provide patient-centered care to this patient? a. No action is necessary at this time. b. Examine his own feelings about alcoholism and homosexuality. c. Determine the patient's degree of risk for contracting the human immunodeficiency virus. d. Discuss homosexuality and alcoholism with the patient.

b. Examine his own feelings about alcoholism and homosexuality. A Not acting will not facilitate patient-centered care. The nurse needs to reflect on his feelings about this patient. B Examining one's own feelings about alcoholism and gay men specifically and identifying one's own beliefs and values generally is part of the process for developing cultural competence, which should allow one to recognize the uniqueness of patients and to demonstrate respect for individuals. C Determining the patient's degree of risk for contracting the human immunodeficiency virus is based on an assumption that because the patient is gay, he has or will have the human immunodeficiency virus. D A discussion with the patient may help the nurse understand him, but will not necessarily provide insight into the nurse's beliefs and feelings.

In contrasting the assessment of mental status from mental health, a nurse recognizes that data for the mental status examination are obtained using which techniques? a. Asking them about their relatives who have mental health disorders b. Having them demonstrate their ability to reason and calculate c. Asking them to recall how they have coped with daily stress d. Having them describe their mood and emotions

b. Having them demonstrate their ability to reason and calculate A This obtains information from patients for the histories, but does not ask patients to demonstrate mental abilities. B The mental status examination asks patients to perform calculations and other tasks to show their abilities, rather than asking them about their abilities. C This obtains information from patients for the histories, but does not ask patients to demonstrate mental abilities. D Having them describe their mood and emotions does not ask patients to demonstrate mental abilities.

A nurse is preparing to take a patient's blood pressure. The blood pressure cuff is 5 inches wide and the patient's upper arm circumference is 20 inches. How accurate will this patient's blood pressure be using this blood pressure cuff? a. Accurate, the actual value b. Higher than the actual value c. Lower than the actual value d. Unable to determine accuracy with available data

b. Higher than the actual value A For an arm circumference that is 20 inches, the proper size cuff is at least 8 inches (20 ´ 0.40 = 8). Therefore the blood pressure measurement will not be accurate. B For an arm circumference that is 20 inches, the proper size cuff is at least 8 inches (20 ´ 0.40 = 8). The cuff is 5 inches, which is too narrow. A cuff that is too narrow will overestimate the blood pressure and report a falsely high value. C For an arm circumference that is 20 inches, the proper size cuff is at least 8 inches (20 ´ 0.40 = 8). Therefore the blood pressure measurement will be higher than the actual value. D Sufficient data provided to determine accuracy. For an arm circumference that is 20 inches, the proper size cuff is at least 8 inches (20 ´ 0.40 = 8).

Which neurotransmitters are decreased in patients with depression? Select all that apply. a. Acetylcholine (Ach) b. Histamine c. Norepinephrine (NE) d. Dopamine (DA) e. Gamma aminobutyric acid (GABA)\ f. Serotonin (5 HT)

b. Histamine c. Norepinephrine (NE) d. Dopamine (DA) f. Serotonin (5 HT) Correct: Histamine, norepinephrine (NE), dopamine (DA), and serotonin (5 HT) are neurotransmitters that are decreased in depression. Drugs prescribed for people with depression may provide therapy by increasing these neurotransmitters. Incorrect: Acetylcholine (Ach) is increased in depression. Gamma aminobutyric acid (GABA) is decreased in schizophrenia and anxiety states.

A nurse is inspecting the skin of a patient who has had skin problems after multiple piercings. How will the nurse recognize the characteristics of keloids? a. Roughened and thickened scales involving flexor surfaces b. Hypertrophic scarring extending beyond the original wound edges c. Thin, fibrous tissue replacing normal skin following injury d. Loss of the epidermal layer, creating a hollowed-out or crusted area

b. Hypertrophic scarring extending beyond the original wound edges A Roughened and thickened scales involving flexor surfaces is a description of lichenification. B Hypertrophic scarring extending beyond the original wound edges is a description of a keloid. C Thin, fibrous tissue replacing normal skin following injury is a description of a scar. D Loss of the epidermal layer, creating a hollowed-out or crusted area is a description of excoriation.

A patient is admitted with edema of the occipital lobe following a head injury. The nurse correlates which finding with damage to this area? a. Ipsilateral ptosis b. Impaired vision c. Pupillary constriction d. Increased intraocular pressure

b. Impaired vision A Ipsilateral ptosis (drooping of the eye lid) is controlled by the oculomotor cranial nerve (CN III) that is located in the midbrain. The nurse must correlate anatomy with function and assessment. B The occipital lobe contains the visual context. C Pupillary constriction is controlled by the oculomotor cranial nerve (CN III) that is located in the midbrain. D This abnormality is associated with glaucoma rather than injury to the occipital lobe.

A patient complains of a lesion in his nose. Which technique does a nurse use to inspect the nasal mucosa? a. Inserts a nasal speculum horizontally into the patient's affected nares b. Inserts a nasal speculum obliquely into the patient's affected nares c. Uses a light source from the ophthalmoscope d. Inserts a nasal speculum vertically into the patient's affected nares

b. Inserts a nasal speculum obliquely into the patient's affected nares A Horizontal insertion puts pressure on the nasal septum, which is painful. B This is the appropriate technique for inspecting the nares. C The alternate light source is from an otoscope, rather than an ophthalmoscope. The otoscope has an ear speculum that can be used when a nasal speculum is unavailable. D Vertical insertion obstructs the nurse's view of the internal nares.

A patient expresses concern that a new lesion may be melanoma. Which finding suggests a malignant melanoma? a. Nonblanching lesion b. Irregular border c. Diameter less than 5 mm d. Black color of the lesion

b. Irregular border A Blanching is not assessed in malignant skin lesions. B Irregular border or poorly defined border is an indication of a malignancy. C Diameter of a malignant skin lesion is usually greater than 6 mm. D Melanoma is a variety of colors.

The nurse palpates the patient's jaw movement, placing two fingers in front of each ear and asking the patient to slowly open and close the mouth. What additional request does the nurse ask the patient to do to assess the jaw? a. Clinch the jaws together as tightly as possible. b. Move the lower jaw from side to side. c. Open the mouth as wide as possible, like a yawn. d. Move the lower jaw forward and backward several times.

b. Move the lower jaw from side to side. A This is not an assessment technique for the jaw. B This is the technique to complete assessment of the motion of the jaw. C This was completed when the nurse asked the patient to open and close the mouth. D This is not an assessment technique for the jaw.

During an eye examination of an Asian patient, a nurse notices an involuntary rhythmical, horizontal movement of the patient's eyes. How does a nurse document this finding? a. An expected racial variation b. Nystagmus c. Exophthalmus d. Myopia

b. Nystagmus A This is not a racial variation. B An involuntary rhythmical, horizontal movement of the patient's eyes is a description of nystagmus. C Exophthalmus is the bulging of the eyeball forward, seen in patients with hyperthyroidism. D Myopia is an elongated eyeball found in patients who are nearsighted.

A nurse calculates a patient's body mass index (BMI) as 33. This measurement indicates which class of weight? a. Overweight b. Obesity class I c. Obesity class II d. Obesity class III

b. Obesity class I A Overweight is a BMI of 25 to 29.9. B Obesity class I is a BMI of 30 to 34.9. C Obesity class II is a BMI of 35 to 39.9. D Obesity class III is a BMI greater than 40.

6. Which assessment technique is appropriate to measure the 8-month-old's vital signs during a well-baby check? a. Assess temperature using a rectal thermometer. b. Observe the infant's abdomen when counting respirations. c. Take the infant from the parent's arms to assess pulse. d. Measure blood pressure in the leg.

b. Observe the infant's abdomen when counting respirations. Feedback A Rectal temperatures should be taken as a last resort because children tend to fear intrusive procedures and because of the risk for rectal perforation. The recommended sites for temperature measurement in newborns, infants, and children to age 5 are the axillary or tympanic sites. B Infants usually breathe diaphragmatically, which requires observation of abdominal movement. C For the older infant ( 6 months) and toddler, the nurse may find that having the caregiver hold the baby or toddler decreases fear and distress, thus making it easier for the nurse to conduct the examination. D This infant is too young for blood pressure measurement. The National High Blood Pressure Education Program recommends that blood pressure be measured in children from age 3 through adolescence as part of routine health care visits.

Which method of temperature measurement indirectly reflects inner core temperature? Select all that apply. a. Axillary temperature b. Oral temperature c. Tympanic temperature d. Rectal temperature e. Temporal artery temperature

b. Oral temperature e. Temporal artery temperature Correct: Inner core temperature is measured indirectly because the probe is placed near an artery. For oral temperature, the probe is placed near the carotid artery and the temporal artery is used for the temporal artery temperature. Incorrect: For axillary, tympanic, and rectal temperatures, the probe is not placed close to any major blood vessels.

Which of the components described below represent the Filipino culture? Select all that apply. a. Tagalog and Cebuano are the primary dialects spoken. b. Orientation to the past is evident in their respect for elders. c. The family, rather than the individual, is the unit. d. Most Filipinos are Catholic. e. Filipinos like to eat rice with most meals. f. Sharing is common since interdependence is important.

b. Orientation to the past is evident in their respect for elders. c. The family, rather than the individual, is the unit. f. Sharing is common since interdependence is important. Correct: Respect for elders is an example of values and beliefs, a part of culture. Value of family represents beliefs and customs of culture. Interdependence is a value of Filipinos. Incorrect: Language, religion, and food preferences are part of ethnicity, rather than culture.

A patient has had chronic back pain for several years. On assessment, the nurse notes that the patient sits quietly in a chair, reads a book, talks with a companion, and does not appear to be in pain. When questioned, the patient rates the pain as a 6 on a scale of 0 to 10. How does the nurse interpret these data? a. Many patients cannot be believed when they complain of severe pain lasting many months. b. Patients may not have the same objective responses to chronic pain because of compensation over time. c. The patient probably has already taken a very effective pain medication. d. This patient is probably not having as much pain as reported initially, and more assessment is required.

b. Patients may not have the same objective responses to chronic pain because of compensation over time. A Pain is whatever the patient says it is. Patients with chronic pain adapt to the pain and have more subtle manifestations than patients with acute pain. B Clinical manifestations of chronic pain are not those of physiologic stress because the patient adapts to the pain. C Patients with chronic pain adapt to the pain and have more subtle manifestations than patients with acute pain despite the effects of pain medication. D Pain is whatever the patient says it is. Patients with chronic pain adapt to the pain and have more subtle manifestations than patients with acute pain.

1. What is the most important nursing action to reduce transmission of microorganisms during a physical assessment? a. Clean the bell and diaphragm of the stethoscope between patients. b. Perform hand hygiene. c. Wear gloves when anticipating exposure to body fluids. d. Wear eye protection when anticipating spatter of body fluids.

b. Perform hand hygiene A Cleaning the bell and diaphragm of the stethoscope between patients is important to prevent the spread of microorganisms when auscultating only. B Consensus recommendations of the World Health Organization include use of hand hygiene techniques to prevent spread of microorganisms before palpating, percussing, or auscultating patients, and during patient care. C Wearing gloves when anticipating exposure to body fluids is important to prevent the spread of microorganisms from the patient while giving care. D Wearing eye protection when anticipating spatter of body fluids is important to prevent the spread of microorganisms from the patient while giving care.

A nurse is assessing joint function of a patient with severe rheumatoid arthritis. Which instrument/tool does the nurse use to measure the degree of flexion and extension of the patient's knee joints? a. Calipers b. Ruler or tape measure c. Goniometer d. Doppler

c. Goniometer A Calipers are used to estimate the amount of body fat. B A ruler or tape measure cannot accurately measure the degree of flexion and extension of joints. C A goniometer is used to measure the degree of flexion and extension of a joint. D Doppler is used to detect the presence of pulses.

A nurse shines a light toward the bridge of the patient's nose and notices that the light reflection in the right cornea is at the 2 o'clock position and in the left cornea at the 10 o'clock position. Based on these data, the nurse should take what action? a. Document these findings as normal. b. Perform the cover-uncover test. c. Perform the confrontation test. d. Document these findings as abnormal.

b. Perform the cover-uncover test. A The findings are abnormal. The light should appear in the same location in each cornea. B The nurse is performing the corneal light reflex test and the findings are abnormal. Thus, when the corneal light reflex is asymmetric, the cover-uncover test is performed to determine which eye has the weak extraocular muscle(s). C The confrontation test is used to assess peripheral visual fields and is not appropriate to perform when the corneal light reflex is asymmetric. D The asymmetric corneal light reflex is abnormal, but the cover-uncover test should follow the abnormal finding to determine which eye has the weak extraocular muscle(s).

How does the nurse perform a Weber test to assess hearing function? a. Whispers three to four words into the patient's ear and asks him to repeat the words heard b. Places a vibrating tuning fork in the middle of the head and asks the patient if the sound is heard the same in both ears c. Places a set of headphones over both ears, plays several tones, and asks the patient to identify the sounds d. Places a vibrating tuning fork on the mastoid process and asks the patient to signal when he can no longer hear the sound

b. Places a vibrating tuning fork in the middle of the head and asks the patient if the sound is heard the same in both ears A This technique describes the whisper test. B This technique describes the Weber test. C This technique describes the use of an audiometer. D This technique describes part of the Rinne test.

To assess jaw movement of an adult patient, the nurse uses which technique? a. Asking the patient to open the mouth and then passively moving the patient's open jaw from side to side b. Placing two fingers in front of each ear and asking the patient to slowly open and close the mouth c. Asking the patient to open the mouth and to resist the nurse's attempt to close the mouth d. Using the pads of all fingers to feel along the mandible for tenderness and nodules

b. Placing two fingers in front of each ear and asking the patient to slowly open and close the mouth A The patient's jaw movement should be active, not passive. B This is the correct technique for palpating the jaw. C This technique assesses strength of the jaw, which is not typically evaluated. D Palpating under the middle of the mandible may reveal the submental lymph node.

A patient admitted with pneumonia reports that she takes insulin for diabetes mellitus. In which section of the history does the nurse document the insulin and diabetes? a. Past health history b. Present health status c. Reason for seeking care (chief complaint) d. History of present illness

b. Present health status A The past health history includes categories of childhood illness, surgeries, hospitalizations, accidents or injuries, immunizations, and obstetric history. B The present health status documents the current health conditions, which include chronic diseases and medications taken. In this case, diabetes and taking insulin are not the reason for seeking care, but need to be managed while the patient's pneumonia is being treated because they may affect the patient's recovery from pneumonia. C The reason for seeking care (chief complaint) is a brief statement of the patient's purpose for requesting the services of a health care provider. D History of present illness further investigates the history of the present problem; best accomplished by conducting a symptom analysis.

1. An adolescent patient appears reluctant to discuss sensitive issues with her parents present. What is the nurse's most appropriate intervention? a. Tell the patient that it is very important to be honest and specific. b. Provide time when the adolescent is alone with the nurse. c. Reassure the patient that anything said in the interview is considered confidential. d. Ask the parents to answer the questions if the patient is not willing to answer.

b. Provide time when the adolescent is alone with the nurse. A Although this statement is true, the adolescent should have time alone with the nurse, if needed, to answer or ask personal questions. B As children reach adolescence, they should be given the option to provide sensitive parts of the history without their parents present. C Although this statement is true, the adolescent should have time alone with the nurse, if needed, to answer or ask personal questions. D This intervention is not appropriate when the patient is present and able to answer questions. In addition, the parents may not know the information needed by the nurse about the adolescent.

A patient complains of nasal drainage and sinus headache. The nurse suspects a nasal infection and anticipates observing which finding during examination? a. Foul-smelling drainage b. Purulent green-yellow drainage c. Bloody drainage d. Watery drainage

b. Purulent green-yellow drainage A Foul-smelling drainage is consistent with a foreign object in the nose. B Purulent green-yellow drainage is consistent with a nasal or sinus infection. C Bloody drainage is consistent with trauma to the nose. D Watery drainage is consistent with a nasal allergy.

Which statement is correct regarding taking or interpreting axillary temperatures? a. Axillary temperatures should not be used in patients less than 2 years of age. b. Readings may be less accurate. c. The thermometer is left in place for no more than 3 minutes. d. The thermometer is placed in the axilla with the shoulder abducted.

b. Readings may be less accurate A The axilla is a common site for temperature measurement on infants and children. B Multiple studies have shown temperature measurements at the axillary site are less accurate compared with alternative sites. C The thermometer is left in place until the audible signal occurs and the temperature appears on the screen. D Place the probe in the middle of the axilla, with the arm held against the body (adducted).

During the history, a patient reports blurred vision, seeing double at times, and a glare from headlights from oncoming cars at night. Based on this information, what finding does the nurse expect to find on assessment of this patient's eyes? a. Anterior chamber depth is shallow. b. Red reflex is absent. c. Extraocular muscle movement is asymmetric. d. Retinal arteries are wider than retinal veins.

b. Red reflex is absent. A Shallow anterior chamber depth occurs in glaucoma. B The symptoms suggest cataracts. The red reflex cannot be seen because the light cannot penetrate the opacity of the lens. C Extraocular muscle movement is asymmetric. Cataracts affect the lens rather than the eye muscles. D Retinal arteries are wider than retinal veins. Cataracts affect the lens rather than the retinal vessels.

A patient has had an infected facial wound for more than 3 months. How does the nurse expect the patient's enlarged lymph nodes to feel? a. Soft, edematous, and tender b. Round, tender, and movable c. Hard, nontender, and nonmobile d. Irregularly shaped, tender, and firm

b. Round, tender, and movable A These are not characteristics of lymph nodes associated with inflammation. B These are characteristics of enlarged lymph nodes associated with inflammation. C These are characteristics of enlarged lymph nodes associated with a malignancy. D These are not characteristics of lymph nodes associated with inflammation.

What findings does the nurse expect when assessing skin, hair, and nails of a healthy male adult? Select all that apply. a. Transverse depression noticed across nails b. Scalp is bald c. Elevated, firm, circumscribed area less than 1 cm wide found on the fingers d. Purpura and ecchymosis are noticed on arms and legs e. Freckles are noted on face, back, arms, and legs f. Skin turgor is elastic

b. Scalp is bald e. Freckles are noted on face, back, arms, and legs f. Skin turgor is elastic Correct:Scalp is bald; freckles are noted on face, back, arms and legs; and skin turgor is elastic . These are expected findings for a healthy adult male. Incorrect: Transverse depression across the nails describes Beau lines. It results from a stressor that temporarily impairs nail formation. An elevated, firm, circumscribed area, less than 1 cm wide on the fingers describes a papule, such as a wart. Purpura and ecchymosis on arms and legs are indications of bleeding.

A community organization sponsors a health fair to increase awareness of colon cancer. At the health fair, colorectal cancer screening kits are distributed, and health care professionals answer questions, take blood pressure, and distribute literature. These activities are examples of _____ prevention. a. Primary b. Secondary c. Tertiary d. Risk factor

b. Secondary A Primary prevention is focused on preventing disease from developing through the promotion of a healthy lifestyle. B Secondary prevention consists of screening efforts to promote early detection of disease—in this scenario, colorectal cancer and hypertension. C Tertiary prevention is directed toward minimizing the disability from chronic disease and helping the patient maximize his or her health. D Risk factor prevention is part of primary prevention that focuses on preventing disease by managing risk factors.

While assessing a patient's ability to consume food, the nurse recalls which types of foods are the easiest to chew and swallow? a. Thin liquids b. Soft foods c. Dry foods d. Chewy foods

b. Soft foods A Thin liquids and foods requiring forceful chewing (such as meat) may not be tolerated well. B Foods that are soft and highly viscous are most easily chewed and swallowed. C Foods that are soft and highly viscous are most easily chewed and swallowed. D Thin liquids and foods requiring forceful chewing (such as meat) may not be tolerated well.

A patient with gout is complaining of severe, throbbing pain in the great toe. What type of pain is this patient experiencing? a. Neuropathic pain b. Somatic pain c. Referred pain d. Visceral pain

b. Somatic pain A Neuropathic pain is caused by abnormal processing of sensory input from the peripheral nervous system. B Somatic pain arises from bone, joint, muscle, skin, or connective tissues and is usually aching or throbbing in quality and well located. C Referred pain is pain felt at a site different from that of an injured or diseased organ. D Visceral pain occurs with obstruction of a hollow organ and causes intermittent cramping pain.

A nurse is performing an admission physical examination on a patient who has been bedridden for a month. The nurse notices a pressure ulcer on the patient's left trochanter area that involves partial-thickness skin loss with damage to the subcutaneous tissue. The nurse reports this ulcer at what stage? a. Stage I b. Stage II c. Stage III d. Stage IV

b. Stage II A Stage I ulcers have persistent redness, but the epidermis is intact. B Stage II ulcers have partial-thickness skin loss of dermis. It appears as a shiny or dry shallow open ulcer with pink wound bed without slough or bruising. C Stage III ulcers have full-thickness skin loss involving damage to or necrosis of subcutaneous tissue that may extend to, but not through, underlying fascia. D Stage IV ulcers have full-thickness tissue loss with exposed bone, tendon, or muscle. Slough or eschar may be within the wound bed.

A nurse observes a student using the whisper test to screen a patient with hearing loss. Which behavior by the student requires a corrective comment from the nurse? a. Instructing the patient to cover the ear not being tested b. Standing beside the patient on the side of the ear being tested c. Shielding the mouth to prevent the patient from reading lips d. Whispering one or two syllable words and ask the patient to repeat what is heard

b. Standing beside the patient on the side of the ear being tested A This is the correct technique. B The student nurse should stand 1 to 2 feet in front or to the side of the patient. C This is the correct technique. D This is the correct technique.

What are the characteristics of one's culture? a. Color of skin and hair b. System of beliefs and practices c. Food preferences d. Language and religion

b. System of beliefs and practices A Skin and hair color are examples of racial characteristics based on genetics. B System of beliefs and practices is part of the definition of culture. C Food preferences are an example of ethnicity. D Language and religion are examples of ethnicity.

During the Rinne test, a nurse determines that the patient hears the tuning fork held on the mastoid process for 15 seconds and hears the tuning fork held in front of the ear for 30 seconds. The same results are found in both ears. Based on this finding, what is the most appropriate response of the nurse? a. Repeat the test again using a 2000 Hz tuning fork. b. Tell the patient that this represents an expected finding. c. Refer the patient for additional testing to detect hearing abnormality. d. Perform a Weber test to confirm the findings of the Rinne test.

b. Tell the patient that this represents an expected finding A This is unnecessary because the finding of the Rinne test was normal. B This is a normal finding. Air conduction (30 seconds) is twice as long as bone conduction (15 seconds). C This is unnecessary because the finding of the Rinne test was normal. D This is unnecessary because the finding of the Rinne test was normal.

What assessment data do nurses obtain through striking a hand directly against the flank or costovertebral angle of a patient's body? a. Fluid in the lungs b. Tenderness over the kidneys c. Air in the abdomen d. Tenderness over the liver

b. Tenderness over the kidneys A Fluid in the lungs is detected by indirect percussion. B Tenderness over the kidneys is detected by direct percussion over the costovertebral angle. C Air in the abdomen is detected by indirect percussion. D Tenderness over the liver is detected by palpation.

What part of the stethoscope do nurses use to auscultate the chest? a. Press the bell firmly against the skin to hear sounds and vibrations. b. The bell of the stethoscope is used to hear breath sounds. c. The diaphragm of the stethoscope is used to hear heart sounds. d. Either the bell or the diaphragm is used to auscultate the chest.

b. The bell of the stethoscope is used to hear breath sounds A The bell should be pressed lightly on the skin with just enough pressure to ensure that a complete seal exists around the bell. If the bell is pressed too firmly on the skin, the concave surface is filled with skin, and the bell functions like a diaphragm and inhibits vibrations. B The bell is used to hear soft, low-pitched sounds such as extra heart sounds or vascular sounds (bruit). C The diaphragm is used to hear breath sounds, bowel sounds, and normal heart sounds (high-pitched sounds). D Either the bell or the diaphragm is used to auscultate the chest. The diaphragm is used to hear breath sounds, bowel sounds, and normal heart sounds (high-pitched sounds).

During an examination of the head and neck of a healthy adult, the nurse expects which findings? Select all that apply. a. Small red lesions with white flakes scattered on the scalp b. The head and facial bones are proportional for the size of the body c. Depressions palpated on the right and left sides over the parietal bones d. Head held flexed 15 degrees to the left e. Face and jaw are symmetric and proportional f. Temporomandibular joint moves smoothly

b. The head and facial bones are proportional for the size of the body e. Face and jaw are symmetric and proportional f. Temporomandibular joint moves smoothly Correct: These are expected findings from an assessment of the head of a healthy adult. Incorrect: Small red lesions with white flakes scattered on the scalp is an abnormal finding. The scalp should be intact without lesion or flakes. Depressions palpated on the right and left sides over the parietal bones is an abnormal finding. Perhaps this patient had skull tongs from cervical traction at one time. Head held flexed 15 degrees to the left is an abnormal finding. The head should be erect.

Where does a nurse palpate to assess the posterior tibial pulse? a. Behind the knee in the popliteal fossa b. The inner aspect of the ankle below and slightly behind the medial malleolus c. Over the dorsum of the foot between the extension tendons of the first and second toes d. The outer side of the ankle below and slightly behind the lateral malleolus

b. The inner aspect of the ankle below and slightly behind the medial malleolus

Which patient in the eye clinic should the nurse assess first? a. The patient who reports a gradual clouding of vision b. The patient who complains of sudden loss of vision c. The patient who complains of double vision d. The patient who complains of poor night vision

b. The patient who complains of sudden loss of vision A A gradual clouding of vision is a symptom of cataracts that develop slowly and do not require immediate assessment. B Sudden vision loss may indicate a detached retina and requires immediate referral. C Double vision is a symptom of cataracts that develop slowly and do not require immediate assessment. D Poor night vision is a symptom of cataracts that develop slowly and do not require immediate assessment.

Which patient has pain caused by abnormal processing of sensory input from the peripheral nervous system? a. The patient who has aching pain from muscle strain b. The patient who has burning pain along the sciatic nerve c. The patient who has cramping pain from a tumor in the colon d. The patient who has throbbing pain from arthritis

b. The patient who has burning pain along the sciatic nerve A The patient who has aching pain from muscle strain has nociceptor, somatic pain. B The patient who has burning pain along the sciatic nerve has neuropathic pain. C The patient who has cramping pain from a tumor in the colon has nociceptor, visceral pain. D The patient who has throbbing pain from arthritis has nociceptor, somatic pain

For which person is a screening assessment indicated? a. The person who had abdominal surgery yesterday b. The person who is unaware of his high serum glucose levels c. The person who is being admitted to a long-term care facility d. The person who is beginning rehabilitation after a knee replacement

b. The person who is unaware of his high serum glucose levels A A shift assessment is most appropriate for the person who is recovering in the hospital from surgery. B A screening assessment is performed for the purpose of disease detection. In this case this person may have diabetes mellitus. C A comprehensive assessment is performed during admission to a facility to obtain a detailed history and complete physical examination. D An episodic or follow-up assessment is performed after knee replacement to evaluate the outcome of the procedure.

How does the nurse detect a pulse when using a Doppler? a. The pulsation is felt. b. The pulsation is heard. c. The pulse wave is seen on a screen. d. The pulse wave is printed out on special paper.

b. The pulsation is heard A A Doppler is used when the pulses cannot be palpated. B A Doppler amplifies sounds difficult to hear with an acoustic stethoscope. C A Doppler amplifies the sound of the pulsation. D A Doppler amplifies the sound of the pulsation

A nurse screens every adult and adolescent patient for alcohol consumption. Which patient drinks more than recommended? a. The man who reports drinking 3 beers and one shot of whiskey each day b. The woman who reports drinking 2 glasses of wine and 2 vodka martinis each day c. The older adult man who reports drinking one glass of sherry before going to bed each night d. The woman who reports drinking one glass of wine with lunch and dinner each day.

b. The woman who reports drinking 2 glasses of wine and 2 vodka martinis each day A The man who reports drinking 3 beers and one shot of whiskey each day. This amount of alcohol is within the National Institute on Alcohol Abuse and Alcoholism recommendations for men to drink fewer than 5 standard drinks daily. B The woman who reports drinking 2 glasses of wine and 2 vodka martinis each day. The National Institute on Alcohol Abuse and Alcoholism recommends women drink fewer than 4 standard drinks daily. C The older adult man who reports drinking one glass of sherry before going to bed each night. This amount of alcohol is within the National Institute on Alcohol Abuse and Alcoholism recommendations for men to drink fewer than 5 standard drinks daily. D The woman who reports drinking one glass of wine with lunch and dinner each day. This amount of alcohol is within the National Institute on Alcohol Abuse and Alcoholism recommendations for women to drink fewer than 4 standard drinks daily.

To test deep tendon reflexes, the nurse uses which instrument? a. Goniometer b. Calipers c. Reflex hammer d. Monofilament

c. Reflex hammer A A monofilament is used to test for sensation on the lower extremities. B Calipers are used to measure thickness of subcutaneous tissue to estimate the amount of body fat. C A reflex hammer is used to test deep tendon reflexes. D A monofilament is used to test for sensation on the lower extremities.

A nurse notices that the angle of the patient's proximal nail fold and the nail plate are almost a flat line; about 160 degrees. How does the nurse interpret this finding? a. This patient has chronic pulmonary disease. b. This is an expected finding. c. This is due to stress to the nails. d. This is associated with anemia.

b. This is an expected finding A This patient has chronic pulmonary disease, which causes clubbing (when the angle of the nail base exceeds 180 degrees). B The expected angle of the nail base is 160 degrees. C This answer describes Beau lines, which appear as a groove or transverse depression running across the nail. It results from a stressor that temporarily impairs nail formation. D This is associated with anemia, which causes koilonychia, a thin, depressed nail with the lateral edges turned upward.

When does a nurse use a Pederson or Graves speculum for examination of a patient? a. To inspect the external ear b. To assess the vaginal canal c. To inspect nasal passages d. To assess the oropharynx

b. To assess the vaginal canal A The external ear is inspected using an otoscope. B The vaginal canal and cervix are inspected using a Pederson or Graves speculum or a pediatric or virginal speculum. C The nasal passages are inspected using a nasal speculum. D The oropharynx is inspected using a tongue blade and penlight.

A patient is complaining of pain over the maxillary sinuses. Which device does the nurse use to determine if there is air or fluid in the patient's sinuses? a. Magnification device b. Transilluminator c. Monofilament d. Wood lamp

b. Transilluminator A A magnification device helps visualize the tissue, but will not determine if sinuses are filled with air or fluid. B A transilluminator disseminates its light source under the surface of the skin to determine if the areas under the surface, such as the sinuses, are filled with air, fluid, or tissue. C A monofilament is used to test for sensation on the lower extremities. D A Wood lamp is used to detect fungal infections.

A nurse is preparing to assess a patient's ability to detect vibrations. Which piece of equipment is appropriate for this assessment? a. Reflex hammer b. Tuning fork c. Goniometer d. Monofilament

b. Tuning fork A A reflex hammer is used to test for deep tendon reflexes. B The tuning fork is used to assess the patient's ability to detect vibration. C A goniometer is used to measure the degrees of flexion and extension of a joint. D A monofilament is used to test for sensation on the lower extremities.

A teenager comes to the clinic complaining about the whiteheads and blackhead on his face interfering with his social life. During the examination the nurse palpates an enlarged submental lymph node. Where is this lymph node located? a. In front of the ear b. Under the mandible c. At the base of the skull d. Along the angle of the jaw

b. Under the mandible A This is the location of the preauricular lymph nodes. B This is the location of the submental lymph node. C This is the location of the occipital lymph nodes. D This is the location of the parotid lymph nodes.

In assessing a patient with head injury, the nurse should be most concerned with which finding? a. Pain on palpation of the scalp b. Unilateral clear, watery nasal discharge c. A scalp laceration at the sight of injury d. Complaints of dizziness

b. Unilateral clear, watery nasal discharge A This is expected after a head injury and is not a cause for concern. B This may be cerebrospinal fluid, indicating a skull fracture. C This is expected after a head injury and is not a cause for concern. D This is expected after a head injury and is not a cause for concern.

A male patient is very talkative and shares much information that is not relevant to his history or the reason for his admission. Which action by the nurse improves data collection in this situation? a. Terminate the interview. b. Use closed-ended questions. c. Ask the patient to stay on the subject. d. Ask another nurse to complete the interview

b. Use closed-ended questions. A Terminating the interview is not beneficial to the patient and does not allow data collection. B Using closed-ended questions is useful to obtain specific data when open-ended questions are not obtaining the needed data. C Asking the patient to stay on the subject is not therapeutic and may result in less data collection. D Asking another nurse to complete the interview may not be practical and interrupts the nurse-patient relationship that has been established.

Which technique used by the nurse encourages a patient to continue talking during an interview? a. Laughing and smiling during conversation b. Using phrases such as "Go on," and "Then?" c. Repeating what the patient said, but using different words d. Asking the patient to clarify a point

b. Using phrases such as "Go on," and "Then?" A Laughing and smiling during conversation may show attentiveness during the interview, but does not encourage more talking. B Using phrases such as "Go on" and "Then?" encourages the patient to continue talking. C Rephrasing what the patient has said is restatement. It confirms your interpretation of what they said, but does not encourage additional talking. D Asking the patient to clarify a point is done when the information is conflicting, vague, or ambiguous.

A patient who keeps his fat consumption at 10% of his total caloric intake is at risk for deficiency of which nutrient(s)? a. Iron b. Vitamins A, D, and K c. Zinc d. B and C vitamins

b. Vitamins A, D, and K A Iron absorption is not affected by low fat intake. B Vitamins A, D, and K are fat soluble vitamins. If the patient does not have enough fat intake, adequate amounts of these vitamins cannot be absorbed. Fat soluble vitamins should be linked to a lack of body fat. C Zinc absorption is not affected by low fat intake. D Vitamins B and C are water soluble vitamins and their absorption is not affected by low fat intake.

When examining a patient, the nurse remembers to follow which principle of Standard Precautions? a. Wear gloves throughout the entire examination of the patient. b. Wear gloves when in contact with the patient's mucous membranes. c. Wear gloves to reduce the need for handwashing. d. Wear eye protection and a gown during the examination of the patient.

b. Wear gloves when in contact with the patient's mucous membranes. A Wearing gloves throughout the examination of the patient is unnecessary. Referring to the Standard Precautions for the correct answer; nurses use judgment to determine when contact with body fluids is possible. B Specifically, this applies to contact with blood, body fluids (e.g., urine, feces, sputum, wound drainage), nonintact skin, and mucous membranes. C Hands must be washed after removal of gloves. D The nurse should wear a mask with eye protection or a face shield during procedures that may result in splashes or sprays of the patient's blood, body fluids, secretions, or excretions.

During a health fair, which recommendation is appropriate as a primary prevention measure to reduce the risk for skin cancer? a. Use a tanning booth instead of sunning outside if a tan is desired. b. Wear protective clothing while in the sun. c. Perform self-examination of skin monthly. d. Use sunscreen with a sun protection except on overcast days.

b. Wear protective clothing while in the sun. A Avoiding tanning and sunning are part of primary prevention. B Wearing protective clothing while in the sun provides primary prevention for skin cancer. C Performing self-examination of skin monthly is secondary prevention. D Sunscreen also needs to be used on overcast days.

In which situation is the nurse's use of closed-ended questions most appropriate? a. When clarifying vague or conflicting data b. When obtaining a history from an overly-talkative patient c. When encouraging a patient to elaborate on details of his or her history d. When collecting data about the current health problem

b. When obtaining a history from an overly-talkative patient A When clarifying vague and conflicting data, the nurse needs to use open-ended questions to obtain data. B When obtaining a history from an overly-talkative patient, a nurse can resort to closed-ended questions to complete the data collection in a timely manner. C When encouraging the patient to elaborate on details of his or her history, the nurse needs to use open-ended questions to obtain the details. D When collecting data about the current problem, the patient needs to describe the symptoms that brought him or her to seek help. These details are not collected with closed-ended questions.

A nurse assessing a patient with liver disease expects to find which manifestation during the examination? a. Yellowish color in the axilla and groin b. Yellow pigmentation in the sclera c. Very pale skin on the palms d. Ashen-gray color in the oral mucous membranes

b. Yellow pigmentation in the sclera A Instead of the axilla and groin, assess the sclera of the eyes, fingernails, palms of hands, and oral mucosa. B Jaundice is manifested by a yellowish color in the sclera of the eyes and palms of the hands in both light- and dark-skinned patients. C Pale skin may indicate anemia, but not jaundice. Yellow color of the palms indicates jaundice. D Ashen-gray color may be seen in dark-skinned patients who are cyanotic.

A nurse is asking questions about the present health status of a young woman who has lost weight recently. Which question is most appropriate when inquiring about present health status? a. "What concerns have you had in the past regarding your weight?" b. "Do you have anorexia?" c. "Describe the recent changes in your weight." d. "Do you have a family history of eating disorders?"

c. "Describe the recent changes in your weight." A This question implies the patient has or should have concerns about weight loss and is not focused on the present health status. B This is not an appropriate question. More data are needed about this patient before anorexia can be suspected or determined. C Asking the patient to describe recent changes in weight is an appropriate, open-ended question to begin data collection so that the patient can report her current health status related to weight. D This question is asked in the history rather than in the present health status part of the health history.

a. "How frequently do you eat fast food or junk food?" b. "Which carbonated drinks do you drink most often?" c. "Do you have any food restrictions or diet routines?" d. "What are your favorite fruits and vegetables?

c. "Do you have any food restrictions or diet routines?" Feedback A Asking the frequency of fast food or junk food consumption does not give data about what food is eaten. B Knowing the amount of carbonated drinks provides more useful data. C Adolescents should be asked specifically about their perception of their current weight and behaviors associated with eating disorders, including food restrictions, extreme diet/exercise routines, binging or purging, and the use of laxatives to screen for eating disorders. D Knowing how frequently these foods are eaten provides more useful data.

What instructions does the nurse give the patient before palpating the right supraclavicular lymph nodes? a. "Lean your head backward and toward the right as far as comfortably possible." b. "Lie supine and turn your head away from the right side." c. "Draw up your shoulders forward, and flex your chin toward the right side." d. "Sit up, raise both arms over your head, and flex your chin away from the right side."

c. "Draw up your shoulders forward, and flex your chin toward the right side." A This is incorrect. The patient should draw up (hunch) the shoulders forward rather than leaning back. B The patient should be sitting, rather than lying down. C This is the technique for palpating the supraclavicular nodes. D The shoulders should be drawn up (hunched) forward, rather than raising the arms.

A nurse is admitting a new patient. Which statement by the patient suggests a bipolar disorder? a. "The last time I had blood drawn at the office, I fainted dead away." b. "No matter how hard I try, I just can't get into an elevator of any kind." c. "Everyone knows I can control the financial health of this town with a snap of my fingers." d. "I worked for Frank Sinatra's band for several months when I lived in New Jersey years ago."

c. "Everyone knows I can control the financial health of this town with a snap of my fingers." A This statement does not indicate bipolar disorder and may be a true statement. B This statement is an example of a phobia. C Patients in the manic phase of bipolar disorder have delusions of grandeur, which is described in the statement. D This may be a true statement depending on the age of the patient.

What instructions does the nurse give the patient before using the Snellen visual acuity chart? a. "Remove your eyeglasses before attempting to read the lowest line." b. "Stand 10 feet from the chart and read the first line aloud." c. "Hold a white card over one eye and read the smallest possible line." d. "Squint if necessary to improve the ability to read the largest letters."

c. "Hold a white card over one eye and read the smallest possible line." A Patients should wear their glasses when visual acuity is tested. B The patient should stand 20 feet from the Snellen chart. C This is the appropriate technique for using the Snellen chart. D The patient should not squint to see the chart.

The nurse is taking a health history on a patient who reports frequent headaches with pain in the front of the head, but sometimes felt in the back of the head. Which statement by the patient is most indicative of tension headaches? a. "I usually have nausea and vomiting with my headaches." b. "My whole head is constantly throbbing." c. "It feels like my head is in a vice." d. "The pain is on the left side over my eye, forehead, and cheek."

c. "It feels like my head is in a vice." A This is descriptive of migraines rather than tension headaches. B This is descriptive of migraines rather than tension headaches. C This is descriptive of tension headaches, which is consistent with the rest of the data reported by the patient. D This is consistent with cluster headaches rather than tension headaches.

During symptom analysis, the nurse helps the patient distinguish between dizziness and vertigo. Which description by the patient indicates vertigo? a. "I felt faint, like I was going to pass out." b. "I just could not keep my balance when I sat up." c. "It seemed that the room was spinning around." d. "I was afraid that I was going to lose consciousness."

c. "It seemed that the room was spinning around." A This is a description of lightheadedness, a form of dizziness. B This is a description of disequilibrium, a form of dizziness. C This is consistent with vertigo because it includes a sensation of motion. D This is a description of syncope, a form of dizziness.

A patient tells the nurse that she tries to keep her fat intake at less than 15% of her total caloric intake per day. What is the nurse's most appropriate response to this patient's comment? a. "That is admirable; how do you accomplish fat intake that low on a daily basis?" b. "Eating fat is essential for good health, and you should consume about 40% of your fats as monounsaturated fat." c. "Limiting fat prevents some diseases, but your fat intake is much lower than the 25% recommended." d. "If you want to bring your fat intake down further, you might want to eliminate eating fast foods."

c. "Limiting fat prevents some diseases, but your fat intake is much lower than the 25% recommended." A The patient's fat intake is too low. The recommended amount of fat is 25% to 30% of total calories. B The recommended amount of fat is 25% to 30% of total calories. C The recommended amount of fat is 25% to 30% of total calories. D The patient's fat intake is already too low. The recommended amount of fat is 25% to 30% of total calories.

A patient who had an amputation of his lower leg comes to the clinic with a complaint of pain. He asks, "How I can be feeling pain in my foot—my foot is gone!" What is the appropriate response from the nurse? a. "After your amputation, pain perception increases." b. "Amputating your leg caused abnormal processing of sensory input by the peripheral nervous system." c. "Stimulation of nerves from your leg sends impulses to the brain so that you feel pain even though your leg is no longer there." d. "When sensory nerves enter the spinal cord, they stimulate nerves from unaffected organs in the same spinal cord segment as those neurons in areas where injury or disease is located."

c. "Stimulation of nerves from your leg sends impulses to the brain so that you feel pain even though your leg is no longer there." A "After your amputation, pain perception increases" is a definition of pain threshold. B "Amputating your leg caused abnormal processing of sensory input by the peripheral nervous system" is a definition of neuropathic pain. C "Stimulation of nerves from your leg sends impulses to the brain so that you feel pain even though your leg is no longer there" is a definition of phantom pain. D "When sensory nerves enter the spinal cord, they stimulate nerves from unaffected organs in the same spinal cord segment as those neurons in areas where injury or disease is located" is a definition of referred pain.

Which statement by the nurse demonstrates a patient-centered interview? a. "I need to complete this questionnaire about your medical and family history." b. "The hospital requires me to complete this assessment as soon as possible." c. "Tell me about the symptoms you've been having." d. "I've had the same symptoms that you've described."

c. "Tell me about the symptoms you've been having." A "I need to complete this questionnaire about your medical and family history" focuses on the nurse's need to complete the assessment rather than the needs of the patient. B "The hospital requires me to complete this assessment as soon as possible" focuses on the nurse's need to meet hospital requirements rather than the needs of the patient. C "Tell me about the symptoms you've been having" focuses on the needs of the patient so that the patient is free to share concerns, beliefs, and values in his or her own words. D "I've had the same symptoms that you've described" focuses on the nurse rather than on the patient.

A nurse is conducting an assessment of an American Indian woman who has come to the clinic complaining of persistent headaches. The patient tells the nurse that the medicines prescribed by the tribal healer have done "some good." What is the appropriate response of the nurse at this time? a. "I advise you to stop taking those medicines from the tribal healer." b. "Perhaps you should increase the frequency of the healer's medicines." c. "Tell me about these medicines and how often you are using them." d. "Could your headaches be caused by the healer's medicines?"

c. "Tell me about these medicines and how often you are using them." A Advising the patient to stop taking any nonprescription medicines is inappropriate until the nurse knows the details about all medicines used by the patient. B Telling the patient to increase the frequency of the healer's medicines is inappropriate until the nurse knows details about the medicines. C Asking the patient about the nature of these medicines and how often the patient uses them allows the nurse to collect data about the medicines and their uses, to learn more about the practices used by this patient to improve her health, and to check for potential drug interaction before prescribing other medications or treatment. D Suggesting the patient's headaches are caused by the healer's medicines is inappropriate until the nurse knows details about the medicines.

A deep tendon reflex with a normal response is scored as: A. 0 B. 1+ C. 2+ D. 3+

c. 2+ 2+ is considered an active or expected response for deep tendon reflex; this is a normal response. 0 indicates no response for deep tendon reflex; this is an abnormal response. 1+ indicates a sluggish or diminished response for deep tendon reflex; this is an abnormal response. 3+ is a brisker-than-expected or slightly hyperactive response; this is an abnormal response.

How does a nurse assess movements of the eyes? a. By assessing peripheral vision b. By noting the symmetry of the corneal light reflex c. By assessing the cardinal fields of gaze d. By performing the cover-uncover test

c. By assessing the cardinal fields of gaze A This tests the function of cranial nerve I (optic). B This indicates symmetry of eye muscles. C This tests the movement of the eye in all directions, which assesses the functions of the cranial nerves III (oculomotor), IV (abducens), and VI (trochlear). D This is performed after the corneal light reflex is abnormal, indicating asymmetric eye muscles.

What is the desired body weight for a male who is 7 feet tall? a. 178 lb b. 225 lb c. 250 lb d. 275 lb

c. 250 lb A This incorrect result is obtained when 106 lb is used for the first 6 feet rather than 5 feet (84 inches - 72 inches = 12); thus, 106 + (12 ´ 6) = 178 lb. B This incorrect result is obtained when the scale for women is used rather than the scale for men (84 inches - 60 inches = 24); thus, 105 + (24 ´ 5) = 225. C The patient is 84 inches tall. Use 106 lb for the first 5 feet (60 inches) and 6 lb/ inch for the remaining 24 inches (84 inches - 60 inches = 24); thus, 106 + (24 ´ 6) = 250 lb. Students must perform a calculation to find the answer. D A result of 275 lb uses an incorrect calculation.

A patient tells the nurse that she has smoked two packs of cigarettes a day for 20 years. The nurse records this as how many pack-years? a. 10 b. 20 c. 40 d. 60

c. 40 A This incorrect calculation was made by dividing 20 years by 2 packs. B This is correct if the patient smoked 1 pack per day for 20 years. C Two packs of cigarettes ´ 20 years = 40 pack-years. D This is correct if the patient smoked 3 packs per day for 20 years or 2 packs a day for 30 years.

4. A nurse is assessing a child who is able to dress herself, jump rope, identify colors, and follow rules when playing games. These are expected developmental achievements of a child of what age? a. 3 years old b. 4 years old c. 5 years old d. 6 years old

c. 5 years old Feedback A These are developmental behaviors too advanced for a 3-year-old child. B These are developmental behaviors too advanced for a 4-year-old child. C These are developmental behaviors consistent with a 5-year-old child. D These developmental behaviors are typically achieved and surpassed by a 6-year-old child.

A patient's blood pressure has been averaging 120/72 when using the upper arms. Today the nurse uses this patient's thigh to measure the blood pressure. What is the expected systolic pressure using the thigh that is equivalent to a systolic pressure of 120? a. A systolic reading of 110 mm Hg b. A systolic reading of 120 mm Hg c. A systolic reading of 140 mm Hg d. A systolic reading of 170 mm Hg

c. A systolic reading of 140 mm Hg A A systolic reading of 110 mm Hg is too low. B A systolic reading of 120 mm Hg is too low. C Normally the systolic blood pressure is 10 to 40 mm Hg higher in the leg than in the arm. D A systolic reading of 170 mm Hg is too high.

Which patient has the greatest risk for hypertension? a. An Asian man who is 5 ft 5 in (165 cm) tallk, weighs 125 lbs (56.7 kg), and complains of a headache over his forehead and eyes b. A Cheyenne Indian woman who complains of a gnawing, burning epigastric pain radiating to her neck and jaw c. An African American man who has type 2 diabetes mellitus, exercises once month, and drinks two-to-three alcoholic drinks a bight with dinner d. A Caucasian woman who has a family history of heart disease and complains of pain in her chest when she takes a deep breath

c. An African American man who has type 2 diabetes mellitus, exercises once month, and drinks two-to-three alcoholic drinks a bight with dinner

A nurse suspects the patient has an infection of the maxillary sinuses. How can this suspicion be confirmed? a. Using a flashlight to illuminate the floor of the mouth b. Pressing gently with both thumbs into the eyebrow ridges c. Applying firm pressure with the thumbs below the cheekbones d. Standing behind the patient and asking him or her to slowly rotate the head

c. Applying firm pressure with the thumbs below the cheekbones A To transilluminate the maxillary sinuses, the nurse places the source of light lateral to the nose, just beneath the medial aspect of the eye, and looks through the patient's open mouth for illumination of the hard palate. B This palpates the frontal sinuses rather than the maxillary sinuses. C This palpates the maxillary sinuses to detect tenderness, which may indicate sinus congestion or infection. D This is not a correct technique to confirm infection of the maxillary sinuses.

During inspection of a patient's upper back, the nurse notices three small, elevated superficial lesions filled with purulent fluid. How does the nurse document this finding? a. As three cysts on the upper back b. As several bullae on the back c. As three pustules on the upper back d. As three wheals on the upper back

c. As three pustules on the upper back A Cysts are elevated, circumscribed, encapsulated lesions. B Bullae are vesicles greater than 1 cm in diameter. This documentation is not specific to the number or exact location. C Pustules are elevated, superficial lesions similar to vesicles but filled with purulent fluid. This is a specific documentation of what the nurse saw (three pustules) and their location (upper back). D Wheals are elevated irregular-shaped areas of cutaneous edema that are solid, transient, and of variable diameter.

During a Weber test, a patient with right ear hearing loss reports hearing sound longer in the right ear than the left ear. What results should the nurse expect to find from this patient during a Rinne test? a. Air conduction will be twice as long as bone conduction (2:1 ratio). b. Air conduction will be 1.5 times as long as bone conduction (1.5:1 ratio). c. Bone conduction will be longer than air conduction. d. Bone conduction will be equal to air conduction.

c. Bone conduction will be longer than air conduction. A This is an expected finding. B This finding is consistent with a sensorineural hearing loss, but this patient has a conduction hearing loss based on the results of the Weber test. C This finding from the Rinne test indicates a conduction hearing loss, which is consistent with the finding from the Weber test described in the question. D This finding is not consistent with the conductive hearing loss described.

A patient complains of right ear pain. What findings does the nurse anticipate on inspecting the patient's ears? a. Redness and edema of the pinna of the right ear b. Report of pain when the nurse manipulates the right ear c. Bulging and red tympanic membrane in the right ear d. Increased cerumen in the right ear canal

c. Bulging and red tympanic membrane in the right ear A Redness and edema of the pinna of the right ear is consistent with external ear pain that may be associated with otitis externa or swimmer's ear. B Report of pain when the nurse manipulates the right ear is consistent with external ear pain that may be associated with otitis externa or swimmer's ear. C Bulging and red tympanic membrane in the right ear is consistent with internal ear pain that may be associated with otitis media. D Increased cerumen in the right ear canal is not consistent with internal ear pain.

As a nurse is inspecting the nails of a patient with chronic hypoxemia and notices enlargements of the ends of the fingers and angles of the nail base greater than a straight line (exceeding 180 degrees). How does the nurse document these findings? a. An expected finding b. Koilonychia (spoon nail) c. Clubbing d. Leukonychia

c. Clubbing A This is clubbing, which is not an expected finding. B Koilonychia is a thin, depressed nail with the lateral edges turned upward and is associated with anemia. C Clubbing is present when the angle of the nail base exceeds 180 degrees. It is caused by proliferation of the connective tissue resulting in an enlargement of the distal fingers and is most commonly associated with chronic respiratory or cardiovascular disease. D Leukonychia appears as white spots on the nail plate, usually caused by minor trauma or manipulation of the cuticle.

A patient reports the mole on the scalp has started itching and it bleeds when scratching it. What other finding is a danger sign for pigmented skin lesions? a. Symmetry of the lesion b. Rounded border c. Color variation d. Size less than 6 mm wide

c. Color variation A Symmetry is an expected finding for moles. Asymmetric lesions are an early sign of malignant melanoma. B A rounded border is an expected finding. A border that is poorly defined or irregular is an early sign of malignant melanoma. C Uneven, variegated color is an early sign of malignant melanoma. D A size of less than 6 mm wide is an expected finding. A lesion greater than 6 mm is an early sign of malignant melanoma.

The nurse suspects an irregularity in the rhythm of the patient's radial pulse. What is the most appropriate action for this nurse to take at this time? a. Document this rhythm as normal for the patient. b. Use a Doppler to check the brachial pulse. c. Count the patient's apical pulse for a full minute. d. Count the radial pulse again for 15 seconds and multiply by 4.

c. Count the patient's apical pulse for a full minute. A An irregular rhythm is not a normal finding. The pulsation between each beat should be the same or regular. B A Doppler is not indicated in this case; it is used when the pulse cannot be palpated. C When an irregular pulse is palpated, the nurse counts the number of pulsations for a full minute. D Counting the radial pulse again for 15 seconds and multiplying by 4 may reconfirm the initial findings, but does not provide additional data for the nurse on this patient.

A patient is in a sitting position as the nurse palpates the temporal arteries and feels smooth, bilateral pulsations. What is the appropriate action for the nurse at this time? a. Auscultate the temporal arteries for bruits. b. Palpate the arteries with the patient in supine position. c. Document this as an expected finding. d. Measure the patient's blood pressure.

c. Document this as an expected finding. A This is not necessary for this patient at this time. B This is not necessary for this patient at this time. C These are consistent with expected assessment. D This will be done as a part of the assessment, but does not relate to the expected palpation of this patient's temporal arteries.

A nurse shines a light in the right pupil to test constriction and notices that the left pupil constricts as well. Based on these data, the nurse should take what action? a. Document this finding as an abnormal finding. b. Assess the patient for accommodation. c. Document this finding as a consensual reaction. d. Assess the patient's corneal light reflex.

c. Document this finding as a consensual reaction A This is a description of an expected finding—consensual reaction. B Accommodation is not assessed in response to consensual reaction; it tests the function of the oculomotor cranial nerve (CN III). C This is a description of expected consensual reaction. D This item describes a consensual reaction rather than a corneal light reflex.

How does a nurse determine jugular vein pulsation? a. Elevates the head of the bed about 90 degrees and looks for the jugular vein pulsation parallel to the sternocleidomastoid muscle as the head of the bed is slowly lowered b. Looks for jugular vein pulsations at the jaw line as the patient turns from supine to a side-lying position c. Elevates the head of the bed until the external jugular vein pulsation is seen above the clavicle d. Position the patient supine and asks him or her to cough; looks for jugular vein pulsations during the cough

c. Elevates the head of the bed until the external jugular vein pulsation is seen above the clavicle

During a physical examination, the nurse notes that the patient's skin is dry and flaking, with patches of eczema. Which nutritional deficiency might be present? a. Vitamin C b. Vitamin B c. Essential fatty acid d. Protein

c. Essential fatty acid A Vitamin C deficiency causes bleeding gums, arthralgia, and petechiae. B Vitamin B deficiency is too large a category to consider. Specific categories of vitamin B deficiency have been identified, such as pyridoxine and thiamine. C Dry and scaly skin is a manifestation of essential fatty acid deficiency. D Protein deficiency causes decreased pigmentation and lackluster hair.

A nurse's presentation to patients on risk factors for oral cancer includes which fact? a. The peak incidence oral cancer is before 40 years of age. b. Women have a higher risk than men. c. Excessive alcohol consumption is a risk factor. d. Eating a low fiber diet is a risk factor.

c. Excessive alcohol consumption is a risk factor. A There is increased incidence after age 40 with peak incidence between ages 64 and 74. B There is a 2:1 men-to-women incidence. C Seventy-five to eighty percent of individuals who develop oral cancer consume excessive amounts of alcohol. D A low fiber diet increases the risk for colon cancer, but not oral cancer.

A patient who had extensive surgery asks the nurse for pain medication for a pain of 9 on a scale of 0 to 10. The nurse completes an assessment of this patient's pain and agrees to give pain medication. When the nurse returns to the patient with the ordered intravenous pain medication, she notices the patient's eyes are closed and he appears to be sleeping. What is the nurse's appropriate action at this time? a. Lock up the medication in a safe location until the patient awakens. b. Arouse the patient to confirm he still wants the medication. c. Give the medication as ordered and agreed to. d. Consult a colleague about what action to take.

c. Give the medication as ordered and agreed to. A The patient needs to receive the ordered pain medication now. B There is no reason to confirm the need for pain medication requested earlier. C Sleep is not synonymous with pain relief. When the patient reports a pain of 9 and asks for medication for which there is an order, he needs to receive the medication. D There is no reason to ask a colleague about giving the pain medication requested earlier.

A male patient scores 125 on the Holmes Social Readjustment Scale. How does the nurse interpret this score? a. He is experiencing a great deal of stress in his life and needs hospitalization. b. At this time he has no stress in his life and is healthy both mentally and physically. c. He has relatively low stress in his life and use of daily relaxation can be beneficial. d. He has a moderate chance of developing a stress-related illness and can reduce this by practicing stress management.

c. He has relatively low stress in his life and use of daily relaxation can be beneficial. A A score on the Holmes Social Readjustment Scale greater than 300 is needed for hospitalization. B This does not apply to this person. The lowest score possible on the Holmes Social Readjustment Scale (less than 150) indicates the amount of stress experienced is a result of normal changes in life and is manageable. C A score on the Holmes Social Readjustment Scale of below 150 indicates the amount of stress experienced is a result of normal changes in life and is manageable. D A moderate chance of developing a stress-related illness and reduction through stress management applies to a patient who scores between 150 and 300.

A male patient weighs 205 lb and his desired body weight (DBW) is 190 lb. How should the nurse counsel this patient about his weight? a. He has mild obesity and needs to increase exercise and assess his diet for nutrients and calories. b. He has moderate obesity and needs to consult a health care provider about weight loss therapy. c. He is within normal limits and need not be concerned at this time. d. Further data are needed before an interpretation can be determined

c. He is within normal limits and need not be concerned at this time. A Using the DBW scale, mild obesity is 20% to 40% above DBW. This patient is 9% above his DBW. B Using the DBW scale, moderate obesity is 40% to 100% greater than DBW. This patient is 9% above his DBW. C Ideally, the patient will fall between 90% and 110% of DBW, which for this patient is between 171 lb and 209 lb. A calculation is required to answer the question. D According to the DBW scale, the patient will ideally be between 90% and 110% of DBW; therefore, the nurse has enough information to make an interpretation.

How does the nurse recognize jaundice in a dark-skinned patient? a. Inspect the conjunctiva for ashen-gray color. b. Inspect the nail beds for a deeper brown or purple skin tone. c. Inspect the palms and soles for yellowish-green color. d. Inspect the oral mucous membrane for yellow color.

c. Inspect the palms and soles for yellowish-green color. A Ashen-gray color may be seen in dark-skinned patients who are cyanotic. B Brown or purple tone is seen in dark-skinned patients with erythema. C In dark-skinned patients, jaundice manifests as a yellowish-green color that can be seen most obviously in the sclera, palms of hands, and soles of feet. D Mucous membranes do not change color from jaundice.

A patient's current body weight (304 lb) and his desirable body weight of 190 lb. How does the nurse classify this patient's weight? a. Within expected range b. Mildly obesity c. Moderate obesity d. Morbid obesity

c. Moderate obesity A A range of 90% to 110% of DBW is considered normal. This patient's weight of 304 is 60% above his DBW. B Mildly obesity is 20% to 40% above DBW. This patient's weight of 304 is 60% above his DBW. C Moderate obesity is 40% to 100% above the DBW. This patient's weight of 304 is 60% above his DBW. D Morbid obesity is 100% greater than DBW. This patient's weight of 304 is 60% above his DBW.

A patient with type 2 diabetes mellitus has an infected lesion on his foot. During the history of his present illness, he reports, "I had a cut on my foot, but I did not even feel it." What equipment does the nurse use to gather more data about his foot? a. A Wood lamp b. Transilluminator c. Monofilament d. Reflex hammer

c. Monofilament A A Wood lamp is used to detect fungal infection on the skin. B A transilluminator differentiates the characteristics of tissue, fluid, and air within a specific body cavity. C A monofilament is used to test for sensation on the lower extremities. Because this patient could not feel the cut on his foot, perhaps he has lost sensation. D A reflex hammer is used to test for deep tendon reflexes.

What findings does a nurse expect when inspecting and palpating a patient's nails? a. A nail base angle of not more than 90 degrees b. Whitish to clear nails in darker-skinned patients c. Nail surface is smooth and rounded d. Transverse depression running across the nails

c. Nail surface is smooth and rounded A The expected angle of the nail base is 160 degrees. B Patients with darker-pigmented skin typically have nails that are yellow or brown, and vertical banded lines may appear. C Nail surface that is smooth and rounded is an expected finding. D This is a description of Beau lines.

Which nurse is performing the technique of light palpation appropriately? a. Nurse A applies the bimanual technique to determine size and location of the patient's heart. b. Nurse B uses the fingertips to feel for temperature differences on the patient's legs. c. Nurse C places the ulnar surface of the hands on the patient's thorax to detect vibrations. d. Nurse D depresses the patient's abdomen approximately 4 cm to assess pulsations.

c. Nurse C places the ulnar surface of the hands on the patient's thorax to detect vibrations. A The bimanual technique is used to entrap an organ or mass (such as the uterus or a growth) between the fingertips to determine size and location and is not palpation. B Temperature differences are best detected using the dorsal surface of the hand; this technique is not palpation. C Nurse C places the ulnar surface of the hands on the patient's thorax to detect vibrations. This is considered a light palpation. D Light pulsation is performed by pressing in to a depth of approximately 1 cm, rather than 4 cm.

While inspecting the legs of a male patient, the nurse notes that the skin is shiny and taut with little hair growth. Which additional data would the nurse find to indicate that this patient has peripheral arterial disease? a. Pitting edema of one or both feet or legs b. Increased circumference in the thighs bilaterally c. Pale, cool legs with diminished-to-absent dorsalis pulses d. Pain when legs are dependent that is relieved when legs are elevated

c. Pale, cool legs with diminished-to-absent dorsalis pulses

What changes in using the ophthalmoscope should the nurse need to make when inspecting the eye of a patient who is nearsighted? a. Holding the ophthalmoscope in the right hand when inspecting the patient's right eye b. Using the grid light of the lens aperture to visualize the internal structures of the eye c. Rotating the diopter to the red (minus) numbers d. Asking the patient to look directly into the ophthalmoscope light

c. Rotating the diopter to the red (minus) numbers A This procedure is performed with all patients having an internal eye examination. B The grid is used to estimate the size of lesions. C This compensates for the longer eyeball of a myopic patient. D This is an instruction given to the patient to visualize the macula.

During a visit to the clinic for an annual gynecologic examination, a patient tells the nurse that she had a bad experience on an airplane, saying, "When I sat down, my heart started racing, I was short of breath and sweaty, and I felt as if I was going to die." She stated that her husband helped her to calm down after a few minutes. The nurse recognizes that the patient was describing which problem? a. Bipolar disorder, manic phase b. Moderate anxiety c. Panic d. Delusions

c. Panic A Characteristics of the manic phase are excessive emotional displays, excitement, euphoria, hyperactivity accompanied by elation, boisterousness, impaired ability to concentrate, decreased need for sleep, and limitless energy, often accompanied by delusions of grandeur. B The moderately anxious person has a narrower field of perception and uses selective inattention to ignore stimuli in the environment to focus on a specific concern. C Physical manifestations of a panic attack represent sympathetic nervous system stimulation. The person experiences muscle tension, tachycardia, dyspnea, hypertension, increased respiration, and profuse perspiration. D Delusions are persistent abnormal beliefs or perceptions held by a person despite the evidence that refutes it.

A nurse assesses neck range of movement of several adults. Which patient has an expected range of motion of the neck? a. Patient A is unable to resist the nurse's attempt to move the head upright. b. Patient B bends the head to the right and left (ear to shoulder) 15 degrees. c. Patient C flexes chin toward the chest 45 degrees. d. Patient D hyperextends the head 30 degrees from midline.

c. Patient C flexes chin toward the chest 45 degrees A This finding is abnormal. B This finding is abnormal. The patient should be able to laterally bend the head 40 degrees from midline in each direction. C This is an expected finding. D The patient should be able to hyperextend the head 55 degrees from midline.

A patient reports a history of snorting cocaine and is concerned about his bloody nasal drainage. What does the nurse expect to see on inspection of his nose? a. Deviated septum b. Pale turbinates c. Perforated nasal septum d. Localized erythema and edema

c. Perforated nasal septum A Deviated septum may be from birth or trauma to the nose, but not from cocaine use. B Pale turbinates are an indication of allergies. C Perforated nasal septum develops from cocaine use. D Localized erythema and edema are nonspecific and indicate inflammation somewhere in the nose.

During an eye assessment, a nurse asks the patient to cover one eye with a card as the nurse covers his or her eye directly opposite the patient's covered eye. The nurse moves an object into the field of vision and asks the patient to tell when the object can be seen. This assessment technique collects what data about the patient's eyes? a. Symmetry of extraocular muscles b. Visual acuity in the uncovered eye c. Peripheral vision of the uncovered eye d. Consensual reaction of the uncovered eye

c. Peripheral vision of the uncovered eye A Symmetry is tested by the corneal light reflex. B Visual acuity is tested using the Snellen chart. C This describes the confrontation test, which assesses peripheral vision. D Consensual reaction is tested by noticing the pupillary constriction of one eye when a light is being shown into the other eye.

How does a nurse assess for fluid in a patient's abdomen? a. Placing the nondominant hand (pleximeter) over the area to be percussed, and striking the index finger of the pleximeter with the pad of the middle finger of the dominant hand b. Applying indirect percussion by tapping one finger lightly on the abdominal wall with the plexor c. Placing the middle finger of the nondominant hand (pleximeter) over the area to be percussed, and striking that finger with the tip of the middle finger of the dominant hand d. Using direct percussion by placing one hand over the abdomen and striking lightly with the other hand

c. Placing the middle finger of the nondominant hand (pleximeter) over the area to be percussed, and striking that finger with the tip of the middle finger of the dominant hand A Only the finger being struck touches the area to be percussed; the other fingers are raised off the skin and the middle finger is struck with the tip of the finger of the other hand. B Percussing the abdomen requires both hands, one as the plexor and the other as the pleximeter. C Placing the middle finger of the nondominant hand (pleximeter) over the area to be percussed, and striking that finger with the tip of the middle finger of the dominant hand describes the correct technique. D Using direct percussion by placing one hand over the abdomen and striking lightly with the other hand does not describe the correct technique.

What technique does a nurse use when palpating the right lobe of a patient's thyroid gland using the posterior approach? a. Pushes the cricoid process to the left with the right thumb and feels the right lobe with the left hand b. Uses the left hand to push the sternocleidomastoid muscle to the right and feels the lobe with the right hand c. Pushes the trachea to the right with the left hand and feels the right lobe with the right hand d. Places the fingers on either side of the trachea above the cricoid cartilage and feels the right lobe

c. Pushes the trachea to the right with the left hand and feels the right lobe with the right hand A This description is not a correct technique. In the posterior approach, the right lobe is felt with the right hand. B This description is not a correct technique. The trachea, not the muscle, is moved to the side. C This is the correct technique. D This description is not a correct technique. The fingers are placed below the cricoid cartilage.

A patient tells the nurse at the clinic, "I can never seem to get warm lately and feel tired all the time." The nurse records these data under which section of the health history? a. Past health history b. Present health status c. Reason for seeking care (chief complaint) d. Subjective assessment data

c. Reason for seeking care (chief complaint) A The past health history includes data about immunizations, surgeries, accidents, and childhood illnesses. B The present health status includes data the nurse obtains from the patient, often using a symptom analysis in which more data are collected about the patient's reason for seeking care. C The reason for seeking care (chief complaint) is the patient's reason for seeking care (also called the presenting problem). The patient's reason for seeking care is often recorded as a direct quote. D Subjective assessment data include information from the patient. In this example, the patient expresses the reason for seeking care, which is directly quoted and placed in quotation marks in the chief complaint section of the data sheet so that the patient's reason for seeking care can be easily identified.

A nurse reads in the history that a patient has a new onset of acute otitis media. Based on this information, how does the nurse expect this patient's tympanic membrane to appear? a. Dull b. Shiny c. Red d. Blue to deep red

c. Red A This indicates fibrosis or scarring. B This is normal for the tympanic membrane. C This indicates infection in the middle ear, such as otitis media. D This indicates blood behind the tympanic membrane, which may have occurred secondary to injury.

During conversation, the nurse observes that the patient is talking continuously and excitedly, and is switching rapidly from one topic to another with seemingly no relationship between topics. This behavior is often associated with which disorder? a. Depression b. Obsessive-compulsive disorder c. Schizophrenia d. Bipolar disorder

c. Schizophrenia A This behavior is flight of ideas, which occurs in patients with schizophrenia rather than depression. B This behavior is flight of ideas, which occurs in patients with schizophrenia rather than obsessive-compulsive disorder. C This behavior is flight of ideas, which occurs in patients with schizophrenia. D This behavior is flight of ideas, which occurs in patients with schizophrenia rather than bipolar disorder.

Which body system does the nurse assess primarily by inspection? a. Respiratory b. Gastrointestinal c. Skin d. Cardiovascular

c. Skin A The respiratory system is assessed primarily using auscultation, but also percussion and inspection when observing pale or cyanotic skin from hypoxia. B The gastrointestinal system is assessed primarily by auscultation and palpation, but also with inspection when looking at the contour of the abdomen. C Skin is assessed primarily using inspection, but also palpation. D The cardiovascular system is assessed primarily with auscultation and palpation, but also by inspection when looking at the color of extremities for evidence of perfusion or edema.

A nurse is teaching a patient how to manage chronic obstructive pulmonary disease (COPD). This intervention is an example of which level of health promotion? a. Primary prevention b. Secondary prevention c. Tertiary prevention d. Risk factor prevention

c. Tertiary prevention A The focus of primary prevention is to prevent a disease from developing by promoting a healthy lifestyle. B Secondary prevention consists of efforts to promote early detection of disease. C Teaching a patient how to live with a chronic disease is an example of tertiary prevention directed toward minimizing the disability from chronic disease and helping the patient maximize his or her health. D Risk factor prevention is part of primary prevention that focuses on preventing disease by managing risk factors.

On auscultation of the heart, the nurse recognizes which expected finding? a. A low-pitched blowing sound is heard over the abdominal aorta b. A high-pitched vibration is heard over the base of the heart c. The S1 heart sound is louder at the apex of the heart d. The S1 heart sound sounds like "Ken-tucky"

c. The S1 heart sound is louder at the apex of the heart

The nurse notes in the patient's history that the patient has persistent, malignant pain. What is the meaning of this type of pain? a. The pain has been present for at least 2 weeks. b. The pain began after recent surgery and is associated with healing incisions. c. The pain has been present for 6 or more months. d. The pain has been present since surgery to remove cancer.

c. The pain has been present for 6 or more months A This time frame is too short. Chronic pain may be intermittent or continuous pain lasting more than 6 months. B This is a description of acute pain rather than chronic. C This is the definition of persistent or chronic pain. D Surgery to remove malignant tissue does not necessarily equate to malignant pain.

Which example below best characterizes a patient's race? a. The language spoken in the patient's home is Tagalog. b. The patient's family follows a kosher diet. c. The patient and his family have blonde hair and fair skin. d. The patient's grandparents came to the United States from Germany.

c. The patient and his family have blonde hair and fair skin A The language spoken at home refers to ethnicity. B A kosher diet refers to ethnicity. C Blonde hair and fair skin indicate genetics and race. D Although the patient and grandparents may share the same race, that the grandparents came to the United States from Germany does not relate to race.

For which person is a comprehensive assessment indicated? a. The person who had abdominal surgery yesterday b. The person who is unaware of his high serum glucose levels c. The person who is being admitted to a long-term care facility d. The person who is beginning rehabilitation after a knee replacement

c. The person who is being admitted to a long-term care facility A A shift assessment is most appropriate for the person who is recovering in the hospital from surgery. B A screening assessment is performed for the purpose of disease detection, in this case diabetes mellitus. C A comprehensive assessment is performed during admission to a facility to obtain a detailed history and complete physical examination. D An episodic or follow-up assessment is performed after knee replacement to evaluate the outcome of the procedure.

Which patient behavior indicates to the nurse that the patient's facial cranial nerve (CN VII) is intact? a. The patient's eyes move to the left, right, up, down, and obliquely. b. The patient moistens the lips with the tongue. c. The sides of the mouth are symmetric when the patient smiles. d. The patient's eyelids blink periodically

c. The sides of the mouth are symmetric when the patient smiles.

While taking a history, the nurse observes that the patient's facial cranial nerves (CN VII) are intact based on which behaviors of the patient? a. The patient's eyes move to the left, right, up, down, and obliquely during conversation. b. The patient moistens the lips with the tongue. c. The sides of the mouth are symmetric when the patient smiles. d. The patient's eyelids blink periodically.

c. The sides of the mouth are symmetric when the patient smiles. A This represents movement of the extraocular muscles, which are controlled by the oculomotor, trochlear, and abducens cranial nerves (CN III, IV, and VI, respectively). B This represents movement of the tongue, which is controlled by the hypoglossal cranial nerve (CN XII). C This represents facial symmetry, which is controlled by the facial cranial nerve (CN VII). D This represents function of the oculomotor cranial nerve (CN III).

How do nurses prevent a latex allergy? a. They use nonlatex gloves for all procedures. b. They protect their hands using oil-based hand lotion applying latex gloves. c. They use a powder-free, low-allergen latex gloves. d. They wash their hands with mild soap and dry thoroughly before applying latex gloves.

c. They use a powder-free, low-allergen latex gloves A Nonlatex gloves may be used only for activities that are not likely to involve contact with infectious materials. B NIOSH recommends not using oil-based hand lotions when wearing latex gloves. C Use of these types of gloves is recommended by The National Institute for Occupational Safety and Health (NIOSH). D NIOSH recommends washing hands after removing latex gloves, not before applying them.

Which nursing behaviors indicate culturally competent care? a. Recognizing that there are different definitions of health and illness b. Complying with the stated plan of treatment despite the patient's differing opinion c. Understanding that there is diversity even among people of the same cultural group d. Helping patients of different cultures adopt the beliefs and behaviors of the dominant culture

c. Understanding that there is diversity even among people of the same cultural group A Recognizing that there are many different definitions of health and illness is not sufficient for culturally competent care. B Complying with the stated plan of treatment despite the patient's differing opinion is not a culturally competent behavior. The patient needs to understand, support, and participate in the plan of care. C Understanding that there is diversity even among people of the same cultural group prevents assumptions and stereotypes that inhibit culturally competent care. D Helping patients of different cultures adopt the beliefs and behaviors of the dominant culture is an example of the opposite of cultural competence; it assumes that all persons should adopt certain beliefs and behaviors.

A nurse is using the finger pads to palpate a patient's dorsalis pedis pulses and is unable to feel any pulses. Which action is appropriate for the nurse to perform next? a. Document that the dorsalis pedis pulses are not palpable. b. Have the patient stand and try again to palpate the pulses. c. Use a Doppler to detect the presence of the pulses. d. Palpate the dorsalis pedis pulses using the ulnar surface of the hand.

c. Use a Doppler to detect the presence of the pulses. A Document that the dorsalis pedis pulses are not palpable. Although the pulse may not be palpable, the nurse always tries a Doppler to determine if the pulse can be heard, even when it cannot be felt. B Have the patient stand and try again to palpate the pulses. Changing positions will not facilitate palpation of a pulse. C Use a Doppler to detect the presence of the pulses. The Doppler uses ultrasonic waves to detect difficult-to-hear vascular sounds, such as peripheral pulses. D Palpate the dorsalis pedis pulses using the ulnar surface of the hand. The ulnar surface of the hand is used to palpate for vibrations rather than pulsations.

While inspecting the skin, a nurse notices a lesion on the patient's upper right arm. What is the best way to document the size of this lesion? a. Compare its size to the size of a coin. b. Estimate its size to the nearest inch. c. Use a centimeter ruler to measure the lesion. d. Trace the lesion onto a piece of paper.

c. Use a centimeter ruler to measure the lesion. A Comparing its size to the size of a coin can be done if no measurement tool is available, but the best way is to measure the lesion. B Estimating size to the nearest inch is not recommended due to inaccuracy. C A centimeter ruler to measure the size of lesions may be helpful. The lesion is documented based on its characteristics, including location, distribution, color, pattern, edges, flat or raised, and size. D Tracing the lesion onto a piece of paper can be done if no measurement tool is available, but the best way is to measure the lesion.

A nurse notices multiple lesions on a patient's left hand that are 0.5 cm in width, elevated, circumscribed, and filled with serous fluid. What kind of primary lesions are these? a. Macules b. Patches c. Vesicles d. Bullae

c. Vesicles A Macules are flat, circumscribed areas that are a change in the color of the skin and are less than 1 cm in diameter. B Patches are flat, nonpalpable, irregular-shaped macules greater than 1 cm in diameter. C Vesicles are elevated, circumscribed, superficial (do not extend into dermis), filled with serous fluid, and less than 1 cm in diameter. This documentation tells the number and location of the lesions. D Bullae are large vesicles greater than 1 cm in diameter.

A nurse uses which technique to assess a patient's peripheral vision? a. The nurse asks the patient to keep the head still and by moving the eyes only, follow the nurse's finger as it moves side to side, up and down, and obliquely. b. The nurse covers one of the patient's eyes with a card and observes the uncovered eye for movement, then removes the card and observes the just uncovered eye for movement. c. With the patient and nurse facing each other and a card covering their corresponding eyes, the nurse moves an object into the visual field and the patient reports when the object is seen. d. The nurse shines a light on both corneas at the same time and notes the location of the reflection in each eye.

c. With the patient and nurse facing each other and a card covering their corresponding eyes, the nurse moves an object into the visual field and the patient reports when the object is seen. A This technique tests extraocular muscle symmetry. B This cover-uncover technique is performed when the corneal light reflex is asymmetric. C This is the confrontation test that tests peripheral vision. D This describes the corneal light reflex that tests the symmetry of the eye muscles.

The patient asks about the meaning of his visual assessment of 20/40 using a Snellen visual acuity chart. What is the nurse's appropriate response? a. "20/40 means your vision is about two times normal." b. "A person with corrected vision can see at 20 feet what you can see at 40 feet." c. "A person with normal vision can see at 20 feet what you can see at 40 feet." d. "A person with normal vision can see at 40 feet what you can see at 20 feet."

d. "A person with normal vision can see at 40 feet what you can see at 20 feet." A This is an incorrect interpretation of the data. B This is an incorrect interpretation of the data. C This is an incorrect interpretation of the data. D The top number of the recording indicates the distance between the patient and the chart, and the bottom number indicates the distance at which a person with normal vision should be able to read certain letters of the chart.

A male patient tells the nurse that he rarely sleeps more than 4 hours a night and has not experienced any problems because of the lack of sleep. Which response by the nurse is most appropriate? a. "That is interesting." b. "Only 4 hours of sleep? How do you stay awake during the day?" c. "Really? Everyone needs more sleep than that." d. "Did I understand that you sleep 4 hours every night?"

d. "Did I understand that you sleep 4 hours every night?" A "That is interesting" does not provide an opportunity for the patient to explain any reason for the number of hours of sleep. B "Only 4 hours of sleep? How do you stay awake during the day?" questions the accuracy of the patient's data and may not encourage the patient to give further details. C "Really? Everyone needs more sleep than that" can be perceived as argumentative, but does not encourage further data from the patient. D "Did I understand that you sleep 4 hours every night?" is a reflection technique that allows the nurse to confirm and obtain additional information.

Which question is the most appropriate to learn about a patient's religious practices? a. "How often do you go to church?" b. "Where is your church located?" c. "Do you mind telling me about your religion?" d. "Do you have any specific religious or spiritual practices or beliefs?"

d. "Do you have any specific religious or spiritual practices or beliefs?" A Asking the patient about his or her church makes the assumption that the patient worships in a church, which may not be correct. B Asking the patient about the location of his or her church makes the assumption that the patient worships in a church, which may not be correct. C The question is not an efficient way to learn about religious or spiritual practices. D "Do you have any specific religious or spiritual practices or beliefs?" is a broad, open-ended question that allows the patient to describe his or her religious beliefs. The question makes no reference to any particular faith.

Which question is an example of an open-ended question? a. "Have you experienced this pain before?" b. "Do you have someone to help you at home?" c. "How many times a day do you use your inhaler?" d. "What were you doing when you felt the pain?"

d. "What were you doing when you felt the pain?" A "Have you experienced this pain before?" is closed-ended, which can obtain a "yes" or "no" answer to the question without any additional data. B "Do you have someone to help you at home?" is closed-ended, which can obtain a "yes" or "no" answer to the question without any additional data. C "How many times a day do you use your inhaler?" is closed-ended, which can obtain an answer of a specific number without any additional data. D What were you doing when you felt the pain?" is a broadly-stated question that encourages a free-flowing, open response.

A patient answers questions quietly and appears sad. While answering questions about her marriage, she begins to cry. Which response by the nurse is appropriate in this situation? a. "Don't cry! I'll come back when you've settled down." b. "I only have a few more questions to ask, and then I'll leave you alone for a while." c. "Everyone has ups and downs in their marriage. What problems are you having?" d. "I see that you are upset. Is there something you'd like to discuss?"

d. "I see that you are upset. Is there something you'd like to discuss?" A "Don't cry! I'll come back when you've settled down" is not a therapeutic response. The nurse needs to support the patient rather than leave her. B "I only have a few more questions to ask, and then I'll leave you alone for a while" is not a therapeutic response. The nurse is more concerned about getting the history than the patient's response. C "Everyone has ups and downs in their marriage. What problems are you having?" is not a therapeutic response. The nurse is assuming there are problems in the marriage instead of collecting more data. D "I see that you are upset. Is there something you'd like to discuss?" shows that the nurse is attentive to the patient's feelings and does not make assumptions about the reason why the patient is crying. The crying may signify additional data the nurse needs to collect during this interview.

A nurse suspects a female patient is a victim of physical abuse. Which response is most likely to encourage the patient to confide in the nurse? a. "You've got a huge bruise on your face. Did your husband hit you?" b. "That bruise looks tender. I don't know how people can do that to one another." c. "If your boyfriend hit you, you can get a restraining order against him." d. "I've seen women who have been hurt by boyfriends or husbands. Does anyone hit you?"

d. "I've seen women who have been hurt by boyfriends or husbands. Does anyone hit you?" A "You've got a huge bruise on your face. Did your husband hit you?" assumes that domestic violence did occur, and the comment does not encourage the patient to divulge additional information. B "That bruise looks tender. I don't know how people can do that to one another" assumes that domestic violence did occur, and the comment does not encourage the patient to divulge additional information. C "If your boyfriend has hit you, you can get a restraining order against him" assumes that domestic violence did occur, and the comment does not encourage the patient to divulge additional information. D "I've seen women who have been hurt by boyfriends or husbands" is an example of a technique referred to as "permission giving" in which the nurse communicates that it is safe to discuss uncomfortable topics.

A patient reports "right shoulder pain that comes and goes" as the chief complaint. During the physical examination, the patient asks why the upper right abdomen is being examined for shoulder pain. What is the appropriate response from the nurse? a. "A comprehensive examination is required to determine the cause of your pain." b. "There may be associated problems that have not produced any symptoms yet that we want to identify." c. "Yes, this can be confusing, but if you will be patient I'm sure we can find something to help you." d. "It does seem odd, but the gallbladder doesn't have pain receptors of its own, so the pain shows up in the shoulder."

d. "It does seem odd, but the gallbladder doesn't have pain receptors of its own, so the pain shows up in the shoulder." A A focused examination is indicated at this time, not a comprehensive examination. B This patient's pain is due to referred pain, not to associated problems that have not produced any symptoms of pain. C This response reflects concern for the patient's pain, but does not address the patient's questions about examining the abdomen. D Referred pain is pain felt at a site different from that of an injured or diseased organ. It commonly occurs during visceral pain because many organs have no pain receptors; thus, when afferent nerves enter the spinal cord, they stimulate sensory nerves from unaffected organs in the same spinal cord segment as those neurons in areas where injury or disease is located.

The nurse is taking a health history on a patient who reports frequent stabbing headaches occurring once a day lasting about an hour. Which statement by the patient is most indicative of cluster headaches? a. "I usually have nausea and vomiting with my headaches." b. "My whole head is constantly throbbing." c. "It feels like my head is in a vice." d. "The pain is on the left side over my eye, forehead, and cheek."

d. "The pain is on the left side over my eye, forehead, and cheek." A This is descriptive of migraines rather than cluster headaches. B This is descriptive of migraines rather than cluster headaches. C This is descriptive of tension rather than cluster headaches. D This description is consistent with cluster headaches.

7. An American Indian mother expresses concern about an irregularly shaped, dark area over her neonate's sacrum and buttocks. What is the nurse's most appropriate response to this mother? a. "This area will continue to grow until the infant is 10 to 15 months old." b. "This is a birth mark, which usually disappears by age 5 years." c. "This skin abnormality will require follow-up care." d. "This is a birth mark and they usually disappear by age 1 or 2 years.

d. "This is a birth mark and they usually disappear by age 1 or 2 years. Feedback A This description refers to cavernous hemangioma that requires frequent reassessment. B This description refers to a "stork bite" (telangiectasis). C This is an inappropriate response. D This description refers to a Mongolian spot.

The patient reports having a persistent cough for the past 2 weeks and that the cough disrupts sleep and has not been helped by over-the-counter cough medicines. Which question is most appropriate for the nurse to ask next? a. "So what do you think is causing this persistent cough?" b. "Have you tried taking sleeping pills to help you sleep?" c. "Did you think this will just go away on its own?" d. "What other symptoms have you noticed related to this cough?"

d. "What other symptoms have you noticed related to this cough?" A The answer to the question "So what do you think is causing this persistent cough?" is a guess by the patient and does not provide useful data. B "Have you tried taking sleeping pills to help you sleep?" does not focus on the cough, which is what is disturbing the patient's sleep. C "Did you think this will just go away on its own?" does not provide useful data and criticizes the patient's lack of action. D "What other symptoms have you noticed related to this cough?" is part of a symptom analysis to provide more data.

A nurse's presentation to patients on risk factors for macular degeneration includes which fact? a. The peak incidence is before 60 years of age. b. Women have a higher risk than men. c. Eating a low fat diet causes a vitamin A deficiency, which increases risk. d. Cigarette smokers have twice the risk as nonsmokers.

d. Cigarette smokers have twice the risk as nonsmokers. A Macular degeneration exists in 25% of those between ages 65 and 74 years, and 33% of those older than 75 years. B There are no differences between genders for macular degeneration. C A diet high in monosaturated, polyunsaturated, and vegetable fats increases risk. D Smoking is a risk factor for macular degeneration.

A patient complains of itching on her feet. On inspection the nurse observes weeping vesicles and skin that is softened and broken down between the toes? What explanation does the nurse give the patient about the cause of this skin disorder? a. "Your itching is caused by a bacterial infection." b. "Your itching is caused by an allergic reaction." c. "Your itching is caused by a viral infection." d. "Your itching is caused by a fungal infection."

d. "Your itching is caused by a fungal infection." A Bacterial infections such as cellulitis cause redness, warmth, and tenderness, rather than itching. B Allergic reactions such as contact dermatitis cause itching, but they appear as localized erythema, and may also form edema, wheals, scales, or vesicles. C Viral infection such as herpes form grouped vesicles that are painful, rather than itching. D This is a description of tinea pedia, which is caused by a number of dermophyte fungal infections.

A patient with mild renal disease has been put on a 2200-calorie per day diet plan with the lowest recommended amount of protein. During discharge teaching, the nurse explains to this patient how to use nutrition labels to determine the amount of protein in the product. The nurse explains, however, that the label is based on 2000 calories. Which is the appropriate formula to teach this patient the least amount of protein he can eat on his prescribed diet? a. 2200 calories ´ 0.15 = 330/9 calories/gram = 36.6 g b. 2200 calories ´ 0.10 = 220/4 calories/gram = 55 g c. 2200 calories ´ 0.20 = 440/9 calories/gram = 48.8 g d. 2200 calories ´ 0.12 = 264/4 calories/gram = 66 g

d. 2200 calories ´ 0.12 = 264/4 calories/gram = 66 g A Each gram of protein yields 4 calories rather than 9. B Twelve percent is the least recommended percentage for proteins, rather than 10%. C Twelve percent is the least recommended percentage for proteins. Each gram of protein yields 4 calories rather than 9. D Proteins should account for 12% to 20% of total calories. Each gram of protein yields 4 calories.

Which tool is the best choice for a nurse to use as a quick screening tool to assess a patient's dietary intake? a. Food diary b. Calorie count c. Comprehensive diet history d. 24-hour recall

d. 24-hour recall A A food diary provides detailed information, but is not convenient and requires a follow-up visit. B A calorie count requires several days to collect data and requires a trained dietitian to analyze the results. C A comprehensive diet history may provide more accurate reflection of nutrient intake, but is time-consuming to acquire and requires a trained/skilled dietary interviewer. D Twenty-four-hour recall is useful as a quick screening tool to assess dietary intake.

While taking a history, the nurse notices that the patient's family member repeats most of the questions to the patient in a loud voice. Based on this information, what finding does the nurse anticipate when assessing this patient's hearing using an audioscope? a. 5 dB hearing loss at all frequencies b. 10 dB hearing loss at all frequencies c. 20 dB hearing loss at all frequencies d. 40 dB hearing loss at all frequencies

d. 40 dB hearing loss at all frequencies A This decibel level is not tested by an audioscope . B A 10 dB loss in high frequencies results in difficulty hearing quiet sounds, such as a heartbeat. C A 20 dB loss in high frequencies results in difficulty hearing high-pitched consonants, such as a whisper. D A 40 dB loss in all frequencies causes moderate difficulty in hearing normal speech.

A female Korean patient accompanied by her husband and son comes to the emergency department (ED) complaining of abdominal pain. The patient speaks and understands Korean only. Which person is the appropriate choice for the nurse to use to get a history from this patient? a. The patient's husband who speaks Korean and English b. The patient's son who speaks Korean and English c. A male technician who works in the ED who speaks Korean and English d. A female interpreter who speaks Korean and English and is available by phone

d. A female interpreter who speaks Korean and English and is available by phone A The patient's husband who speaks Korean and English is not the best choice because he is a family member and may alter the meaning of what is said. B The patient's son who speaks Korean and English is not the best choice because he is a family member and may alter the meaning of what is said. C A male technician working in the ED who speaks Korean and English is not a good choice because the patient may feel uncomfortable giving a history to a stranger who is male. D A female interpreter who speaks Korean and English and is available by phone is the best choice because she can communicate with the patient and is the same gender as the patient

Which patient may be experiencing severe anxiety? a. A woman who tells the nurse she is terrified of cats b. A man who tells the nurse he feels worthless and is always tired c. A woman who reports that she is sleeping very lightly each night because her child has an ear infection d. A man who phones the nurse five times asking for instructions about how to take his new medication

d. A man who phones the nurse five times asking for instructions about how to take his new medication A Being terrified of cats describes fear or a phobia rather than anxiety. Unlike fear, which is a response to an actual object or event, anxiety is a response to no specific source or actual object. B Although fatigue is a characteristic of anxiety, in this case, the patient also verbalizes feelings of worthlessness, which suggests depression rather than anxiety. C Although problems falling and staying asleep are characteristics of anxiety, in this example the patient can identify the cause of the sleeplessness—her ill child—thus anxiety is not the cause of the sleep disturbance. D A man who phones the nurse five times asking for instructions about how to take his new medication shows characteristics of anxiety, which includes forgetfulness and difficulty concentrating or making decisions.

Which patient needs to be taught about how diet and exercise can lower lipids to reduce the risk for coronary artery disease? a. A woman with a high-density lipoprotein (HDL) level of 53 mg/dl b. A man with an HDL level of 43 mg/dl c. A woman with a low-density lipoprotein (LDL) level of 125 mg/dl d. A man with an LDL level of 200 mg/dl

d. A man with an LDL level of 200 mg/dl A This patient's HDL level is in the expected range. B This patient's HDL level is in the expected range. C This patient's LDL level is in the expected range. D The healthy range of LDL for men and women is <130 mg/dl.

What tool does the nurse use to assess the patient's near vision? a. A Snellen eye chart placed about 12 inches from the patient's face. b. An ophthalmoscope with the diopter set at 0 (zero). c. A Jaeger or Rosenbaum chart placed about 2 feet from the patient's face. d. A newspaper held about 14 inches from the patient's face.

d. A newspaper held about 14 inches from the patient's face. A A Snellen chart is used to assess distant vision. B An ophthalmoscope is used to assess the internal eye. C This is incorrect because of the distance specified. These charts can be used to assess near vision when placed at 14 inches from the patient's face. D This can be an alternative to using a Jaeger or Rosenbaum chart held at 14 inches from the face.

A patient comes to the clinic for evaluation after a sinus infection. To evaluate the therapy, the nurse uses transillumination to assess the sinuses and notes which finding indicating recovery from a frontal sinus infection? a. The soft palate illuminates brightly when the light source is placed against the lateral nose. b. No illumination is noted when the light source is placed firmly against the lateral nose. c. A bright glow illuminates the hard palate when the light source is placed against each temporal bone. d. A reddish light is noted above the eyebrows when the light is placed against each supraorbital rim.

d. A reddish light is noted above the eyebrows when the light is placed against each supraorbital rim. A This describes incorrect technique for transillumination. B An absence of a glow during transillumination of the sinuses may indicate that the sinuses are congested. C This describes incorrect technique for transillumination. D Finding a reddish light above the eyebrows when the light is placed against each supraorbital rim is consistent with frontal sinuses free of infection.

A nurse examines a patient's auditory canal and tympanic membrane with an otoscope and observes which finding as normal? a. Clear fluid lining the auditory canal b. A firm tympanic membrane without fluctuation with puffs of air c. A small hole within the cone of light d. A shiny, translucent tympanic membrane

d. A shiny, translucent tympanic membrane A Clear fluid or bloody drainage following a head injury may indicate a basilar skull fracture. B An expected response is that the tympanic membrane slightly fluctuates with puffs of air. C A cone of light is expected, but a hole indicates perforation. D A shiny, translucent tympanic membrane is an expected finding.

A patient complains of sore throat, pain with swallowing, fever, and chills. The nurse suspects tonsillitis and plans to palpate the anterior cervical lymph nodes. Where does the nurse place his fingers to palpate these nodes? a. In front of the ears b. Under the mandibles c. Along the angle of the mandibles d. Adjacent to the sternocleidomastoid muscles

d. Adjacent to the sternocleidomastoid muscles A This is the location of the preauricular lymph nodes. B This is the location of the submental and submandibular lymph nodes. C This is the location of the parotid lymph nodes. D This is the location of the anterior cervical lymph nodes.

Where does the nurse attach the sensor probe of the pulse oximeter to measure a patient's oxygen saturation? a. The chest over the patient's heart b. Over the patient's abdominal aorta c. Over the patient's radial pulse d. Around the patient's index finger nail

d. Around the patient's index finger nail A The chest over the patient's heart is an incorrect option because the LED would not be able to reflect off oxygenated and deoxygenated hemoglobin molecules circulating in blood. B Over the patient's abdominal aorta is an incorrect option because the LED would not be able to reflect off oxygenated and deoxygenated hemoglobin molecules circulating in blood. C Over a patient's radial pulse is an incorrect option because the LED would not be able to reflect off oxygenated and deoxygenated hemoglobin molecules circulating in blood. D The sensor is taped to a highly vascular area, such as around the index finger nail that allows the light-emitting diode (LED) to reflect off oxygenated and deoxygenated hemoglobin molecules circulating in blood.

On inspection of the external eye structures of an African American patient, the nurse notices the sclerae are not white, but appear a darker shade with tiny black dots of pigmentation near the limbus. How does the nurse document this finding? a. As an indication of a type of anemia b. As a hordeolum or sty c. As jaundice d. As an expected racial variation

d. As an expected racial variation A This may cause a pale conjunctiva. B This is an acute infection originating in the sebaceous gland of the eyelid. C Jaundice is a yellow color of the sclera associated with liver or gallbladder disease. D This as an expected racial variation.

A patient is visiting an urgent care center after being hit in the back with a baseball. Upon examination, the nurse notes a flat, nonblanchable spot 2.25 cm wide that is reddish-purple in color. How does the nurse document this lesion? a. As an angioma b. As purpura c. As petechiae d. As ecchymosis

d. As ecchymosis A An angioma is characterized by a small central red area with radiating spider-like legs that blanches with pressure. The lesion of this patient does not blanch. B Purpura is flat, reddish purple, non-blanchable, and greater than 0.5 cm in size. It is caused by infection or a bleeding disorder, not trauma. C Petechiae are tiny, flat, reddish-purple, non-blanchable spots in the skin less than 0.5 cm in diameter and appear as tiny red spots that are pinpoint to pinhead in size. D Ecchymosis is reddish purple in color, nonblanchable and is caused by trauma (being hit with a baseball) to the blood vessel which results in bleeding underneath the skin. The size of the ecchymotic area varies depending on the level of trauma.

1. A patient comes to the emergency department and tells the triage nurse that he is "having a heart attack." What is the nurse's top priority at this time? a. Determine the patient's personal data and insurance coverage. b. Ask the patient to take a seat in the waiting room until his name is called. c. Request that a nurse collect data for a comprehensive history. d. Ask a nurse to start a focused assessment of this patient now.

d. Ask a nurse to start a focused assessment of this patient now. A Personal data and insurance information will be obtained, but in this situation, these data can wait until after the patient is assessed. B Rather than asking the patient to wait, the nurse needs to begin data collection, such as vital signs, immediately to determine the patient's health status. C A comprehensive history is not indicated in this situation at this time. Some subjective data will be collected, such as allergies and medical history related to cardiovascular disease. D The nurse needs to begin an assessment as soon as possible that is focused on this patient's cardiovascular system. The type of health assessment performed by the nurse is also driven by patient need.

How does the nurse perform a Rinne test of hearing function? a. Whispers several words to the patient and requests that the patient repeat the words heard b. Places a vibrating tuning fork in the middle of the head and asks the patient if the sound is heard the same in both ears or if it is louder in one ear than the other c. Places a set of headphones over both ears, plays several tones, and asks the patient to identify the sounds d. Places a vibrating tuning fork on the mastoid process until the patient no longer hears it, and then moves it in front of the ear until the patient no longer hears it

d. Places a vibrating tuning fork on the mastoid process until the patient no longer hears it, and then moves it in front of the ear until the patient no longer hears it A This technique describes the whisper test. B This technique describes the Weber test. C This technique describes the use of an audiometer. D This technique describes the Rinne test.

A nurse is interviewing a male patient who reports he has not had a tetanus immunization in about 15 years because he had a "bad reaction" to the last tetanus immunization. What is the most appropriate response by the nurse in this case? a. Notify the health care provider that this immunization cannot be given. b. Document that the patient is allergic to the tetanus vaccine. c. Give the vaccine after explaining that adverse reactions are rare. d. Ask the patient to describe the "bad reaction" to the vaccine in more detail.

d. Ask the patient to describe the "bad reaction" to the vaccine in more detail. A The immunization should not be eliminated at this time. Additional facts are needed to determine the type of reaction the patient experienced. B Documenting an allergy to the tetanus vaccine may be an error because there are insufficient data to make that determination at this time. C Giving the vaccine may be an error if the patient is allergic to the vaccine and additional data indicates that may be the case. D The nurse needs to collect more data about the reaction from the patient to determine the type of reaction experienced. The nurse is trying to assess the relationship between the "reaction" reported by the patient and an allergic reaction.

A patient tells the nurse that her religion prohibits her from eating food prepared outside of a special kitchen. What is the nurse's appropriate action to meet this patient's needs? a. Call the dietary department to cancel the patient's meal tray. b. Tell the patient that her diet must be carefully monitored and prepared at the hospital. c. Tell the patient that because of her illness, a few changes to her religious requirements will be necessary. d. Ask the patient to describe the requirements for the special kitchen.

d. Ask the patient to describe the requirements for the special kitchen. A Cancelling the patient's food tray does not meet the patient's needs. B Telling the patient that her diet must be carefully monitored and prepared at the hospital is inappropriate because it does not consider the ethnicity of the patient and is not culturally competent care. C Asking the patient to change her religious requirements does not respect her ethnicity and is not culturally competent care. D Asking the patient to describe the requirements for the special kitchen allows the nurse to collect more data about the needs of the patient.

Which technique should the nurse use to obtain more data about a patient's vague or ambiguous statement? a. Laughing and smiling during conversation b. Using phrases such as "Go on," and "Then?" c. Repeating what the patient has said, but using different words d. Asking the patient to explain a point

d. Asking the patient to explain a point A Laughing and smiling during conversation may show attentiveness during the interview, but does not help to clarify vague information. B Using phrases such as "Go on" and "Then?" encourages patients to continue talking, but does not help clarify. C Rephrasing what the patient has said is restatement. It confirms your interpretation of what they said, but does not encourage additional talking. D Asking the patient to explain a point is clarification, which is used to obtain more information about conflicting, vague, or ambiguous statements.

Nurses use which measurement as the most highly correlated with risk of morbidity and mortality? a. Waist-to-hip ratio b. Triceps skinfold measure c. Desirable body weight d. Body mass index (BMI)

d. Body mass index (BMI) A The waist-to-hip ratio is an indication of the risk of unhealthy fat distribution. A ratio that exceeds the desired ratio is indicative of upper body obesity. This increases the risk of developing health problems related to obesity (e.g., diabetes, hypertension, coronary artery disease, gallbladder disease, osteoarthritis, and sleep apnea). B Triceps skinfold measurement provides an estimate of total body fat, which is only one measure of risk for obesity, and not, by itself, the most reliable and valid measurement to determine risk. C Calculating desirable body weight and comparing it to actual body weight does not consider height in the assessment of obesity. D The BMI considers both weight and height in the calculation. The U.S. Preventive Services Task Force advocates the BMI assessment as reliable and valid for identifying adults at risk of morbidity and mortality because of being overweight or obese. For this reason, calculating BMI is recommended for all individuals on a periodic basis.

A patient is sitting slightly forward bracing his arms on his knees in a tripod position. This position is associated with which symptom? a. Abdominal pain b. Spinal deformity c. Back pain d. Breathing difficulty

d. Breathing difficulty A Positions used by patients with abdominal pain vary depending upon what organ is involved. For example, patients with appendicitis tend to lie very still; those with acute pancreatitis prefer the fetal position for pain relief. B Spinal deformity usually affects the patient's gait or causes a slumped posture. C Back pain usually affects the patient's gait or causes a slumped posture. D Breathing difficulty is associated with the tripod position, which allows maximal expansion of the muscles of respiration.

How do nurses assess a patient's pain? a. By assessing physiologic changes of the patient b. By understanding the sensory experience related to the amount of tissue damage c. By the patient's medical diagnosis or surgical procedure d. By asking the patient to rate the pain being experienced

d. By asking the patient to rate the pain being experienced A The pain perceived is unrelated to the physiologic changes of the patient. B Although pain occurs when tissues are damaged, there is no correlation between the amount of tissue damage and the degree and intensity of pain experienced. C There is no correlation between pain perceived and a medical diagnosis or surgical procedure. D Pain is whatever the patient says it is. One person cannot judge the perception or meaning of pain of another person.

A toddler patient has a small, slightly raised bright red area on the trunk. The child's mother reports that the lesion has been present since birth and has become a little larger. What type of lesion does the nurse suspect? a. Vascular nevi b. Purpura c. Ecchymosis d. Cherry hemangioma is a benign tumor consisting of a mass of small blood vessels and can vary in size. These are typically small, slightly raised lesions that are bright red in color appearing on the face, neck and trunk of the body. These lesions increase in size with age.

d. Cherry hemangioma is a benign tumor consisting of a mass of small blood vessels and can vary in size. These are typically small, slightly raised lesions that are bright red in color appearing on the face, neck and trunk of the body. These lesions increase in size with age. A Vascular nevi is a type of angioma that involves the capillaries within the skin producing an irregular macular patch that can vary from light red to dark red to purple in color. B Purpura is a flat, reddish purple, nonblanchable discoloration in the skin greater than 0.5 cm in diameter. C Ecchymosis is a reddish purple, nonblanchable spot of variable size. D Cherry hemangioma is a benign tumor consisting of a mass of small blood vessels and can vary in size. These are typically small, slightly raised lesions that are bright red in color appearing on the face, neck and trunk of the body. These lesions increase in size with age.

A Hispanic patient tells an African American nurse, "You are African American and can't possibly understand how a person like me feels." What is an appropriate response by the nurse at this time? a. Find a nurse who is not African American to interview the patient. b. Ask the patient, "Why do you think that, since we just met?" c. Note that the patient is very defensive about being racially different. d. Encourage the patient to describe what he means by his statement.

d. Encourage the patient to describe what he means by his statement A Finding a nurse who is not African American to interview the patient is neither necessary nor practical. B Asking the patient "Why do you think that, since we just met?" sounds defensive and may not facilitate further communication. C Making an assumption "that the patient is very defensive about being racially different" is not based on any data. D By encouraging the patient to describe what he means by his statement, the nurse is demonstrating cultural awareness and sensitivity by gathering more data about the unique beliefs and value systems of this patient.

When does a nurse choose to use skinfold calipers when collecting assessment data? a. Calculating the patient's body mass index b. Inspecting the patient's skin c. Determining the amount of the patient's lean body tissue d. Estimating the amount of the patient's body fat

d. Estimating the amount of the patient's body fat A Body mass index is a formula for determining obesity that is calculated by dividing a person's weight in kilograms by the height in meters. B Calipers estimate body fat. They are not needed to inspect skin. C There is no specific method to determine the amount of lean body tissue. D Estimating the amount of the patient's body fat is the purpose of using skin calipers.

Which activity illustrates the concept of primary prevention? a. Monthly breast self-examination b. Annual cervical (Papanicolaou test) examination c. Education about living with asthma d. Exercising three times a week

d. Exercising three times a week A Monthly breast self-examination is an example of secondary prevention and screening efforts to promote early detection of disease. B Annual cervical (Papanicolaou test) examination is an example of secondary prevention and screening efforts to promote early detection of disease. C Teaching a patient how to live with a chronic disease such as asthma is an example of tertiary prevention directed toward minimizing the disability from chronic disease and helping the patient maximize his or her health. D Exercising is an example of primary prevention that prevents disease from developing by maintaining a healthy lifestyle.

During shift report, a nurse learns that a patient has a macular rash. As the nurse inspects the patient's skin, what finding will confirm the rash? a. Elevated, firm, well-defined lesions less than 1 cm in diameter b. Depressed, firm, or scaly, rough lesions greater than 1 cm in diameter c. Elevated, fluid-filled lesions less than 1 cm in diameter d. Flat, well-defined, small lesions less than 1 cm in diameter

d. Flat, well-defined, small lesions less than 1 cm in diamete A Elevated, firm, well-defined lesions less than 1 cm in diameter is a description of a papule. B Depressed, firm, or scaly, rough lesions greater than 1 cm in circumference is an incorrect description. C Elevated, fluid-filled lesions less than 1 cm in diameter is a description of a vesicle. D Flat, well-defined, small lesions less than 1 cm in diameter is a description of a macule.

What is the most accurate technique for detecting a deep vein thrombosis at the bedside? a. Dosiflex the calf and note if the patient complains of pain b. Elevate one leg above the level of the heart to determine if the veins empty c. Palpate the pulses distal to the areas of the suspected thrombosis d. Measure the thigh circumference to detect an increase from the baseline

d. Measure the thigh circumference to detect an increase from the baseline

A patient in the waiting room appears anxious and moves around the room cleaning surfaces with a disinfectant cloth. This behavior is consistent with which disorder? a. Bipolar disorder b. Delirium c. Schizophrenia d. Obsessive-compulsive disorder

d. Obsessive-compulsive disorder ixed moods. B Delirium has manifestations that include attention deficits, disorganized thinking, confusion, disorientation, restlessness, incoherence, anxiety, and excitement. C Schizophrenia is characterized by gross distortion of reality, disturbances of language and communication, withdrawal from social interaction, and the disorganization and fragmentation of thought perception and emotional reaction. D This patient was concerned about contamination. Compulsions are unwanted, repetitive behavior patterns or mental acts that are intended to reduce anxiety. The person recognizes that the behaviors are excessive or unreasonable but continues them because of the relief from the discomfort of anxiety that they provide.

During the history, a 65-year-old male patient reports smoking two packs of cigarettes a day for more than 40 years. With this knowledge, what does the nurse expect for during the examination of this patient's mouth? a. Cracks and erythema in the corners of the mouth b. Slightly rough papillae on the dorsal surface of the tongue c. Smooth or beefy, red-colored, edematous tongue d. Painless, nonhealing mouth ulcers

d. Painless, nonhealing mouth ulcers A This may be caused by vitamin B deficiencies. B These are an expected finding on the tongue. C This may be an indication of anemia. D This may indicate oral cancer.

Wearing gloves, the nurse grasps the patient's tongue with a gauze pad and palpates a small, firm nodule on the left side of the tongue. Based upon this finding, what is the nurse's appropriate response? a. Document that the patient's tongue is normal on palpation. b. Inspect the left submandibular salivary glands for redness. c. Ask the patient to move the tongue in all directions. d. Palpate cervical and submental lymph nodes for enlargement.

d. Palpate cervical and submental lymph nodes for enlargement. A The nodule is not an expected finding. B The salivary glands are not affected by a nodule of the tongue. C This assesses the hypoglossal cranial nerve or movement of the tongue, which is not related to the nodule found. D The nodules may indicate a malignancy of the tongue, which may also cause enlarged cervical or submental lymph nodes.

Which finding warrants a referral for additional evaluation? a. Earlobes hanging freely from the base of the pinna b. Ears having painless nodules less than 1 cm in diameter at the helix c. Ears measuring 8 cm in length d. Pinna is 20 degrees lower than the outer canthus of the eye

d. Pinna is 20 degrees lower than the outer canthus of the eye A Earlobes hanging freely from the base of the pinna is an expected finding. B This is called a Darwin tubercle. It is a normal deviation and may be noted at the helix of the ear. C A length of 8 cm is an expected finding. D The pinna of the ear should align directly with the outer canthus of the eye and be angled no more than 10 degrees from a vertical position.

A patient comes to the ambulatory surgery center for an elective procedure this morning. While giving the admission history, the patient states she is allergic to latex. What is the most appropriate response by the nurse at this time? a. Removing all latex products from the patient's room b. Using powdered gloves when providing care to this patient c. Informing the surgeon that the patient has type I hypersensitivity to latex d. Questioning the patient about symptoms experienced in the past with latex

d. Questioning the patient about symptoms experienced in the past with latex A Removing all latex products from the patient's room is unnecessary at this time because the latex allergy has not been confirmed. B Using powdered gloves when providing care to this patient is unnecessary at this time because the latex allergy has not been confirmed. C Informing the surgeon that the patient has type I hypersensitivity to latex is unnecessary at this time because the latex allergy has not been confirmed. D Questioning the patient about symptoms experienced in the past with latex is the appropriate response. When patients indicate an allergy to a medication or substance, ask them to describe what happens with exposure to determine whether the reaction is a side effect or an allergic reaction.

When performing a skin assessment of an adult patient, the nurse expects what finding? a. Reddened area does not blanch when gentle pressure is applied b. Indentation of the finger remains in the skin after palpation c. Flaking or scaling of the skin d. Return of skin to its original position when pinched up slightly

d. Return of skin to its original position when pinched up slightly A This is an indication of a stage I pressure ulcer. B This is a description of edema. C This may be an indication of dry skin, systemic disease, or nutritional deficiency. D This is an assessment of skin turgor; skin should return to its original position.

Which action by the nurse describes the correct technique for using an otoscope on an adult? a. Using the pneumatic attachment to observe for tympanic fluctuation b. Striking the otoscope against the hand to engage c. Instructing the adult to raise one finger when a sound is heard d. Selecting the largest size speculum that fits into the adult's ear canal

d. Selecting the largest size speculum that fits into the adult's ear canal A The pneumatic attachment is used to evaluate the fluctuation of the tympanic membrane in children. B The otoscope is not struck. The instrument that is struck before hearing assessment is a tuning fork. C Instructing the patient to raise one finger when a sound is heard is done when using an audiometer to assess hearing. D Using the largest speculum allows visualization, while using a smaller speculum limits inspection and using a speculum that is too large is uncomfortable to the adult.

When palpating the right lobe of the patient's thyroid gland using the anterior approach, the nurse feels the tissue between which two structures? a. Sternocleidomastoid and the trapezius muscles b. Trapezius muscle and the trachea c. Cricoid process and the trachea d. Sternocleidomastoid muscle and the trachea

d. Sternocleidomastoid muscle and the trachea A This is not the correct location for palpating the thyroid gland using the anterior approach. B This is not the correct location for palpating the thyroid gland using the anterior approach. C This is not the correct location for palpating the thyroid gland using the anterior approach. D This is the correct location for palpating the thyroid gland using the anterior approach.

The nurse documents which information in the patient's history? a. The patient's skin feels warm to the touch. b. The patient is scratching his arm. c. The patient's temperature is 100° F. d. The patient complains of itching.

d. The patient complains of itching. A The patient's warm skin is objective information gathered by the nurse through palpation, is also a sign, and is documented in the physical examination. B The patient's scratching is objective information gathered by the nurse through observation, is also a sign, and is documented in the physical examination. C The patient's elevated temperature is objective information gathered by the nurse through measurement, is also a sign, and is documented in the physical examination. D A patient's complaint of itching is subjective information, which means it is a symptom and is documented in the history.

How does a nurse recognize a patient's mydriasis? a. The lens of each of the patient's eyes is opaque. b. There is involuntary rhythmical, horizontal movement of the patient's eyes. c. There is a white opaque ring encircling the patient's limbus. d. The patient's pupils are 7 mm and do not constrict.

d. The patient's pupils are 7 mm and do not constrict. A An opaque lens is an abnormality that occurs when cataracts are present. B An involuntary rhythmical, horizontal movement of the patient's eyes is a description of nystagmus. C A white opaque ring encircling the patient's limbus is a description of corneal arcus seen in patients older than 60 years of age. D Mydriasis is pupil size greater than 6 mm and the pupil fails to constrict.

How does a nurse recognize normal accommodation? a. The patient has peripheral vision of 90 degrees left and right. b. The patient's eyes move up and down, side to side, and obliquely. c. The right pupil constricts when a light is shown in the left pupil. d. The patient's pupils dilate when looking toward a distant object.

d. The patient's pupils dilate when looking toward a distant object. A Normally a patient has 90 degrees peripheral vision temporally, but only 60 degrees nasally. B This is an expected finding, but is not a test for accommodation. It is a test of extraocular muscle function in the six cardinal fields of gaze. C This is an expected finding for consensual reaction, rather than accommodation. D This is an indication of accommodation.

For which person is an episodic or follow-up assessment indicated? a. The person who had abdominal surgery yesterday b. The person who is unaware of his high serum glucose levels c. The person who is being admitted to a long-term care facility d. The person who is beginning rehabilitation after a knee replacement

d. The person who is beginning rehabilitation after a knee replacement A A shift assessment is most appropriate for the person who is recovering in the hospital from surgery. B A screening assessment is performed for the purpose of disease detection, in this case diabetes mellitus. C A comprehensive assessment is performed during admission to a facility to obtain a detailed history and complete physical examination. D An episodic or follow-up assessment is performed after the knee replacement to evaluate the outcome of the procedure.

Nurses understand that a patient's diastolic pressure represents which physiologic function? a. The pressure needed to open the aortic and pulmonic valves b. The pressure in blood vessels when the ventricles contract c. The pressure of the blood returning to the heart from the venous system d. The pressure in blood vessels when the ventricles are relaxed

d. The pressure in blood vessels when the ventricles are relaxed A The pressure needed to open the aortic and pulmonic valves is called the afterload. B The pressure in blood vessels when the ventricles contract is the definition of the systolic pressure. C The pressure of the blood returning to the heart from the venous system is incorrect. D The pressure in blood vessels when the ventricles are relaxed is the definition of the diastolic pressure.

Which patient has the least risk for unhealthy fat distribution? a. The man whose triceps skinfold is at the 25th percentile b. The woman whose triceps skinfold is at the 72nd percentile c. The man whose waist circumference is 46 inches and hip circumference is 40 inches d. The woman whose waist circumference is 30 inches and hip circumference is 38 inches

d. The woman whose waist circumference is 30 inches and hip circumference is 38 inches A Triceps skinfold is an estimate for total body fat, rather than risk. The expected value is near the 50th percentile. B Triceps skinfold is an estimate for total body fat, rather than risk. The expected value is near the 50th percentile. C This man's waist-to-hip ratio is 1.15, which is higher than the 1.0 or less expected value for a man. D This woman's waist-to-hip ratio is 0.789, which is below 0.8, the expected value for women.

A patient has been complaining of abdominal cramping and gas; the nurse notes that his abdomen is slightly distended. Which sound does the nurse expect to hear during percussion of this patient's abdomen? a. Flatness b. Dullness c. Resonance d. Tympany

d. Tympany A Flatness is heard over bones and muscle. B Dullness is heard over the liver. C Resonance is heard over normal lung tissue. D Tympany is a loud, high-pitched sound heard over the abdomen.

How does the nurse perform the bimanual technique of palpation to assess organs? a. Using the palmar surface of the dominant hand to press inward to a depth of about 1 cm b. Holding a light source in one hand while stroking the skin lightly with the dominant hand c. Using the ulnar surfaces of both hands to press inward 4 to 5 cm d. Using both hands, one anterior and one posterior, to entrap an organ between the fingertips

d. Using both hands, one anterior and one posterior, to entrap an organ between the fingertips A Using the palmar surface of the dominant hand to press inward to a depth of about 1 cm describes light palpation, which is different from the bimanual technique. B Holding a light source in one hand while stroking the skin lightly with the dominant hand is used when inspecting rather than palpating. C Using the ulnar surfaces of both hands to press inward 4 to 5 cm describes an incorrect technique. D Using both hands, one anterior and one posterior, to entrap an organ between the fingertips is the correct technique for bimanual palpation.

5. A 4-year-old child has had a tonsillectomy and the nurse is preparing to ask him about his pain. Which technique is the most appropriate method for pain assessment for this patient? a. Asking him if the pain hurts "a little or a lot" b. Asking him to rate the pain on a scale of 0 to 10 c. Using the visual analog scale to rate the pain d. Using the Wong/Baker FACES rating scale

d. Using the Wong/Baker FACES rating scale Feedback A Using adjectives such as these is not reliable to assess pain in patients of any age. B This scale is appropriate for adolescents and adults, but a child cannot understand the concept of using numbers to rate pain. C This type of scale is appropriate for adults, but a child cannot understand the concept of using a straight line to rate pain. D This tool is appropriate for children who can point to the child's face that best represents how they are feeling.

A patient with a partial small bowel obstruction describes the pain as "cramping, off-and-on pain that spreads over my stomach." What type of pain is this patient experiencing? a. Referred pain b. Phantom pain c. Somatic pain d. Visceral pain

d. Visceral pain A Referred pain is felt at a site different from that of an injured or diseased organ. B Phantom pain is associated with amputations. C Somatic pain arises from bone, joint, muscle, skin, or connective tissues and is usually aching or throbbing in quality and well located. D Visceral pain occurs with obstruction of a hollow organ and causes intermittent cramping pain.

A nurse suspects that a large skin lesion on the patient's forearm is a fungal infection. Which device does the nurse use to confirm his suspicion? a. Pen light b. Magnification device c. Transilluminator d. Wood lamp

d. Wood lamp A A pen light is used to highlight a lesion for inspection, but will not determine if it is caused by a fungus. B A magnification device helps visualize the lesion, but will not determine if it is caused by a fungus. C A transilluminator disseminates its light source under the surface of the skin to determine if the area under the surface is filled with air, fluid, or tissue. D Skin lesions caused by a fungal infection exhibit a fluorescent yellow-green or blue-green color when examined with a Wood lamp.

Which question gives the nurse additional information about a patient's report of his hands shaking for the last two months

does the shaking occur when your hands are at rest or when you are picking up an item?"


Kaugnay na mga set ng pag-aaral

Debt Securities: Municipal Bonds

View Set

Chapter 5 Psychology: States of Consciousness

View Set

Principles of Microeconomics (Eco101) Exam II Review

View Set

A&P 4--Structural/Functional Classification of Neurons

View Set